Está en la página 1de 276

www.eltemario.

com Oposiciones Secundaria – Matemáticas


© Antonio J. Abrisqueta Valcárcel Hoja de Problemas nº1 – Algebra 1

1. Hallar un número cuadrado perfecto de cinco cifras sabiendo que el producto


de esas cinco cifras es 1568.

Solución:

Sea n2 = x1 · 104 + x2 · 103 + x3 · 102 + x4 · 10 + x5 el número que buscamos y


sea n = a · 102 + b · 10 + c la raíz cuadrada del número que buscamos.

Entonces debe ocurrir que:

x1 · 104 + x2 · 103 + x3 · 102 + x4 · 10 + x5 =


= a2 · 104 + 2ab103 + (b2 + 2ac) · 102 + 2bc10 + c2

Por otro lado sabemos que 1568 = 72 · 25 , entonces se tiene que n2 debe estar
formado por las siguientes posibles cifras:

1) 7, 7, 8, 4, 1
2) 7, 7, 8, 2, 2
3) 7, 7, 4, 4, 2

La opción 2) no es posible porque un cuadrado no puede tener nunca como cifra de


las unidades un 7, un 8 o un 2.

Por otro lado la opción 3) tampoco puede ser, pues la suma de sus cifras es 24, que
es dursible por 3, pero no por 9, por lo tanto no pueden formar un cuadrado perfecto.

Por lo tanto las cifras de n2 deben ser 7, 7, 8, 4, 1 y bajo estas condiciones se tiene
que x5 debe ser 1 o 4 y x4 debe ser par. Entonces se tiene que los posibles valores de n2
son:

17784 77841 74781


71784 78741 77481
77184 87741 47781

Pero de todos ellos el único que es un cuadrado perfecto es:

77841 = (279)2
Entonces:
N2 = 77841

2. Encontrar un número “abcd” de 4 cifras en base 12, tal que es cuadrado


perfecto y además los números “ab” y “cd” son consecuentes en base 12.

Solución:

Como bien nos dice el problema, se tiene que ab + 1 = cd,

1/7
www.eltemario.com Oposiciones Secundaria – Matemáticas
© Antonio J. Abrisqueta Valcárcel Hoja de Problemas nº1 – Algebra 1

y por otro lado1000012) ≤ abcd < 10000012)

⇒ Pasando la desigualdad a base 10, se tiene que 1728 ≤ n2 < 20738

donde denotamos por n2 el número abcd pero en base 10.

Entonces tenemos que 42 ≤ n < 144

Por otro lado tenemos que abab = abcd – 1 ⇒


⇒ a · 123 + b · 122 + a · 12 + b = 145(12ª + b) = n2 – 1 ⇒
⇒ 29 · 5 · (12ª + b) = (n – 1)(n + 1).

Por lo tanto (n – 1) o (n + 1) deben ser múltiplos de 29 y como 42 ≤ n < 144,


entonces:

• Si n – 1 es múltiplo de 29, y como (n – 1)(n + 1) debe ser múltiplo de 5


entonces n = 59 ⇒ n2 = 3481 = 202112) que verifica las condiciones.

• Si n + 1 es múltiplo de 29 y como (n – 1)(n + 1) debe ser múltiplo de 5


entonces n = 86 ⇒ n2 = 7396 = 434412) que verifica las condiciones del problema.

Por lo tanto hay dos soluciones posibles para este problema:

n1 = 202112) y n2 = 434412)

3. En un sistema de numeración cuya base se desconoce, dos números se escriben


302 y 402. El producto de ambos números es 75583 en el sistema de numeración
de base 9. Hallar la base desconocida.

Solución:
Tenemos que:

302n) · 402n) = 755839)

Entonces pasándolo a base diez se tiene que:

(3 · n2 + 2)(4 · n2 + 2) = 7 · 94 + 5 · 93 + 5 · 92 + 8 · 9 + 3 ⇒

⇒ (3n2 + 2)(4n2 + 2) = 50052 ⇒

⇒12n4 + 6n2 + 8n2 + 4 = 50052 ⇒ 12n4 + 14n2 – 50048 = 0 ⇒

⇒ 6n4 + 7n2 – 25024 = 0, haciendo n2 = m tenemos:

2/7
www.eltemario.com Oposiciones Secundaria – Matemáticas
© Antonio J. Abrisqueta Valcárcel Hoja de Problemas nº1 – Algebra 1

− 7 ± 49 + 600576
6m2 + 7m – 25024 = 0 ⇒ m = =
12
64

− 7 ± 775
= =
12
)
-65´1 6 lo rechazamos

⇒ n2 = 64 ⇒ n1 = 8 o n2 = -8 que lo rechazamos ⇒ la soluciónes n1 = 8

4. Demuestre que para todo número natural n, n ≥ 1, se tiene:

2n
1 2 n ( −1) m +1
∑ =∑
k =n +1 k m =1 m

Solución:
 2n
1 2 n (−1) m +1 
Sea Α = n ∈ ΙΝ / ∑ = ∑ 
 k = n +1 k m =1 m 

ι) ¿1∈Α?

∑k = 2
1 1
k=2

⇒ 1∈Α
2
( −1) m +1
(−1)2
( −1) 3
1 1
∑ m
=
1
+
2
=1 − =
2 2
m =1

2n
1 2 n ( −1) m +1
ιι) Supongamos cierto que n∈Α, es decir: ∑ =∑ .
k =n +1 k m =1 m

2 n +2
1 2 n +2 (−1) m +1
Veamos si (n + 1)∈Α, ¿ ∑ =∑ ?
k =n + 2 k m =1 m

2 n +2 2 n +2 2n

∑ ∑ ∑
1 1 1 1 1 1 1 1
= − + = − + + =
k =n + 2 k n + 1 n + 1 k =n + 2 k k =n +1 k n + 1 2n + 1 2 n + 2

1
2n
(−1) m +1
1 1 2n
( −1) m +1 (−1) 2 n + 2 ( −1) 2 n +3
=∑ + − + 2 =∑ + + =
m =1 m 2 n + 1 n + 1 n + 1 m =1 m 2n +1 2n + 2

(−1) m +1
2 n +2
= ∑ m
⇒ (n + 1)∈Α
m =1
⇒ Α = ΙΝ

3/7
www.eltemario.com Oposiciones Secundaria – Matemáticas
© Antonio J. Abrisqueta Valcárcel Hoja de Problemas nº1 – Algebra 1

5. Hallar un número de cinco cifras diferentes que sea igual a la suma de todos los
de tres cifras que se pueden obtener formando todas las variaciones ordinarias
de dichas cinco cifras tomadas de tres en tres.

Solución:
Supongamos que N = x1 x2 x3 x4 x5 es el número pedido con x1 ≠ 0, porque si
no, no tendría cinco cifras.

Como con esas cinco cifras queremos formar números de tres cifras se tiene que
hay V5,3 = 60 posibilidades, de las cuales hay 12 que tienen una cifra determinada
en una posición determinada.

Por tanto tenemos que:

12(x1 + x2 + x3 + x4 + x5 ) + 12(x1 + x2 + x3 + x4 + x5 ) · 10 +

+ 12(x1 + x2 + x3 + x4 + x5 ) · 100 = 1332(x1 + x2 + x3 + x4 + x5 )

y por lo tanto:

N = 1332(x1 + x2 + x3 + x4 + x5 )

Entonces N es múltiplo de 9 porque 1332 lo es, y utilizando el criterio de


divisibilidad del 9 se tiene que:

x1 + x2 + x3 + x4 + x5 = 9t

y dado que todas las cifras son distintas entre si, su suma estará entre:

1 + 2 + 3 + 4 + 5 = 15 y 5 + 6 + 7 + 8 + 9 = 35

⇒ 15 ≤ x1 + x2 + x3 + x4 + x5 ≤ 35

Por lo tanto los únicos valores posibles de t son t=2 y t = 3.

• Si t = 2 entonces N = 1332 · 9 · 2 = 23976, y como la suma de sus cifras


es 27 ≠ 18 = 9 · 2 entonces contradice que x1 + x2 + x3 + x4 + x5 = 9t
⇒ Este número no es el buscado.

• Si t=3 entonces N = 1332 · 9 · 3 = 35964 y como

3 + 5 + 9 + 6 + 4 = 27 = 9 · 3 entonces N = 35964 es la solución del problema.

6. Dados los códigos ordenados de cinco letras entre las ocho: A, B, C, D, E, F, G,


H (repetidas o no, se pide hallar:

a) Número total de códigos.


b) 1) Número de ellos con una sola letra repetida dos veces. Ejemplo: ABACH.

4/7
www.eltemario.com Oposiciones Secundaria – Matemáticas
© Antonio J. Abrisqueta Valcárcel Hoja de Problemas nº1 – Algebra 1

2) Número de ellos con dos letras repetidas dos veces cada una. Ejemplo:
ABBCA.
3) Número de ellos con una letra repetida tres veces. Ejemplo: ABAAE.
4) Número de ellos con una letra repetida tres veces y otra dos. Ejemplo:
AABAB.
5) Número de ellos con una letra repetida cuatro veces.
6) Número de ellos con una letra repetida cinco veces.
7) Número de los que no estén comprendidos en los grupos anteriores.
8) Supuestas ordenadas las letras alfabéticamente, calcular el número de
códigos formados por cinco letras consecutivas en dicho orden. Ejemplo:
DGFHE.
c) Supuesto el orden lexicográfico entre los códigos, hallar el que corresponde
al 1729.

Solución:
a) El número total de códigos viene dado por las variaciones con repetición de 8
elementos tomados de 5 en 5.

VR8,5 = 85 = 32768

b) 1) C5,2 · 8 · V7,3 = 16800

2) C5,2 · C3,2 · C8,2 · 6 = 5040

3) C5,3 · 8 · V7,2 = 3360

4) C5,3 · 8 · 7 = 560

5) C5,4 · 8 · 7 = 280

6) C5,5 · 8 = 8

7) Le restamos al total de códigos posibles la suma de los anteriores:


32768 – 26048 = 6720

8) Elegidas cinco letras consecutivas el número de formas diferentes de


ordenarlas es P5 = 5! = 120 y como hay cuatro formas diferentes de elegir
cinco letras consecutivas entre ocho, se tiene que:
4 · 120 = 480

c) Si definimos la aplicación f = {A, B, C, D, E, F, G, H} → {0, 1, 2, 3, 4, 5, 6, 7}


dada por: f(A) = 0, f(B) = 1, f(C) =2, f(D) = 3, f(E) = 4, f(F) = 5, f(G) = 6, f(G) = 7.
El orden lexicográfico de los códigos coincide con el orden de los números en base a 8,
por ejemplo: el 00000 corresponde a AAAAA entonces la posición 1729 en verdad es el
número 1728 según nuestra aplicación, es decir 1728 = 033008) = ADDAA.

Entonces la solución es ADDAA

7. Dos mujeres y tres hombres suben a un ascensor en la planta baja de un


edificio de seis pisos. Averiguar de cuantas maneras se pueden bajar del

5/7
www.eltemario.com Oposiciones Secundaria – Matemáticas
© Antonio J. Abrisqueta Valcárcel Hoja de Problemas nº1 – Algebra 1

ascensor, sabiendo que en un mismo piso no pueden bajar personar de distinto


sexo.

Solución:
Hay que tener en cuenta dos posibilidades a la hora de contar, y es que las mujeres
se pueden bajar en la misma planta o en distintas plantas.

• Si las mujeres se bajan, las dos, en la misma planta tenemos cinco formas
diferentes de que se bajen, ya que como se suben en la planta baja, se pueden bajar en
cualquiera de las otras cinco plantas. Para cada una de las cinco formas de bajarse las
 3 + 4 − 1 6!
mujeres hay   = = 20 formas de bajarse los hombres, que son las
 3  3!·3!
combinaciones con repetición de 4 elementos tomados de 3 en 3.

 5
• Si las mujeres se bajan en pisos distintos se tiene que hay   = 10 formas
 2
 3 + 3 − 1
diferentes de bajarse, y los hombres tendrán   = 10 formas diferentes de
 3 
bajarse, combinaciones con repetición de 3 elementos tomados de 3 en 3.

Por lo tanto:

N = 5 · 20 + 10 · 10 = 200 ⇒ El resultado es N = 200

8. Determinar el número máximo de puntos de intersección de las diagonales de


un polígono convexo de n lados:
a)Contenidos en el interior de aquel.
b) Situados en su exterior.

Solución:
a) Cada cuatro vértices distintos del polígono definen dos diagonales que se cortan
en un punto de intersección interior. Entonces:

 n
Pi =  
 4

b) Para poder calcular los puntos exteriores de intersección entre las diagonales,
vamos a calcular los puntos de intersección totales entre las diagonales y luego
restaremos las interiores, obtenemos así los puntos de intersección exteriores entre las
diagonales.

 n n( n − 3)
Como tenemos n vértices y n lados, entonces tenemos   − n =
 2 2

6/7
www.eltemario.com Oposiciones Secundaria – Matemáticas
© Antonio J. Abrisqueta Valcárcel Hoja de Problemas nº1 – Algebra 1

Diagonales distintas (que no son los lados del triángulo). Como en cada vértice
 n − 3
concurren n - 3 diagonales, tenemos por lo tanto que hay  
 2 
diagonales que se cortan en cada vértice. Como cada dos diagonales hay un punto de
intersección se tiene que:

 n (n − 3   n − 3 n( n − 3)( n 2 − 7n + 14)
PT =  2  − n =
   2  8
 2 

Entonces tenemos los puntos exteriores de intersección entre las diagonales vienen
dados por:

n( n − 3)( n 2 − 7 n + 14)  n  n( n − 3)( n − 4)( n − 5)


Pe = PT – Pi = −   =
8  4 12

Entonces las soluciones son:

 n
a) Pi =  
 4

n( n − 3)( n − 4)( n − 5)
b) Pe =
12

7/7
www.eltemario.com Oposiciones Secundaria – Matemáticas
© Antonio J. Abrisqueta Valcárcel, 2001 Hoja de Problemas nº2 – Algebra II

9. Hallar un número de cuatro cifras que sea igual al cubo de la suma de las
cifras.

Solución:

Sea x1 x2 x3 x4 el número buscado con x1 ≠ 0 ya que si no, no seria de cuatro cifras.

Tenemos que x1 x2 x3 x4 = (x1 + x2 + x3 + x4 )3 y como

1000 ≤ x1 x2 x3 x4 < 10000 ⇒ 1000 ≤ (x1 + x2 + x3 + x4 )3 < 10000 ⇒

⇒ 10 ≤ (x1 + x2 + x3 + x4 ) ≤ 21

Como todo número se puede escribir de la forma 3n – 1, 3n o 3n + 1, entonces


el cubo de dicho número, es decir, x1 x2 x3 x4 se puede escribir de la forma 9K – 1,
9K, 9K + 1, y aplicando el criterio de divisibilidad del 9, se tiene que
x1 + x2 + x3 + x4 es de la forma 9K – 1, 9K o 9K + 1, es decir, que
x1 + x2 + x3 +x4 tiene que ser el 10, el 17, el 18 o el 19. Entonces:

103 = 1000 ≠ 13 → Este número no es solución.

173 = 4913 = (4 + 9 + 1 + 3)3 = 173 ⇒ Es solución del problema.

183 = 5832 = (5 + 8+ 3 + 2)3 = 183 ⇒ Es solución del problema.

193 = 6859 ≠ (6 + 8 + 5 + 9)3 = 283 ⇒ Este número no es solución.

La solución del problema es:

n1 = 4913 n2 = 5832

10. Hallar en el sistema de base 9 un número formado por tres cifras significativas,
tal que transportado al sistema de base 13 se escriba con las tres mismas cifras.

Solución:
Sea N el número buscado, como tiene las tres mismas cifras significativas en
base 9 y en base 13 se tiene que 132 ≤ N < 93 ⇒ 169 ≤ N ≤ 728, o lo que es lo
mismo 10013) ≤ N ≤ 44013).

Supongamos que N = x1 x2 x3 entonces por lo dicho anteriormente se tiene


13)

que 1 ≤ x1 ≤ 4, 0 ≤ x2 ≤ 8 y 0 ≤ x3 ≤ 8.

Entonces:

a) Si N = x1 x2 x3 ⇒ 169x1 + 13x2 + x3 = 81x1 + 9x2 + x3 ⇒


9)

⇒ 88x1 + 4x2 = 0 que no tiene solución.

1/8
www.eltemario.com Oposiciones Secundaria – Matemáticas
© Antonio J. Abrisqueta Valcárcel, 2001 Hoja de Problemas nº2 – Algebra II

b) Si N = x1 x3 x2 ⇒ 169x1 + 13x2 + x3 = 81x1 + 9x3 + x2 ⇒


9)

⇒ 88x1 + 12x2 = 8x3 pero 8x3 ≤ 64 < 88x1 + 12x2 ⇒


⇒ No tiene solución.
c) 160x1 = 68x2 ⇒ 40x1 = 17x2 pero 40x1 con 1 ≤ x1 ≤ 4 no es múltiplo de 17 ⇒
⇒ No tiene solución.
d) Si N = x2 x3 x1 9 ) ⇒ 169x1 + 13x2 + x3 = 81x2 + 9x3 + x1 ⇒

⇒ 42x1 = 17x2 + 2x3 donde x2 ha de ser par:

• para x1 = 1 ⇒ 42 = 17x2 + 2x3 con x2 = 2 y x3 = 4


• para x1 = 2 ⇒ 84 = 17x2 + 2x3 ⇒ x2 = 4 y x3 = 8
• para x1 = 3 ⇒ 126 = 17x2 + 2x3 que no tiene solución
• para x1 = 4 ⇒ 168 = 17x2 + 2x3 que no tiene solución.
e) Si N = x3 x1 x2 9 ) ⇒ 169x1 + 13x2 + x3 = 81x3 + 9x1 + x2 ⇒

⇒ 160x1 + 12x2 = 80x3 ⇒ 40x1 + 3x2 = 20x3 ⇒ 3x2 = 20(x3 – 2x1 ) ⇒


⇒ x3 – 2x1 tiene que ser múltiplo de 3 ⇒
⇒ tenemos:
• x1 = 1, x2 = 0, x3 = 2
• x1 = 2, x2 = 0, x3 = 4
• x1 = 3, x2 = 0, x3 = 6
• x1 = 4, x2 = 0, x3 = 8

porque si x3 – 2x1 ≥ 3 ⇒ x2 > 8

f) Si N = x3 x2 x1 9 ) ⇒ 169x1 + 13x2 + x3 = 81x3 + 9x2 + x1 ⇒


⇒ 168x1 + 4x2 = 80x3 ⇒ 42x1 + x2 = 20x3 ⇒ x2 tiene que ser par, es decir,

x2 = 2(10x3 – 21x1 ), pero como 1≤ x1 ≤ 4 ⇒ 10x3 – 21x1 es mayor que 6 ⇒

⇒ No hay solución.

Las soluciones son:

Base 9 214 482 210 420 630 840


Base 13 124 248 102 204 306 408

2/8
www.eltemario.com Oposiciones Secundaria – Matemáticas
© Antonio J. Abrisqueta Valcárcel, 2001 Hoja de Problemas nº2 – Algebra II

11. Probar que 11n – 4n es múltiplo de 7.

Solución:

Sea A = {n∈Ð/11n – 4n = 7k}

ι) ¿1∈A?

11 – 4 = 7 ⇒ 1∈A

ιι) Supongamos cierto que n∈A, es decir: 11n – 4n = 7k

Veamos si n + 1∈A, ¿11n+1 – 4n+1 = 7k1 ?

11n+1 – 4n+1 = 11 · 11n – 4 · 4n = (7 + 4) · 11n – 4 · 4n =

= 7 · 11n + 4 · 11n – 4 · 4n = 7 · 11n + 4(11n – 4n ) =

= 7 · 11n + 4 · 7k = 7(11n + 4k) = 7k ⇒ n + 1 + 1∈A

⇒A=Ð

12. Los coeficientes de los términos tn , tn+1 , tn+2 que ocupan los lugares n, n+1, n+2
en el desarrollo de (a + b)14 están en progresión aritmética. Calcular n sabiendo
que es menor que 7.

Solución:
14
14  j 14 − j
El desarrollo del binomio de Newton para (a + b)14 es (a + b)14 = ∑ 
j= 0
a b
j 
Entonces las posiciones n, n + 1 y n + 2 las ocupan los términos:

 14  n −1 15−n 14   14  n +1 13 −n
tn =   a b ; t n +1 =   a n b14 −n ; t n + 2 =  a b
 n − 1 n  n + 1

y como los coeficientes están en progresión aritmética, se tiene que:

14   14   14  14 
  −   =   −  
 n   n − 1  n + 1  n 

14! 14! 14! 14!


⇒ − = −
n!(14 − n )! ( n − 1)(15 − n)! ( n + 1)! (13 − n )! n! (14 − n)!

(n − 1)! (13 − n)!


pero multiplicando por nos queda:
14!

1 1 1 1
− = − ⇒
n(14 − n) (14 − n)(15 − n) n( n + 1) n(14 − n )

3/8
www.eltemario.com Oposiciones Secundaria – Matemáticas
© Antonio J. Abrisqueta Valcárcel, 2001 Hoja de Problemas nº2 – Algebra II

⇒ (n + 1)(15-n) – n(n + 1) = (14 – n)(15 – n) – (n + 1)(15 – n) ⇒

9
14 ± 196 − 180 14 ± 4
⇒ n2 – 14n + 45 = 0 ⇒ n = = =
2 2
5

pero como n < 7 ⇒ n = 5

13. Se colocan al azar n bolas en n urnas. Calcular las probabilidades siguientes:


a) de que las n urnas queden ocupadas
b) de que quede una sola urna vacía

Solución:
a) El número de cosas favorables es n! que son las n formas diferentes de colocar n
bolas en n urnas (permutaciones sin repetición de n elementos).
Entonces:
n!
P= n
n

b) El número de cosas posibles es nn que es el número de variaciones con repetición


de n elementos tomados de n en n.

El número de cosas favorables viene dado por:

• Primero elegimos que urnas vamos a dejar vacía entre las n que hay y esto es
 n
  = n .
 1

• Segundo elegimos que urna va a tener dos bolas, porque las demás van a tener
 n − 1
una, y esto es   = n − 1.
 n 

• Tercero elegimos que dos bolas de las n que hay vamos a situar en la urna que
 n  n( n − 1)
tendrá dos bolas, esto es,   = .
 2 2

• Y por último situemos las (n – 2) bolas restantes en las (n – 2) urnas restantes ⇒


(n – 2)!

n (n − 1)
Entonces las cosas favorables son: n (n − 1) ( n − 2)!
2

4/8
www.eltemario.com Oposiciones Secundaria – Matemáticas
© Antonio J. Abrisqueta Valcárcel, 2001 Hoja de Problemas nº2 – Algebra II

Entonces la probabilidad es:

n( n − 1)
n( n − 1) ( n − 2)! ( n − 1) 2 (n − 2)!
P= 2 =
nn 2·n n −2

Soluciones:

n!
a) P =
nn

( n − 1) 2 (n − 2)!
b) P =
2n n − 2

14. En un armario hay n pares de zapatos distintos, es decir, cada par es diferente
de los restantes pares. Se toman r zapatos al azar. Se pide la probabilidad de
que entre los zapatos elegidos aparezcan exactamente h pares.

Solución:
 2n 
El número de casos posibles es   que son el número de combinaciones de 2n
 r 
elementos tomados de r en r.

El número de casos favorables viene dado por:

 n
• Primero elegimos h pares entre los n que hay, esto es:  
 h
• En segundo lugar debemos elegir entre los (n – h) pares que quedan los r – 2h de
n − h 
los cuales vamos a sacar un zapato de cada uno, esto es:  
 r − 2h 
 2
• Y por último, hay   formas de elegir un zapato de las dos que hay en cada par,
1 
de manera que nunca elijamos un par completo.

 n  n − h 
Entonces hay n =    2 casos favorables.
 n  r − 2h 
Entonces la probabilidad viene dada por:

 n  n − h 
  2
 n  r − 2h 
P=
 2n
 
r 

5/8
www.eltemario.com Oposiciones Secundaria – Matemáticas
© Antonio J. Abrisqueta Valcárcel, 2001 Hoja de Problemas nº2 – Algebra II

15. Resolver en 9 7 el sistema:


x − 2y + 5z = 2

3x + y − 2 z = 1
5 x + 4 y + z = 5

Solución:

 x − 2 y + 5z = 2 1 − 2 5 2 1 − 2 5 2 
    
3x + y − 2 z = 1 ⇒  3 1 − 2 1   →  0 7 − 17 − 5  →
F2' = F2 −3 F1
5 x + 4 y + z = 5 5 4 1 5  F3 '=F3 − 5 F1  0 14 − 24 − 5 
 

 1 − 2 5 2 1 − 2 5 2
    x − 2 y + 5z = 2
→  0 0 4 2   → 0 0 4 2  ⇒
 0 0 4 2
F '3 = F3 − F2
 0 0 0 0 4z = 2
   

⇒ z = 4 ⇒ x – 2y + 20=2 → x = 2y – 18 ⇒ x = 2y + 3

x = 2y + 3
⇒ con y∈97
z = 4

16. Demostrar que siendo n un número entero, la expresión

n5 − 5n 3 + 4n
n+2
siempre es divisible por 24.

Solución:

n5 − 5n 3 + 4n
Sea xn =
n+2

Vamos a intentar factorizar el numerador ya que, si uno de los factores fuese el


propio (n + 2) podríamos simplificar la expresión.

n5 – 5n3 + 4n = n(n4 – 5n2 + 4) = n(n2 – 4)(n2 – 1) = n(n – 2)(n + 2)(n – 1)(n + 1)

entonces xn = (n – 2)(n – 1) n(n + 1).

6/8
www.eltemario.com Oposiciones Secundaria – Matemáticas
© Antonio J. Abrisqueta Valcárcel, 2001 Hoja de Problemas nº2 – Algebra II

Como queremos demostrar que xn es divisible por 24 = 8 · 3, es suficiente que


demostremos que xn es divisible por 8 y por 3.

• xn será divisible por ocho, porque como tenemos que xn es un producto de 4


número consecutivos, entonces como mínimo dos de ellos son pares, y como cada
cuatro números hay uno que es divisible por 4 entonces uno de ellos será divisible por 4
y el otro al ser par por 2, entonces xn es divisible por 8.

• xn es divisible por 3 porque cada tres números consecutivos hay uno que es
divisible por 3 ⇒ xn es divisible por 3.

⇒ xn es divisible por 24.

17. a) Sea x un número racional, ¿qué condición debe cumplir para que existan y
1 1
sean distintos x, − x, ,− ?.
x x
b) Sean x e y dos números racionales que cumplen la condición del apartado a),
1 1
y además las siguientes: x < y, < . Indicar que números son positivos
x y
del conjunto.
 1 1 1 1
H =  x,− x , ,− , y, − y , ,− 
 x x y y

1
c) Si además se cumple y = MaxH, < x, ordenar de menor a mayor los
x
números del conjunto HU{0, -1,1}.

Solución:

a) Para que x y –x sean distintos, como x = - x ⇒ 2x = 0 ⇒ x = 0 debe ocurrir que


x ∉{0}. Pero por otro lado se tiene que si x = ⇒ x 2 = 1 ⇒ x = ±1 ⇒ x ∉ {− 1,0,1} y
1
x
1 1
como x ≠ − y − x ≠ ⇒ la solución de este apartado es:
x x
x ∉ {− 1,0,1}

1 1
b) Como x < y y además < ⇒ x e y tienen distinto signo, ya que si no
x y
1 1
x<y ⇒ < . En concreto x < 0 < y ⇒ Se tiene que los elementos positivos del
y x
conjunto H (que los denotaremos como el conjunto H+ ) serán:

1 1
H+ = {- x, − , y, }
x y

7/8
www.eltemario.com Oposiciones Secundaria – Matemáticas
© Antonio J. Abrisqueta Valcárcel, 2001 Hoja de Problemas nº2 – Algebra II

1 1
c) Como < x con x< 0, se tiene que x > -1 y por lo tanto < -1 y además
x x
1 1
y = MaxH ⇒ y > 1 ⇒ < 1. Pero al ser y = MaxH ⇒ y > − . Por lo tanto, el orden
y x
de los elementos es:
1 1 1 1
- y< < -1 < x < − < 0 < < - x < 1 < - < - y
x y y x

8/8
www.eltemario.com Oposiciones Secundaria – Matemáticas
© Antonio J. Abrisqueta Valcárcel, 2001 Hojas de Problemas – Algebra III

18. a) Demostrar que si n es par, los números naturales n2 – 1 y 3n + 1 son primos


entre si.
b) Demostrar que si n = 30m, entonces la cantidad de números enteros positivos
distintos de cero que no son mayores que m y que no se dividen por ninguno de
los números 6, 10, 15 es igual a 22m.

Solución:

a) Supongamos que a es un divisor primo de n2 – 1, entonces a divide a n – 1 o a


n + 1. En caso de que divida a n – 1 entonces divide a 3n – 3, pero si divide a n + 1
entonces también divide a 3n + 3.

Supongamos que divide también a 3n + 1, entonces divide a (3n + 1)–(3n – 3)=4 o


divide a (3n + 3) – (3n + 1) = 2, entonces a = 2.

Pero como 3n + 1 y n2 – 1 son impares (por ser n par), entonces a = 2, no puede


dividirlos, entonces son primos entre si.

b) Como hay 30m enteros positivos no mayores que n y se cumple que:

n 30m
• hay = = 5m múltiplos de 6 no mayores que n
6 6
n 30m
• hay = = 3m múltiplos de 10 no mayores que n
10 10
n 30m
• hay = = 2m múltiplos de 15 no mayores que n
15 15

n
Pero como 30 es en mcd de 6 y 10, de 6 y 15 y de 10 y 15 ⇒ hay =m
30
múltiplos de 30 no mayores de n que los hemos contado como múltiplos de 6, 10 y 15.

El número de enteros positivos buscado es:

30m – (5m + 3m + 2m – m – m) = 22m

La solución es: 22m

19. Un número tiene 24 divisores, su mitad tiene 18 divisores y su triple 28


divisores. Hallar el número.

Solución:

Supongamos que n = 2 a1 ⋅ 3a 2 ⋅ n3a 3 ⋅ .... ⋅ n5a 5 es el número buscado, con 2, 3,


n3 ,…, n3 primos.

Entonces se tiene que como:

1/8
www.eltemario.com Oposiciones Secundaria – Matemáticas
© Antonio J. Abrisqueta Valcárcel, 2001 Hojas de Problemas – Algebra III

n
= 2 a1 −1 ⋅ 3a 2 ⋅ n3a 3 ⋅ ... ⋅ n5a 5
2
3n = 2 a1 ⋅ 3a 2 +1 ⋅ n3a 3 ⋅ ... ⋅ n5a 5

Se verifica que:

24 = (a1 + 1)( a2 + 1) ⋅ ... ⋅ ( a5 + 1)



18 = a1 ( a 2 +1) ⋅ ... ⋅ ( a5 + 1)
28 = ( a + 1)( a + 2) ⋅ ... ⋅ ( a + 1)
 1 2 5

Dividiendo la 1ª ecuación por la 2ª ecuación tenemos:

24 a1 + 1
= ⇒ a1 = 3
18 a1

Dividiendo la 3ª ecuación por la 2ª ecuación tenemos:

28 a2 + 2
= ⇒ a2 = 5
18 a 2 + 1

Como tenemos que 24 = (3 + 1)(5 + 19 entonces a3 = a4 =….= a5 = 0 ⇒


⇒ n = 23 ⋅ 35 = 1944 → n = 1944

20. ¿Cuántas cifras tiene el menor número natural que cumple que, cuando la
primera cifra de la izquierda se coloca en el último lugar de al derecha, el
número que resulta es una vez y media el número inicial?.

Solución:

Si llamamos m al número de cifras, nm a la primera cifra y N al número que queda


después de haber suprimido la primera cifra. Entonces.

10N + nm =
3
2
(nm ⋅ 10 m −1 + N )

⇒ nm(3 ⋅ 10m – 1 – 2) = 17N.

Entonces 3 · 10m – 1 – 2 debe ser múltiplo de 17 pues el número 17 es primo ⇒

⇒ 3 · 10m – 1 – 2 ≡ 0(17) ⇒ 3 · 10m – 1 ≡ 2(17) ⇒ 3 · 10m ≡ 20(17) ⇒ 3 · 10m ≡ 3(17) ⇒

⇒ 10m ≡ 1(17) ⇒ m 16 por el teorema de Fermat.

Como 10 y 17 son primos entre si, y el indicador de 17 es 16, el menor número que
verifica la periodicidad de los restos potenciales tiene que ser un divisor de 16.

2/8
www.eltemario.com Oposiciones Secundaria – Matemáticas
© Antonio J. Abrisqueta Valcárcel, 2001 Hojas de Problemas – Algebra III

Entonces como.

101 ≡ -7 (17) 102 ≡ -2 (17)

104 ≡ 4 (17) 106 ≡ -1 (17)

⇒ El número buscado es el propio 16.

Entonces la cifra que buscamos tiene 16 cifras.

21. Demostrar que 33n+3 – 26n – 27 es múltiplo de 169 para todo n entero positivo.

Solución:

Sea xn = 33n+3 – 26n – 27.

1) Efectivamente si n = 1 entonces x1 = 36 – 26 – 27 = 676 = 4 · 169


entonces xn es múltiplo de 169 para n = 1.

2) Si demostramos que xn+1 ≡ xn (169) entonces aplicando el principio de


inducción tendremos que la propiedad se verifica ∀n∈9+. Demostrar que xn+1 ≡ xn
(169) es lo mismo que comprobar que xn+1 – xn ≡ 0(169).

Xn+1 – xn = 33(n+1) + 3 – 26(n + 1) – 27 – 33n+3 + 26n + 27 =

= 33n+3 · 33 – 26n – 26n – 26 – 27 – 33n+3 + 26n + 27 = (33 – 1)33n+1 – 26 =

= 26(33n+3 – 1) = 26((33 )n+1 – 1) = 26(27n+1 – 1) = {Aplicando la fórmula del

polinomio ciclotómico} = 26(27 – 1)(27n + 27n-1 + ….. + 27 + 1) =

= 262 (27n + 27n-1 + … + 27 + 1) = 676 · (27n + 27n-1 + …. + 27 + 1) =

= 169 · 4 · (27n + 27n+1 + …. + 27 + 1) = 0(169) ⇒ xn+1 ≡ xn (169)

Entonces por el principio de inducción 33n+3 – 26n – 27 es múltiplo de 169


∀n∈9 . +

22. Probar que si m y n son enteros primos entre si, y a y b enteros cualesquiera,
existe un entero x tal que
x ≡ a(m) y x = b(n)

Solución:

Como m y n son enteros primos entre si, tenemos entonces, utilizando la identidad
de Bezout que:
1 = αm + βn

3/8
www.eltemario.com Oposiciones Secundaria – Matemáticas
© Antonio J. Abrisqueta Valcárcel, 2001 Hojas de Problemas – Algebra III

Pero multiplicando esta expresión por b – a se tiene que:

b – a = (b – a) αm + (b – a)βn

⇒ b + (a – b) βn = a + (b – a) α m

⇒ Definimos x como este número, es decir.

x = b + (a – b)βn que es lo mismo que

x = a + (b – a)α m porque son iguales,

y como se ve, se verifica que:

x ≡ a(m) y x ≡ b(n)

23. Sea Q(K) = {xy-1 ∈ K; x, y ∈ Z(K), y ≠ 0}, a los elementos de este subconjunto se
les llama racionales del cuerpo K. Demostrar que:

a) Q(K) es un subcuerpo de K.
b) Demostrar que es el menor subcuerpo de K.

Solución:

x
Tomaremos los elementos de Q(K) como xy-1 = , que nos será más cómodo.
y

a c ad ± bc
a) i) ± = con b ≠ 0 y d ≠ 0 y como se cumple que ad, bc y bd ∈ (K) ⇒
b d bd
ad ± bc
∈ Q(k ) .
bd

 a  ⋅  c  = ac con b ≠ 0 y d ≠ 0 y como ac y bd ∈ Z(K) ⇒ ac ∈ Q(K ).


ii)    
 b   d  bd bd

a
 
iii)  b 
ad ad
= con b ≠ 0, c ≠ 0 y d ≠ 0 y como ad y bc ∈ Q(K) ⇒ ∈ Q(K )
c bc bc
 
d 
⇒ Q(K) es un subcuerpo de K

b) Sea K´ un subcuerpo cualquiera de K, entonces como 1∈ K´ (por ser un


subcuerpo de K) y si x ∈ K´ ⇒ x´+ 1 ∈ K´ (por propiedad aditiva), entonces por el
principio de inducción N(K) ⊂ K´. Es evidente también que O ∈ K´(por ser un
subcuerpo de K) y si x ∈ K´ ⇒ (- x) ∈ K´ entonces Z(K) ⊂ K´. Y por la propia
definición de Q(K) podemos decir que Q(K)⊂ K´ ⇒

4/8
www.eltemario.com Oposiciones Secundaria – Matemáticas
© Antonio J. Abrisqueta Valcárcel, 2001 Hojas de Problemas – Algebra III

⇒ Q(K) es el menor subcuerpo de K.

24. Demostrar que 2 , 3, 5 no pueden ser términos de una progresión


aritmética.

Solución:

Supongamos que si son términos de una progresión aritmética de diferencia d,


entonces existen términos m y n tales que m ≠ 0, n ≠ 0 con:

3 − 2 = md y 5 − 3 = nd

Por lo tanto, podemos asegurar que:

3 − 2 md m 5 − 3 nd n
= = ∈ Q y que = = ∈Q
5 − 3 nd n 3 − 2 md m


3− 2
=
( 3− 2 )( 5+ 3 )
=
15 + 3 − 10 − 6
∈Q ⇒
5− 3 ( 5− 3 )( 5+ 3 ) 2


5− 3
=
( 5− 3 )( 3+ 2 )
= 15 + 10 − 3 − 6 ∈ Q ⇒
3− 2 ( 3− 2 )( 3+ 2 )
⇒ ( 15 + )( )
10 − 6 y 15 − 10 − 6 ∈ Q, entonces su diferencia es un nº racional
⇒ 2 10 ∈ Q (contradicción) ⇒ 2 , 3, 5 no son términos de una progresión
geométrica.

25. Demostrar que para todo número natural n, el número dado por
An = (n5 – n)(n4 + n2 – 6 es divisible por 210.

Solución:

Como tenemos que:

An = (n5 – n)(n4 + n2 – 6) = n(n4 – 1)(n4 + n2 – 6) = n(n2 – 1)(n2 + 1)(n4 + n2 – 6)

= n(n – 1)(n + 1)(n2 + 1)(n2 + 3)(n2 – 2).

Además tenemos que 210 = 2 · 3 · 5 · 7, bastará ver que An es divisible por


2, 3, 5, 7.

• Es divisible por 2 y por 3 puesto que n –1, n y n + 1 son tres números


consecutivos ⇒ uno o dos de ellos son pares y uno de ellos es múltiplo de 3, ya que

5/8
www.eltemario.com Oposiciones Secundaria – Matemáticas
© Antonio J. Abrisqueta Valcárcel, 2001 Hojas de Problemas – Algebra III

cada dos números consecutivos uno es par y cada tres números consecutivos uno es
múltiplo de 3.

• Es divisible por 5 puesto que:

a) Si n = 5p, ya está
b) Si n ≠ 5p utilizando el teorema de Fermat ⇒ n4 – 1 es múltiplo de 5.

• Es divisible por 7 puesto que:

a) Si n = 7p, ya está
b) Si n = 7p + 1 ⇒ n2 – 1 = (7p + 1)2 – 1= 49p2 + 14p + 1 – 1 =

= 7(7p2 + 2p) que es múltiplo de 7.

c) Si n = 7p + 2 ⇒ n2 + 3 = (7p + 2)2 + 3 = 49p2 + 28p + 4 + 3 =

= 7(7p2 + 4p + 1) que es múltiplo de 7.

d) Si n = 7p + 3 ⇒ (n2 – 2) = (7p + 3)2 – 2 = 49p2 + 42p + 9 – 2 =

= 7(7p2 + 6p + 1) que es múltiplo de 7.

e) Si n = 7p + 4 ⇒ (n2 – 2) = (7p + 4)2 – 2 = 49p2 + 56p + 16 – 2 =

= 7(7p2 + 8p + 2) que es múltiplo de 7.

f) Si n = 7p + 5 ⇒ (n2 + 3) = (7p + 5)2 + 3 = 49p2 + 70p + 25 + 3 =

= 7(7p2 + 10p + 4) que es múltiplo de 7.

g) Si n = 7p + 6 ⇒ (n2 – 1) = (7p + 6)2 – 1= 49p2 + 84p + 36 – 1 =

= 7(7p2 + 12p + 5) que es múltiplo de 7.

Luego An es divisible por 2, 3, 5, 7 ⇒ An es divisible por 210.

26. a) Demostrar que para todo número natural n y p (con p < n y n ≠ 0) el número
p −2
 n
An = (n + 1) - ∑   ni
n
es decir, por np.
i= 0  
i
b) Demostrar que para todo número natural n y p (con p < n y n ≠ 0) el número
p −2
 n
Bn = p ( p −1) n − ∑   ni es divisible por (pn – 1)p.
n

i =0  i 

Solución:

6/8
www.eltemario.com Oposiciones Secundaria – Matemáticas
© Antonio J. Abrisqueta Valcárcel, 2001 Hojas de Problemas – Algebra III

a) Tenemos que demostrar que:

n i
( )
p− 2
(n + 1)n − ∑  n ≡ 0 n p . Entonces:
i =0  i 

p −2
 n n
 n p− 2
 n n
 n
( n + 1) n − ∑   n i = ∑   ni − ∑   ni = ∑   ni =
i =0  i  i =0  i  i =0  i  i = p −1 i 

 n  p −1  n  p  n  p +1 n
=   n +  n +   n + ..... +  n n =
 p − 1  p  p + 1 n

n! n  n  p +1  n  n
= n p −1 +  n p +  n +  n =
( p − 1)! (n − p + 1)!  p  p + 1 n

 n 
 
 p − 1 p  n  p  n  p +1 n
= n +  n +   n + ...... +  n n =
n  p  p + 1 n

 n  
  p − 1 
n  n   n  n− p  n
( )
p− 2
p  
=n +   +   n + ..... +  n ⇒ (n + 1) − ∑  n i ≡ 0 n p
n

 n  p   p + 1 n  i= o  i 
 
 

p− 2
n 
⇒ (n + 1)n − ∑  n i es divisible por np .
i =0  i 

b) Sea a = p n − 1 ⇒ a p = p n − 1 ( ) p

( )
⇒ (a + 1)a = p n − 1 + 1 = p na = p n ( p
a n
−1 )

p −2
 n p− 2
 n p− 2
n
⇒ (a + 1)a − ∑   n i = p n ( p −1 ) − ∑   ni ⇒ (a + 1)a − ∑  n i = Bn
n

i =0  i  i =0  i  i= 0  i 

p− 2
 n
Y por el apartado a) tenemos que Bn es divisible por a p ⇒ p ( p −1 )n − ∑  n i es
n

i =0  i 
n p
divisible por (p – 1) .

27. a) Demostrar que existen infinitos números de la forma An = 10n + 3 que son
compuestos.
b) Hallar el menor número natural A tal que, dividido por 2 da de resto 1,
dividido por 3 da de resto 2, dividido por 4 da de resto 3, dividido por 5 da de

7/8
www.eltemario.com Oposiciones Secundaria – Matemáticas
© Antonio J. Abrisqueta Valcárcel, 2001 Hojas de Problemas – Algebra III

resto 4, dividido por 6 da de resto 5, dividido por 7 da de resto 6, dividido por 8


da de resto 7 y dividido por 9 da de resto 8.

Solución:

a) Vamos a demostrar que existen infinitos An compuestos viendo que existen


infinitos An que son divisibles por 7. Sea n = 6K + 4:

An = 10 n + 3(7 ) ⇒ An ≡ 106 K + 4 + 3(7 ) ⇒

⇒ An ≡ 10 4 ⋅10 6 K + 3(7) ⇒ An ≡ 3 4 ⋅ (10 6 ) + 3(7 ) ⇒


K

⇒ An ≡ 81 ⋅ (106 ) + 3(7 ) y como 106 ≡ 1(7) por el teorema de Fermat


K

⇒ A n ≡ 4 ⋅ 1K + 3(7 ) ⇒ An ≡ 4 + 3(7 ) ⇒ An ≡ 0(7) ⇒

⇒ An es divisible por 7. Si n = 6K + 4 ⇒

⇒ An = 106K+4 + 3 ∀K∈Ð es divisible por 7 y por lo tanto es compuesto de


∀K∈Ð.

b) • Como al dividir A por 2 queda resto 1 ⇒ A + 1 es divisible por 2.


• Como al dividir A por 3 queda resto 2 ⇒ A + 1 es divisible por 3.
• Como al dividir A por 4 queda resto 3 ⇒ A + 1 es divisible por 4.
• Como al dividir A por 5 queda resto 4 ⇒ A + 1 es divisible por 5.
• Como al dividir A por 6 queda resto 5 ⇒ A + 1 es divisible por 6.
• Como al dividir A por 7 queda resto 6 ⇒ A + 1 es divisible por 7.
• Como al dividir A por 8 queda resto 7 ⇒ A + 1 es divisible por 8.
• Como al dividir A por 9 queda resto 8 ⇒ A + 1 es divisible por 9.

Como buscamos el menor valor de A que verifique estas condiciones ⇒


⇒ A + 1 = m.c.m (2, 3, 4, 5, 6, 7, 8, 9) = 2520
⇒ A = 2519

8/8
www.eltemario.com Oposiciones Secundaria – Matemáticas
© Antonio J. Abrisqueta Valcárcel, 2001 Hoja de Problemas nº 4 – Algebra IV

28. Hallar un número entero A que no tenga mas factores primos que 2, 5 y 7,
sabiendo además que 5ª tiene divisores más que A y que 8ª tiene 18 divisores
mas que A. Calcular también la suma de todos los divisores de A.

Solución:

a) Como no tiene mas factores primos que 2, 5 y 7 ⇒ A = 2ª · 5b · 7c entonces


5A = 2ª · 5b+1 · 7c y 8A = 2ª+3 · 5b · 7c

Utilizando la fórmula que nos proporciona el número de divisores se tiene que:

 (a + 1)(b + 2)(c + 1) = (a + 1)(b + 1)(c + 1) + 8


 ⇒
(a + 4)(b + 1)(c + 1) = (a + 1)(b + 1)(c + 1) + 18

 (a + 1)(b + 1 + 1)(c + 1) = (a + 1)(b + 1)(c + 1) + 8


⇒ ⇒
(a + 1 + 3)(b + 1)(c + 1) = (a + 1)(b + 1)(c + 1) + 18

 (a + 1)(b + 1)(c + 1) + (a + 1)(c + 1) = (a + 1)(b + 1)(c + 1) + 8


⇒
(a + 1)(b + 1)(c + 1) + 3(b + 1)(c + 1) = (a + 1)(b + 1)(c + 1) + 18

 (a + 1)(c + 1) = 8 ( a + 1)(c + 1) = 18
⇒ ⇒ (1) entonces dividiendo
3(b + 1)( c + 1) = 18  (b + 1)( c + 1) = 6

a +1 4
tenemos que: = ⇒ a = 4α − 1 y b = 3α − 1
b +1 3

que sustituyendo en (1) nos da que α = 1 ⇒ a = 3, b = 2, c = 1

⇒ A = 23 · 52 · 71 = 1400 ⇒ A = 1400

b) Utilizando la fórmula para la suma de divisores en teste problema tenemos que:

2 4 − 1 53 − 1 72 − 1
S= + + = 15 ⋅ 31 ⋅ 8 = 3720
2 −1 5 −1 7 − 1

S = 3720

29. Demuestra que si K es un cuerpo de característica p, siendo p primo, entonces


∀ x, y ∈ K, se verifica:
(x + y)p = xp + yp

Solución:

Tenemos que:

1/8
www.eltemario.com Oposiciones Secundaria – Matemáticas
© Antonio J. Abrisqueta Valcárcel, 2001 Hoja de Problemas nº 4 – Algebra IV

 p  p  p  p  p −1  p  p
(x + y)p =   x p +   x p −1 y +   x p −2 y 2 + ..... +   xy +   y
0  1  2  p − 1  p

 p   p
Por un lado tenemos que   =   = 1 , pero por otro lado si 1≤ K < p y si e =
 0   p
 p
elemento unidad, entonces como K!   = p ( p − 1)( p − 2) ⋅ ...... ⋅ ( p − K + 1) se obtiene
K 
que:

 p 
( K! e) e  = p ( p − 1)( p − 2 ) ⋅ ...... ⋅ ( p − K + 1)e = 0
K  

⇒ Como K!e = e(2e)(3e) ·……· (Ke) y he ≠ 0 si 1 ≤ h < p ⇒

 p
⇒ K!e = 0 ⇒  e = 0
K 

Por lo tanto (x + y) p = xp + yp

30. Dado un subconjunto A de la recta real 3 , se dice que un punto x∈ ∈ 3 es un


punto de condensación de A si cualquier entorno de x contiene una infinidad no
numerable de puntos de A.
Se pide:
a) Demostrar que cualquier subconjunto no numerable de 3 admite como
mínimo un punto de condensación.
b) Si P simboliza el conjunto de todos los puntos de condensación de un
subconjunto A de 3 , demuestre que P es un cerrado de 3 .

Solución:

a) Sea A un conjunto de 3 no numerable, y supongamos un intervalo cerrado y


acotado I = [-n, n] ⊂ A.

Supongamos que A no tienen puntos de condensación ⇒ ∀x∈[-n, n] ⇒ ∃∪(x) tal


que U(x)∩A es como mucho numerable.

⇒ Como esto ocurre ∀x∈I ⇒ existe una familia {U(x) :x∈I} de entornos
abiertos que recubres a I, y como I es compacto ⇒ existe un subrecubrimiento finito
U(x1 ),…….U(xr) cuya unión es I.

r
⇒ I∩A = U (U ( x j ) ∩ A) que es como mucho numerable, porque es la unión finita
j =1

de conjunto como mucho numerable.

2/8
www.eltemario.com Oposiciones Secundaria – Matemáticas
© Antonio J. Abrisqueta Valcárcel, 2001 Hoja de Problemas nº 4 – Algebra IV

⇒ Como A = U ([− n, n] ∩ A) es a lo más numerable, por si la unión numerable de


n∈N

conjuntos a los más numerables ⇒ A es numerable (contradicción).

⇒ A tiene como mínimo un punto de condensación.

b) Vamos a demostrar viendo que 3 - P es un abierto.

Sea x∉ P ⇒ ∃ ∪ ( x ) abierto tal que U(x)∩A es a lo sumo numerable. Entonces si


y∈U(x), como U(x) es un entorno de y que es a lo sumo numerable ⇒ y∉P, que es el
conjunto de los puntos de condensación ⇒ U(x)∩a = Ø si x∉P ⇒ 3 - P es un
abierto de 3 ⇒ P es un cerrado de 3.

31. Demostrar que todo conjunto B de números reales con un solo punto de
acumulación es numerable.

Solución:

Sea A = B – {p} donde p es un punto de acumulación, en concreto, el único punto


de acumulación de B.

Como el espacio topológico (3, T) con T la topología usual es Hausdorff ⇒


∀x∈B con x ≠ p se tiene que x es un punto aislado ⇒ C es un conjunto de puntos
aislados.

Como (3,T) verifica el 2º Axioma de numerabilidad ⇒ (C, Tc) también verifica ⇒


Existe una base numerable de Tc ⇒ los elementos de C que son subconjuntos unitarios
y abiertos de Tc son elementos de dicha base ⇒ C es numerable.

⇒ Se tiene que:
• Si p∉B ⇒ B = C que es numerable
• Si p ∈ B ⇒ B = C∪{p} que será numerable porque C es numerable y p es un
elemento.

Entonces B es un conjunto numerable.

32. Demostrar que para dos números reales x<1, y<0, existe un único número
entero n tal que xn-1≤ y<xn .

Solución: supongamos por reducción al absurdo que no existe ningún entero n

tal que y<xn . Esto significa que y>xn ∀n∈Z ⇒ y es cota superior del conjunto

M={xn ∈R: n∈Z}. Sea µ el supremo de M (ya que al estar acotado superiormente tiene

3/8
www.eltemario.com Oposiciones Secundaria – Matemáticas
© Antonio J. Abrisqueta Valcárcel, 2001 Hoja de Problemas nº 4 – Algebra IV

µ µ
supremo). como x>1 ⇒ es menor que µ ⇒ no es cota superior de M ⇒ ∃p∈Z /
x x
µ
< x p ⇒ µ<xp+1 ⇒ µ no es cota superior (contradicción) ⇒∃n∈Z / y<xn .
x

Análogamente podemos demostrar que existe un entero p tal que xp <y con p<n

evidentemente (por ser x>1). Entonces tomando la sucesión finita xp , xp+1 ,..., xn-1 , xn y

comparando término a término con y podremos encontrar uin entero m tal que p<m≤n

de modo que: xm-1≤y< xm.

33. Sean a,a´,b,b´ números racionales y γ un número irracional. Probar que el


a b
número (aγγ +b) / (a´γγ +b´) es racional sii = .
a´ b´

aγ + b
Solución: "⇒" Como r = es racional ⇒ r(a´γ+b´)=(aγ+b) ⇒ γ(a-ra´) =
a´γ + b´
=rb´-b entonces como rb´-b es un número racional y a-ra´ también es racional ⇒ (a-a´)=
rb´−b
= 0, porque si a-ra´≠0 ⇒(a-ra´)∈Q, (rb´-b)∈Q y tendríamos que γ = ∈Q
a − ra´
a
(contradicción) entonces efectivamente a-ra´=0 y en consecuencia rb´-b=0 ⇒ r= y

b a b
r= ⇒ =
b´ a´ b´

a a b
"⇐" Como ∈Q⇒ sea r= ⇒ r= y por lo tanto a=ra´ y b=rb´. Por lo tanto:
a´ a´ b´
aγ + b ra´γ + rb´ r ( a´γ + b´) aγ + b
= = = r∈Q ⇒ ∈Q
a´γ + b´ a´γ + b´ a´γ + b´ a´γ + b´

34. Comprobar que es racional el número: r = 3 20 + 14 2 + 3 20 − 14 2

Solución: Sean x= 3 20 + 14 2 e y = 3 20 − 14 2 entonces haciendo:

(x+y)3 =x3 +3x2 y+3xy2 +y3 =

=( 3 20 + 14 2 )3 +3xy(x+y)+ ( 3 20 − 14 2 )3 =

= 20+14 2 +3 3 20 + 14 2 3
20 − 14 2 r+20-14 2 =

= 40+3 3 ( 20 + 14 2 )( 20 − 14 2 ) r = 40+3 3 400 − 392 r = 40+6r

4/8
www.eltemario.com Oposiciones Secundaria – Matemáticas
© Antonio J. Abrisqueta Valcárcel, 2001 Hoja de Problemas nº 4 – Algebra IV

⇒ r3 = 40+6r ⇒ r3 -6r-40 = 0.

Haciendo Ruffini tenemos:

1 0 -6 -40

4 4 16 40

1 4 10 0

r = 4
⇒ r3 -6r-40 = (r-4)(r2 +4r+10) = 0 ⇒  2 con r2 +4r+10=0 →
r + 4r + 10 = 0

− 4 ± 16 − 40
r= que no tiene solución real ⇒ la única solución real de la ecuación es
2

r=4 que es racional ⇒ 3 20 + 14 2 + 3 20 − 14 2 =4∈Q.

35. Demuéstrese que la recta racional es un subespacio de la recta real.

Solución: Demostraremos que la base BQ, subordinada por la base B de R y la

base B´ de Q son equivalentes.

Sea x∈(a,b)∩Q∈BQ con a,b∈R entonces ∃p,q∈R tales que (p,q)⊂(a,b) con x∈(p,q)∈B´.

Recíprocamente, sea y∈(p,q)∈B´ con p,q∈Q entonces y∈(p,q)=(p,q)∩Q∈BQ.

Con lo cual queda demostrado que la recta racional es un subespacio de la recta real.

36. Si el dividendo está aproximado por defecto y el divisor por exceso demostrar
que el cociente está aproximado por defecto y su error relativo es menor que
la suma de los errores relativos del dividendo y el divisor.

Resolución:
A− E
Sea A el dividendo y A´ el divisor. El cociente aproximado será: y el error
A´+ E´
absoluto es:

A A − E A( A´+ E´) − A´( A − E) AE´+ A´ E AE´+ A´E


− = = <
A´ A´+ E´ A´( A´+ E´) A´( A´+ E´) A´ 2

5/8
www.eltemario.com Oposiciones Secundaria – Matemáticas
© Antonio J. Abrisqueta Valcárcel, 2001 Hoja de Problemas nº 4 – Algebra IV

A
Si dividimos ambos miembros por , el primer miembro representa el error relativo

del cociente , y el segundo miembro viene dado por:

AE´+ A´E A´ AE´+ A´E E´ E


· = = +
A´ 2 A AA´ A´ A

Por lo tanto, podemos concluir que el error relativo del cociente es menor que la suma

de los errores relativos.

37. El error relativo de la raíz de un número aproximado por defecto es menor


que el error relativo de el número aproximado, dividido por el índice de la
raíz.

Resolución:

Sea A-E el valor aproximado de A, entonces el error absoluto cometido al aproximar


A , por ejemplo, es más grande que 5(A-E)4 ·E.
5

Si tomamos la raíz quinta de A elevada a % se tiene que ( A ) =A y el error


5
5

absoluto es más grande que:

5(5 A − E ) 4 ·E

5
donde E1 es el error absoluto cometido cuando aproximamos por defecto a A con
5
A − E , por lo tanto:

E E1 E
E> 55 ( A − E ) 4 ·E1 ⇒ E1 < ⇒ <
55 ( A − E ) 4 55 A − E 5( A − E )

E1 E
y como el error relativo es más pequeño que se tiene que:
5
A 5( A − E )

E1 E
<
5
A 5( A − E )

38. Demostrar que la sucesión definida por x n +1 = 1 − 1 − x n con 0<x1 <1, es


xn +1
convergente. Calcular su límite. Calcular el limn→
→∞ .
xn

Resolución:

Vamos a demostrar lo primero que xn es una sucesión que está acotada entre 0 y

6/8
www.eltemario.com Oposiciones Secundaria – Matemáticas
© Antonio J. Abrisqueta Valcárcel, 2001 Hoja de Problemas nº 4 – Algebra IV

1. La demostración de que está acotada la haremos por inducción.

Sea A={n∈N / 0<xn <1}.

i) ¿1∈A?

Como 0<x1 <1 ⇒ 1∈A

ii) Supongamos que n∈A, 0<xn <1. Veamos si (n+1)∈A, ¿0<xn+1 <1?

Como 0<xn <1 ⇒ 0<1-xn <1 ⇒ 0< 1 − xn <1 ⇒ 0<1- 1 − xn <1 ⇒ 0<xn+1 <1 ⇒

⇒ (n+1)∈A.

Entonces A=N.

Por lo tanto xn es una sucesión que está acotada entre 0 y 1. Vamos a demostrar

ahora que xn es decreciente ∀n.

Sea A={n∈N / xn+1 <xn }

i) ¿1∈A?

x2 <x1 ⇔ 1- 1 − x1 <x1 ⇔1-x1 < 1 − x1 es cierto pues como 1-x1 ∈(0,1) ⇒

⇒ 1 − x1 >1-x1 ⇒1∈A.

ii) Supongamos que es cierto que n∈A, xn+1 <xn . Veamos que (n+1)∈A,

¿xn+2 <xn+1 ?.

Como xn+1 <xn ⇒ -xn <-xn+1 ⇒ 1- xn <1-xn+1 ⇒ 1 − x n < 1 − xn +1 ⇒

− 1 − x n +1 < − 1 − x n ⇒ 1 − 1 − x n +1 < 1 − 1 − x n ⇒ xn+2 <xn+1 ⇒(n+1)∈A ⇒

⇒ A=N ⇒ xn es decreciente ∀n∈N.

Como xn es decreciente y está acotada inferiormente por 0 ⇒xn es convergente.

Ya que xn es convergente, se cumple pues que:

limn→∞xn = limn→∞xn+1 = L.

Entonces:
L=1- 1 − L ⇒ 1 − L =1-L ⇒ (1-L) = (1-L)2 ⇒ (1-L)-(1-L)2 = 0 ⇒

7/8
www.eltemario.com Oposiciones Secundaria – Matemáticas
© Antonio J. Abrisqueta Valcárcel, 2001 Hoja de Problemas nº 4 – Algebra IV

⇒(1-L)(1-1+L) = 0 ⇒L(1-L) = 0 ⇒ L=0 ó L=1 pero L≠1 porque 0<x1 <1, y xn

es decreciente ⇒ L=0.

xn +1 1− 1 − xn  0  (1 − 1 − x n )(1 + 1 − x n )
lim n →∞ = lim n → ∞ =   = lim n → ∞ =
xn xn  0 x n (1 + 1 − xn )

lim n →∞
(
12 − 1 − x n )
2

= lim n → ∞
1 − 1 + xn
= lim n →∞
xn
=
xn (1 + 1 − x n ) x n (1 + 1 − x n ) x n (1 + 1 − x n )

1 1 1
lim n →∞ = = =
1 + 1 − xn 1 + 1 2

8/8
www.eltemario.com Oposiciones Secundaria – Matemáticas
© Antonio J. Abrisqueta Valcárcel Hojas de Problemas nº 5 – Álgebra V

n2
39. Comprobar que la serie ∑ n =1
+∞
es convergente, y en caso de serlo, calcular su
2n
suma.

Resolución:

Para comprobar su convergencia aplicaremos el criterio del cociente:

( n + 1) 2 n 2 2n 1
+ 2 + 2
2 n +1 = lim n2 + 2n +1 n 2
n n =
lim n →∞ n →∞ = lim n →∞ =
n2 2n 2 2n 2
2n n2

2 1
1+ +
= lim n →∞ = n n 2 = 1 < 1 por lo tanto la serie es convergente.
2 2

Vamos a sumar la serie:


+∞
n2 n
j2 n
j2

n =1 2
n
= lim n →∞ ∑
j =1 2
j
= lim n→ ∞ S n donde S n = ∑
j =1 2
j

1 4 9 16 25 n2
Sn = + 2 + 2 + 4 + 5 + ... + n
2 2 3 2 2 2

1 1 4 9 16 (n − 1) 2 n 2
Sn = + + + + ... + + n
2 22 32 24 25 2n 2

1 1 3 5 7 9 2n −1 n 2
Restando: S n = + 2 + 3 + 4 + 5 + ... + + n +1
2 2 2 2 2 2 2n 2

1 1 3 5 7 2n − 3 2n − 1 n 2
Sn = + + + + ... + + n +1 + n + 2
4 22 23 2 4 25 2n 2 2

1 1 2 2 2 2 2 ( n − 1) 2 n2
Sn = + 2 + 3 + 4 + 5 + ... + n + −
4 2 2 2 2 2 2 2 n +1 2n +2

1 1  1 1 1 1  ( n − 1) 2 n2
Sn = + 2 2 + 3 + 4 + ... + n  + −
4 2 2 2 2 2  2 n +1 2 n+ 2
www.eltemario.com Oposiciones Secundaria – Matemáticas
© Antonio J. Abrisqueta Valcárcel Hojas de Problemas nº 5 – Álgebra V

1 1 
 − n +1  ( n − 1) 2
1 1 4 2 n2
Sn = +2 + −
4 2  1− 1  2 n +1 2 n+ 2
 
 2 

1 1 
 − n +1  ( n − 1) 2 n2
S n = 2 + 8 4 2 + −
 1  2 n +1 2 n +2
 
 2 

1 1  (n − 1) 2 n2 1
S= limn→∞Sn = limn→∞ 2+16  − n +1  + n +1
− n+ 2
= 2 + 16 = 6
4 2  2 2 4

40. Hallar cuántos números menores que 1000 son primos con 29 y 13. Hallar
también la suma de los números no primos con 29 y 13 menores que 1000.

Resolución:
Como hay 999 números menores que 1000, vamos a ver cuáles de ellos
no son primos con 13 y 29.
Como 13 y 29 son primos entre sí, vamos a calcular el mínimo común múltiplo
de 13 y 29, que es: 377, que como es menor que 1000 hay que descontarlo . Pero
2·3777=754, que también es menor que 1000, y habrá que descontarlo. Y para
terminar tenemos 3·377 = 1131 que es mayor que 1000.
Entonces hay 997 número menores que 1000 y primos con 29 y 13.
Por otro lado, la suma de los no primos con 29 y 13 menores que 1000 es
377+754=1131.

41. Calcular la arista del menor cubo en el que se pueden acoplar sin dejar
espacios vacíos un número entero de ortoedros iguales cuyas dimensiones
expresadas en cm son C5,2 , P3 y el módulo de 8+6i. ¿Cuántos ortoedros pueden
acoplarse?
Resolución:
Calcularemos primero las dimensiones de los ortoedros:
www.eltemario.com Oposiciones Secundaria – Matemáticas
© Antonio J. Abrisqueta Valcárcel Hojas de Problemas nº 5 – Álgebra V

5!
C5,2 = = 10
3! 2!
P3 =3! = 6.

18+6i = 64 + 36 = 10
Buscaremos ahora quién es el mínimo común múltiplo de estos valores.
10 = 2·5 

6 = 2·3 m.c.m.(10,6) = 30
10 = 2·5

Por lo tanto la arista del menor cubo será de 30 cm. El número de


ortoedros vendrá dado por:
30 30 30
· · = 3·5·3 = 45 → se podrán acoplar 45 ortoedros.
10 6 10

42. Hallar el exponente de 3 en 2121.


Resolución:
Si dividimos sucesivamente el 2121 por 3, tenemos que:

2121 3
021 77 3
0 17 25 3
2 1 8 3
2 2

Por lo tanto el exponente de 3 en 2121 es: 77+25+8+2=112.


La solución es 112.

43. Hallar la última cifra en la que acaba el número 45932 .


Resolución:
Veamos cuáles son los restos potenciales de 4 módulo 10, para intentar
establecer, en caso de que sea posible, una recurrencia en dichos restos.

40 = 1 (mod 10)
www.eltemario.com Oposiciones Secundaria – Matemáticas
© Antonio J. Abrisqueta Valcárcel Hojas de Problemas nº 5 – Álgebra V

41 = 1 (mod 10)
42 = 1 (mod 10)
43 = 1 (mod 10)
44 = 1 (mod 10)
45 = 1 (mod 10)
46 = 1 (mod 10)

Como podemos observar cuando el exponente del 4 es par el resto


módulo 10 es 6, pero si el exponenete es impar el resto módulo 10 es 4. Como el
exponente en este problema es 5932 = 2·2966 que es par se tiene que la última
cifra en la que acaba el número 45932 es el 6.

44. Encontrar el criterio de divisibilidad por 5 en base 12, y aplicárselo al número


12x7512 hallando x para que sea divisible por 5.
Resolución:
Tomando los restos potenciales de 12 módulo 5, se tiene que:

120 = 1 (mod 10)


121 = 1 (mod 10)
122 = 1 (mod 10)
123 = 1 (mod 10)
124 = 1 (mod 10)
Entonces si tomamos un número N= abcde12) es divisible por 5 ⇔ e+2d-
0
c-2b+a =  . Aplicando el caso 12x7512) es divisible por 5 ⇔
5&

0
⇔ 5+14-x-4+1 =  ⇔ 16-x = 5& ⇒ x=1 ó x=6.
5&
www.eltemario.com Oposiciones Secundaria – Matemáticas
© Antonio J. Abrisqueta Valcárcel Hojas de Problemas nº 5 – Álgebra V

45. Hallar cuántos números menores que 1342 son primos con 19 y 53. Hallar
también la suma de los números no primos con 19 y 53 menores que 1342.
Resolución:
Como sabemos, hay 1341 números menores que 1342. Las que no son
primos on 19 y 53 a la vez, son aquellos que sean múltiplos de m.c.m.(19,53) =
1007. Como 1007<1342 y no es pirmo con 19 y 53, habrá que descontarlo.
Por lo tanto, números menores que 1342 primos con 19 y 53 hay 1340.
La suma de los números no primos 19 y 53 y que sean menores que 1342 es
1007, porque sólo hay ese.

46. Al número natural m0 no cuadrado perfecto, se le suma su exce3so sobre el


máximo cuadrado en él contenido y se obtiene el número m1 . Operando de
modo análogo con m1 , resulta el número m2 , y así sucesivamente.
Demostrar que la sucesión m1 ,m2 ,m3 ,... es indefinida, es decir, no contiene
ningún cuadrado.

Solución:

Sea m0 el número antes expuesto, entonces:


n2 <m0 <(n+1)2 ⇒ m1 =m0 +(m0 -n2 )+2n+1 = n2 +4n+2<(n+2)2
Como m1 = 2m0 -n2 ⇒ dado que 2m0 es par ⇒ m1 será par o impar dependiendo
de que n2 sea par o impar respectivamente. ⇒ m1 tiene la misma paridad que n2 ⇒ m1
tiene distinta paridad que (n+1)2 ⇒ m1 no es de la forma (n+1)2 ⇒ m1 no es un
cuadrado perfecto. Y reiterando el proceso se tiene que mi no es cuadrado perfecto
∀i∈N.

47. En una provincia española el número correspondiente a la última matrícula de


vehículos concedido hasta esta fecha es el 46000. Hállese el número de vehículos
cuyos números de matrícula son capicuas de cinco cifras.

Solución:
www.eltemario.com Oposiciones Secundaria – Matemáticas
© Antonio J. Abrisqueta Valcárcel Hojas de Problemas nº 5 – Álgebra V

Como los números capicúas son de la forma abcba una vez fijadas las tres
primeras cifras las otras dos vienen obligadas, por lo tanot bastará con estudiar las
posibilidades que tenemos con las tres primeras, es decir:

V10,3 =103 =1000

pero:
a) Hay que descontar los que empiezan por x>4 (5,6,7,8 y 9) o por 0, es decir:
5·V10,2 =6·100=1000
b) Las que tienen por primera cifra un 4 y por segunda un 6,7,8 ó 9.
4·V10,1 =40.
Entonces las solución es: 1000-600-40 = 360
N=360.
www.eltemario.com Oposiciones Secundaria – Matemáticas
© Antonio J. Abrisqueta Valcárcel, 2001 Hoja de Problemas nº 6

48. a) ¿De cuántas maneras se pueden repartir n bolas idénticas en urnas de


manera que r urnas prefijadas estén vacías?
b) ¿De cuántas maneras distintas se pueden repartir n bolas idénticas en k
≤ n)?
urnas de manera que una urna prefijada contenga r bolas (r≤

Resoluciòn:
a) Habrá tantas configuraciones como maneras distintas de repartir todas las
bolas en k-r urnas. Por lo tanto habrá
 n + k − r − 1
 
 k − r −1 
b) Para construir estas configuraciones tomamos r bolas y las colocamos en la
urna prefijada. Luego, las restantes n-r bolas se repartirán, de todas las maneras
posibles, entre las n-1 urnas que quedan. Por lo tanto, habrá
 n + k − r − 2
 
k − 2 

49. Discutir y resolver el siguiente sistema de ecuaciones (Andalucía 98):

ax + y + z = 1

 x + ay + z = 1
 x + y + az = 1

Resolución:
Aplicando el método de Gauss, vamos a triangularizar superiormente la matriz
de coeficientes ampliada:
1 1 a | 1 1 1 a 1 
   
{ } {
1 a 1 | 1 ≈ F2 = F2 − F1 // F3 = F3 − aF1 ≈  0 a − 1 1 − a | 0  ≈ F3 = F3 + F2 ≈
´ ´ ´
}
 a 1 1 | 1  0 1 − a 1 − a2 1 − a 
   

1 1 a 1 
 
≈  0 a −1 1− a | 0 
0 2 − a − 2 1 − a 
 0

Los valores conflictivos del sistema son:


a-1=0 → a=1

1/6
www.eltemario.com Oposiciones Secundaria – Matemáticas
© Antonio J. Abrisqueta Valcárcel, 2001 Hoja de Problemas nº 6

1 ± 1 + 8 1 ± 3 → 1
2-a-a2 =0 → a = = =
−2 − 2 → −2 → a = −2
Caso 1: a=1

1 1 1 1 Rg ( A) = 1 
  
 0 0 0 | 0 Rg ( A | B ) = 1 S .C.I . (2 parámetros ).
 0 0 0 0 n º de incógnitas = 3
 

x+y+z=1 → z=1-x-y
x,y∈R
Caso 2: a=-2

1 1 − 2 1
  Rg ( A) = 2 
0 −3 3 | 0 Sistema Incompatible.
0 0 Rg ( A | B) = 3
 0 3 

Caso 3: a≠1 y a≠z

1 1 a 1  Rg ( A) = 3 
  
 0 a −1 1− a | 0  Rg ( A | B ) = 3  S.C.D.
0 (1 − a)( a + 2) 1 − a  n º de incógnitas = 3
 0


 x + y + az = 1

 1
( a − 1) y + (1 − a ) z = 0 → y − z = 0 → y = z → y =
 a+2
 1
(1 − a)( a + 2) z = (1 − a ) → z = a + 2

1 1 1 a
⇒ x+y+az=1 → x+ +a =1 → x =1− −
a+ 2 a+2 a +2 a+ 2
a + 2 −1 − a 1
→ x= → x=
a+2 a+2

2/6
www.eltemario.com Oposiciones Secundaria – Matemáticas
© Antonio J. Abrisqueta Valcárcel, 2001 Hoja de Problemas nº 6

50. Calcular el valor del determinante:


1+ x 1 1... 1 1+ x 1 1... 1
1 1 + x 1... 1 1+ x 1+ x 1... 1
D= 1 1 1 + x... 1 = {F1 ´= F1 + ... + Fn } = 1 + x 1 1 + x... 1 =
... ... ... ... ... ... ... ...
1 1 1 1+ x 1+ x 1 1 1+ x

1 1 1... 1 1 1 1... 1
1 1+ x 1... 1 0 x 0... 0
(1 + x) = 1 1 1 + x... 1 = {Fi ´= Fi − F1 , i > 1} = (1 + x ) 0 0 x... 0 = (1 + x)· x n −1
... ... ... ... ... ... ... ...
1 1 1 1+ x 0 0 0 x

⇒ D=(1+x)·xn-1

∈ Z}.
51. Sea G el grupo cíclico infinito generado por x. Sea Y={y=x12a+30b+42c; a,b,c∈
Llamemos H al subgrupo cíclico de G generado por x6 . Probar que H=Y.

Resolución:
(⊇) Es fácil ver que como ∀y∈Y y=x12a+30b+42c=(x6 )2a+5b+7c ⇒ y∈H
Por lo tanto, se tiene que Y⊆H.

(⊆) Como el m.c.d.(2,5)=1, es decir, 2 y 5 son pirmos entre sí entonces


aplicando la identidad de Beront tenemos que existen u y v∈Z tales que 1=2u+5v, por
lo tanto:
∀n∈Z tenemos que n=2nu+5nv y en concreto 6n=12nu+30nv+42·0 ⇒ x6n ∈Y,
entonces H⊆Y. Entonces H=Y.

→ G difinida por:
52. Sea G un grupo. Probar que la aplicación fa : G→
→ fa (x) = axa-1 ∀ x∈
x→ ∈ G y a∈
∈ G es un automorfismo.
Sea F el conjunto de todos los automorfismos anteriores. Estudiar la estructura
que confiere a dicho conjunto la ley de composición de aplicaciones.

Resolución:

3/6
www.eltemario.com Oposiciones Secundaria – Matemáticas
© Antonio J. Abrisqueta Valcárcel, 2001 Hoja de Problemas nº 6

A) 1) Como ∀x,y∈G, fa(sy) = a(xy)a-1 = a(xa -1ay)a-1 = (axa -1)(aya -1) = fa (x)· fa(y) ⇒
fa es un homomorfismo.

2) ∀x∈G (fa-1°fa)(x) = fa-1(axa-1) = a-1axa-1a = x


(fa°fa-1)(x) = fa(fa-1(x)) = fa (a-1xa) = aa-1xaa1 = x
⇒ fa tiene inverso
⇒ fa es un automorfismo

b) 1) fb fa(x) = fb (axa -1) = baxa -1b-1 = bax(ba)-1 = fba(x) ∀x∈G ⇒ fb°fa = fba

2) Como la composición de aplicaciones ya es asociativa, en este caso también


lo es.
3) Si e∈G es el elemento neutro de G ⇒ fe es el elemento neutro de (F,°), pues
∀fa∈F ⇒ fe°fa = fea = fa y fa°fe=fae = fa.

4) Por lo visto anteriormente fa tiene anteriormente inverso, que es fa-1.


⇒ (F,°) es un grupo.

53. Sea Q[x] el conjunto de los polinomios de una indeterminada con


coeficientes racionales. Demostrar:
a) Siendo xm -1, xn -1, dos polinomios de Q[x] tales que: m.c.d.(m,n) = d, se
verifica m.c.d.(xm -1, xn -1) = xd-1.
≠ 0, p≠
b) Que no existe ningún polinomio p de Q[x] tal que p≠ ≠ 1 y que la
función polinómica p(x) cumpla p(x+y)=p(x)p(y) ∀ x,y∈
∈ Q.

Resolución:
a) Las raíces en los complejos de xm-1 y xn -1 son las raíces m-ésimas y n-ésimas de
la unidad.
Los ceros o raíces comunes de xm-1 y xn -1 son los ceros del m.c.d.( xm-1, xn -1),
es decir, las raíces d-ésimas de la unidad donde d=m.c.d.(m,n), esto viene basado en e
teorema de Beront con d=am+bn y rm=1 y rn =1 ⇒ rd=ram+bn =1 ⇒ m.c.d.(xm-1, xn -1) =
= xd-1.

4/6
www.eltemario.com Oposiciones Secundaria – Matemáticas
© Antonio J. Abrisqueta Valcárcel, 2001 Hoja de Problemas nº 6

b) Si tomamos x=y ⇒ p(zx) = (p(x))2 ∀x∈Q.


Si p(x) es de grado d ⇒ p(2x) es de grado d (p(x))2 de grado 2d ⇒ d=2d ⇒ d=0.
Como d=0 ⇒p(x)=c ⇒ p(x+y) = d = p(x)p(y) = d2 ⇒ d=d2 ⇒ d(d-1) = 0 ⇒
d = 0
⇒ ⇒ p = 0 o p =1.
d = 1

54. Sea U=(ui j) la matriz cuadrada de orden n donde ui j=1 para todo (i,j). Sea
∈ Rn,n , AU=UA= α (A)U, α (A)∈
F={A∈ ∈ R}. Pruébese:
1) F es un subespacio de Rn,n .
∈ F y existe A-1, entonces A-1∈ F y α (A)≠
2) Si A∈ ≠ 0. Relacionar α (A) y α (A-1).
∈ F, entonces AB∈
3) Si A, B∈ ∈ F. Calcular α (AB).
n n
∈F ⇔
4) A∈ ∑a
i =1
ij = ∑ a ij = α( A) ∀(i , j )
j =1

→α (A) es una forma lineal.


5) La aplicación A→

Resolución:
1) F≠0 pues la matriz nula está en F.
Si tomamos A,B∈F y α,β∈R, entonces:
(αA+βB)U = αAU+βBU = αUA+βUB = U(αA+βB)
(αA+βB)U = αα(A)U+βα(B)U = (αα(A)+β α(B))U
⇒ ∀A,B∈F y α,β∈R ⇒ αA+βB ∈F con α(αA+βB) = αα(A)+β α(B). Entonces F es
un subespacio vectorial de Rn,n .

2) Sea A∈F y sea A-1, que existe.


AU = UA = α(A)U ⇒ U = A-1UA = α(A)A-1U ⇒ α(A)≠0 puesto que si α(A)=0
⇒ U sería nula.
1 1
Entonces U = AU = UA .
α( A) α( A)
AU = UA ⇒ A-1AUA-1 = A-1UAA-1 ⇒ UA-1 = A-1U
 1  1
⇒ UA-1 =  UA A −1 = U
 α( A)  α( A)

1 1
⇒ A-1U = UA-1 = U y α ( A −1 ) =
α( A) α( A)

5/6
www.eltemario.com Oposiciones Secundaria – Matemáticas
© Antonio J. Abrisqueta Valcárcel, 2001 Hoja de Problemas nº 6

3) AU = UA = α(A)U
AU = UA = α ( A)U 
 ⇒ ABU = AUB = α( B ) AU ⇒ ABU = UAB = α(B)α(A)U
BU = UB = α ( B)U 

⇒ A,B∈F y α(AB)= α(A)α(B)

n n
4) AU = α(A)U ⇔ ∑ aij = ∑ a ij = α ( A) i, j ∈ {1,..., n}
j =1 i 01

5) Sea f: F→R aplicación: f(λA+µB) = α(λA+µB) = λα(A)+ µα(B) ⇒ f es una


forma lineal.

6/6
www.eltemario.com Oposiciones Secundaria – Matemáticas
© Antonio J. Abrisqueta Valcárcel, 2001 Hoja de Problemas – Álgebra VII

55. Supongamos que la función g está definida y es derivable en [0,1]. Supongamos


≤ g´(x)≤
que g(0)<0<g(1), 0<a≤ ≤ b donde a y b son constantes. Probar que existe una
constante M tal que la solución de la ecuación g(x)=0 se puede econtrar aplicando
el algoritmo de iteración a la función f(x)=x+Mg(x), ¿para qué valores de M la
sucesión definida por xn+1 =f(xn ) converge cuadráticamente a la solución α de
g(x)=0?

a) |f´(x)| = |1+Mg´(x)|<1
−2 −2 −2 −2
-1<1+mg´(x)<1 ⇒ -2<Mg´(x)<0 ⇒ <M<0 como ≤ ≤
g (x ) a g ( x) b

−2
⇒ < M <0
b
x0 =2
f(0)=Mg(0)>0
f(1)=Mg(1)<0
−1 −1 −1 −1 −1
f´(x)=1+Mg´(x)≥0 ⇒M≥ como ≤ ≤ ⇒ M ≥
g´( x) a g´( x) b b

−1
⇒ ≤ M < 0 para que f sea Lipschitziana de [0,1]→[0,1] con constante de
b
Lipschitz M

b) Debe cumplir que:


con f(α)=α; f´(x)=0
f´(x)=1+Mg´(x) = 0
−1
⇒ M =
g´(x )

56. Mostrar que la función F(x)=x-e -x satisface las condiciones de convergencia


global del método de Newton en el intervalo [0,1]. Resuelve la ecuación F(x)=0
comenzando en x0 =0´5.

i) F(a)·F(b)<0 F(0)= 0 -e0 =-1


1
F(0)·F(1)<0 F(1) = 1-e-1= 1 - = 0´632120558
e

1/8
www.eltemario.com Oposiciones Secundaria – Matemáticas
© Antonio J. Abrisqueta Valcárcel, 2001 Hoja de Problemas – Álgebra VII

F(0)·F(1) = (-1)·0´632120558= -0´6321...

ii) F´(x)≠0 ∀x∈[a,b]


F´(x)≠0 ∀x∈[0,1]
1 1 1
F´(x)=1+e-x = 1+ x
como 1>0 y x >0 ⇒ 1+ x >0 ∀x∈[0,1]
e e e
⇒ F´(x)≠0 ∀x∈[0,1]

iii) F´´(x)≥ ∀x∈[a,b] ó F´´(x)≤0 ∀x∈[a,b]


−1 1
⇒ F´´(x) = e-x= x
como x >0 ∀x∈R ⇒F´´(x)<0 ∀x∈[0,1]
e e

F (c )
iv) Si c es el extremo del intervalo [a,b] donde |F´(x)| es menor entonces ≤b−a
F ´(c)
F´(x)=1+1/e donde F´(x) es menor es para x=1 con F´(1) = 1+1/e. Entonces c=1
1 1
1− 1−
F (1) 1− e −1
e = e = e − 1 ≤ 1 Si
≤1− 0 ⇒ ≤ 1⇒
F´(1) 1 + e −1 1 e +1 e +1
1+
e e

F(x)=x-e-x
F´(x)=1+e-x x0 =0´5
F ( x0 ) 0´5 − e −0 ´5
x1 =x0 - =0´5- = 0´566311003
F´( x 0 ) 1 + e − 0 ´5
x2 =0567143
x3 =0´567143
⇒ La x∈[0,1] / F(x)=0 es x=0´567143

57. Calcular π con nueve cifras decimales exactas.


Utilizaremos F(x)=x2 -π en [1,3].
Veremos que cumple las condiciones del Teorema global de Newton.
i) F(1)=1-π<0, F(3)=9-π>0 ⇒ F(1)·F(3)<0
ii) F´(x)=2x
F´(x)≠0 ∀x∈[1,3]

2/8
www.eltemario.com Oposiciones Secundaria – Matemáticas
© Antonio J. Abrisqueta Valcárcel, 2001 Hoja de Problemas – Álgebra VII

iii) F´´(x)=2 ≥0 ∀x∈[1,3]


iv) c=1

F (1) 1 −π
= = 1´07079 ≤ 2
F´(1) 2
Sea x0 =2
x1 =1785398163
x2 =1´772500775
x3 =1´772453852
x4 =1´772453851
x5 =1´772453851

⇒ π =1´772453851
Hay que calcular el error:
Ln 1 π
|xn -α|≤ | x 0 − x1 |< 10 −q f(x)= ( x + ) |f´(x)<1 [1´5,2]
1− L 2 x
x0 =2
x1 =1´783 ⇒ n=18 habrá que afinar decimales exactas
1 1 π
f´(x)= − L=f´(2)
2 2 x2

58. Determinar el único cero positivo del polinomio p(x)=x3 +x2 -x-1 por el método
de Newton.

Cogemos F(x) = x3 +x2 -x-1 en [1´5,2´5].


i) F(1´5)=-1´375<0
F(2´5)=5´875>0
F(1´5)·F(2´5)<0

ii) F´(x)=3x2 -2x-1


F´(x)≠0 ∀x∈[1´5,2´5]
F´(1´5) = 2´75 y es creciente.

iii) F´´(x)=6x-2 >0 ∀x∈[1´5,2´5]

3/8
www.eltemario.com Oposiciones Secundaria – Matemáticas
© Antonio J. Abrisqueta Valcárcel, 2001 Hoja de Problemas – Álgebra VII

F (1´5) − 1´375 1´375


= = <1
F´(1´5) 2´75 2´75

Sea x0 =2
x1 =1´857142857
x2 =1´839544513
x3 =1´839286810
x4 =1´839286755
x5 =1´839286757
Su raíz positiva es x=1´839286757

59. Supongamos que y está definido por la fórmula y= z + z + z + ... donde

∈ (0,+∞
z∈ ∞ ). a) Busca un método iterativo para calcular y determinando para que
valores la iteración converge. b) Calcular y.

a) f(x)= z + x
x0 =0

x1 =f(0)= z

x2 =f(x1 )= z + z
.
.
.

xn =f(xn-1)= z + z + z + ... ⇒ limn→∞ fn (x)

1
f´(x)= decreciente
2 z+x
1
f´(0)=
2 z
1 1 1
f´(x)<f´(0)<1 ⇒ <1 ⇒ < z ⇒z>
2 z 2 4
1
f:[0,+∞)→ [0,+∞) L=
2 z

z+x =x
x2 =z+x

4/8
www.eltemario.com Oposiciones Secundaria – Matemáticas
© Antonio J. Abrisqueta Valcárcel, 2001 Hoja de Problemas – Álgebra VII

x2 -x-z

1 ± 1 + 4z 1 + 1 + 4z
x= ⇒ y=
2 2

b) z y= z + x

y=x

-z

− z

1
z< f:[a,+∞)→[a,+∞)
4
1
f´(x)<f´(a)<1 f´(a)=
2 a+ z
1 1
< a + z → − z < a y ∀x0 ∈[a,+∞)
4 4

xn = z + z + ... + x0 → y

porque el punto fijo es único.


60. Probar que la ecuación x=1-tg-1(x) tiene solución α . Encontrar un ijtervalo [a,b]
que contenga a α tal que para cualquier x0 ∈ [a,b], la iteración xm+1 =1+tg-1(xm ) m
≥ 0 tienda hacia x. Calcular las primeras iteradas y ver la rapidez de convergencia.

x=1+arctg(x)
1) ¿Tiene solución?
Sea f(x)=1+arctg(x)-x, cojamos [1,3]:
f(1)=0´7854
f(3)-0´7509
⇒ como f es continua en [1,3] y se tiene que f(1)·f(3)<0, aplicando el teorema de
Bolzano ⇒∃c∈[1,3] tal que f(c)=0 ⇒f(c)=1+arctgc-c = 0 ⇒ c = 1+ arctg c

5/8
www.eltemario.com Oposiciones Secundaria – Matemáticas
© Antonio J. Abrisqueta Valcárcel, 2001 Hoja de Problemas – Álgebra VII

2) Para el intervalo [1,3] tenemos que si f(xn )=1+tg-1(xn ) ⇒ se cumple que


1
f:[1,3]→[1,3] y [1,3] contiene c / f(c)=c y además f´(x)= y se cumple que:
1 + x2
1
f´(x)= <1 ∀x∈[1,3]⇒f es lipschitziana (contractura)⇒∀x0 ∈[1,3] xm+1 =1+tg-1(xm)
1 + x2
m ≥0 tiende hacia c.

61. Probar que el grupo simétrico Sm puede obtenerse a partir del ciclo de m
elementos y de una transposición.

Solución:
 1 2 3.... m − 1 m 
Sea el ciclo C =   y consideremos la transposición
 2 3 4.... m 1 
 1 2
T=   ; el ciclo inverso C-1 que operado con C nos da la identidad es:
 2 1
 1 2 3.... m 
C-1 =   . Vamos a considerar las sustituciones obtenidas mediante el
 m 1 2.... m −1
producto C-h. T Ch siendo h un exponenete que puede tomar valores entre 0 y m-1. Así
por ejemplo para h=0, resulta la misma T, ya que C0 =identidad. Para h=1 tendremos:

 1 2 3.... m   1 2   1 2 3.... m − 1 m 
     
 m 1 2.... m − 1  2 1   2 3 4.... m 1 
 1 m
Es una transposición   ya que todos los elementos permanecen
m 1 
invariantes excepto esos dos.
1→2→1→m
m→ 1 → 2 → 1
Vamos a generalizarlo para una potencia cualquiera.
 1 2 3.... m − 1 m − h + 1 m − h + 2.... m − 1 m 
Ch =  
1 + h 2 + h 3 + h.... m 1 2....... h − 1 h 

 1 2 3.... h h + 1 h + 2 h + 3... m 
C-h=  
 m − h + 1 m − h + 2 m − h + 3.... m 1 2 3... m − h 

6/8
www.eltemario.com Oposiciones Secundaria – Matemáticas
© Antonio J. Abrisqueta Valcárcel, 2001 Hoja de Problemas – Álgebra VII

El producto C-h ·T· Ch mantiene invariantes todos los elementos excepto el m-


h+1 y el m-h+2. En efecto:
m-h+1 → 1 → 2 → m-h+2
m-h+2 → 2 → 1 → m-h+1

Es decir, haciendo h=2 obtenemos la transposición:


 m m − 1
 
 m −1 m 

 m −1 m − 2   3 2
Haciendo h = 3 tendremos   . Para el valor h=m-1 resulta   .
 m − 2 m −1   2 3
Es decir, tendresmos el conjunto de transposiciones:
(2,1); (1,m); (m,m-1); .....................(5,4); (4,3); (3,2)
Recordemos que según un teorema toda sustitución de un grupo simétrico Sm puede
escribirse como producto de transposiciones. Por otra parte, una transposición
cualquiera (a,b) puede expresarse mediante un producto de transposiciones del conjunto
anterior:

Supongamos a>b
(a,b) = (a,a-1) (a-1,a-2)(a-2,a-3)...(b+2,b+1), (b+1,b) (b+1,b+2)...(a-2,a-1) (a-1,a)
En efecto:
a → a-1 → a-2 → ... → b+2 → b+1 → b+1 → b
a-1 → a → a-1 → ...................................................
a-2 → a-1 → a-2 ....................................................
.................................................................................

b+2 → b+3 → b+2


b+1 → b+2 → b+1
b → b+1 → b+2 .................. → a-2 → a-1 → a
Resumiendo una sustitución cualquiera puede expresarse como producto de
transposiciones y éstas, a su vez, vienen dadas como productos de trasnposiciones del
conjunto anterior.

7/8
www.eltemario.com Oposiciones Secundaria – Matemáticas
© Antonio J. Abrisqueta Valcárcel, 2001 Hoja de Problemas – Álgebra VII

62. En el conjunto P de los números enteros pares se definen dos operaciones: una
de ellas es la adición ordinaria y la otra hace corresponder a dos elementos x, y, el
elemento producto xy/2. Demostrar que P tiene estructura de anillo.

Solución:
Veamos que el conjunto P con la adición ordinaria es un subgrupo del grupo
aditivo de los enteros.
Bastará comprobar que si x∈P, y∈P entonces:
x*y-1∈P. En efecto:
x ∈ P ⇒ x = 2K 
 x*y = 2k+(-2h) = 2(k-h), luego x*y ∈P.
-1 -1
Si
y ∈ P ⇒ y = 2h 

Por consiguiente P respecto a la primera operación tiene estructura de grupo, que


además es abeliano ya que la adición ordinaria es conmutativa.
La segunda operación definida en P es cerrada; basta observar que si x=2, y=2
también x⊗ y= xy/2 es múltiplo de dos.
Se verifica la propiedad asociativa:
y· z

x ⊗ (y ⊗ z)= x⊗ (yz/2) = 2 = xyz
2 4
xy
z
xyz
(x ⊗ y) ⊗ z = (xy/2 ⊗ z) = 2 =
2 4

Es fácil ver que se cumplo la propiedad distributiva a derecha e izquierda, puesto


x( y + z )
que: x⊗ (y*z) = x⊗ (y+z) =
2
x( y + z )
(x ⊗ y)*(x⊗ z) = xy/2+xz/2 =
2
Análogamente se comprobaría a la derecha:
(y*z) ⊗ x= (y⊗ x)*(z⊗ x)

8/8
www.eltemario.com Oposiciones Secundaria – Matemáticas
© Antonio J. Abrisqueta Valcárcel, 2001 Hoja de Problemas – Álgebra VIII

63. Dados tres vectores cualesquiera del espacio ordinario de la geometría clásica
de tres dimensiones, demostrar que la condición necesaria y suficiente para que
sean linealmente independientes es que no sea nulo su producto mixto.

Solución:
Sean los vectores
a=(x1 y1 z1 ); b=(x2 y2 z2 ); c= (x3 y3 z3 )
La independencia lineal exige:
λ1 a+λ2 b+λ3 c=0 ⇔ λ1 =λ2 =λ3 =0. Expresando las igualdades escalares:

λ1 x1 + λ2 x 2 + λ3 x3 = 0 

λ1 y1 + λ2 y 2 + λ3 y 3 = 0 
λ1 z1 + λ2 z 2 + λ3 z 3 = 0 

La condición necesaria y suficiente para que un sistema homogéneo tenga


soluciones diferentes de la impropia, es decir, soluciones distintas de λ1 =λ2 =λ3 =0 es que
el determinante
x1 x2 x3
y1 y2 y3 = 0
z1 z2 z3

Por consiguiente, si dicho determinante es diferente de cero, la igualdad


vectorial λ1 a+λ2 b+λ3 c=0 sólo se cumple cuando λ1 =λ2 =λ3 =0; pero dicho determinante
es la expresión del producto mixto de dichos vectores a,b,c.

∈ R} con una operación suma


64. Demuéstrese que el conjunto C={(x,y,z,-x): x,y∈
definida así:
(x,y,y,-x)+(w,z,z,-w) = [x+w,y+z,y+z, -(x+w)] y con el producto por un
∈ R definido por:
escalar k∈
k(x,y,y,-x)=(kx,ky,ky,-kx) constituye un espacio vectorial de dos
dimensiones.

Solución:
Demostremos que C es un subespacio del espacio vectorial R4 . Para ello basta
comprobar el carácter cerrado respecto a la suma de vectores y respecto al producto por
un escalar.

1/11
www.eltemario.com Oposiciones Secundaria – Matemáticas
© Antonio J. Abrisqueta Valcárcel, 2001 Hoja de Problemas – Álgebra VIII

En efecto:
(x,y,y,-x)+(w,z,z,-w) = [x+w,y+z,y+z,-(x+w)]∈C
k(x,y,y,-x) = (kx,ky,ky,-kx)∈C
Una base de dicho espacio vectorial está formada por los vectores (0,1,1,0) y
(1,0,0,-1).
1º) Son linealmente independientes. Es obvio, ya que ninguno de ellos puede
obtenerse multiplicando el otro por un escalar.
2º) Además, cualquier vector de C puede expresarse como una combinación de
estos dos:
(x,y,y,-x) = x(1,o,o,-1)+y(0,1,1,0)
Como todas las bases de un espacio tienen el mismo número de vectores, queda
demostrado que la dimensión es dos.

65. Sea A un anillo conmutativo y con uno, NA su nilradical. Demostrar que las
condiciones siguientes son equivalentes:
i) A tiene un único idel primo.
ii) Todo elemento de A es unidad o está en NA (es decir, es nilpotente).
iii) A/ NA es un cuerpo.

Solución:
i) ⇒ii)
Sea x∈A, si x es unidad ya está demostrado, si no es unidad entonces (x) es un
ideal propio, luego está contenido en un ideal maximal y por tanto en un ideal primo,
como solo existe un ideal primo, P, se sigue que
x∈∩Pi = P, Pi primo ⇒ x∈ NA,
y en consecuencia x es nilpotente.
Obsérvese que la intersección de todos los ideales primos de un anillo es el
nilradical.

ii) ⇒iii)
Sea x+ NA ≠ 0 entonces x∉ NA ⇒ x no es nilpotente
⇒ x es unidad
⇒∃ y + NA = (x+ NA)-1,

2/11
www.eltemario.com Oposiciones Secundaria – Matemáticas
© Antonio J. Abrisqueta Valcárcel, 2001 Hoja de Problemas – Álgebra VIII

entonces A/ NA es un cuerpo.

iii) ⇒i)
Supongamos que A tiene más de un ideal primo, sean P y Q dos ideales primos distintos
de A, entonces existe un x∈A que está en uno de ellos y no en el otro.
Sea x∈P, x∉Q, entonces x∉ NA =∩Pi, Pi primo, luego
x+ NA ≠ 0 ⇒ x + NA es una unidad en A/ NA
⇒ ∃x+ NA / (x+ NA)(y+ NA) = 1 + NA.
⇒ xy-1 ∈ NA
⇒ xy-1∈P
⇒ 1∈P, pues x∈P
luego P=A, contradicción.

66. Resolver la ecuación x2 +2y2 =z2 en el anillo de los números enteros.

Solución:
* Sea d el máximo común divisor de x e y, es decir, d=m.c.d.(x,y), entonces
x=dx1 , y=dy1
y sustituyendo en la ecuación dada, resulta:
d2 x2 1 +2d2 y2 1 =z2
es decir d2 (x2 1 +2y2 1 )=z2
de donde se sigue que d es un divisor de z, y por tanto, z=dz1 . Sustituyendo en la
ecuación anterior resulta:
d2 (x2 1 +2y2 1 )=d2 z2 1
x2 1 +2y2 1 )=z2 1 (1)
La ecuación dada y la ecuación (1) son de la misma forma, y las soluciones de ambas
son proporcionales, es decir, si (a,b,c) es una solución de (1) la terna (da,db,dc) es una
solución de la dada.
Por tanto, supondremos que m.c.d.(x,y,z) = 1, m.c.d.(x,y) = 1, y también que m.c.d.(x,z)
= 1 y m.c.d.(y,z)=1.

* De lo anterior se deduce que x es un número impar. Puesto que 2y2 es par, z2 es impar
y, por tanto, z es impar.

3/11
www.eltemario.com Oposiciones Secundaria – Matemáticas
© Antonio J. Abrisqueta Valcárcel, 2001 Hoja de Problemas – Álgebra VIII

* De la ecuación dada se sigue que:


2y2 = z2 -x2 = (z+x)(z-x), (2)
Puesto que x y z son impares, los números z+x y z-x son pares. Veamos además que el
máximo común divisor de z+x y z-x es 2. En efecto si d es su máximo común divisor se
tiene:
z+x = da, z-x = db
con a y b números primos entre sí. De estas relaciones se obtienen:
2z = d(a+b)
2y = d(a-b)
Puesto que m.c.d.(x,z) = 1 se sigue que m.c.d.(2z,2y) = 2 y por tanto, d=2. De la
ecuación (2) y teniendo en cuenta el resultado anterior, deducimos que:
1 1
( z + x ) ó ( z − x) es impar.
2 2
Por tanto, son primos entre sí los números
1
z+x y (z − x )
2
ó los números
1
z-x y (z + x ) .
2
En el primer caso de la igualdad,
z − x
y2 =(z+x)  
 2 
se deduce que y2 es un cuadrado que

 z + x = n 2
 (3)
 z − x = 2m 2

y de la segunda posibilidad tenemos que:

 z − x = n 2
 (4)
 z + x = 2m2

siendo m y n números enteros, m impar.


i) Del sistema (3) y de la ecuación (2) obtenemos:
1 2 1
* z= ( n + 2m 2 ), x = ( n 2 − 2 m 2 ), y = nm, ( 5)
2 2
ii) Del sistema (4) y de la ecuación (2) obtenemos:

4/11
www.eltemario.com Oposiciones Secundaria – Matemáticas
© Antonio J. Abrisqueta Valcárcel, 2001 Hoja de Problemas – Álgebra VIII

1 2 1
* ( n + 2m 2 ), x = ( 2m 2 − n 2 ), y = nm, ( 6)
2 2
Las fórmulas (5) y (6) pueden reunirse en la siguiente:
1 1
* x = ± ( n 2 − 2m 2 ), z = ( n 2 + 2 m 2 ), y = nm, ( 7)
2 2
con m, como ya hemos dicho, impar. Más aún, para que x y z sean números enteros, los
paréntesis de (7) deben ser números pares, lo que implica que n debe ser para ya que
2m2 lo es.

* Poniendo n=2u y m=v, las fórmulas generales que dan las soluciones de la ecuación
dada, suponiendo que x,y,z son enteros positivos sin divisor común mayor que 1, son:
* x=±(u2 -2v2 ), y=2uv, z=u2 +2v2 , (8)
siendo u y v números enteros positivos primos entre sí y v impar.
Por tanto, las fórmulas (8) dan todas las soluciones de la ecuación dada en números
enteros positivos y primos entre sí, para ello basta elegir a y b de mod que x sea
positivo. Por otra parte, estas ecuaciones satisfacen, como puede comprobarse
sustituyendo la ecuación dada.
Finalmente señalemos que las restantes soluciones se obtienen a partir de éstas
multiplicando por un número cualquiera entero d o poniendo signos arbitrariamente.

67. Dada la ecuación x2 +y2 =z2 , (1), se pide:


a) Resolver dicha ecuación en el anillo de los enteros.
b) A cada solución s(a,b,c) encontrada se le asocia el número complejo
a + bi
w= . Probar que la multiplicación en el conjunto de los números complejos
c
así construidos subordina en el conjunto de soluciones enteras de la ecuación una
ley interna. Estudiar las propiedades de esta ley.

Solución:
a) Para números enteros positivos, se trata evidentemente de valores que verifican el
teorema de Pitágoras. Por eso cada terna que verifica la ecuación se dice que está
formada por números pitagóricos.

5/11
www.eltemario.com Oposiciones Secundaria – Matemáticas
© Antonio J. Abrisqueta Valcárcel, 2001 Hoja de Problemas – Álgebra VIII

* Sea d el máximo común divisor de x e y, es decir, d=m.c.d.(x,y), entonces


x=dx1 , y=dy1
y sustituyendo en la ecuación pitagórica, resulta:
d2 x2 1 +d2 y2 1 =z2
es decir d2 (x2 1 +y2 1 )=z2
de donde se sigue que d es un divisor de z, y por tanto, z=dz1 . Sustituyendo en la
ecuación anterior resulta:
d2 x2 1 +d2 y2 1 =d2 z2 1
x2 1 +y2 1 =z2 1 (2),
La ecuación (2) es también de la misma forma que la dada, y las soluciones de amas son
proporcionales, es decir, si (a,b,c) es una solución de (2), (da,db,dc) es una solución de
(1).
Por tanto, para resolver la ecuación (1) es suficiente limitarse al caso en que los valores
x e y son primos entre sí, es decir, m.c.d.(x,y)=1.

* Si m.c.d.(x,y)=1 entonces al menos uno de los números es impar; sin perder


generalidad podemos suponer que es, por ejemplo, x. De la ecuación (1) se tiene:
x2 =z2 -y2 =(z+y)(z-y)
Sea d1 =m.c.d.(z+y,z-y) entonces podemos escribir que
z+y=d1 a, z-y=d1 b, m.c.d.(a,b)=1
luego x2 =abd2 1 , (3).
Puesto que los números a y b son primos entre sí, la ecuación anterior es cierta si a y b
son cuadrados perfectos, es decir,
a=m2 y b=n2
Sustituyendo estos valores en la ecuación (3) obtenemos:
x2 =m2 n2 d2 1 ,
es decir x=mnd 1 (4)
De las relaciones

 z + y = ad1 = m 2 d 1

 z − y = bd1 = n 2 d1

se obtiene: 2z=(m2 +n2 )d1


2y=(m2 -n2 )d1 (5)
y de aquí

6/11
www.eltemario.com Oposiciones Secundaria – Matemáticas
© Antonio J. Abrisqueta Valcárcel, 2001 Hoja de Problemas – Álgebra VIII

1 2
z= (m + n 2 )d 1 (6)
2
1 2
y= ( m − n 2 ) d1 (7)
2
Es evidente que los números m,n y d1 son impares ya que x=mnd1 es un número impar,
según hemos supuesto. Más aún, d1 =1, ya que en caso contrario d1 sería un divisor de x e
y, basta tener en cuenta las relaciones (4) y (5), ya que en (5) d1 dividiría a y por no ser
d1 par. Contradicción con la hipótesis de que x e y son primos entre sí.
Puesto que d1 =1, las relaciones (4), (6) y (7) se reducen a:
1 2 1 2
x=mn, y= ( m − n 2 ) d1 z= ( m + n 2 ) d 1 , (m>n), (8)
2 2
siendo m y n números impares y primos entre sí. Estas fórmulas nos permiten obtener
todas las ternas de números enteros positivos sin divisores comunes, y que verifican la
ecuación pitagórica (1).
Ejemplos:
*Si n=1 y m=3 se tiene la terna clásica: (3,4,5)
*Si n=1 y m=5 se obtiene la terna: (5, 12, 13).
*Si n=3 y m=5 se obtiene la terna: (15,8,17).
Las soluciones que se derivan de las fórmulas (8) son aquellas que no tienen divisores
comunes; las restantes se obtienen a partir de estas multiplicando por un entero
arbitrario d o poniendo arbitrariamente signos a x,y,z.

b) Sea (a,b,c) una solución de la ecuación pitagórica, entonces el número complejo


a + bi
asociado es: w= .
c
Para que este cociente tenga sentido c debe ser evidentemente distinto de cero, luego la
solución nula (0,0,0) de la ecuación pitagórica no tiene asociado ningún número
complejo.
Por otra parte, es también evidente que dos soluciones proporcionales tienen el mismo
número complejo por imagen, es decir, la aplicación de S*=S-{0,0,0} en C no es
inyectiva.
Teniendo en cuenta que
a + bi
(a,b,c) →
c

7/11
www.eltemario.com Oposiciones Secundaria – Matemáticas
© Antonio J. Abrisqueta Valcárcel, 2001 Hoja de Problemas – Álgebra VIII

d + ei
(d,e,f) →
f
y que,
a + bi d + ei ( ad − be ) + ( ae + bd ) i
· =
c f cf
se puede definir formalmente una operación de SxS en S de la siguiente forma,
obviando las dificultades anteriores:
(a,b,c)*(c,d,e) = (ad-be, ae+bd, cf)

• a) * es una operación interna.


En efecto,
(ad-be)2 +(ae+bd)2 =a2 d2 + b2 e2 + a2 e2 + b2 d2 - 2abde + 2abde =
=(a2 +b2 )(d2 +e2 )
=c2 f2 =(cf)2
• b) * es asociativa por serlo la multiplicación en C.
• c) * es conmutativa por serlo la multiplicación en C.
• d) * tiene elemento neutro y es la terna (1,0,1).
En efecto,
(a,b,c) * (1,0,1) = (a,b,c)
• e) * no tiene elemento inverso.
En efecto, si (x,y,z) es el elemento inverso de (a,b,c) se tiene:
(a,b,c)*(x,y,z) = (ax-by, ay+bx, cz) = (1,0,1)
de donde cz=1, ecuación que no tiene solución en Z si |c|≠1.
Por tanto, (S,*) es un semigrupo conmutativo y con unidad.

b´) Hemos visto que dos soluciones proporcionales dan lugar al mismo número
complejo, se tiene así la siguiente relación de equivalencia en el conjunto de solucioes
S*:
(a,b,c) R (d,e,f) ⇔ (d,e,f) = (ta, tb,tc)
Designaremos por [(a,b,c)] la clase de equivalencia y por T el conjunto cociente S*/R.
La operación * en T viene definida por:
[(a,b,c)]*[(d,e,f)] = [(ad-be, ae+bd,cf)]

8/11
www.eltemario.com Oposiciones Secundaria – Matemáticas
© Antonio J. Abrisqueta Valcárcel, 2001 Hoja de Problemas – Álgebra VIII

Ahora la aplicación de T en C es evidentemente inyectiva. Operar, por tanto, en T, es lo


mismo que operar en C.
Veamos ahora las propiedades que verifica esta operación en T.
i) * es una operación interna por lo visto en b)
ii) * es asociativa por serlo la multiplicación en C.
iii) * es conmutativa por serlo la multiplicación en C.
iv) * tienen elemento neutro y es [(1,0,1)].
v) * tiene elemento inverso.
En efecto, si [(x,y,z)] es elemento inverso de [(a,b,c)] se tiene que cumplir que:
[(a,b,c)]*[(x,y,z)] = [(ax-by,ay+bx,cz)] = [(1,0,1)]
La ecuación ay+bx = 0 tiene solución tomando y=-b, x=a.
Por tanto para que (a,-b,z) sea una terna pitagórica, hemos de tomar z=c. Se tiene
así que:
[(a,b,c)]*[(a,-b,c)] = [(a2 +b2 ,0,c2 )] = [c2 (1,0,1)] = [(1,0,1)]

Por tanto, (T,*) es un grupo conmutativo.

68. Determinar un número entre 400 y 500 tal que al dividirlo por 6 se obtenga de
resto 5, y al dividirlo por 11 el resto es 2.

Solución:
Si A es el número que verifica las condiciones dadas se tiene que cumplir:
 A = 6x + 5 (1)

 A = 11 y + 2 (2)
por tanto, 6x+5=11y+2
de donde 6x+3 = 11y
y de aquí 6x+3≡ mod 11
es decir, 2x+1 ≡ mod 11

Si x =5 entonces 2·5+1 ≡ 0 mod 11


luego, x=5+11t
Sustituyendo este valor en la ecuación (1) resulta que:
A=35+66t
*Para t=0 se tiene A=35 que no cumple las condiciones.

9/11
www.eltemario.com Oposiciones Secundaria – Matemáticas
© Antonio J. Abrisqueta Valcárcel, 2001 Hoja de Problemas – Álgebra VIII

*Para t=1 se tiene A = 101, tampoco cumple las condiciones.


*Para t=2 se tiene A=167, tampoco válida.
----------------------------------------------------------------------------------------------------------
*Para t=6 se tiene A=35+396 = 431
*Para t=7 se tiene A = 35+462 = 497
que son las soluciones válidas.

69. Dado el sistema de ecuaciones:


x 3 + y 3 + z 3 = 3

x + y + z = 3
hallar las soluciones enteras, es decir, resolverlo en Z.

Solución:
x 3 + y 3 + z 3 = 3 ( x + y )( x 2 − xy + y 2 ) + (3 − ( x + y )) 3 = 3
El sistema  es equivalente a, 
x + y + z = 3  z = 3 − ( x + y)
( x + y )(3( x + y ) − ( 9 + xy)) = −8
de donde,  .
 z = 3 − ( x + y)
Teniendo en cuenta la primera ecuación, las soluciones de x e y se obtienen resolviendo
los sistemas que resultan al descomponer –8 en producto de dos números enteros de
todas las formas posibles e igualando cada uno de los factores del primer miembro a
dichos números enteros. Los casos posibles son:
x + y = −8  x + y = −8
1) ⇔  No tiene soluciones en Z.
3( x + y ) − (9 + xy) = 1 XY = −34 

x + y = −8  x + y = 8 x = 4
2) ⇔ ⇔ 
3( x + y ) − (9 + xy) = −1 xy = 16  y = 4
Por tanto, la solución del sistema es: x=4, y=4, z=-5.
x + y = −4  x + y = −4
3) ⇔  No tiene solución en Z.
3( x + y ) − (9 + xy) = −2 xy = −23 

x+y=4  x + y = 4
4) ⇔  No tiene solución en Z.
3( x + y ) − (9 + xy) = −2 xy = 5 

x + y = −2  x + y = 2
5) ⇔  No tiene soluciones en Z.
3( x + y ) − (9 + xy) = 4 xy = −19 

10/11
www.eltemario.com Oposiciones Secundaria – Matemáticas
© Antonio J. Abrisqueta Valcárcel, 2001 Hoja de Problemas – Álgebra VIII

x+y=2  x + y = 2 x = 1
6) ⇔ ⇔ 
3( x + y ) − (9 + xy) = −4  xy = 1  y = 1
Por tanto la solución del sistema es: x=y=z=1.
x + y =1  x + y = 1
7) ⇔  No tiene solución en Z.
3( x + y ) − (9 + xy) = −8 xy = 2 

x + y = −1  x + y = −1 x1 = −5  x2 = 4
8) ⇔ ⇔ ó
3( x + y ) − (9 + xy) = 8 xy = 20  y1 = 4   y 2 = −5

Por tanto las soluciones del sistema son: (-5,4,4,) ó (4,-5,4).

11/11
www.eltemario.com Oposiciones Secundaria – Matemáticas
© Antonio J. Abrisqueta Valcárcel, 2001 Hojas de Problemas – Álgebra IX

70. Sea A la familia de funciones fn:R+* → R definidas por fn(x)=lognx, ∀ n∈


∈ N (R+*={x∈
∈ R:x>0}).
a) Estudiar su dependencia lineal en el R-espacio vectorial Aplic(R+*,R).
b) Hallar la dimensión y una base del subespacio que engendran.
Solución:
a) Es log nx = log n +log x.
Si n≠ m, fn y f m son linealmente independientes:
α fn + β f m = 0, α,β∈R ⇒ ∀x∈R+*, αlog n x + βlog mx = 0 ⇒
∀x∈R+*, (αlog n + βlog m)+(α+β)logx = 0 ⇒ α+β=0 y αlog n + βlog m = 0 ⇒ β=-α,
α(log n - log m) = 0 ⇒ α=β=0, al ser log n ≠ log m.
Si además k es distinto de n y m, fk depende linealmente de fn y fm, pues para que sea fk =αfn +βf m,
α,β∈R se ha de verificar que ∀x∈R+*:
log kx =αlog nx + βlog mx ⇔ log k = αlog n + βlog m t 1 = α + β,
por lo que basta tomar β = 1-α y α=logk (log n - log m)-1.
En resumen, cualquier par de funciones de A son linealmente independientes y tres cualesquiera son
linealmente dependientes.

b) Por el apartado anterior es dim L(A) = 2 y si n≠m, {fn ,f m} es una base de L(A).
Además L(A) = {f:R+*→R | f(x) = α+βlogx, α,β∈R}, luego {id, log} es otra base de L(A)
(obsérvese que la función id(x) = x ∀x∈R+* no está en A).

71. Sean a1 ,...,an∈ R dos a dos distintos y Rn-1[x] el espacio vectorial de polinomios de grado menor
que n.
a) Encontrar la familia de polinomios de grado n-1, {p1 ,...,pn} caracterizada por pi (aj) = δ ij.
b) Probar que {p1 ,...,pn } es una base de Rn-1[x].
c) Hallar respecto de dicha base las coordenadas de q(x) = xn-1+xn-2+...+x+1.

Solución:
a) Para j≠i aj es un cero de p i y al ser p i (ai ) = 1, necesariamente:

1
p i (x) = ( x − a1 )...( x − ai −1 )( x − ai +1 )...( x − a n )
(a i − a1 )...( ai − a i −1 )( ai + ai +1 )...( ai − a n )

b) Por ser dim Rn-1[x]=n, bastará probar que {p 1 ,...,p n } es un sistema de generadores:
n
Cualquiera que sea q(x) ∈ Rn-1[x]=n es q(x) = ∑ q(a ) p (x ), ya que ambos miembros son polinomios
i i
i =1

de grado menor que n que coinciden en n puntos: a1 ,...,an ; luego son polinomios idénticos.

c) Según el apartado anterior:

1/7
www.eltemario.com Oposiciones Secundaria – Matemáticas
© Antonio J. Abrisqueta Valcárcel, 2001 Hojas de Problemas – Álgebra IX

n n n
ain − 1
q(x) = ∑ q(a i ) pi (x ), = ∑ (ain−1 + ain−2 + ... + a i + 1) p i (x ) = ∑
i =1 i =1 a j −1 i
p ( x) ,
i =1

ya que xn -1 = (x-1)(xn-1+xn-2+...+x+1).

 a in − 1 
Así, supuestos los ai≠1, i=1,...,n, las coordenadas de q(x) son 
 a − 1  . Y si para algún i es ai =1,
 i  i =1 ,..., n
la coordenada i-ésima es ai n-1+ ain-2 +...+ ai +1 = 1n-1+ 1n-2+...+1+1 = n, permaneciendo las restantes.

≥ 1.
72. Sea E=Rn [x] el R-espacio vectorial de los polinomios de grado menor o igual que n, siendo n≥
Sean B = {1,x,x2 ,...,xn} y B´={v0 , v1 ,..., vn } con vi = xi (1-x)n-1 para i=0,1,...,n.
a) Demostrar que B y B´ son bases de E.
b) Hallar las matrices de los cambios de base de B a B´ y de B´ a B.

Solución:
a) Es inmediato demostrar que los elementos de B son linealmente independientes y que consituyen un
sitema generador de E.
Para ver que B´ es una base, teniendo en cuenta que el cardinal de B´ es n+1 = =dim E, basta ver
que los elementos de B´ son linealmente independientes. Veámoslo:

 n  n   n
v 0 = (1 − x) n = 1 −   x +   x 2 − ... + ( −1) n   x n
1   2  n
 n − 1 2  n − 1 3  n − 1 n
v1 = x(1 − x) n −1 = x −   x +   x − ... + (−1) n −1   x
 1   2   n − 1
 n − 2 3  n − 2  4  n − 2 n
v 2 = x 2 (1 − x) n − 2 = x 2 −   x +   x − ... + ( −1) n − 2   x
 1   2   n − 2
........................................ ........................................ ........................................
v n = xn = xn
n− k
n − k  r
Es decir: v = x ∑ ( −1)   x , k = 0,1,..., n
r
k
 r 
k
r =0

La matriz que expresa las coordenadas de los vectores de B´ respecto de B es:

2/7
www.eltemario.com Oposiciones Secundaria – Matemáticas
© Antonio J. Abrisqueta Valcárcel, 2001 Hojas de Problemas – Álgebra IX

 1 0 0 ..........1
 n 
 −   1 0 ..........0
   1 
 n  n − 1 
  2  −   1 ..........0
M= 
   1  
 . . . . 
 . . . . 
 
 . . . . 
  n   n − 1   n − 1 
( −1)   ( −1)   ( −1) n −1  
n n −1
..........1
  n  n − 1  n − 1 

Evidentemente |M| = 1 ≠ 0 ⇒ B´ es un conjunto de vectores linealmente independientes.


b) La matriz del cambio de base de B a B´ es M.
La matriz del cambio de base de B´ a B será M -1. Para la operación de M -1 es muy cómodo hallar
directamente la expresión de cada xk respecto de los vi .
n− k
n − k  r
k k
Para cada k∈{0,1,...,n} es x = x (1-x+x) = x k k ∑   x (1 − x) n − k − r =
r =0 r 
n− k
 n − k  r+ k n− k n − k
 
= ∑ 
r =0 r 
 x (1 − x ) n − k − r = ∑ 
r= 0 
 .
r  v r+ k

Así:

 1 0 0 ..........1
 n  
  1 0 ..........0
 1  
 n   n − 1 
 2    1 ..........0
   1  
M -1 =  n − 2 
 . .   ..........0
  1  
 . . . . 
 . . . . 
 n   n − 1 n − 2 
      ..........1
 n   n − 1 n − 2 

73. Sean V, W dos C-espacios vectoriales. Sea f un homomorfismo de V en W como R-espacios


∈ Homc(V,W) sii f(iz)=i f(z) para todo z de V.
vectoriales. Probar que f∈

Solución:
Si f∈HomC(V,W) entonces, por restricción de la ley externa, es f ∈ HomC(V,W) y además f(i z)
= i f(z).

3/7
www.eltemario.com Oposiciones Secundaria – Matemáticas
© Antonio J. Abrisqueta Valcárcel, 2001 Hojas de Problemas – Álgebra IX

Recíprocamente, si f∈HomR(V,W) y f(i z)= i f(z), se tiene que:


f(z+w) = f(z)+f(w) para z,,w∈V
f((a+bi)z) = f(az+ibz) = f(az)+f(ibz) = f(az) + i f(bz) = af(z) + i b f(z) = (a+bi) f(z) para a+bi ∈ C y z∈V.
En consecuencia, f∈HomC(V,W).

74. Sea M2 el espacio vectorial sobre R de las matrices cuadradas de orden 2 con elementos en R.
Consideremos las aplicaciones:
a) f: M2 → M2 b) g: M2 → M2

a b   pa pb  a b   pa 0
 c d  →  pc pd  c d  →  0 pd 
     
donde p es un cierto elemento fijo de R. Se pide demostrar que son endomorfismos ¿Son
automorfismos?

Solución:

 a b  a´ b´   λa + µa´ λb + µb´    pλa + pµa´ pλb + pµb´ 


a) f  λ  + µ   = f 


 λc + µc´ λd + µd´  =  pλc + pµc´ pλd + pµd ´ =
 c d   c´ d ´     
 pa pb   pa´ pb´   a b   a´ b´ 
= λ  + µ =λ f     + µ f     cualesquiera que sean
 pc pd   pc´ pd ´  c d     c´ d ´ 
a b   a´ b´ 
λ,µ∈R y  c d ,  c´ d´ ∈ M 2
  

Luego f es un endomorfismo. Es la homotecia de razón p, pues f(x) = px ∀x∈M 2 .


Si p=0 f es el endomorfismo nulo. Por ello ni es inyectivo ni suprayectivo.
Si p≠0 f es automorfismo, pues:
f(x)=f(y) ⇔ px=py ⇔ x=y (por ser p≠0)

1
f-1(y)= y para todo y ∈M2 .
p
 a b a´ b´    λa + µa´ λb + µb´    pλa + pµa´ 0 
g  λ  + µ   = f  =
   pλd + pµd ´
b) =
 c d  c´ d´    λc + µc´ λd + µd´   0

 pa 0   pa´ 0   a b    a´ b´  
= λ  + µ  = λ g     + µ g 


 c´ d´  cualesquiera que sean λ,µ∈R
0 pd   0 pd ´  c d   
a b   a´ b´ 
y  c d ,  c´ d´ ∈ M 2
  
Así, g es un endomorfismo.

4/7
www.eltemario.com Oposiciones Secundaria – Matemáticas
© Antonio J. Abrisqueta Valcárcel, 2001 Hojas de Problemas – Álgebra IX

  0 1   0 0
g nunca es inyectiva ya que g     = 0 0 , luego g no es automorfismo. Además, g no es
 1 0   
0 1 
sobreyectiva, ya que 1 0 ∉ g ( M 2 ) .
 

75. Sea F(R,R) el R-espacio vectorial de todas las aplicaciones de R en R. Sea E el subespacio
→ senx, x→
vectorial engendrado por la función constante igual a 1 y las funciones x→ → cosx.
Definimos las aplicaciones α ,β
β : E→
→ E como:

π
α (f)(x) = f( − x ), β =(f)(x)=f´(x).
2
a) Demostrar que α ,β
β son aplicaciones lineales.
b) Obtener α ° β y β ° α y comprobar que en este caso α ° β = - β ° α .

Solución:
a)Veamos que α(f) es un elemento de E, para todo f∈E.
En efecto: Sea f(x) = a+bsen(π/2-x)+c cos(π/2-x) = a+b cosx+c senx, luego α(f)∈E.
Análogamente:β(f)(x)=f´(x) = bcosx-csenx, que también pertenece a E.

*Veamos que α es lineal:


Sean f(x) = a+bsenx+ccosx, g(x) = m+nsenx+pcosx y λ,µ∈R. Se verifica:
α(λf+µg)(x) = (λf+µg)(π/2-x) = λa +µm+(λb+µn) cosx +(λc+µp)sen x = λa +λbcosx + +λcsen x + µm+µn
cosx+ µpsenx = λa +λbsen(π/2-x)+λc cos(π/2-x)+ µm+ µnsen(π/2-x) +µp cos(π/2-x) = λα(f)(x)+
µα(g)(x).
Luego α(λf+µg) = λα(f)+µα(g) y α∈End (E).

* Veamos que β es lineal:


β(λf+µg)(x) = (λf+µg)´(x) = (λb+µn) cosx - (λc+µp) senx= λ b cosx - λ c sen x + µ n cosx - µp senx =
λf´(x)+µg´(x) = λ β (f)(x) + µβ(g)(x).
Así: β(λf+µg) = λβ(f)+µβ(g) y β∈End E.

b) (α°β)(f) = α[β(f)] = α(f´) ⇒ (α°β)(f)(x) = f´(π/2-x).


(β°α)(f) = β[α(f)] ⇒ (β°α)(f)(x) = -f´(π/2-x).

Luego α°β=−β°α.

76. Sea V un espacio vectorial de dimensión 2 y {u1 ,u2 } una base de V. Se consideran f,g ∈ EndV tales
que:
f(u1 )=u1 +u2 , g(u1 )=-2u1 + 2u2 .

5/7
www.eltemario.com Oposiciones Secundaria – Matemáticas
© Antonio J. Abrisqueta Valcárcel, 2001 Hojas de Problemas – Álgebra IX

f(u2 )=f(u1 ), g(u2 )= u1 - u2 .

a) Demostrar que f y g conmutan.


b) Demostrar que son divisores de cero del anillo End V.
c) ¿Es cierto que fn ° gm = g m ° fn cualesquiera que sean los números naturales m, n?

Solución:
a) Para ver que f°g = g°f, como son aplicaciones lineales, basta ver que los dos miembros
transforman del mismo modo los elementos de una base:
f°g(u 1 ) = f(-2u 1 +2u 2 ) = -2(u 1 +2u 2 )+2(u 1 +2 u 2 ) = 0
g°f(u 1 )= g(u 1 +2u 2 ) = -2u 1 +2u 2 +2(u 1 -u 2 ) = 0
f°g(u 2 ) = f(-2u 1 +2u 2 ) = 0
g°f(u 2 )= g(u 1 +2u 2 ) = -2u 1 +2u 2 +2u 1 -2u 2 ) = 0

b) Es evidente, pues según el apartado anterior f°g = g°f = 0 (endomorfismo cero).


c) Sí es cierto. Lo demostraremos por inducción en dos etapas:

1ª etapa: Sea n un natural; entonces g°fn = fn °g.


Por inducción sobre n:
* Para n=1 ya está probado (apartado a)).
* Supongamos que es cierto para n y demostrémoslo para n+1:
fn+1 °g = f°(fn °g) = f°(g°fn ) = (f°g)°fn = (g°f)°fn = g°fn+1 .

2ª etapa: ∀n natural, ∀m natural, fn °g m = g m°fn . En efecto: consideramos n fijo y hacemos inducción en


m:
* Para m=1 es cierto (1ª etapa).
* Supongamos que es cierto para m y lo probamos para m+1:
fn °g m+1 =( fn °g m)°g = (g m°fn ) ° g = g m°(fn °g m) = g m°(g°fn ) = (g m°g)°fn = g m+1 °fn .

∈ EndV designemos por fn = f ° ...°° f ( n veces). Establecer las


77. Sea V un espacio vectorial. Si f∈
relaciones siguientes:
1) Im f = Im f2 ⇒ V=Im f + ker f.
2) Ker f = Ker f2 ⇒ V = Im f ∩ Ker f = {0}.
3) V = Im f ⊕ Ker f ⇔ Im f=Imf 2 , Ker f = Ker f2 .

Solución:
1) Sea x∈V; f(x) ∈Im f = Im f2 ⇒ ∃y∈V tal que f(x) = f2 (y) ⇒ f(x-f(y) = 0 ⇒ z=x-f(y) ∈ker f ⇒
x=f(y)+z ∈ Im f + Ker f.

6/7
www.eltemario.com Oposiciones Secundaria – Matemáticas
© Antonio J. Abrisqueta Valcárcel, 2001 Hojas de Problemas – Álgebra IX

Luego efectivamente V⊂Im f + Ker f. Como el otro contenido es evidente, la igualdad V = Im f + Ker f es
cierta.
2) Sea x∈Im f ∩Ker f; entonces existe y ∈ V tal que x=f(y), pero f(x) = 0 ⇒ f2 (y) = 0 ⇒ y ∈ Ker f2 = Ker
f ⇒ f(y) = 0 ⇒ x= 0.
Es decir, Im f ∩Ker f = {0}.
3) ⇐ Es la conjunción de 1) y 2)
⇒ *Es evidente que Imf 2 ⊂ Im f.
*Veamos que Im f⊂Imf2 :
x∈Im f ⇒ ∃y∈V tal que f(y) = x. Por ser V = Im f ⊕ Ker f, existe u∈Ker f, f(z)∈Im f tales que y
= f(z) + u.
Luego x = f(y) = f(f(z))+f(u) = f2 (z) ⇒ x∈Im f2 .
Así, de todo lo anterior se deduce que Im f2 = Im f.

*También es evidente que Ker f ⊂ Ker f2 , pues x∈Ker f ⇒ f(x) = 0 ⇒f2 (x) = f(0) = 0 ⇒ x∈Ker f2 .
Veamos que Ker f2 ⊂Ker f:
x∈Ker f2 ⇒ f2 (x)=0 ⇒ f(f(x)) = 0 ⇒ f(x) ∈Ker f ⇒ f(x) ∈Ker f∩Im f = {0} ( por ser V= Ker f
⊕ Im f).
Así x∈Ker f.
En resumen: Ker f = Ker f2 . Esto completa la demostración.

7/7
www.eltemario.com Oposiciones Secundaria – Matemáticas
©Antonio Jesús Abrisqueta Valcárcel.2001. Hoja Problemas – Análisis I

n =1
78.- Hallar ∑ sen (a + nj ) = sen a + sen (a + j) + sen (a + 2 j ) + ... + sen (a + (n − 1) j) .
j= 0

Solución:

Como sabemos que senp·sen q = 1/2 (cos(p-q)-cos(p+q)) entonces se tiene que:

1  2 j −1   2 j + 1 
cos a + u  − cos a + u 
n −1 n −1
sen(a + uj )sen(u / 2 ) n −1
2  2   2 
∑ sen(a + uj ) = ∑
j= 0 j= 0 u
=∑
j= 0 u
=
2sen 2 sen
2 2
 2 j −1   2 j +1 
n −1
cos a + u  − cos  a + u
    = (*)
=∑
2 2
j= 0 u
2sen
2

Si j=0 cos a − u  − cos  a + u 


1 1
 2   2 

Si j=1 cos a + u  − cos a + u 


1 3
 2   2 

Si j=2 cos a − u  − cos  a + u 


3 5
 2   2 
. . . . . .
2n − 5  2n − 3 
Si j=n-2 cos a + u  − cos a + u
 2   2 
2n − 3  2n − 1 
Si j=n-1 cos a + u  − cos a + u
 2   2 
2n − 1 
cos a − u  − cos  a +
1
u
 2   2 

 1   2n - 1 
cos  a - u  - cos  a + u  usando la igualdad cos p − cos q =
1  2   2 = 
(*) = ·   p +q  p−q 
2 sen
u = 2 sen 2 ·sen 2  
2  

1/8
www.eltemario.com Oposiciones Secundaria – Matemáticas
©Antonio Jesús Abrisqueta Valcárcel.2001. Hoja Problemas – Análisis I

 2a 2 n − 2   2n   n −1   n 
2sen + u  sen u  sen a + u  sen u 
1  2 4   4   2  4 
= · =
2 u u
sen sen
2 2

 n −1  n 
n −1
sen a +  sen  u 
 2  2 
∑ sen( a + uj ) =
 n
j= 0
sen 
 2

79.- Hallar el volumen del sólido generado al girar alrededor del eje OX, la región
del plano que resulta de la intersección del interior x2 +y2 =17x.

Resolución:
Las dos circunferencias se cortan en 17=17x ⇒x=1, y para x=1 ⇒ y2 =16 ⇒
⇒ y=±4 ⇒ P1 (1,-4) y P2 (1,4).

El volumen pedido es la mitad del volumen de la esfera de radiio 17 es decir,


14 34
V1 = V1 = · π ( 17 ) 3 = π 17 , más del volumen resultante de girar entorno al eje ox
23 3
γ, menos el de γ2 .
1
(
V2 = ∫0 π  17 − x 2

) − ( 17 x − x ) dx = ∫ π(17 − x
2
2
2 1

0
2
− 17 x + x 2 ) dx =
1
 x2  17
= ∫ π(17 − 17 x) dx =17π ∫ (1 − x ) dx = 17π x −  = π
1 1

0 0
 2 0 2

2/8
www.eltemario.com Oposiciones Secundaria – Matemáticas
©Antonio Jesús Abrisqueta Valcárcel.2001. Hoja Problemas – Análisis I

 68 17 + 51  3
VT = π u

 6 

80.- Sea la función y=f(x) , definida en todo R, de modo que los incrementos
correspondientes de x e y son proporcionales de razón a en cualquier punto. Se
supone que además y0 =f(x0 ) es conociedo. Calcular la expresión general de y=f(x).

Solución:
Tengamos en cuenta que ∆f(x) = f(x+∆h)-f(x) siendo ∆x el incremento de la
variable x.
Por lo tanto:
∆f ( x ) f ( x + ∆x ) − f ( x )
= = a para todo x.
∆x ∆x
De aquí se deduce que esta función es derivable en cualquier punto ya que su
derivada en x será:

f´(x)= lim∆x→0 f ( x + ∆x ) − f ( x ) = a
∆x
Además si f´(x)=a para todo x, la función es de la forma
f(x)= ax+m
falta, finalmente, calcular m. Para ello, obligamos a que y0 =f(x0 ):
y0 =f(x0 )=ax0 +m
es decir, m=y0 -ax0 =f(x0 )-ax0 . Por lo tanto la función es:
f(x) =ax+f(x0 )-ax0 , o bien
f(x)-f(x0 ) = a(x-x0 ).

81.- Separar las raíces de la ecuación siguiente:


2x3 +3x2 -72x+12=0.

Solución:
Sea f(x) = 2x3 +3x2 -72x+12.
f´(x)= 6x2 +6x-72 = 6(x2 +x-12).
La ecuación f´(x)=0 tiene raíces x1 =-4, x2 =3. Puesto que f(-4) = 220 >0 y f(3) =-123 <0,
resulta que f(x) = 0 tiene una única raíz en el intervalo (-4,3). Además f(0)=12>0, así

3/8
www.eltemario.com Oposiciones Secundaria – Matemáticas
©Antonio Jesús Abrisqueta Valcárcel.2001. Hoja Problemas – Análisis I

que dicha raíz está en el intervalo (0,3). Por otra parte, f(6)>0 y f(3)<0, lo que nos
permite asegurar que existe una única raíz en el intervalo (3,6). Finalmente, f(-4)>0 y
f(-7)<0 así que la raíz se encuentra en el intervalo (-7,-4).

82.- Aplicar el teorema de los incrementos finitos a la función f(x)=(x2 +9)1/2 en el


intervalo [0,4]. Aplicar la fórmula de Cauchy a las funciones f(x)=senx, g(x)=cosx,
π /4,3π
en el intervalo [π π /4]. En ambos casos se pide hallar el valor o valores del
"punto intermedio".

Solución:

1) La fórmula de los incrementos finitos se escribe en esete caso: f(4)- f(0)=f´(c)(4-0).


Es decir: (16+9)1/2 -91/2 = f´(c)·4. De donde resulta f´(c)=1/2. Busquemos el punto
c∈(0,4).
1 2 x
f´(x) = ( x + 9)1 / 2 2 x = .
2 x2 + 9
1 x
Resolviendo la ecuación = obtenemos c= 3 .
2 x2 + 9
2) La fórmula de Cacuhy es:
 3π  π  3π π
f − f  sen − sen
 4   4  = f ´(c) o bien 4 4 = cos c = − cot gc
 3π   π  g´(c ) 3π π − senc
g  − g  cos − cos
 4  4 4 4

de donde cotg c = 0 y c=π/2.

83.- Un depósito está inicialmente lleno con 1000 litros de agua salada cuya
concentración o salinidad es de dos gramos de sal por litro. Para reducir la
salinidad se hace entrar agua pura en el depósito a razón de 5 litros por minuto, al
tiempo que por un orificio el depósito evacua el mismo caudal. Determinar la
cantidad de sal contenida en el depósito en función del tiempo y calcular el tiempo
que debe transcurrir para que queden sólo 200 gramos de sal.

Denotemos por S(t) a la función pedida, es decir, la cantidad de sal (en gramos) que
contiene el depósito en función del tiempo t(en minutos). La concentración de sal en el

4/8
www.eltemario.com Oposiciones Secundaria – Matemáticas
©Antonio Jesús Abrisqueta Valcárcel.2001. Hoja Problemas – Análisis I

instante t vale S(t)/1000=10-3S(t) gr/litro. En el instante t+∆t la cantidad de sal será


S(t+∆t) y por consiguiente los gramos de sal que han salido del depósito en el intervalo
de tiempo [t,t+∆t] son S(t)-S(t+∆t). Pero por otra parte sabemos que durante ese
intervalo de tiempo el depósito ha evacuado 5∆t litros de agua salada. Si el incremento
∆t es muy pequeño, la concentración del agua del depósito puede considerarse constante
en el transcurso del intervalo de tiempo meniconado. El producto del agua que ha salido
5∆t por dicha concentración 10-3S(t) nos dará también la cantida de sal desaparecida en
dicho intervalo de tiempo. Planteamos así la ecuación:
S(t)-S(t+∆t)= 5·10-3S(t)∆t
esto es
S ( t + ∆t ) − S (t ) − 1
= S (t )
∆t 200
Tomando límites cuando ∆t→0 obtendremos (supuesta derivable la función S)
S´(t) = -S(t)/200
es decir
S´(t ) − 1
=
S (t ) 200
de modo que
d −1
log S (t ) =
dt 200
luego
S(t)=e-1/200 t +k
donde k es una constante a determinar. Para t=0 sabemos que S(0) = 2000 luego
ek =2000 y la función buscada es, por fin:
S(t)= 2000e-t/200.

Llamemos T al instante en el cual la sal que queda en el depósito son 200


gramos. Sustituyendo en la igualdad anterior, será:
200=2000e-T/200
de donde se obtiene T=200log10 y como log10 = 2,30 resulta aproximadamente
T=460m = 7h 40m.

5/8
www.eltemario.com Oposiciones Secundaria – Matemáticas
©Antonio Jesús Abrisqueta Valcárcel.2001. Hoja Problemas – Análisis I

84.- Demostrar que todos los triángulos con la misma base y el mismo ángulo
opuesto, el isósceles tiene área máxima.
Probar que entre todos los triángulos inscritos en una circunferencia dada,
el equilátero tiene área máxima.

Solución:
Sean a y A fijos. El área del triángulo es:

S=ah/2.
Ahora bien:

h=bsenĈ

Aˆ + Bˆ + Cˆ = π
a b c
= =
senAˆ senBˆ sen Cˆ

sistema de ecuaciones que nos permite deducir que


a a
h= senBˆ ·sen Cˆ = sen Cˆ ·sen(π − Aˆ − Cˆ ).
senAˆ senAˆ
Sustituyendo en la expresión del área resulta

1 a2
S= senCsen (π − A − C)
2 senA
y buscamos el máximo de S en función de Ĉ :

1 a2
S= [cos Csen(π − A − C ) − senC cos(π − A − C) ] =
2 senA
1 a2
= sen (π − A − 2C )
2 senA

π− A
La ecuación S´=0 es sen(π-A-2C)=0, es decir, π-A-2C=0 de donde C= .
2

6/8
www.eltemario.com Oposiciones Secundaria – Matemáticas
©Antonio Jesús Abrisqueta Valcárcel.2001. Hoja Problemas – Análisis I

a2
Puesto que S´= − cos(π − A − 2C) , es sencillo comprobar que este valor de C
senA
π− A
corresponde a un máximo de S. Pero entonces, B=π-A-C= , es decir, B=C así
2
que, efectivamente, el triángulo que buscábamos es un triángulo isósceles.

Segunda parte:
En el primer lugar, hemos de hacer notar que si consideramos los triángulos
inscritos en una circunferencia con un lado fijo, se verifica que el ángulo opuesto a
dicho lado tendrá el mismo valor para todos ellos, ya que será un ángulo inscrito a una
circunferencia abarcando un arco fijo.
Deducimos entonces de la primera parte que de todos los triángulos inscritos en
una misma circunferencia que tengan un lado fijo, el de mayor área es el isósceles .
Entonces, resulta evidente que:
Dado un triángulo cualquiera inscrito en una circunferencia fija o bien este
triángulo es isósceles o bien puedo encontrar un isósceles también inscrito en dicha
circunferencia, que tenga área mayor que el triángulo inicial. (Basta para ello con tomar
como fijo uno de los lados del primer triángulo y aplicar el párrafo anterior). Nuestro
problema se reduce pues a demostrar que entre todos los triángulos isósceles en una
circunferencia fija, el de área máxima es el equilátero.

Calculemos el área de este triángulo isósceles en función de x.


A

B C

S=bh/2.
h=r+x.
Para calcular b/2, tendremos en cuenta que el triángulo ABP es rectángulo en B,
así que b/2 es la altura relativa a la hipotenusa en dicho triángulo, cumpliéndose que:

7/8
www.eltemario.com Oposiciones Secundaria – Matemáticas
©Antonio Jesús Abrisqueta Valcárcel.2001. Hoja Problemas – Análisis I

r + x b / 2 b2
= = ( r + x )( r − x) = r 2 − x 2
b/2 r − x 4
es decir

b/2= r 2 − x 2 .
Se obtiene finalmente:
b
S= h = ( r + x ) r 2 − x 2
2
Calculemos el valor de x que hace máxima esta expresión:

−x − 2 x 2 − rx + r 2 r+x
S´= r 2 − x 2 + ( r + x ) = = (r − 2 x )
r 2 − x2 r 2 − x2 r−x

El valor x=r/2 anula S´ y además hace negativa S´´, así que corresponde a un máximo.

Pero si x=r/2, se obtiene que b=2 r 2 − x 2 =r 3 , es decir, el triángulo ABC es


equilátero.
En efecto:

9r 2 + 3r 2
AB = AC = ( r + x ) 2 + (b / 2) 2 = (3r / 2) 2 + ( r 3 / 2) 2 = = r 3.
4

8/8
www.eltemario.com Oposiciones Secundaria – Matemáticas.
©Antonio Jesús Abrisqueta Valcárcel, 2001. Hoja Problemas – Análisis II

85.- Sea Bj(t) la función polinómica:


 n
Bj(t)=  t j (1 − t ) n − j , t ∈ R, j ≤ n, jn ∈ N
 j
Demostrar que:
n
i) ∑Bj =1
j (t ) = 1

n
ii) ∑ jB (t ) = nt
j =1
j

n
iii) ∑ j ( j − 1)B
j= 2
j (t ) = n( n − 1) t 2

n
iv) ∑ ( j − nt )
j= 2
2
B j (t ) = nt (1 − t )

Solución:
n
n
Consideremos la identidad: (t+x)n= ∑  j t
j= 0
j
x n− j , n ∈ N , t, x ∈ R .

Derivando en ambos miembros respecto de t y multiplicando después por t se obtiene:


n
n
n(t+x)n-1t= ∑  j  jt
j =1
j
x n− j

Derivando nuevamente y multiplicando por t2 resulta:


n
 n
n(n-1)(t+x)n-2t2 = ∑  j  j( j − 1)t
j= 2
j
xn − j

Las igualdades i), ii) y iii) se obtienen inmediatamente sustituyendo x por 1-t en las tres
identidades anteriores.
Operando en las igualdades ii) y iii) resulta:
n n n

∑j= 2
j ( j − 1) B j (t ) = ∑ j 2 B j (t ) − ∑ jB j (t ) = n( n − 1)t 2
j= 2 j =2

de donde:
n n

∑j
j= 2
2
B j (t ) = n( n − 1)t + ∑ jB j (t ) = n( n − 1) t 2 + nt − B1 ( t )
2

j= 2

Entonces:

1/9
www.eltemario.com Oposiciones Secundaria – Matemáticas.
©Antonio Jesús Abrisqueta Valcárcel, 2001. Hoja Problemas – Análisis II

n n n n

∑ ( j − nt )2 B j (t) = ∑ j 2 B j (t) + n 2t 2 ∑ B j (t) − 2nt ∑ jB j (t) =


j= 0 j =0 j =0 j =0

= n(n-1)t2+nt+n2t2-2n2t2 = nt-nt2 = nt(1-t), que es la igualdad iv).

86.- Probar que la función f(x) = x/(1+x2) definida en toda la recta real, es
uniformemente contínua.

Solución:
Calculemos en primer lugar |f(x1)-f(x2)|, siendo x1,x2 dos números reales arbitrarios:

x1 x 1 − x1 x 2
|f(x1)-f(x2)| = − 2 2 =| x1 − x1 |
1 + x1 1 + x 2
2
(1 + x12 )(1 + x 22 )

1 − x1 x 2
Comprobemos que ≤1 para todo par x1,x2∈R.
(1 + x12 )(1 + x 22 )

Llevaremos a cabo dicha comprobación teniendo en cuenta los posibles valores de |1-
x1x2|.
a) |1-x1x2| = 1-x1x2.
1 − x1 x 2 1 − x1 x2
Entonces = ≤ 1 es una desigualdad equivalente a 1-x1x2≤
(1 + x )(1 + x )
2
1
2
2 (1 + x12 )(1 + x22 )

≤(1+x12) (1+x22), o bien x2+x22+x12 x22+x1x2 ≥0. Ahora bien, el primer miembro de esta

x  3x
desigualdad se puede escribir en la forma  1 + x 2  + 1 + x1 x 2 , que evidentemente, es una
2  4
expresión positiva o nula.
b) |1-x1x2| = 0. Es un caso de comprobación trivial.
c) |1-x1x2| = x1x2-1.
Razonando igual que en a), se llega a una expresión de la forma x12+x22+x12 x22+2-x1x2 =

 x1 
2
3x
 + x 2  + 1 + x12 x22 + 2 , que también es positiva o nula.
2  4

Volviendo de nuevo a nuestro problema, hemos obtenido que, para todo x1,x2∈R es:
|f(x1)-f(x2)|≤ |x1-x2|.
Entonces, fijado cualquier número real E>0, si tomamos η=E, resulta que si |x1-x2|≤η, será
|f(x1)-f(x2)|≤E, lo que demuestra la continuidad uniforme de la función f.

2/9
www.eltemario.com Oposiciones Secundaria – Matemáticas.
©Antonio Jesús Abrisqueta Valcárcel, 2001. Hoja Problemas – Análisis II

87.- Establecer la igualdad siguiente:

1− x
2·arctg + arcsen ( 2 x − 1) = π / 2
x

Solución:
Si utilizamos la notación:

1− x 1− x
α = arctg (o bien , tg α = )
x x
β = arcsen (2 x − 1) ( o bien , 2 x − 1 = sen β)

se tratará de probar que 2α+β = π/2. Pero obsérvese que esto equivale a:

π  1 − tg 2α
sen β = sen − 2α = cos 2α =
2  1 + tg 2α

y esta igualdad es cierta, según se compruba inmediatamente sustituyendo las expresiones de


tg α y sen β en función de x.

88.- Sea p(x) una función polinómica de grado n con coeficientes reales tal que la
ecuación p(x)=0 admite n raíces distintas x1,...,xn ninguna de las cuales es nula.

i) Expresar en función de estas raíces:


2
p´( x) p´´( x) p´( x)
f(x) = y g(x)= −
p (x ) p( x) p (x )

ii) Calcular las sumas:


n n
S-1= ∑ x −j 1 , S − 2 = ∑ x −j 2 , S = ∑ xi x j
j =1 j =1 i< j

Solución:
1º.- Si an es el coeficiente de xn en p(x), podemos escribir:
p(x) = an(x-x1)(x-x2)...(x-xn)
Derivando:
p´(x) = an[(x-x2)...(x-xn)+(x-x1)...(x-xn)+...+(x-x1)...(x-xn-1)] y dividiendo aquí por
p(x) (puesto que an≠0), se obtiene:

3/9
www.eltemario.com Oposiciones Secundaria – Matemáticas.
©Antonio Jesús Abrisqueta Valcárcel, 2001. Hoja Problemas – Análisis II

n
p´( x ) 1 1 1
= + + ... + para x≠xj, j=1,2,...,n. Luego f(x) = = ∑ ( x − x j ) − 2
p ( x ) x − x1 x − x2 x − xn j =1

2º.- Las sumas S-1 y S-2 son respectivamente los valores de las funciones f y -g para x=0. Así
pues:

 p´(0) 
2
p´(0) p´´(0)
S-1= y S-2=   − .
p (0)  p (0)  p( 0)

Para hallar la suma S observaremos que:


2
 n −1  n n
 ∑ x j  = ∑ xj − 2 + s ∑ x i−1 x −j 1
 j =1 
  j =1 i< j

es decir, S-12 = S-2+2S, de donde:


1 1 p´´(0)
S= ( S −21 − S − 2 ) = .
2 2 p (0)

89.- Se considera el determinante de orden n:


1+ x 1 ... 1
1 1 + x ... 1
... ... .... ...
1 1 ... 1 + x

Establecer la fórmula:
F´n(x)=nFn-1(x)
y deducir que
Fn(x)=xn+nxn-1.

Solución:
Emplearemos la regla obtenida en el problema anterior para calcular la derivada de un
determinante. Observemos, además, que, al derivar la fila o la columna i-ésima se obtiene una
fila o una columna que tiene todos sus elementos nulos salvo el aij que es igual a 1. Es decir:

4/9
www.eltemario.com Oposiciones Secundaria – Matemáticas.
©Antonio Jesús Abrisqueta Valcárcel, 2001. Hoja Problemas – Análisis II

1, 0, ..., 0 1+ x 1 ..., 1 1 + x 1, ..., 1


1, 1 + x ..., 1 0 1 ..., 0 1, 1 + x ..., 1
F´ n ( x ) = + + ... +
... ... ..., ... ... ... ..., ... ... ... ..., ...
1, 1, ..., 1 + x 1, 1, ..., 1 + x 0, 0, ..., 1

Cada uno de los determinantes qe aparecen en esta suma se desarrolla por la fila que
tiene todos los elementos nulos menos uno, y como este elementos se sencuentra situado
siempre en la diagonal principal, su adjunto llevará signo positivo. Este adjunto es,
evidnentemente el determinante Fn-1(x), y entonces:
1 + x, 1, ..., 1 1+ x 1 ..., 1 1 + x 1, ..., 1
1, 1+ x ..., 1 1 1+ x ..., 1 1, 1 + x ..., 1
F´ n ( x ) = + 1· + ... + 1· =
... ... ..., ... ... ... ..., ... ... ... ..., ...
1, 1, ..., 1 + x 1, 1, ..., 1 + x 1, 1, ..., 1 + x

= nFn-1(x).
Estableceremos la fórmula Fn(x) = xn+nxn-1 por inducción. Desde luego, esta fórmula
es cierta para n=2: F2(x)=x2+2x.
En efecto:
1+ x 1
F2(x) = = (1 + x ) 2 − 1 = x 2 + 2 x + 1 − 1 = x 2 + 2 x
1 1+ x

Supongamos que es cierta para n: Fn(x) = xn+nxn-1 y comprobemos que también es


cierta para n+1.
Según la fórmula establecida en la primera parte:
F´n+1(x)= (n+1)Fn(x) = (n+1)(xn+nxn-1) = (n+1)xn+(n+1)nxn-1, es decir, Fn+1(x) es una
función cuya derivada es la expresión anterior. Según la regla del cálculo de derivadas, resulta
inmediato que:
Fn+1(x)= xn+1+(n+1)xn
así que la fórmula también es cierta para n+1, c.q.d.

90.- Se considera la función real f definida por


1 1
f ( x ) = x·sen sen
x 1
sen
x

5/9
www.eltemario.com Oposiciones Secundaria – Matemáticas.
©Antonio Jesús Abrisqueta Valcárcel, 2001. Hoja Problemas – Análisis II

≠ 0 y x ≠ 1/kπ
para x≠ π , con k=±
± 1, ± 2,... completándose su definición en toda la recta
del siguiente modo:
f(0)=0 y f(1/k π )=0 para k=±
± 1,±
± 2,...
∈ R pero carece de
Probar que esta función es continua en todo punto x∈
π ,-1/π
derivada en los puntos del conjunto M={0,1/π π ,1/2π
π ,-1/2π
π ,...}.

Solución:
En un punto cualquiera x∈R-M la función f es continua en virtud del teorema de la
continuidad de las funciones compuestas. La continuidad en el punto x=0 resulta de la
mayoración evidente |f(x)|≤|x|, que no sdice que si x→0 también f(x)→0 = f(0). La
continuidad en un punto de la forma x=1/kπ con k∈Z-{0} resulta de la mayoración
|f(x)|≤|x|·|sen 1/x| ya que limx→1/kπ |sen 1/x| = 0.
Así pues, la función f es contínua en toda la recta real. Su derivabilidad es cualquier
punto x∈R-M se obtiene como consecuencia del teorema de derivabilidad de las funciones
compuestas. Estudiemos la derivabilidad en el punto x=0. Para ello, formaremos el siguiente
cociente incremental

f ( h) − f (0) f (h ) 1 1
= = sen sen
h h h 1
sen
h
donde se observa que si h=1/kπ y hacemos k→∞, el límite que se obtiene es 0, porque la
función f se anula en estos puntos, mientras que si tomamos h=2/(4k+1)π y hacemos k→∞, el
límite que resulta es sen 1. Al no coincidir estos límites, se ve que no existe el limh→0f(h)/h, por
lo cual la función f no es derivable en el punto x=0. Estudiemos por último la derivabilidad en
un punto de la forma x=1/kπ. Para ello formaremos el cociente incremental

 1   1   1 
f + h − f  f + h
 kπ   kπ  =  kπ  = 1 + hkπ sen kπ sen 1
h h hkπ 1 + hkπ kπ
sen
1 + hkπ
para estudiar su límite cuando h→0. Es inmediato que limh→0(1+hkπ)/kπ = 1/kπ. Por otro
lado,

6/9
www.eltemario.com Oposiciones Secundaria – Matemáticas.
©Antonio Jesús Abrisqueta Valcárcel, 2001. Hoja Problemas – Análisis II

limh→0 1 sen kπ = ( −1) k +1 k 2 π 2


h 1 + hkπ
como se deduce de la relación

kπ kπ ( 2 + hkπ) kπ − hk 2 π 2
sen = sen − senkπ = 2 cos sen .
1 + hkπ 1 + hkπ 2 + 2 hkπ 2 + 2hkπ
Ahora bien, no es difícil comprobar que el límite de
1
sen

sen
1 + hkπ
cuando h→0 no existe. Luego no puede existir el límite del cociente incremental que venimos
estudiando, es decir, la función f no es derivable en ningún otro punto de la forma x01/kπ, con
k∈Z-{0}. Así pues, la función dada f es continua en R y derivable en todo punto de R-M,
careciendo de derivada en los puntos del conjunto M.

91.- Dada la parábola de ecuación y= 2 px , respecto de un sistema cartesiano

rectangular XOY de ejes coordenados, hallar las ecuaciones de la tangente y de la


normal en un punto cualquiera P=(x0,y0) así como las longitudes de la tangente, de la
normal, de la subtangente y de la subnormal.

Solución:

2p p
La derivada de y= 2 px vale y´= =
2 2 px y

7/9
www.eltemario.com Oposiciones Secundaria – Matemáticas.
©Antonio Jesús Abrisqueta Valcárcel, 2001. Hoja Problemas – Análisis II

p
La ecuación de la tangente en P es y-y0= ( x − x0 ) , o sea:
y0

px-y0y-px0+y02=0.
La normal es la perpendicular a la tangente en el punto P. Su ecuación será:
y0
y-y0=- ( x − x0 )
p

es decir,
y0x+py-y0x0-py0 = 0.
El punto T de intersección de la tangente con el eje OX se obtiene haciendo y=0 en la
ecuación de la tangente y despejando x; resulta:
px0 − y 02 y2
xT = = x0 − 0
p p

El punto N de intersección de la normal con el eje OX se obtiene haciendo y=0 en la


ecuación de la normal y despejando x; resulta:
y 0 x0 + py0
xN= = x0 + p
y0

La subtangente es, por definición, la longitudf del segmento HT (donde H=(x0,0) es la


proyección ortogonal de P sobre el eje OX). Valdrá pues, St=HT=xT -x0=-y02/p =-2x0. La
subnormal es, por definición, la longitud del segmento HN. Valdrá, pues, Sn= HN = xN-x0=p.
Observamos entonces que en una parábola la subnormal es constante (independiente del
punto P desde el cual se considera); puede demostrarse con facilidad que esta propiedad
caracteriza a las parábolas. La longitud de la tangente es, por definición, la del segmento PT.

Se calcula por el teorema de Pitágoras PT = = y02 + S t2 = y 02 + 4x 02 . Análogamente, la

longitud de la normal es PN = = y02 + S N2 = y 02 + p 2 .

92.- Supongamos que g es una función derivable en R con derivada acotada:


≤ M. Fijado E>0, consideremos la función f(x)=x+Eg(x). Demostrar que f es
|g´(x)|≤
inyectiva si E es suficientemente pequeño. Determinar un conjunto de valores
admisibles de E dependiente sólo de M.

Solución:

8/9
www.eltemario.com Oposiciones Secundaria – Matemáticas.
©Antonio Jesús Abrisqueta Valcárcel, 2001. Hoja Problemas – Análisis II

La función f es derivable y por lo tanto continua. Así que, para ser inyectiva debe ser
estrictamente creciente o estrictamente decreciente en R, es decir, debe ser f´(x)>0 o bien
f´(x)<0 para todo x∈R.
f´(x) = 1+Eg´(x)
La condición f´(x)<0 es incompatible con la condición |g´(x)| ≤ M. (haga el lector los
cálculos). Así pues, exijamos que f´(x)>0 para todo x:
f´(x) = 1+Eg´(x)>0
equivale a:
g´(x)>-1/E para todo x
Por otra parte, |g´(x)|≤M equivale a
-M≤g´(x)≤M
así que basta con fijar E de forma que -M>-1/E, o bien M<1/E, que proporciona para E el
conjunto de valores: E<1/M.

9/9
www.eltemario.com Oposiciones Secundaria – Matemáticas
© Antonio J. Abrisqueta Valcárcel, 2001 Hoja de Problemas – Análisis III

93. Demostrar que la ecuación

x+ k −a
e 2x =
− x+ k +a

donde | k | < 1, tiene una raíz y sólo una. Determinar una sucesión convergente
hacía esta raíz.
______________________________________________________________________

Solución:

x+ k −a
La ecuación e 2 x = equivale a:
− x+ k +a

a − k + e 2 x (k + a )
x=
1 + e 2x

a − k + e 2 x (k + a )
Sea f ( x ) =
1 + e2 x

e2 x
f ´( x ) = 4 k
(1 + e 2 x ) 2

e2x
Estudiemos g ( x) =
(1 + e 2 x ) 2

2e 2 x − 2e 6 x
g´( x ) =
(1 + e 2 x ) 4

1
en x = 0 tiene un máximo y g ( 0) = . Por lo tanto:
4

2x
e 1
| f ´( x ) |= 4 | k | < 4 | k | = | k | < 1,
(1 + e )
2x 2
4

cualquiera que sea x ∈ ℜ .


Aplicando el método de iteración, puede asegurarse la existencia de una única raíz
de la ecuación f (x) = x. Se construye una sucesión ( x n ) convergente hacia dicha raíz

eligiendo un punto x 0 ∈ ℜ arbitrario y tomando después:


x n +1 = f ( x).

1/13
www.eltemario.com Oposiciones Secundaria – Matemáticas
© Antonio J. Abrisqueta Valcárcel, 2001 Hoja de Problemas – Análisis III

94. Demostrar que la ecuación x = c sen x + k, donde 0 < c < 1, tiene una raíz y sólo
una. Partiendo de un valor cualquiera x 0 ∈ ℜ , se construye la sucesión ( x n )
mediante la fórmula

x n = c sen x n −1 + k n =1,2,…

Probar que esta sucesión converge hacía la raíz.


______________________________________________________________________

Solución:

Veamos en primer lugar que la sucesión ( x n ) es convergente. Para ello, tomemos


x n = c sen x + k :

x k +1 − x k = f ( x k ) − f ( xk −1 ) f ´(ξk ), ξk ∈ ]x k −1 , xk [

tomando valores absolutos:

| x k +1 − x k |=| x k − x k −1 | | c cos ξk | ≤ c | x k − x k −1 |

Sea

d= | x1 − x0 |= |c sen x0 + k − x0 |

Entonces:

| x 2 − x1 |≤ c | x1 − x0 |= cd
| x3 − x2 |≤ c | x2 − x1 |≤ c 2 d
…………………………….
| x k − x k −1 |≤ c k −1 d

Con ayuda de esta desigualdad, comprobaremos que la sucesión ( xn ) satisface el


criterio de Cauchy:

| x m − x n |≤ x m − x m −1 + xm −1 − x m − 2 + ... + xn +1 − xn ≤ c m −1 d + c m −2 d + ... + c n d =
(m > n )
c m −1 c − c n cm − cn cn − cm cn
=d =d =d <d
c −1 c −1 1− c 1− c

y puesto que 0 < c < 1, es evidente que | x m − x n |≤ ε siempre que sea m, n ≥ v.


Así pues, la sucesión converge. Llamaremos a = lim n → ∞ xn .

A continuación, comprobemos que a es una raíz de la ecuación x = c sen x + k:

2/13
www.eltemario.com Oposiciones Secundaria – Matemáticas
© Antonio J. Abrisqueta Valcárcel, 2001 Hoja de Problemas – Análisis III

Como f es continua, se verifica que lim n →∞ f ( x n ) = f ( a) . Pero


lim n → ∞ f ( xn ) = lim n →∞ f ( x n +1 ) = a , así que resulta a = f (a), que es lo que queríamos
probar.
Finalmente, demostraremos que la raíz de la ecuación es única:
Si b fuese otra raíz, tendríamos que f (b) = b, así que | b – a | =| f(b) - f(a) | =
f ´(ξ)(b − a ) = f ´(ξ) || b − a |< c | b − a |<| b − a | , lo cual es una contradicción.

95. Sea f una función real convexa definida en un intervalo I de la recta real. Sean
x1 ,..., x p puntos de I y λ1 ,...λp puntos del intervalo ]0,1[ tales que
p

∑ λ = 1.
i
i =1
p
Se pide: 1.º) Probar que ∑λ x ∈ I i i
i =1
p p
2.º) Demostrar que f (∑ λi xi ) ≤ ∑ λ f (x ) i i
i =1 i =1
p p
3.º) Establecer la desigualdad log ( ∑ λi xi ) ≥ ∑ λi log xi
i =1 i =1

______________________________________________________________________

Solución:

1.º) Sean α y β el menor y el mayor de los números x1 ,..., x p respectivamente. Es claro


que

p p p
α = α∑ λi ≤ ∑ λi xi ≤ β∑ λi = β
i =1 i =1 i =1
p
El punto ∑ λ x , por estar comprendido entre α y β pertenecerá al Intervalo I.
i i
i =1
2.º) Para p = 2 la desigualdad es cierta por definición de función convexa. Supongamos
p > 2 y razonemos por inducción. Admitiendo que la desigualdad se verifica para
p = k –1 probémosla para p = k. Se tiene

k −1 k −1 k −1
α∑ λi ≤ ∑ λi xi ≤ β∑ λi
i =1 i =1 i =1

1 k −1
de donde se deduce que el punto x = ∑ λi xi pertenece al intervalo I, siendo
λ i =1
k −1 k −1
λ = ∑ λi . Como ∑ λ / λ = 1 , la hipótesis de inducción nos permite escribir
i
i =1 i =1

k −1 k −1 k −1
λf ( x) = λf ( ∑ ( λi / λ) x i ) ≤ λ∑ (λi / λ) f ( xi ) = ∑ λi f ( xi )
i =1 i =1 i =1

Poniendo

3/13
www.eltemario.com Oposiciones Secundaria – Matemáticas
© Antonio J. Abrisqueta Valcárcel, 2001 Hoja de Problemas – Análisis III

k k −1

∑λ x = ∑λx i i i i + λk x k = λx + λk x k
i =1 i =1

y observando que λ + λk = 1 tendremos, por ser f una función convexa, que

k
f (∑ λi x i ) = f ( λx + λk xk ) ≤ λf ( x ) + λk f ( x k )
i =1

Utilizando un resultado obtenido más arriba concluimos que

k k −1 k
f (∑ λi xi ) ≤ ∑ λi f ( xi ) + λk f ( x k ) = ∑ λi f ( xi )
i =1 i =1 i =1

quedando la propiedad demostrada para p = k y con ello establecida en general.


3.º) Aplicando el resultado anterior a la función f(x)=- log x que es convexa
tendremos

p p
− log ∑ λi xi ≤ ∑ λi ( − log xi )
i =1 i =1

de donde resulta la desigualdad a demostrar multiplicando por –1.

96. Escribir el desarrollo de Taylor de la función f ( x ) = (log x ) 2 en el punto x0 = 1


hasta el lugar de las derivadas de orden 4, con diversas fórmulas del término
complementario.

Solución:

La fórmula de Taylor hasta las derivadas de orden 4 es

f ´( x o ) f ´´(x0 ) f ´´´(x 0 )
f(x)= f ( x0 ) + (x − x 0 ) + ( x − x0 ) 2 + ( x − x 0 ) 3 + T4 ( x0 )
1! 2! 3!

En donde T4 ( x0 ) representa el término complementario.


Calculemos las derivadas que aparecen:
2
f(x)=(log x)

1 2
f ´( x ) = 2 log x = log x
x x

2
f ´´( x) = (1 − log x )
x2

4/13
www.eltemario.com Oposiciones Secundaria – Matemáticas
© Antonio J. Abrisqueta Valcárcel, 2001 Hoja de Problemas – Análisis III

4 2 1 2
f ´´´(x ) = − 3
(1 − log x) + 2 ( − ) = − 3 ( 3 − 2 log x)
x x x x

2
f ( 4) ( x ) = (11 − 6 log x)
x4

En el punto x0 = 1 , se tiene

f (1) = 0, f ´(1) = 0, f ´´(1) = 2, f ´´´(1) = −6

Por lo tanto:

0 2 −6
(log x) 2 = 0 + ( x − 1) + ( x − 1) 2 + ( x −1) 3 + T4 (1) = ( x − 1) 2 − ( x − 1) 3 + T4 (1)
1! 2! 3!

Daremos dos formas distintas del término complementario:

a) Término complementario de Lagrange:

( x − 1)( x − c´) 3 ( 4 ) ( x − 1) 4 2
T4 (1) = f ( c) = (11 − 6 log c ),1 < c < x
3! 4! c 4

b) Término complementario de Cauchy:

( x − 1)( x − c´) 3 ( 4 ) ( x − 1)( x − c´) 3 2


T4 (1) = f ( c´) = (11 − 6 log c´)
3! 3! ( c´) 4

97. Escribir el desarrollo de MacLaurin, de la función f ( x ) = e x hasta el lugar de


las derivadas de orden 10.
______________________________________________________________________

Solución:

El desarrollo de MacLaurin de f será:

f ´(0) f ´´(0) 2 f (9 ) (0) 9 f (10 ) (ξ) 10


f ( x ) = f ( 0) + x+ x + ... + x + x
1! 2! 9! 10!

en donde ξ ∈ ]0, x [ .
Es necesario calcular las derivadas sucesivas de f. Sea f ( x ) = e x = e P ( x ) con
5

P( x) = x 5 .
Las derivadas sucesivas de P(x) son:

P( x) = x 5
P´( x) = 5 x 4

5/13
www.eltemario.com Oposiciones Secundaria – Matemáticas
© Antonio J. Abrisqueta Valcárcel, 2001 Hoja de Problemas – Análisis III

P´´( x) = 5 ⋅ 4 ⋅ x 3
P´´´(x) = 5 ⋅ 4 ⋅ 3 x 2
P ( 4 ) ( x ) = 5! x
P (5 ) ( x ) = 5!
P ( n ) ( x) = 0 n>5

Resulta entonces que la única derivada de P que no se anula para x = 0 es P (5 ) ( x ) = 5! .


Veamos ahora una manera de calcular las derivadas de f, teniendo en cuenta que en la
fórmula de MacLaurin que vamos a emplear, dichas derivadas hasta el orden 9 están
particularizadas para x = 0:

f (x ) = e P ( x ) f (0) = e 0 = 1
f ´( x ) = e P ( x ) P´( x) = f ( x ) P´( x) f ´(0)=0

las derivadas siguientes de f pueden calcularse aplicando la fórmula de Leibnitz sobre la


derivada n-sima de un producto f ⋅ P´.
Para simplificar las notaciones, llamaremos P´ = g. Entonces g ( 4 ) ( x) = p (5 ) ( x) , es
decir, la única derivada de g que no se anula en x=0 es g ( 4 ) (0) = 5!

n −1 n − 1
  (k )
f ( n ) ( x ) = ∑   f ( x) g (n −1− k ) ( x )
k=0  k 

puesto que:

f ´´( x) = ( f ( x) g ( x))´= f ´( x) g ( x) + f ( x) g´( x)


2  2  2
f ´´´(x ) = ( f ( x ) g ( x ))´´=   f ( x) g´´(x ) +   f ´( x) g´( x) +   f ´´( x ) g ( x)
0 1   2
resulta entonces que:

f´(0)=f´´(0)=f´´´(0)=f ( 4 ) (0) = 0

ya que en estas derivadas no aparece en ningún término g ( 4 ) (0) .

 4  4  4
f ( 5 ) ( x) = ( f ( x) g ( x )) ( 4 ) =   f ( x ) g ( 4 ) ( x) +   f ´( x ) g´´´(x ) +   f ´´(x ) g´´( x)
 0 1   2
 4  4
+   f ´´´(x ) g´( x ) +   f ( 4 ) ( x) g ( x )
3   4

expresión en la que se anulan todos los sumandos en X = 0 salvo el primero. Por tanto:

 4
f 5 (0) =   f (0) g ( 4 ) ( 0) = 5!
 0

6/13
www.eltemario.com Oposiciones Secundaria – Matemáticas
© Antonio J. Abrisqueta Valcárcel, 2001 Hoja de Problemas – Análisis III

Obsérvese que, hasta el orden 5, se anulan todas las derivadas de f en x = 0. Por lo tanto
en las derivadas siguientes serán nulos en x = 0 todos los productos en los que figura
alguna derivada de f de orden menor que el quinto, o bien alguna derivada de g de orden
distinto del cuarto.

5
5 
f ( 6 ) ( x) = ∑   f ( k ) ( x ) g ( 5− x ) ( x) , f ( 6 ) ( 0) = 0
k =0  k 

6
6
f ( 7 ) ( x ) = ∑   f (k ) ( x) g (6 − x ) ( x) , f ( 7 ) (0) = 0
k =0  k 

7
7
f ( 8 ) ( x) = ∑   f (k ) ( x) g (7 − x ) ( x) , f ( 8 ) (0) = 0
k=0  k 

8
8 
f ( 9 ) ( x) = ∑   f ( k ) ( x) g (8 − x ) ( x) , f ( 9 ) ( 0) = 0
k =0  k 

9
9 
f (10 ) ( x) = ∑   f ( k ) ( x) g (9 − x ) ( x)
k=0  k 

teniendo en cuenta que P ( 6 ) ( x ) = P (7 ) ( x ) = ... = 0 para todo x, en la expresión anterior se


anularán todos los términos en los que aparezca una derivada de g de orden mayor o
igual que 5.
Por lo tanto:

9
9 
f (10 ) ( x) = ∑   f ( k ) ( x) g (9 − x ) ( x)
k=0  k 

Sustituyendo en la fórmula de MacLaurin, resulta finalmente

9  (k) x10
ξ ∈ ]0, x [
9
5! 5
e = 1 + x + ( ∑   f (ξ) g (ξ))
x5 9− k )
,
5! k =5  k  10!

98.Obtener el desarrollo de MacLaurin, de las siguientes funciones

f(x) = log(1+x), f(x)=(1+x) α

con el término complementario de Cauchy.

1) La derivada n-sima de la función log (1+ x) se obtiene sin dificultad calculando


las primeras y aplicando el método de inducción. Resulta ser
f ( n ) ( x ) = ( −1) n −1 ( n −1)!(1 + x ) − n para n = 1,2,... Sus valores en el punto x = 0 son
f ( n ) (0) = ( −1) n −1 (n − 1)! . Por consiguiente el desarrollo de MacLaurin de esta
función con el término complementario de Cauchy será:

7/13
www.eltemario.com Oposiciones Secundaria – Matemáticas
© Antonio J. Abrisqueta Valcárcel, 2001 Hoja de Problemas – Análisis III

1 1 1 n −1 1−θ n 1 n
log( 1 + x ) = x − x 2 + x 3 − ... + ( −1) n x + ( −1) n +1 ( ) x
2 3 n −1 1 + θx 1 − θ

donde θ es cierto número del intervalo ]0,1[.

2) La derivada n-sima de la función binómica (1 + x )α , donde α es un numero real


cualquiera, se obtiene fácilmente como en el caso anterior, resultando

f ( n ) ( x ) = α(α − 1)(α − 2)...(α − n + 1)(1 + x )α − n

para n=1,2,... Sus valores en el punto x = = son

f ( n ) (0) = α(α − 1)(α − 2)...(α − n + 1)

Ponemos por definición

α α(α −1)...(α − n + 1)


  =
n  n!

generalizando así la notación de los números combinatorios ordinarios. El


desarrollo de MacLaurin de nuestra función, con el término complementario de
Cauchy es:

 α  α  α  n −1 α
(1 + x) α = 1 +   x +   x 2 + ... +   x +   n(1 − θ ) n −1 (1 + θx) α − n x n
1   2   n − 1 n 

donde θ es cierto número del intervalo ]0,1[.

99.Haciendo uso de la fórmula de Taylor para la función (1+x) 3 calcular


1
3
aproximadamente (1,03) . Situando el término complementario en el lugar de
las derivadas terceras, estimar el error cometido.

Solución:

Supondremos conocido el desarrollo de Taylor de la función (1 + x) r . Tomando en


1
dicho desarrollo r = , resulta:
3

1 1 1
( − 1)
f m (ξ) 3
1

(1 + x) = 1 + x + 3
3 3 3 x2 + x
1! 2! 3!

con 0 < ξ < x, y también

8/13
www.eltemario.com Oposiciones Secundaria – Matemáticas
© Antonio J. Abrisqueta Valcárcel, 2001 Hoja de Problemas – Análisis III

−8 −8
1 2 −5
1
1 1 1 −3 10
f m ( x) = ( −1)( − 2)(1 + x) 3 = (− )( )(1 + x) 3 = 3 (1 + x) 3
3 3 3 3 3 3 3

Tomando en el desarrollo de Taylor x = 0,3

1 1 −2
1
1 1
(1,03) 3 = 1 + 0,3 + ( )( 0,3) 2 + T2 = 1 + 0,1 − 0,09 + T3 = 1,1 − 0,01 + T3
3 2! 3 3 9

Por lo tanto:

(1,03) ≅ 1,09
3

El error cometido es precisamente el término complementario:

f ´´´(ξ) 1
T3 = (0,3) 3 = 3310− 3 f ´´´(ξ) con 0 <ξ <0,3
3! 3!

Puesto que:

−8
10 10
f ´´´(ξ) = 3
(1 + ξ) 3 ≤ 3
3 3

Obtenemos, como cota del error cometido:

1 3 10 5
E≤ 3 ⋅10 −3 ⋅ 3 = ⋅10 −3 ≤ 2 ⋅10 −3
3! 3 3

100. Determinar las asíntotas y ramas parabólicas de la curva:

y = cos 2x + 2 sen x

Solución:

1) El campo de existencia de esta curva es todo ℜ , y además es periódica de


período 2π ya que:

cos 2 (x+2π) +2 sen (x + 2π)=cos 2x +2 sen x

Por lo tanto, nos limitaremos a representar gráficamente la función en el intervalo


[-π,π].

2) Cortes con los ejes:


Para x = 0 es y = 1, así que pasa por (0,1).
Para y = 0, resulta la ecuación cos 2x + 2 sen x = 0 o bien:

9/13
www.eltemario.com Oposiciones Secundaria – Matemáticas
© Antonio J. Abrisqueta Valcárcel, 2001 Hoja de Problemas – Análisis III

cos 2 x − sen 2 x + 2senx = 0


1 − 2 sen 2 x + 2 senx = 0

que es una ecuación de segundo grado en sen x cuyas raíces son:

2 ± 12 1 ± 3
sen x= =
4 2

1+ 3 1+ 3
la solución sen x = no nos sirve porque >1 así pues, resulta sen
2 2
1− 3 1− 3
x= , que proporciona dos valores de x en [-π,π], x = arcsen .
2 2

3) Es sencillo comprobar que esta curva no tiene asíntotas.


4) Máximos, mínimos y puntos de inflexión:

y ´=-2 sen 2x +2 cos x(-2 sen x +1)

Calculemos los valores de x que anulan y´: y´ = 0 proporciona la ecuación: 2cos x (1


–2 sen x) = 0 es decir:
cos x = 0

o bien

1
sen x=
2

π π π 5π
cuyas raíces son, en el intervalo [-π,π], x1 = , x2 = − , x3 = , x4 = .
2 2 6 6
Hemos de estudiar ahora el signo de y´´:
y ´´=-4 cos 2x – 2 sen x = 8 sen 2 x − 2 senx − 4

π
y ´´( x1 ) = 2 > 0 así que x1 = corresponde a un mínimo
2

y´´ ( x2 ) = 6 > 0 mínimo


y´´ ( x3 ) = −3 < 0 máximo
y´´ ( x4 ) = −3 < 0 máximo

π −π
Por lo tanto, hay mínimo relativo en los puntos ( ,1)( ,−3) y un máximo relativo en
2 2
π 3 5π 3
los puntos ( , )( , ) .
6 2 2 2

Los posibles puntos de inflexión son los valores de x que anulan la derivada segunda:
y´´ = 0 proporciona la ecuación

10/13
www.eltemario.com Oposiciones Secundaria – Matemáticas
© Antonio J. Abrisqueta Valcárcel, 2001 Hoja de Problemas – Análisis III

8sen 2 x − 2 senx − 4 = 0
cuyas raíces son:

1± 33
sen x =
8

que corresponden a cuatro valores de x.

π
2
101. Calcular ∫ senxdx por el método de las sumas de Riemann, dividiendo el
0

π
intervalo [ 0,  en partes iguales.
 2

Solución:

Si dividimos el intervalo en n partes iguales tendremos que:

π
2 n
π jπ
∫ senxdx = lim ∑ 2 n sen (0 + 2n )
n→ ∞
j =1
(1)
0

π
Si llamamos h = , calculemos la suma
2n

∑ senjh = senh + sen 2h + ... + sennh


j =1

Para ello, emplearemos la igualdad:

2k + 1 2k − 1 h
cos h − cos h = −2 senkh⋅ sen
2 2 2

Dando a k los valores k = 1,...,n, se obtiene:

3 1 h
cos h − cos h = −2senh ⋅ sen
2 2 2
5 ( 2n − 1) h
cos h − cos h = −2sen 2h ⋅ sen
2 2 2
....................................................................
( 2n + 1) ( 2n − 1) h
cos h − cos h = −2sen (nh ) ⋅ sen
2 2 2

y sumando miembro a miembro estas igualdades:

11/13
www.eltemario.com Oposiciones Secundaria – Matemáticas
© Antonio J. Abrisqueta Valcárcel, 2001 Hoja de Problemas – Análisis III

( 2n + 1) 1 h
cos h − cos h = −2sen ( senh + sen2 h + ... + sen( nh))
2 2 2

de donde:

h 2n + 1
cos − cos π
sen h + sen 2h + ... + sen nh= 2 4 n
h
2sen
2

Sustituyendo este resultado en (1):

π 2n + 1
π cos − cos π
π
∫ 2
senxdx = lim 2 4n =
0 n →∞ 2n π
2sen
2

π 2n + 1
cos − cos π
π 4n 4n π 2n + 1
= lim = lim (cos − cos π) = 1.
n →∞ 2n π n → ∞ 4 n 4 n
2
4n

102. Estudiar el cálculo de los siguientes límites, por el método de las sumas de
Riemann

n
1
1.º) lim
n →∞
∑ 2n + j
j =1
n
j(n − j)
2.º) lim ∑
n→ ∞
j =1 n2

Solución:

1.º)
n n
dx = [log( 2 + x ) ]0 = log 3 − log 2 = log
1 1 1 1 3
∑ ∑ =∫
1
= ⋅
1
lim lim
j =1 2n + j j =1 n 2 + j / n
n →∞ n →∞ 0 2+ x 2

2.º)

n
j(n − j) 1  n n
 1  ( n + 1) n n( n + 1)( 2 n + 1) 
lim ∑ = lim ⋅  ∑ jn − ∑ j 2  = lim 2 n −  =
 n→ ∞ n 
n→ ∞ 2 n →∞ n 2
j =1 n  j =1 j =1 2 6
1  n 2n + 1 m( n + 1)( n − 1) 2
= lim 2 n( n + 1)  −  = lim 6n = +∞
n →∞ n
2 6  n →∞

12/13
www.eltemario.com Oposiciones Secundaria – Matemáticas
© Antonio J. Abrisqueta Valcárcel, 2001 Hoja de Problemas – Análisis III

Obsérvese que, en el cálculo de este límite no puede emplearse el método de las sumas
n
j(n − j)
de Riemann porque ∑ no corresponde a la suma de Riemann de ninguna
j =1 n2
integral propia.

13/13
www.eltemario.com Oposiciones Secundaria – Matemáticas
© Antonio J. Abrisqueta Valcárcel, 2001 Hoja de Problemas – Análisis IV

x3 − x2 + 2x
103. ∫ x 4 + x 2 + 1 dx
Sol.

Las raíces de la ecuación x 4 + x 2 + 1 son


1 3 1 3 1 3 1 3
x1 = + i , x2 = − i , x3 = − + i , x4 = − − i , así que el
2 2 2 2 2 2 2 2
polinomio x 4 + x 2 + 1 se descompone de la forma:

 1 3    1 3    1 3     1 3 


x 4 + x 2 + 1 =  x −  + i   x − − i   x − − + i   x − − − i =
  2 2    2 2    2 2     2 2 
 1
2
3   1
2
3
=  x −  +   x +  + 
 2 4   2 4 

Descomponemos en fracciones simples:


x3 − x2 + 2x Ax + B Cx + D
= +
x + x +1 
4 2
1
2
3  1 3
2

x−  + x+  +
 2 4  2 4
identificando coeficientes, se obtienen los valores:

A = 0, B = ½, C = 1, D = -½,

Por lo que:
x3 − x2 + 2x 1 dx x −1/ 2
∫ x 4 + x 2 + 1 dx = 2 ∫  1  2 3 + ∫  1  2 3 dx (1)
x −  + x+  +
 2 4  2 4

Calculemos por separado estas integrales:


1 3 3
a) Hacemos el cambio de variable x − = u de donde, dx = du :
2 2 2
dx 1 3 2 1 2 2 2  1
∫ 1 3
2
=∫
3 2 3 2
dt = ∫
3 t +1
2
dt =
3
arctan t + C =
3
arctan x −  + C
3 2
x −  + t +
 2 4 4 4

1 3 3
b) Hacemos el cambio de variable x + = u de donde, dx = du :
2 2 2

1/11
www.eltemario.com Oposiciones Secundaria – Matemáticas
© Antonio J. Abrisqueta Valcárcel, 2001 Hoja de Problemas – Análisis IV

3 3
u −1 u −1
x −1/ 2 3 2 2  3 u 1 
∫ dx = ∫ ∫ ∫ du − ∫ 2
2 du = 2 du = du  =

1
2
3 3 2 3 2 3 u +1
2
3  4 u +1
2
u +1 
x +  + u +
 2 4 4 4

2  3  1 4  1
2
 2 2  1
=  log( u + 1) − arctan u  + C = log   x +
2
 + 1 − arctan x + +C
3 4  2  3  2  3 3 2

Sustituyendo ambos resultados en (1):

x3 − x2 + 2x 2 2  1 1 4  1
2
 2 2  1
∫ x 4 + x2 + 1 dx = arctan 
3
x −  +
2 2
log   x +  + 1− arctan x + +C
3  3  2  3 3 2

2x2 − x − 2
104. ∫ dx
( x − 3)( x 2 + x + 1)

Sol.

Descomponemos en fracciones simples:

2x2 − x − 2 A Bx + C
= +
( x − 3)( x + x + 1) x − 3 
2
1
2
3
 x +  +
 2 4
identificando coeficientes, resulta A = 1, B = 1, C = 1
(1)

2x2 − x − 2 1 x +1 x +1
∫ (x − 3)( x 2 + x + 1)dx = ∫ x − 3 dx + ∫  1  2 3 dx = log( x − 3) + ∫  1  2 3 dx en
x +  + x +  +
 2 4  2 4
1 3 3
la última integral hacemos el cambio de variable x + = t , dx = dt :
2 2 2
3 1
t+
x +1 3 1  t 1 
∫  1  2 3 dx = ∫ 32 2 32 2 dt = 3  3 ∫ t 2 + 1 dt + ∫ t 2 + 1 dt  =
x +  + t +
 2  4 4 4

1  3  1 4 1
2
 1 2  1
=  log( t 2
+ 1) + arctan t  + C = log   x +  + 1 + arctan x+ +C
3 2  2  3  2  3 3 2

Sustituyendo en (1):

2/11
www.eltemario.com Oposiciones Secundaria – Matemáticas
© Antonio J. Abrisqueta Valcárcel, 2001 Hoja de Problemas – Análisis IV

2x2 − x − 2 1 4  1
2
 1 2  1
∫ (x − 3)( x 2 + x + 1)dx = log( x − 3) + 2 log  3  x + 2  + 1 + 3 arctan 3  x + 2  + C
 

x
105. ∫ (x − 1) 2
( x 2 + x + 1)
dx

Sol.

x d  Ax 2 + Bx + C  D Ex + F
=   + + 2
( x − 1) ( x + x + 1) dx  ( x − 1) ( x + x + 1)  x − 1 x + x + 1
2 2 2 2

operando resulta:
A = -1/3, B = 0, C = 0, D = -1/9, E = 1/9, F = -1/9
1 1 1
− x2 x−
x 1 1
∫ (x − 1) 2 ( x 2 + x + 1)dx = (x − 1)2 ( x 2 + x + 1) − 9 ∫ x − 1 dx + ∫ x 2 + x + 1dx =
3 9 9

1
− x2
1 1 x −1
= 3 − log( x − 1) + ∫ 2 dx
( x − 1) ( x + x + 1) 9
2 2
9 x + x +1
(1)
resolvamos esta última integral: la ecuació n x 2 + x + 1 = 0 tiene las raíces
1 3 1 3
x1 = − + i , x2 = − − i ,
2 2 2 2
así que el polinomio x 2 + x + 1 se descompone de la forma:
  1 3    1 3   
2

 =  x +  + 3
1
x + x + 1 =  x −  − + i
2
  x −  − − i
  2   2
  2 2    2 4
de donde:
x −1 x −1
∫ x 2 + x + 1 dx = ∫  1  2 3 dx
x +  +
 2 4

1 3 3
Hacemos el cambio de variable x + = t , dx = dt :
2 2 2

3/11
www.eltemario.com Oposiciones Secundaria – Matemáticas
© Antonio J. Abrisqueta Valcárcel, 2001 Hoja de Problemas – Análisis IV

3 1 3 3
t − −1 3 t−
x −1 2 2 2 2 2 dt = 2 3 1 log(t 2 + 1) − 2 3 arctant + C =
∫ 1 3
2 dx = ∫ 3 2 3 2
dt =
3 ∫ t + 1 3 2 2 32
x +  + t +
 2 4 4 4

1 4  1
2
 2  1
= log   x +  + 1 − 3 arctan x + +C
2  3  2  3 2

Sustituyendo en (1):
1
− x2
x 1
∫ (x − 1) 2 ( x 2 + x + 1)dx = (x − 1)2 (3x 2 + x + 1) − 9 log( x − 1) +
1 4  1
2
 3 2  1
+ log   x +  + 1 − arctan x + +C
18  3  2  9 3 2

x 3 / 2 (1 − x) −3 / 2
106. ∫ x + x 1/ 2 (1 − x )−1/ 2 dx
Sol.

3/ 2
 x 
 
x (1 − x)
3/2 −3 / 2
 1− x  x
∫ x + x 1 / 2 (1 − x) −1 / 2 dx = ∫  x  dx . Hacemos el cambio 1 − x = t ,
2

x+ 
 1− x 
t2 2t
del cual se obtiene x = y dx = dt. Por lo tanto:
1+ t 2
( 1+ t2) 2

x (1 − x)
3/2 −3 / 2
t 3
2t 2t 4
dx = ∫ 2 dt = ∫ dt =
x + x (1 − x )
1/ 2 −1 / 2
t
+ t (
1+ t2 )2
(1 + t 2 )(t 2 + t 3 + t )
∫ 1+ t2
t3
= 2∫ 2 dt
(t + t + 1)(1 + t 2 )

esta última integral se resuelve descomponiendo en fracciones simples:

t3 At + B Ct + D
= 2 + 2
(t + t + 1)(1 + t ) t + t + 1 t + 1
2 2

4/11
www.eltemario.com Oposiciones Secundaria – Matemáticas
© Antonio J. Abrisqueta Valcárcel, 2001 Hoja de Problemas – Análisis IV

Operando se obtienen los valores:

A = 1, B = 1, C = 0, D = -1

Así pues,

t3 t +1 1 t +1
∫ (t 2 + t + 1)(1 + t 2 ) dt = ∫ t 2 + t + 1 dt − ∫ t 2 + 1 dt = ∫  1  2 3 dt − arctan t
t +  +
 2 4
t +1
La integral ∫ 2
dt se resuelve con ayuda del cambio de
t + 1  + 3
 
 2 4
variable
1 3
t+ = s:
2 2
3 1
s+
t +1 2 3 ds = 1 3s + 1 1 1
∫ ∫ ∫
dt = 2 ds = = log( s 2 + 1) + arctan s + C =
1
2
3 3 2 3 2 3 s +1
2
2 3
t +  + s +
 2 4 4 4

1  4  1 2  1 2  1
= log   t +  + 1 + arctan t +  + C
2  3  2   3 3 2

Finalmente, resulta:

x 3 / 2 (1 − x) − 3 / 2 4  x 1 
2
 2 2  x 1 x
∫ x + x (1 − x)
1/ 2 −1 / 2
dx = log  
 +
 3  1 − x 2   + 1+
 3
arctan  +  − 2 arctan
3  1− x 2  1− x
+C

107. Sea f : [0, π / 2] → 3 una función reglada y λ = lim f ( x) . Demostrar que


x→ π / 2 , x < π / 2
π /2
lim ∫
n →+∞ 0
n cos x sin n xf ( x ) dx = λ

Sol.

Mediante un sencillo cálculo directo observamos que


π /2 n
λ∫ n cos x sin n xdx = λ.
0 n +1

5/11
www.eltemario.com Oposiciones Secundaria – Matemáticas
© Antonio J. Abrisqueta Valcárcel, 2001 Hoja de Problemas – Análisis IV

π/2
Ahora estudiaremos la diferencia I = ∫ n cos x sin n xf ( x ) dx − ( n / n + 1)λ . Por
0
definición del número λ , sabemos que fijado arbitrariamente un número ε > 0,
existe otro número ηcon 0 < η < π / 2 tal que f (x ) − λ ≤ ε siempre que sea
π / 2 − η ≤ x ≤ π / 2 . Entonces descomponemos I en la forma I1 + I2 donde

n cos x sin n x[ f ( x) − λ]dx


π / 2 −η
I1 = ∫
0

n cos x sin n x[ f ( x ) − λ]dx


π /2
I2 = ∫
π / 2 −η

Como la función f (x ) − λ está acotada en [0, π / 2] por ser reglada, podremos


escribir
π
sin n +1  − η
π / 2 −η n
I1 ≤ M ∫ n cos x sin n xdx = M
0 n +1 2 

donde M es una constante, y de aquí se deduce que lim I 1 = 0 . Así que existe un
n →+∞

número natural ν tal que I 1 ≤ ε siempre que sea n ≥ ν . Igualmente se prueba sin
dificultad que
π /2
I 2 ≤ ε ∫π / 2 −nη cos x sin n xdx ≤ ε

para todo n. Entonces lim I 2 = 0 , de donde se deduce lo que queríamos demostrar.


n →+∞

108. Sea I un intervalo compacto de la recta real. A cada intervalo abierto o cerrado
(a, b) ⊂ I y a cada función reglada f definida en él asociémosles un número I(a, b; f) de
modo que se verifiquen las cinco condiciones siguientes:

1) I ( a, b; f + g ) = I (a , b; f ) + I ( a, b; g ) cualesquiera que sean las funciones


f y g.
2) I ( a, b; kf ) = kI (a , b; f ) cualesquiera que sean el número k y la función f.
3) I ( a, b;1) = b − a , donde 1 denota la función constante igual a 1 en (a, b).
4) I ( a, b; f ) = I ( a, c; f ) + I (c, b; f ) , cualquiera que sea el punto c ∈ (a, b) .
5) I ( a, b; f ≤ k sup f ( x) donde k es una constante que depende del
a ≤ x ≤b
intervalo
(a, b) y de la función f.

Demostrar que I ( a, b; f ) = ∫ f
b

Sol.

6/11
www.eltemario.com Oposiciones Secundaria – Matemáticas
© Antonio J. Abrisqueta Valcárcel, 2001 Hoja de Problemas – Análisis IV

Supongamos primero que f es la función característica de un cierto intervalo


( a, b) ⊂ I , esto es, f(x)= 1 para x ∈ ( a, b ) y f(x)= 0 para x ∈ I − ( a, b) . Según la
propiedad 3) es claro que I ( a, b; f ) = b − a = ∫ f . En particular, si a= b, se
a

obtiene I(a, a; f)= 0. Como [a, b] = {a} U ]a, b[ U {b} , aplicando la propiedad 4)
se ve el intervalo (a, b) abierto o cerrado. Supongamos ahora que f es una
función escalonada en [a, b]. Entonces f es, evidentemente, una combinación
lineal de funciones características de subintervalos abiertos (salvo un número
finito de puntos).

Aplicando las propiedades 1) y 2) se obtiene inmediatamente que


b
I(a, b; f) = f .
a

Supongamos por último que la función f es en general reglada. Existe


entonces una sucesión (f n ) de funciones escalonadas en [a, b] que converge
uniformemente hacia f. De las propiedades 1), 2) y 5) deducimos que

I ( a, b; f ) − I ( a , b; f n ) = I ( a, b; f − f n ≤ k sup f ( x ) − f n ( x)
a ≤ x≤ b

Cuando n → +∞ , el último término tiende hacia 0 por la convergencia


uniforme. Luego
lim I ( a, b; f n ) = I ( a, b; f )
n →+∞


b
Ahora bien, según hemos visto más arriba, I ( a, b; f n ) = f n ya que las f n
a


b
son escalonadas. Resulta entonces que lim f n = I (a , b; f ) . Pero por
n →+∞ a

definición, el primer miembro es la integral de la función reglada f en el


b
intervalo [a, b]. Luego f n = I ( a, b; f ) c. q. d.
a

109. Demostrar que lim J n = 0 siendo


n →+∞

π x n (bx − a) n
Jn = ∫ 0
f n ( x) sin xdx y f n ( x ) =
n!

donde a, b, n son números naturales y π ≤ a / b

Sol.

Es inmediato ver que la función polinómica p(x) = x(bx-a) tiene un


a −a2
mínimo relativo en el punto x = y su valor en este mínimo es , que es también
2b 2b

7/11
www.eltemario.com Oposiciones Secundaria – Matemáticas
© Antonio J. Abrisqueta Valcárcel, 2001 Hoja de Problemas – Análisis IV

a 2n
el mínimo absoluto de dicha función en el intervalo [0,a/b]. Luego f n ( x) ≤ para
2n b n n!
todo [0,a/b]. Tendremos, puesto que [0,π]⊂[0,a/b]:

n
π π 1  a2 
J n = ∫ f n ( x ) sin x dx ≤ ∫ f n ( x) dx ≤ π  
0 0 n!  2b 

de donde se deduce que lim J n = 0 y por tanto lim J n = 0 .


n →+∞ n →+∞

110. Determinar la función derivable f definida en [0, 2] que verifique la ecuación


3∫ f ( t ) dt = [ f ( x) + 2 f (0) ]x y es tal que f(1) = 1, f(2) = 7.
x

Sol.

Si f es derivable en [0, 2], será continua, así que, derivando en la igualdad del
enunciado resultará:
3 f ( x ) = f ( x ) + 2 f ( 0) + f ' ( x ) x
es decir:
xf ' ( x ) − 2 f ( x) + 2 f (0) = 0

Vamos a llamar g(x) = f(x) – f(0), por lo que g ' ( x ) = f ' ( x ) , y la igualdad
anterior puede escribirse de la forma:
xg' ( x ) − 2 g ( x ) = 0
o bien;
g ' (x ) 2
=
g (x ) x

Pero se sabe que [log g ( x ) ] =


′ g ' (x )
y también log x 2
′ 2
[ ]
, lo que nos
g (x ) x'
permite escribir:

log g ( x) = log x 2 + c = log x 2 + log k = log kx 2

en donde hemos tenido en cuenta que c es una constante arbitraria, que por lo
tanto puede escribirse en la forma c = log k.

Resulta finalmente:

g ( x ) = kx 2
o bien
f ( x ) − f (0) = kx 2

8/11
www.eltemario.com Oposiciones Secundaria – Matemáticas
© Antonio J. Abrisqueta Valcárcel, 2001 Hoja de Problemas – Análisis IV

Puesto que sabemos que f(1) = 1 y f(2) = 7, obtenemos:

1 − f (0) = k
7 − f (0) = 4k
de donde:
f (x ) = 2 x 2 − 1

111. Sea f una función real con derivada continua en el intervalo I = [a, b]. Supongamos
que f(a) = 0 y que 0 ≤ f ' ( x ) ≤ 1 , para todo x ∈ I . Establecer las siguientes
desigualdades:

[ f ( x )]2 ≤ 2∫a
b
1) f (t )dt para todo x ∈ I
2

∫b [ f ( x)] dx ≤  ∫b f ( t ) dt 
a 3 a
2)

Sol.


1
[ f ( x )]2 de donde:
x
1) Se tiene f (t ) f ' (t ) dt =
a 2

[ f ( x) ]2 = 2∫a
x
f (t ) f ' (t )dt

para todo x ∈ I . Como f ' (t ) ≤ 1 , será f (t ) f ' (t ) ≤ f (t ) para todo t ∈ I .


Luego [ f ( x )] ≤ 2∫ f (t ) dt .
2 x

Por otra parte, al ser f ' ( x ) ≥ 0 en I y f(a) = 0, la función f es monótona


creciente en I y entonces:

∫ f (t ) dt ≤∫ f (t ) dt
x b

a a

resultando
[ f ( x )]2 ≤ 2∫a
b
f (t )dt c. q. d.

2) Llamando F ( x) = ∫ f (t ) dt , tenemos según una desigualdad obtenida en


x

el apartado 1) que [ f ( x) ]2 ≤ 2F ( x ) para todo x ∈ I . Entonces

9/11
www.eltemario.com Oposiciones Secundaria – Matemáticas
© Antonio J. Abrisqueta Valcárcel, 2001 Hoja de Problemas – Análisis IV

∫b [ f ( x)] dx = ∫b [ f (x )] f (x )dx ≤ ∫b 2 F (x )F ' (x )dx =


a 3 a 2 a

2
= [F ( b)] − [F ( a) ] =  ∫b f ( t ) dt  c.q.d
2 2 a

 

112. Sea f una función real continua en toda recta real, y sea F la función:

1 x
2 x ∫− x
F ( x) = f (t )dt si x ≠ 0, F (0) = f (0)

Demostrar que F es continua en IR y derivable en IR – {0}. Aplicación al caso


f (t ) = t sin( 1 / t ) si t ≠ 0, f ( 0) = 0 .

Sol.

1
y f 2 ( x) = ∫ f (t )dt ,
x
Si x ≠ 0 , F es el producto de las funciones f 1 ( x ) =
2x −x

ambas continuas en IR – {0}, así que podemos asegurar la continuidad de F en


IR – {0}.

Examinaremos ahora el caso x = 0:

1 x 1
lim F ( x ) = lim
x→ 0 ∫
x→ 0 2 x − x
f (t ) dt = lim 1 2 xf ( c x ) = lim f (c x )
x →0 2 x x→ 0

en donde c x ∈ [ x,− x ] . Pero f es una función continua, y cuando x → 0 , también


c x → 0 así que lim f (c x ) = f (0) . Resulta finalmente que
x→ 0

lim F ( x ) = f (0) = F (0) , por lo que concluimos que F es continua también en el


x→ 0

punto x = 0.

Por otra parte, las funciones f 1 y f 2 son derivables en todo punto x ≠ 0 , así
que su producto, que es la función F, también lo será. Estudiemos el
comportamiento de F en x = 0.
1 h
F ( h) − F (0) 2 h ∫− h f (t )dt − f (0)
F ' (0) = lim = lim =
h →0 h h→ 0 h
1
2hf (c h ) − f (0)
f (c h ) − f ( 0)
= lim 2 h = lim ⇔ c h ∈ [− h, h ]
h→ 0 h h → 0 h

En principio, no puede asegurarse la existencia del límite anterior, que es


indeterminado, así que tampoco puede asegurarse la existencia de F’(0).

10/ 11
www.eltemario.com Oposiciones Secundaria – Matemáticas
© Antonio J. Abrisqueta Valcárcel, 2001 Hoja de Problemas – Análisis IV

1
Aplicación a la función f (t ) = t sin f si t ≠ 0 , f(0) = 0. La función f es
t
1
continua en todo IR, ya que lim t sin = 0 = f ( 0) . Podemos asegurar entonces
t →0 t
1 x 1
2 x ∫− x
que F ( x) = t sin dt es una función continua en IR, y derivable en
t
3 – {0}.
1
ch sin
f (c h ) − f (0) ch
En x = 0 resulta: F ' (0) = lim = lim que es una
h →0 h h→ 0 h
indeterminación.

11/ 11
www.eltemario.com Oposiciones Secundaria – Matemáticas
© Antonio J. Abrisqueta Valcárcel, 2002 Hoja de Problemas – Análisis V

−1 1 1


8
113. Calcular x 2 ( 2 − x ) dx 3 4
0

Sol.:
1

Mediante la sustitución x 3 = 2t

−1 −1 −3

x = 8t 3
´´ dx = 24t dt ´´ x 2 2
= (8) t 2 2

1 1 1 1 1

( 2 − x ) = ( 2 − 2t ) = 2 (1 − t )
3 4 4 4 4

para x = 8 ´´ 3 8 = 2t ´´ t = 1
para x = 0 ´´ 0 = 2t ´´ t = 0

−1 1 3 5
1
Γ( )Γ( )

8 3 5
x 2 ( 2 − x ) dx 3 4 12
=4 ∫t
1
2
−1
(1 − t ) 4
−1 12 3 5 12
dt = 4 β( , ) = 4 2 4
0 2 0
2 2 4 2 Γ(11)
4

114. Calcular mediante la función β de Euler

−1 −1
dx
∫ = ∫ (1 − x ) dx
1 1
4 2

1− 4 x
0 0

Sol.:

Realizando la sustitución x = t 4

x = t ´´
4
dx = 4t 3 dt
1 −1 −1

(1 − x ) 4 2
= (1 − t ) 2

en x = t 4 para x = 1 ⇒ t = 1
para x = 0 ⇒ t = 0

sustituyendo:

1 −1 −1 1
−1

∫ (1 − x ) dx = ∫ (1 − t ) 4t dt =4∫ (1 − t )
1 1 1
4 2 2 3 2 4 −1
t dt
0 0 0

1/6
www.eltemario.com Oposiciones Secundaria – Matemáticas
© Antonio J. Abrisqueta Valcárcel, 2002 Hoja de Problemas – Análisis V

1
Γ( 4)Γ( )
1 2 3! π 4!2 4 128
= 4 β( 4, ) = 4 =4 = =
2 9 7531 7 ⋅ 5 ⋅ 3 ⋅ 1 35
Γ( ) π
2 5222

∫x
1
a
115. Calcular mediante una función de Euler: ( log x) n dx siendo n un numero
0

natural

Sol.:

Realizamos la sustitución x = e −t ; dx = −e −t dt

Para x = 1 t =0 x a = e − ta
Para x = 0 t =∞ log x = −t ; (log x) n = (−1) n t n

∫x ( log x ) n dx = − ∫ e −ta ( −1) n t n e −t dt = ( −1) n ∫ e −t ( a +1) t n dt =


1 0
a
0 ∞ 0

dz
realizamos la sustitución t (1 + a) = z ; dt =
1+ a

para t = ∞ z=∞
n
z
t=
z
; tn =
1+ a (1 + a ) n
para t = 0 z=0

∞ zn dz ( −1) n ∞ ( −1) n
= ( −1) ∫ = ∫ e z dz = Γ(n + 1) y dado que n
n −z −z n
e
0 (1 + a) n (1 + a ) (1 + a ) n +1 0 (1 + a ) n +1

(−1) n
es natural Γ( n + 1) = n! = n!
(1 + a) n +1

116. Calcular el área comprendida entre la estrofoide y 2 ( a + x) = x 2 ( a − x ) y su


asíntota.

Sol.:

La curva es simétrica con respecto a OX, si despejamos y

x 2 (a − x) x 2 ( a − x) a−x
y = 2
´´ y = ± = ±x
a+x a+ x a+ x
para x = −a y=∞ (La asíntota es x=-a)

2/6
www.eltemario.com Oposiciones Secundaria – Matemáticas
© Antonio J. Abrisqueta Valcárcel, 2002 Hoja de Problemas – Análisis V

Calcularemos el área entre estrofoide y su asíntota por encima de OX y la


multiplicaremos por dos.

a−x a−x a−x x( a − x )


S = 2 ∫−a x dx = 2∫− a x dx = 2∫− a
0 0 0
dx Realizaremos la
a+x a+x a−x a2 − x2
sustitución x = a ⋅ cos⋅ t ´´ dx = −a ⋅ sent ⋅ dt
a 2 − x 2 = a ⋅ sent
π
en x = a ⋅ cos t para x=0 ´´ 0= a ⋅ cos t t=
2
para x=-a ´´ -a= a ⋅ cos t t =π

Sustituyendo:

π π
x ( a − x) a ⋅ cos t ( −a ⋅ cos t )
2∫ dx = 2∫ (− a ⋅ sent ) dt = −2a ∫ cos t (1 − cos t ) dt =
0
2 2 2
−a
a2 − x 2 π a ⋅ sent π

π
π π
1 1  1 1 2
− 2a 2 ∫ 2 (cos t − cos 2 t ) dt = −2a 2 ∫ 2 (cos t − − cos 2t ) dt = −2 a 2  + sent − t − sen2t  =
π π 2 2  2 4 π

 π π 1 π 1  a2
= −2a  ( sen − − ⋅ sen π) − ( senπ − − sen 2π)  = −
2
( 4 + π) =
 2 4 2 2 4  2

a2
S= ( 4 + π)u 2
2

117. Calcular el área encerrada por la curva y 2 = x 2 − x 4

Sol.:

La curva es simétrica con respecto a OX y OY

y = ± x2 − x 4 = ± x 1 − x 2

Hallando sus puntos de corte con OX, haciendo y=0

x=0
± x 1 − x =0 x = +1
2

x = −1

Calcularemos la rama positiva el área sobre OX y la multiplicaremos por 4

3/6
www.eltemario.com Oposiciones Secundaria – Matemáticas
© Antonio J. Abrisqueta Valcárcel, 2002 Hoja de Problemas – Análisis V

1
4 
1 3
1 1 3
S = 4 ∫ x 1 − x dx = 4( − ) ∫ − 2 x (1 − x 2 ) 2 dx = −  (1 − x) 2 
1
2
0 3 0 2 3 0
4 2
S= u
3

118. Calcular el área encerrada por la rosa de tres pétalos ρ = a ⋅ cos 3ω

Sol.:

π
Se puede calcular el área entre 0 y y multiplicar el resto por seis.
6

π
π π π
1 1 1 1 1 6
S = 6∫6 a 2 cos2 3ω⋅ dω= 3a2 ∫ 6 cos2 3ω⋅ dω= 3a 2 ∫ 6 ( + cos6ω) ⋅ dω=3a2  ω+ sen6ω
0 2 0 0 2 2 2 4 ⋅6 0
π ⋅ a2 2
= u
4

119. Calcular el área encerrada por la curva ρ = 2 + cos ω

Sol.:

Se puede calcular el área entre 0 y π y multiplicar el resto por 2

1 π π π 1 1
S=2 ∫ + ω ω = ∫ + ω + ω ω = ∫ ( 4 + 4 cos ω + + cos 2ω)dω =
2 2
(2 cos ) d ( 4 4 cos 4 cos )d
2 0 0 0 2 2

π
 1 1  9π 2
=  4ω + 4 senω + ω + sen 2ω = u
 2 4 0 2

120. Calcular el área encerrada por la cardioide ρ = a (1 + cos ω)

Sol.:

Se puede calcular el área entre 0 y π y multiplicar el resultado por 2.

1 π 2 π π 1 1
S =2 ∫
2 0
a (1 + cos ω) 2 dω =a 2 ∫ (1 + 2 cos ω + cos 2 ω) dω =a 2 ∫ (1 + 2 cos ω + + cos 2ω) dω =
0 0 2 2
π
  3π ⋅ a 2
2
1 1
a 2 ω + 2 senω + ω + sen2ω = u
 2 4 0 2

4/6
www.eltemario.com Oposiciones Secundaria – Matemáticas
© Antonio J. Abrisqueta Valcárcel, 2002 Hoja de Problemas – Análisis V

121. Calcular el área común entre el cardioide ρ = 1 + cos ω y el circulo ρ = 3 cos ω

Sol.:

Calcularemos primero los puntos comunes a ambas curvas resolviendo el sistema

ρ = 1 + cos ω 1 + cos ω = 3 cos ω

1
ρ = 3 cos ω 1 = 2 cos ω ´´ cos ω =
2

π
ω= ±
3

Dada la simetría de la figura respecto al eje podemos calcular el área superior y


multiplicar por dos el resultado

π
El área la podemos calcular como el área de la cardioide desde 0 a mas el área
3
π π
del circulo desde a
3 2

1 π3 1 π2
S = 2 ∫ (1+ cosω) dω + 2 ∫π 9 cos2 ω ⋅ dω =
2

2 0 2 3
π π
1 1
= ∫03 (1 + 2 cos ω + cos 2 ω) dω + 9 ∫π2 + cos 2ω ⋅ dω =
3
2 2
π π
 1 1 3 1 1  2 5π 2
= ω + 2 senω + ω + sen 2ω + 9 ω + sen 2ω = u
 2 4 0 2 4 π 4
3

122. Calcular el volumen engendrado por el área común a la cardioide ρ = 1 − cos ω y


al circulo ρ = 1 al girar alrededor del eje polar

Sol.:

Calcularemos el volumen engendrado por el primer cuadrante de la cardioide al


2
girar alrededor del eje polar y sumaremos a este volumen el de la semiesfera = π13
3

5/6
www.eltemario.com Oposiciones Secundaria – Matemáticas
© Antonio J. Abrisqueta Valcárcel, 2002 Hoja de Problemas – Análisis V

π
2 π2 3 2 π2 2 1
[ π
V = π∫0 ρ senω⋅ dω = π∫0 (1− cosω)3senω ⋅ dω= π (1−cosω)4 02 = (1) =
3 3 3 4 6
]
π
6
π 2π π 4π 5π 3
Volumen = + = + = u
6 3 6 6 6

6/6
www.eltemario.com Oposiciones Secundaria – Matemáticas
© Antonio J. Abrisqueta Valcárcel, 2002 Hoja de Problemas – Análisis VI

123. Calcular el área de la superficie engendrada por la revolución alrededor del


1 1 1

eje OX de la astroide (hipocicloide x3 + y 3 = a3 )

Sol.:

x = a ⋅ cos 3 t para x=a ´´ a = a ⋅ cos 3 t


t =0
y = a ⋅ sen 3 t para x=-a ´´ − a = a ⋅ cos 3 t
t =π

2
dx  dx  = 9 a 2 sen 2 t ⋅ cos 4 t
= a 3( −sent ) cos 2 t ´´  
dt  dt 
 dy 
2
dy
= a3 cos t ⋅ sen 2 t   = 9a cos t ⋅ sen t
2 2 2

dt  dt 

 dx  +  dy  = 9a 2 sen 2 t ⋅ cos 2 t ( sen 2 t + cos 2 t ) = 9 a 2 sen 2 t ⋅ cos 2 t


2 2

   
 dt   dt 

π
integramos y 0 y multiplicamos por 2
2
π π

S = 2π ⋅ 2 ∫ a ⋅ sen t ⋅ 3a ⋅ sent ⋅ cos t ⋅ dt = 12a π ∫ 2 sen 4 t ⋅ cos t ⋅ dt =


2 3 2
0 0

π
para t = z=1
2
haciendo la sustitución sen t=z ´´ cos t dt=dz
para t=0 z=0

1
 z 5  12 a 2 ⋅ π 2
= +12a ⋅ π∫0 z dz = + 12a ⋅ π   =
1
2 4 2
u
 5 0 5

124. Determinar el volumen de una cuña, cortada de un cilindro circular, por un


plano que pasando por el diámetro de la base esta inclinado respecto a ella
formando un ángulo a, siendo el radio igual a a.

Sol.:

El área de la sección ABC, a la distancia x del origen de coordenadas es

1 1 y2
S ( x) = areaABC = AB ⋅ BC = y ⋅ y ⋅ tga = tga
2 2 2

1/7
www.eltemario.com Oposiciones Secundaria – Matemáticas
© Antonio J. Abrisqueta Valcárcel, 2002 Hoja de Problemas – Análisis VI

El volumen podemos, dado que la figura es simétrica, integrarlo entre a y o y


multiplicar el resultado por 2:

1
V = 2 tga ∫ y 2 dx ; pero dado que la ecuación de la circunferencia de la base es
a

2 0

x 2 + y 2 = a2 ; y 2 = a 2 − x de

donde
a
 x3  2
V = tga∫ ( a − x ) dx = tga a 2 x −  = tga ⋅ a 3 u 3
a
2 2
0
 3 0 3

125. Calcular el volumen de un cuerpo cuya base es circular de radio igual a a,


siendo cada sección plana perpendicular a un diámetro fijo un triangulo un
triangulo equilátero

Sol.:
.
La ecuación del circulo considerado en el plano OXY es x 2 + y 2 = a 2 Y ABC
es el triángulo equilátero sección de la figura, cuya base es 2y y su altura (altura de un
triangulo equilátero de lado 2 y ; h = 3y )

(4 y 2 = y 2 + h 2 ) → h = 3 y

El volumen podemos, dada la simetría de la figura integrarlo entre a y 0 y multiplicar el


resultado por 2:
1
S ( x) = 2 y 3 y = 3 y 2 = 3 ( a 2 − x 2 )
2

a
 x3  4a3 3 3
V = 3 ⋅ 2 ∫ ( a − x ) dx = 3 ⋅ 2 a 2 x −  =
a
2 2
u
0
 3 0 3

ex −1
126. Calcular la longitud del arco de la curva y = log entre x=2 y x=4
ex +1

Sol.:

ex −1
y = log x = log( e x − 1) − log( e x + 1)
e +1
e x
ex e x ( e x + 1) − e x ( e x − 1) e 2 x + e x − e 2 x + e x
y´= x − x = =
e −1 e +1 ( e x + 1)( e x − 1) ( e 2 x − 1)

2/7
www.eltemario.com Oposiciones Secundaria – Matemáticas
© Antonio J. Abrisqueta Valcárcel, 2002 Hoja de Problemas – Análisis VI

2e x 4e 2 x 4e 2 x (e 2 x − 1) 2 + 4e 2 x
y´= ´´ ( y´) 2
= 1 + ( y´) = 1 +
2
= =
e2 x − 1 e2 x − 1
2
( ) ( e 2 x − 1) 2 ( e 2 x − 1) 2

e 4 x + 1 − e 2 x + 4e 2 x
=
(e 2x
+1 ) 2

(e 2x
)
−1
2
e −1
2x

e 2x + 1 e2x + 1
L = ∫2
4
= 2x ´´ dx
e −1 e2x − 1

Realizando la sustitución e 2 x = t ´´ 2 ⋅ e 2 x dx = dt

dt dt
dx = = ´´ para x=4 e 2 ⋅4 = e 8 = t
2⋅e 2x
2t

para x=2 e 2 ⋅2 = e 4 = t

e2x + 1 e8 t + 1 dt 1 e8 t + 1
∫ ∫e 4 t − 1 2t 2 ∫e 4 (t − 1)t dt (Racional C-1)
4
dx = =
2 e2 x − 1

t +1 A B t +1 A( t − 1) B⋅t
= + , = +
(t − 1) t t t − 1 ( t − 1)t t ( t − 1) t ( t − 1)

t + 1 = A( t − 1) + B ⋅ t para t=1 , 2=B

para t=0 ´´ 1=-A ´´ A=-1

e8
1 e8 t + 1 1 e8 dt e8 dt  1 

2 e (t − 1)
4
dt = − ∫ 4
2 e t
+ ∫e t −1
4
=  − log t + log( t − 1)  =
 2  e4

 1 1  e8 − 1

 2 log e 8
+ log( e 8
− 1) + log e 4
− log( e 4
− 1) = − 2 + log =L
2 e4 −1

127. Calcular la longitud del cardioide

Sol.:

ρ = 2 a(1 + cos ω)

Dado que la curva es simétrica con respecto al eje polar podemos calcular la
longitud desde ω1 = 0 a ω2 = π

ρ 2 = 4a 2 (1 + cos ω) 2 = 4a 2 + 4a 2 cos 2 ω + 8a 2 cos ω

3/7
www.eltemario.com Oposiciones Secundaria – Matemáticas
© Antonio J. Abrisqueta Valcárcel, 2002 Hoja de Problemas – Análisis VI

dρ  dρ 
2

= −2a ⋅ senω ´´   = 4a ⋅ sen ω


2 2

dω  dω 

 dρ 
2

ρ + 
2
 = 4a 2 + 4a 2 cos ω + 4a 2 ⋅ sen 2 ω = 4a 2 + 4a 2 (cos 2 ω + sen 2 ω) + 8a 2

 dω 

ω
= 8a 2 + 8a 2 cos ω = 8a 2 (1 + cos ω) = 8a 2 2 cos 2
2

 dρ 
2
ω ω
ρ2 +   = 16a 2 cos 2 = 4 a cos
 dω  2 2

π
π ω π 1 ω  ω π
L = 2 ∫ 4 a cos dω = 2 ⋅ 4a ⋅ 2 ∫ cos dω = 16a  sen  = 16a ( sen − sen 0) = 16 a
0 2 0 2 2  2 0 2

128. Calcular la longitud del arco de curva x=cos t desde t=0


y=2sen t a t=π
3
z= t
π

Sol.:

dx dy dz 3
= −2 sent ; = 2 cos t ; =
dt dt dt π

2
 dy 
2
 dy 
2
 dx  = 4sen 2 t ; 9
    = 4 cos t   = 2
2
;
 dt   dt   dt  π

2 2 2
 dx   dy   dz  9 9 4π 2 + 9 4π 2 + 9
  +   +   = 4sen 2 t + 4 cos 2 t + 2 = 4 + 2 = =
 dt   dt   dt  π π π2 π

4π 2 + 9 4π 2 + 9 π
[t]0 = 4π 2 + 9
π
L=∫ dt =
0 π π

129 Calcular la longitud del arco de curva alabeada

x = e θa cos θ
y = e θa senθ desde θ = θ2 a θ = θ1
z = b ⋅e θa

4/7
www.eltemario.com Oposiciones Secundaria – Matemáticas
© Antonio J. Abrisqueta Valcárcel, 2002 Hoja de Problemas – Análisis VI

Sol.:

2
dx  dx  = e 2θa ( a 2 cos 2 θ + sen 2θ − 2a cos θ ⋅ senθ
= aeθa cos θ − e θa senθ ;  
dθ  dθ 
 dy 
2
dy
= aeθa senθ + eθa cos θ ;   = e ( a sen θ + cos θ + 2 a cos θ ⋅ senθ
2θ a 2 2 2

dθ  dθ 
2
dz  dz  = e θa b 2
= b ⋅ ae θa ;  
dθ  dθ 

2 2 2
 dx   dy   dz 
  +  +  = eθa a 2 + 1 + b 2 a 2
 
dθ  dθ   d θ 

[ ]
θ2
θ2 1 1
L=∫ e θa
1 + a + a b ⋅ dθ =
2 2 2
1 + a 2 + a 2 b 2 eθa = 1 + a 2 + a 2 b 2 (e θ 2 a − e θ1a )
θ1 a a
θ1


b
130. Calcular por derivación paramétrica: x 2 cos x ⋅ dx
0

Sol.:

Partimos del resultado conocido:

a cos ax ⋅ dx = [sena ⋅ x] = sena ⋅ b


1 b 1 1
∫0 ∫
b
cos ax ⋅ dx =
a 0 a 0 a
d  b 1  b 1
 ∫0 cos ax ⋅ dx = sena ⋅ b  = − ∫0 x ⋅ senax ⋅ dx = cos ab − 2 senab
b

da  a  a a

derivando nuevamente respecto de a

d  b b 1 
 − ∫0 x ⋅ senax ⋅ dx = cos ab − 2 senab  =
da  a a 
2
b b 2
− ∫ x 2 ⋅ cos ax ⋅ dx = −
b
senab − 2 cos ab + 3 senab
a a a
 b 2
2 2b
∫0
b
x 2
cos ax ⋅ dx = 
 a a 3  senab − a 2 cos ab
− +
 


2 2
131. Calcular por derivación paramétrica la integral e −a x
cos 2bx ⋅ dx
0

Sol.:

5/7
www.eltemario.com Oposiciones Secundaria – Matemáticas
© Antonio J. Abrisqueta Valcárcel, 2002 Hoja de Problemas – Análisis VI

d ∞
I= = −2∫ xe −a x sen2bx ⋅ dx , que vamos a integrar primero por partes:
2 2

db 0

u = sen 2bx dv = xe − a ⋅ dx
2 2
x

du 1
∫ dv = ∫ xe ∫
−a x 2 2
− a 2x 2
= 2b cos 2bx dx = − − 2
2 a xe dx
dx 2a 2
1 −a 2 x 2
du = 2b cos 2bx ⋅ dx v=− e
2a2


∞  1  2b ∞
− 2∫ xe− a sen 2bx ⋅ dx = −2 − 2 e − a x sen 2bx  − 2 ∫ xe− a x cos 2bx ⋅ d x +
2 2 2 2 2 2
x
0
 2a 0 a 0

 1 
+ 2 2 e − a x sen 2bx =
2 2

 2a 0
 1   1 
=   2 + a 2 ∞ sen( 2b ⋅ ∞ )  −  2 a 2 0 sen 2b ⋅ 0   = 0 de donde
  2a e   2a e 

0 acotada cte 0

2b ∞

∫ e −a
2 2
I =− cos 2bx ⋅ dx
x
si llamamos M a la integral inicial
a2 0

2b dM
I =− 2
M y como I =
a db

dM 2b dM 2b
= − 2 M de donde = − 2 db integrando en ambos términos:
db a M a

dM 2b 2 b2 b2
∫ M = ∫ − a 2 db ´´ log M = −
a2

2
= −
a2
+ log C

b2 b2 b2
− + log C − −
M =e a2
=e a2
e log C
= C ⋅e a2


M ( b= 0) = ∫ e − a
2 2
si hacemos b=0 x
dx = C ⋅ e 0 = C
0

 −b 2 
2
b2
 π − a2
dx  ⋅  e a
 

M =  ∫0 e − a 2x 2
de donde M = e
    2a
 
∞ π
∫ e −a dx =
2 2
x
Sabiendo que
0 2a

6/7
www.eltemario.com Oposiciones Secundaria – Matemáticas
© Antonio J. Abrisqueta Valcárcel, 2002 Hoja de Problemas – Análisis VI

x n −1 − 1
132. Calcular por derivación respecto a un parámetro I = ∫
1
dx
0 log x
Sol.:

Derivando I respecto a n:

1
dI 1 x
n −1
log x xn  1
= ∫0 dx = ∫0 x n −1 dx =   =
1

dn log x  n 0 n

dI 1 dn
= ´´ dI = integrando
dn n n

= [log n]1n = log n


dn
∫ dI = ∫
I n

0 1 n

I = log n

1 − e− x
2

133. Calcular la integral ∫
0
x2
dx

Sol.:

La calcularemos por derivación introduciendo el parámetro a como sigue:

1 − e −ax
2

u = ∫0 dx
x2

du  ∞ 1 − e − ax  ∞ − ax 2
2

da  ∫0
dx  = ∫ e dx =
x2  0

dz
realizando la sustitución a x2 = z2 ; dx =
a
para x = ∞ z =∞
para x=0 z=0

∞ 1 1 π ∞ π
=∫ e − z dz = ∫ e − z dz =
2 2
(veremos adelante que )
0
a a 2 0 2

du 1 1 π 1 π
= ; du = da ; u=∫ da
da a a 2 a 2

1 − e− x
2

u= π a+C ; C=0 para a=1 de donde u = ∫0 dx = π
x2

7/7
www.eltemario.com Oposiciones Secundaria – Matemáticas
© Antonio J. Abrisqueta Valcárcel, 2002 Hoja de Problemas – Geometría I

134. En un trapecio ABCD, la base mide 9cm y es fija en posición. El lado AB es


paralelo a la base DC, varia en posición, pero su magnitud es siempre igual a
6cm. La diferencia de los lados no paralelos BC y DA se mantienen siempre
igual a 1cm. Estos dos lados prolongados se cortan en el punto M. Se pide:

a) Hallar el lugar geométrico del punto M, dando las características que


permitan la definición completa de dicho lugar.
b) Dado un punto P situado sobre la mediatriz de DC hacia arriba y a 6cm de
este lado, tazar la normal al lugar sin dibujar este, justificando mediante
demostración los procedimientos gráficos empleados.

Demostración
a) De la semejanza de los triángulos MAB y MBC se deduce:
MA MB AB 6 3 3
= = = , luego MD = MA MC = MB como MD=MA+AD
MD MC DC 9 2 2

MC=MB+BC BC-AD=1 resulta:

3 3
MD = MA = ( MD − AD) ⇒ MD = 3 AD
2 2
MC = MB = (MC − BC ) = (MC − AD − 1) ⇒ MC = 3 AD + 3
3 3 3
2 2 2

Asi MC-MD=3 y el punto M esta en una hipérbola de focos los puntos C y D,


distancia focal 9cm, y semieje real 3/2 cm.

Al ser MC-MD=3, los puntos N tales que ND-NC=3, también pertenecientes a la


hipérbola, no están en lugar geométrico; este se reduce a la rama de la hipérbola
correspondiente al foco D; esto es, la situada en el semiplano izquierdo de las
determinadas por la mediatriz de DC (eje transverso de la hipérbola)

b) Supongamos resuelto el problema.

Sean N y N´ los puntos en los que las normales desde P a la hipérbola la cortan. La
circunferencia, c, que pasa por P, N y N´ corta al eje de la hipérbola en otro punto Q, y
por simetría, PQ es un diámetro de la circunferencia.

Por ser QN perpendicular a PN y QN´ perpendicular a PN´, QN y QN´ son las


tangentes trazadas desde Q a la hipérbola.

Veamos que la circunferencia, c, pasa por los focos D y C. En efecto, por ser
tangente en un punto bisectriz de los radios vectores, son iguales los ángulos DNQ y
QNC y los ángulos DN´Q y QN´C; por simetría, son iguales los ángulos DNQ y QN´D,
luego son iguales los ángulos DNQ y DN´Q, con lo que D esta en la circunferencia, c;
análogamente C esta en la circunferencia.

El radio vector desde C al punto N pasara por un punto H de la circunferencia focal


o directriz con centro en C, tal que ND=NH (por la definición de hipérbola, al ser
HC=3cm). Por otro lado, H esta también en la circunferencia de centro Q que pasa por

1/9
www.eltemario.com Oposiciones Secundaria – Matemáticas
© Antonio J. Abrisqueta Valcárcel, 2002 Hoja de Problemas – Geometría I

D, ya que al ser la tangente bisectriz de los radios vectores y ND=NH, la tangente es la


mediatriz del segmento DH.

El punto de intersección de la mediatriz DH (tangente de la hipérbola en N) y de la


prolongación del radio CH (radio vector del punto N) determinara el punto N.
La normal pedida será la recta PN.

Se tiene el siguiente proceso para trazar la normal:

• Construir la circunferencia que pasa por P y los focos.


• Determinar el punto Q en donde la circunferencia corta al eje.
• Hallar los puntos de intersección H y H´, de la circunferencia con centro en Q y
radio QD, con la circunferencia de centro en C y radio 3 (circunferencia focal)
• Trazar la mediatrices de los segmentos QH y QH´.
• Hallar la intersección de estas mediatrices con las rectas CH y CH´
(respectivamente), se obtienen así los puntos N y N´.
• La rectas PN y PN´ son las normales desde P a la hipérbola.
• La recta Pn es la normal desde P al lugar, rama izquierda de la hipérbola.

135. Dado un cuadrilátero cualquiera ABCD, trazar por el vértice B, una recta que
dividida al cuadrilátero en 2 partes equivalentes. Demuéstralo.

Demostración:

Sea E el punto de intersección de la recta paralela a AD pasando por B, con la recta


paralela a BD pasando por C.

Los cuadriláteros ABCD y ABED son equivalentes, ya que las alturas


correspondientes al lado común BD de los triángulos BCD y BED tienen la misma
longitud.

Sea F el punto medio de ED y sean G y H los puntos de intersección de la recta


paralela a AB pasando por F con BE y AD respectivamente.

Como los triángulos FEG y FDH son iguales, los cuadriláteros ABED y ABGH son
equivalentes.

Al ser ABGH un paralelogramo, su diagonal BH lo divide en dos partes


equivalentes, luego también el área del triángulo ABH es la mitad del área del
cuadrilátero ABCD, por lo que BH es la recta buscada.

136. En una circunferencia de radio R se consideran las cuerdas AB, fija de


longitud R y la cuerda AC variable. Sobre estas dos cuerdas se construye el
paralelogramo BACD. Se pide determinar los siguientes lugares:

a) Del punto medio de AC.


b) Del centro del paralelogramo.
c) Del vértice D.

2/9
www.eltemario.com Oposiciones Secundaria – Matemáticas
© Antonio J. Abrisqueta Valcárcel, 2002 Hoja de Problemas – Geometría I

Demostración:

1
a) Sea M el punto medio de la cuerda AC, AM = AC . Asi M es homotético de C en
2
1
una homotecia de centro A y razón . Cuando AC varia, C recorre la circunferencia
2
R
dada; por tanto, M describe una circunferencia de radio y centro en O´, punto
2
1
medio de AO`, homotética de la dada en la homotecia de centro A y razón .
2
Obsérvese que el lugar pasa, en particular, por el punto medio de AB.
1
b) Sea P el centro del paralelogramo. BP = BC . Así P es homotético de C en una
2
1
homotecia de centro B y razón .
2
Cuando AC varia, C recorre la circunferencia dada; por tanto P describe una
1
circunferencia homotética a la dada en la homotecia de centro B y razón . Su radio
2
R
es y su centro O´´ con 2BO´´=BO. Nótese que el lugar se puede deducir del
2
1
anterior, efectuando una traslación de vector MP= Ab.
2
c) El punto D se deduce de C aplicándole una traslación de vector AB, pues CD=AB.
Cuando AC varia, C recorre la circunferencia dada; por tanto, D describe una
circunferencia trasladada de la dada en una traslación de vector AB.

137. Construir un triangulo, conociendo dos lados “b” y “c”, y la bisectriz “d” de
angulo que forman. Discusión del problema a resolver.

Demostración:

Supuesto el problema resuelto si por el vértice B trazamos BE paralela a la bisectriz


d, al ser iguales los ángulos BAD y ABE, por un lado, AEB y CAD por otro, obtenemos
que el triangulo BAE es isósceles. En consecuencia c=EA.

Considerando las rectas CE y CB cortadas por las paralelas AD y EB se tiene:

CB − CD c
= ⇔ CB =  1 +
c
DB EA
= ⇒
DB c
= ⇔ CD [1]
DC AC DC b CD b  b

La igualdad [1] pone de manifiesto que B es el homotético de D en la homotecia de


centro C y razón 1+c/b.

Por todo ello construimos el segmento EC de longitud c+b. A distancia c de E se


encuentra en el punto A.

3/9
www.eltemario.com Oposiciones Secundaria – Matemáticas
© Antonio J. Abrisqueta Valcárcel, 2002 Hoja de Problemas – Geometría I

Resolveremos únicamente el problema en el caso c ≤ b (lo que presupone d<b para


que exista solución), pues por analogía se resolvería el caso restante. El vértice B se
encontrara fuera de la recta EC y en una circunferencia de centro A y radio c.

El punto D estará situado en una circunferencia de centro A y radio d y fuera de la


recta EC.

Según [1] , aplicando a la circunferencia de centro A y radio d una homotecia de


centro C y razón 1+c/d, obtenemos otra circunferencia; si esta circunferencia
transformada corta a la circunferencia de centro A y radio c en dos puntos obtendremos
solución a nuestro problema pues los puntos de corte son los puntos donde puede estar
situado B ( resultan ser simétricos respecto de la recta EC). Si hubiera un solo punto de
corte no se formaría triangulo, ya que B estaría en la recta EC. Una vez obtenido B
resulta inmediato obtener D.

Designamos por K a la expresión 1+c/b.

Al aplicar la homotecia de centro C y radio K a la circunferencia de centro A y radio


d, obtenemos otra circunferencia de centro A´=E y radio dK tal que
CA´
= K ⇔ CA´= KCA = Kb .
CA

Para que la circunferencia de centro A´ y radio dK corte en 2 puntos a la


circunferencia de centro A y radio c, tomando C como origen y siendo las abcisas de A
y E, b y b+c respectivamente, se ha de verificar:
Kb-dK∈(b-c, b+c)

Pues téngase en cuenta que las abcisas de los puntos de corte de la circunferencia de
centro A y radio d con CE son b-c y b+c.

En consecuencia:
2c 2bc
b − c < b + c − Kd < b + c ⇔ 0 < Kd < 2c ⇔ 0 < d < = luego d ha de
c b+c
1+
b
estar comprendido entre 0 y la media armónica de b y c.

138. Dado un cono equilátero de lado 10, se corta por un plano paralelo a una
generatriz, y se pide el área del segmento parabólico así obtenido cuando esta
área es máxima.

Demostración:

Eligiendo convenientemente los ejes, calculemos el área del segmento parabólico


DEFG, tal y como se muestra en la figura 2.
Suponiendo que la ecuación de la parábola es y = Kx 2 , haciendo FG=a, DF=b, al ser
b
G(a,b) perteneciente a la parábola ha de ser K = 2 .
a

4/9
www.eltemario.com Oposiciones Secundaria – Matemáticas
© Antonio J. Abrisqueta Valcárcel, 2002 Hoja de Problemas – Geometría I

El área del segmento es:


b a3
[1]
a b 2 2
S = 2( ab − ∫0 2 x 2 dx ) = 2( ab − 2 ) = 2ab = EG.DF
a a 3 3 3

En la fig 1, si x=af, utilizando que FBD es un triangulo equilátero se obtiene:

FD=10-x [2]
En el triángulo rectángulo AGB se verifica:
2 2

FG 2 = AF.FB ⇔ 
EG   EG  = x(10 − x)
 = AF .FB Luego   [3]
 2   2 

Sustituyendo [2] y [3] en [1] :

2
S ( x) = 2 x(10 − x) (10 − x)
3 [4]
x ∈ [0,10]

Al ser x → S (x ) continua en [0,10 ] alcanza su máximo absoluto. Como


S(0)=S(10)=0 el máximo absoluto será un máximo relativo y se alcanza en (0,10)

En vez de derivar S derivaremos S 2 pues el valor de x buscado maximiza


simultáneamente ambas funciones; dado que S ( x) ≥ 0 :

16
S 2 (x ) =x(10 − x ) 3
9
dS 2 16 16 16 16
= (10 − x) 3 + x3(10 − x) 2 ( −1) = (10 − x) 2 (10 − x − 3x ) = (10 − x ) 3 (10 − 4 x )
dx 9 9 9 9

Así el x buscado es tal que:


16
x ∈ ( 0,10) y (10 − x ) 2 (10 − 4 x) = 0
9

10
Es decir: x=
4

, es según [4] :
10
El área del segmento, para x =
4
2 10 10 10 1
S= 2 (10 − ) (10 − ) = 300 ⋅ 30 = 25 3
3 4 4 4 12

5/9
www.eltemario.com Oposiciones Secundaria – Matemáticas
© Antonio J. Abrisqueta Valcárcel, 2002 Hoja de Problemas – Geometría I

139. Siendo a, b el menor par de números naturales que verifican la ecuación 32a-
19b=1, y K el valor que hace compatible las ecuaciones del sistema.

2x+y=3
x+3y=5
-x+7y=K
encontrar:
a. Ecuaciones de la inversión de polo el punto p(a,b) y potencia K
b. Ecuación de la circunferencia de puntos dobles de dicha inversión
c. Transformada en dicha inversión de x 2 + y 2 − 6 x − 10 y = 0

Demostración.

Resolvamos la ecuación diofántica 32a-19b=1

32a − 1 13 1
b= =a+ a−
19 19 19

Como b ha de ser natural y a también, 13a-1=19z y b=a+z con z∈N

Luego la solución buscada es a=3 y b=5.

Para que el sistema sea compatible, dado que el rango de la matriz de los coeficientes es
2, el de la ampliada también a de ser 2. luego:
2 1 3
1 3 5 =0 ⇔ K = 9
-1 7 K

a) Por semejanza de triángulos:


x − 3 y − 5 PA PA.PA (PA) 2
= = = = [1]
x´−3 y´−5 PA´ PA.PA´ 9
Teniendo en cuenta que ( PA) 2 = ( x − 3) 2 + ( y − 5) 2 , sustituyendo en [1] , se
obtiene:
[ ]
9( x − 3) = ( x´−3) ( x − 3) 2 + ( y − 5) 2 ⇒ x´=
9( x − 3)
+3
( x − 3) 2 + ( y − 5) 2

[ ]
9( y − 5) = ( y´−5) ( x − 3) 2 + ( y − 5) 2 ⇒ y´=
9( y − 5)
+5
( x − 3) 2 + ( y − 5) 2
que son las ecuaciones de la inversión.

b) Es ( x − 3) 2 + ( y − 5) 2 = 9
c) x 2 + y 2 − 6 x − 10 y = 0 ⇔ ( x − 3) 2 + ( y − 5) 2 = 34

Sustituyendo en las ecuaciones de la inversión:

6/9
www.eltemario.com Oposiciones Secundaria – Matemáticas
© Antonio J. Abrisqueta Valcárcel, 2002 Hoja de Problemas – Geometría I

9( x − 3) 34( x´−3)
x´= +3 x−3=
34 9

9( y − 5) 34( y´−5)
y´= +5 y−5=
34 9

luego la circunferencia transformada es:

34 2 ( x´−3) 2 34 2 ( y´−5) 2 81
+ = 34 ⇔ ( x´−3) 2 + ( y´−5) 2 =
81 81 34

140. Dada la cónica y 2 = px y el haz de rectas y-a=t(x-b), hallar el lugar geométrico


de los puntos en que las rectas de este haz cortan a las tangentes a la cónica en
los puntos de intersección de esta con el haz de rectas y=tx

Demostración.

Hallemos las intersecciones de las rectas y=tx con la cónica.


y = tx  2 p 2p 
⇒ ( 0,0) y  2 ,  son los puntos de corte.
y = 2 px
2
t t 
En el caso t=0 solo existiría un punto de corte: (0,0)

a) Supongamos t ≠ 0

Las tangentes a la parábola en los dos puntos de corte son:


2p t  2p
r1 ≡ x = 0 , r2 ≡ y − = x − 2 
t 2 t 

Al hallar la intersección de r1 con y-a=t(x-b) se obtiene (0,a-tb)

Si b ≠ 0 esos puntos son todos los del eje x=0 , salvo el punto (0,a)

Si b=0 obtenemos (0,a)

Al hallar la intersección de r2 con y-a=t(x-b), eliminando t queda:


2 y ( x − b)( y − a ) − 2 p ( x − b ) 2 − x ( y − a) 2 = 0 [1] ecuación que nos da
puntos pertenecientes al lugar.

b) Si t=0 al hallar la intersección de x=0 con y-a=0(x-b)=0 queda el punto (0,a)

Resumiendo: el lugar buscado esta constituido por los puntos que verifican [1] y
el eje x=0 si b ≠ 0 .
Si b=0 esta formado por (0.a) y [1]

7/9
www.eltemario.com Oposiciones Secundaria – Matemáticas
© Antonio J. Abrisqueta Valcárcel, 2002 Hoja de Problemas – Geometría I

141. Dada una pirámide regular de base cuadrada y altura igual al doble de lado de
la base, se pide determinar la relación de volúmenes (superior e inferior) en
que queda dividida por un plano que pasando por un lado de la base corta a la
pirámide según un polígono cuyo perímetro sea mínimo.

Demostración.

La sección de la pirámide por un plano que pasa por el lado CD es el trapecio


CDEF. El perímetro de CDEF será mínimo cuando, considerando el desarrollo lateral
de la pirámide, sea mínima la longitud de la poligonal DEFC y ello supone que esta
poligonal es un segmento rectilíneo.

Por la semejanza de los triángulos OEF y OAB:

EF ON OD cos 3α 4 cos 3 α − 3 cos α


= = = = 4 cos 2 α − 3
AB OM OA cos α cos α
1
AB
AM AM 2
y como senα = = = 2 =
OA AH 2 + OH 2 3 6
AB
2
7
Haciendo AB=a, resulta EF = ( 4 cos 2 α − 3)a = (1 − 4 sen 2α) a = a.
9

Para obtener la altura, EP=h, del prismatoide ABCDEF:

OH − EP EF 14 14
= ⇒ ( 2a − h) = a ⇒ h = a
OH AB 9 9

17  8 1
Como GJ =  a + a  = a JK = a , por la formula de los tres niveles el volumen
29  9 2
del prismatoide ABCDEF es:
1 11 4 50 3
h( S ABCD + 4 S GJKL ) = a (a 2 + 4 a 2 ) = a
6 64 9 243

2 3
Al se el volumen de la pirámide a , la relación de volúmenes pedida es:
3
2 3 50 3
a − a
3 243 56
50 3 25
a
243

142. Sean 0 y A dos puntos diametralmente opuestos de una circunferencia. Por 0


se traza una secante que corta al círculo en B y a la tangente en A en el punto
C. Por B se traza una perpendicular o OA y por C una paralela a OA. Estas
dos rectas se cortan en un punto M. El lugar geométrico de los puntos M es

8/9
www.eltemario.com Oposiciones Secundaria – Matemáticas
© Antonio J. Abrisqueta Valcárcel, 2002 Hoja de Problemas – Geometría I

una cubica que tiene por asíntota la tangente en el punto 0. Hallar el área
comprendida entre esta curva y su asíntota.

Demostración.

Sea ( x − r )2 + y 2 = r 2 la ecuación de la circunferencia.

π π
Sea m=tgα, con α variando en el intervalo  − , , la pendiente de la recta OC. Se
 2 2
tiene:

Ecuación de la recta OC y=mx coordenadas de C(2r,2mr)

2r
y = mx x=
1+ m2
Coordenadas del punto B: ⇒
( x − r) 2 + y 2 = r 2 2 mr
y=
1+ m2

Ecuación de la recta CM: y = 2 mr

⇒ M  , 2mr 
2r
1+ m 
2

2r
Ecuación de la recta BM: x=
1 + m2

Unas ecuaciones paramétricas de lugar descrito por M, son:


2r
x=
1 + m2
y = 2 mr [1]
m∈R

8r 3
Eliminando m en [1] queda: x =
y 2 + 4r 2

8r 3
La asíntota es la recta x=0 dado que lim 2 = 0 . El área pedida es:
y → ±∞ y + 4r 2

1 1
3
+∞ 8r + ∞ 1 16 + ∞ + ∞
S =∫ = 2 dy = 16r 3 ∫ = 2 dy = 3 ∫ 2r dy = 8r 2 ∫ 2r dy =
y + 4r y + 4r
−∞ 2 2 2 2
 y   y 
0 0 0
r
  +1   +1
 2r   2r 

+∞
 y π 
= 8r  arctg  = 8r 2  − 0  = 4πr 2
2

 r 0 2 

9/9
www.eltemario.com Oposiciones Secundaria – Matemáticas
© Antonio J. Abrisqueta Valcárcel, 2002 Hoja de Problemas – Geometría II

143. a) Ecuación del cono de revolución que tiene su vértice en el punto O, su eje
coincide con OZ y b el ángulo en el vértice es recto.

b) Ecuaciones paramétricas de una curva c definida por un punto del cono


anterior que se mueve de tal manera que su proyección sobre el plano XY es la
espiral x=t cos t, y=t sen t (hélice cónica).

c) Hallar la superficie tangencial a c y comprobar que la binormal a la curva


en cada punto coincide con la normal en él a la superficie tangencial.

d) Demostrar que cada generatriz del cono corta a c en puntos a distancia


constante.

e) Sea c´ la proyección ortogonal de c sobre el cilindro x 2 + y 2 = a 2 .


Demostrar que la curvatura y la torsión de c´ son constantes para todos sus puntos,
y que las tangentes a c´ forman ángulo constante con la dirección del eje Z.

Demostración.

a) Las generatrices del cono situadas en el plano y=0 son x=z, x=- z al ser recto el
ángulo en el vértice.

Las ecuaciones paramétricas del cono de revolución con el vértice en O, eje OZ y


generatriz x=z son:

x = u ⋅ cos φ
y = u ⋅ senφ
z =u

y la ecuación implícita: x2 + y2 = z2 .

b) Las ecuaciones paramétricas de una curva sobre el cono son:

x = u (t ) cos φ(t )
y = u (t ) senφ(t )
z = u( t )

Si su proyección sobre el plano XY ha de ser x=t cos t, y=t sen t, necesariamente


u(t)=t y φ( t ) = t .

Las ecuaciones de c son así:


x = t cos t
c(t) y = t ⋅ sent
z =t

1/10
www.eltemario.com Oposiciones Secundaria – Matemáticas
© Antonio J. Abrisqueta Valcárcel, 2002 Hoja de Problemas – Geometría II

c) La superficie tangencial a una curva es la superficie reglada engendrada por las


tangentes de la curva.

Como c´(t)=(cost-tsent, sent+t, 1) es un vector tangente a c, la superficie


tangencial a c es S(t,v)=c(t) + v c´(t). Sus ecuaciones paramétricas son:

x = t cos t + v(cos t − t ⋅ sent )


y = t ⋅ sent + v( sent + t cos t )
z =t +v

Como St =c´(t)+v c´´(t) y Sv =c´(t), el vector normal a la superficie, es un vector


unitario en la dirección de St ^ Sv =v c´´(t)^c´(t).

Por otro lado c´(t) es un vector tangente a la curva y c´´(t) esta en el plano
determinado por los vectores tangente y normal a la curva. Si n(t) es el vector normal a
la curva sera c´´(t)=α(t)c´(t) + β(t)n(t).

El vector binormal a la curva, unitario en la dirección de c´(t)^n(t), es también


un vector unitario en la dirección de:

c´(t)^c´´(t)=β(t)c´(t)^n(t).

En suma, tanto la recta binormal a la curva, como la recta normal a la superficie


tienen como vector de dirección c´(t)^c´´(t), luego coinciden.

d) Excluyendo el origen, una generatriz del cono cortara a c en puntos obtenidos para
valores de t que se diferencian en múltiplos de 2π. La distancia pedida será la
distancia entre los puntos:

P1 (t cos t, sen t, t) ; P2 ((t + 2π) cos(t + 2π), ( t + 2π ) sen(t + 2π), t + 2π )

Luego d ( P1 , P2 ) = (2π cos t ) 2 + (2πsent )2 + (2π ) 2 = 2 2π .que efectivamente no


depende de t.

e) Como la curva c´ será x 2 + y 2 = a 2 , las ecuaciones de c´ son:


x = a cos t
y = a ⋅ sent
z =t
c´ es así una hélice circular.
c´(t)=(-a sent, a cos t, t) el vector tangente unitario, t es :
t=
1
(− a ⋅ sent , a cos t,1)
a2 +1
Sea s = t a 2 + 1 el parámetro arco.
dt 1
Entonces = t ´= 2 ( −a cos t , −asent ,0)
ds a +1

2/10
www.eltemario.com Oposiciones Secundaria – Matemáticas
© Antonio J. Abrisqueta Valcárcel, 2002 Hoja de Problemas – Geometría II

d 2t 1
= t´´= ( asent , −a cos t ,0)
ds 2 3

( a 2 + 1) 2
a2 a
Luego k = t´⋅t ´= 2
2
, y k =± 2
( a + 1) 2
a +1
−a a
cos t − sent
τ=
[
t , t ´, t]n
=
1 1 + a 2
1 + a 2
( a 2 + 1) 2
= 2
1
t´⋅t´ a a a +1
a +1 sent −
2
2
cos t a
3 3

( a + 1) 2
2
( a + 1) 2
2

que son constantes en todos los puntos.

Además t ⋅ (0,0,1) =
1
, luego el vector tangente forma en cada punto un
a2 + 1
1
ángulo constante con la dirección del eje Z, ángulo igual a arccos .
a2 +1

144. En una circunferencia dada se tienen dos puntos fijos, A y B. Un punto M se


mueve recorriendo la circunferencia. Determinar el lugar geométrico de: a) el
baricentro; b)el circuncentro; c) el ortocentro; d) el incentro, del triangulo móvil
AMB.

Demostración.

a) Sea H el baricentro del triangulo AMB. Si C es el punto medio de AB se verifica


1
que CH = CM . Cuando M recorre la circunferencia salvo A y B, H recorre
3
una circunferencia homotética a la dada en una homotecia de centro C y razón
1
, exceptuando los homotéticos de los puntos A y B.
3

b) Como el triangulo esta inscrito en la circunferencia, cualquiera que sea M, su


circuncentro es siempre el centro de la circunferencia dada, que es el lugar
pedido.

c) Sea H el ortocentro del triangulo AMB. Los ángulos AHB y AMB son
suplementarios ya que AH es perpendicular a BM y BH lo es a AM, luego H
recorre la circunferencia simétrica de la dada respecto de la recta AB.

d) Sea I el incremento del triangulo AMB. Supongamos que M recorre el arco BA


(en sentido positivo), y sea F el punto medio del arco AB. Consideremos la
circunferencia que pasa por los puntos A, I, y B y sea C su centro. C esta en la
mediatriz de AB y se verifica que ICB=2IAB. Como por otro lado
IFB=MAB=2IAB, y además F esta en la mediatriz de AB, se deduce que C=F.

3/10
www.eltemario.com Oposiciones Secundaria – Matemáticas
© Antonio J. Abrisqueta Valcárcel, 2002 Hoja de Problemas – Geometría II

Así cuando M recorre el arco BA, el incentro I recorre el arco BA de la


circunferencia cuyo centro es F, punto medio del arco AB.

Y, análogamente, cuando M recorre el arco AB, el incentro I recorre el arco AB


de la circunferencia cuyo centro es E, punto medio del arco BA.

145. Dada la superficie z = x 2 + y 2 y el plano x+2y-z-4=0 hallar:

a) Mínima distancia de la superficie al plano.


b) Ecuación de la recta r que contiene a los puntos que determinan dicha
distancia mínima.
c) Distancia de la recta a los ejes OZ y OX.

Demostración.

( )
a) La distancia del punto de la superficie, P x, y , x 2 + y 2 , al plano es

2
 x − 1  + ( y − 1)2 + 11
 
x + 2y − x 2 − y 2 − 4 (x 2
) (
− x + y2 − 2y + 4)  2 4
= =
1+ 4 +1 6 6
1 11
cuyo valor mínimo se obtiene para x = , y=1, siendo este valor mínimo
2 4 6

b) La recta r pasara por el punto  ,1,  y será perpendicular al plano. Su ecuación


1 5
2 4
es:
1 5 2x − y = 0
x− z−
2= y − 1 4⇔
= 7
1 2 −1 x+ z=
4

c) Un vector uniendo un punto de OZ con otro de r es  ,1,  , luego la distancia


1 5
2 4
de OZ a r es el valor absoluto de:

0 0 1
1 2 −1
1 5
1
2 4 =0
(0,0,1)^ (1, 2, −1)

Así la diferencia de Oz a r es 0.

Produciendo análogamente, la distancia de Oy a r, será el valor absoluto de

4/10
www.eltemario.com Oposiciones Secundaria – Matemáticas
© Antonio J. Abrisqueta Valcárcel, 2002 Hoja de Problemas – Geometría II

0 0 1
1 2 −1
1 5 7
1 −
2 4 4 7
= =−
(0,0,1)^ (1,2, −1) (− 1,0,−1) 4 2

ya que  ,1,  , es un vector uniendo un punto de OY con otro de r.


1 5
2 4
7
Así la distancia de OY a r es
4 2

146. Calcular la distancia entre los centros de la circunferencia inscrita y


circunscrita a un triangulo cualquiera en función de los radios.

Demostración.

Sean I el centro de la circunferencia inscrita al triangulo ABC y O el de la


circunferencia circunscrita; sean r y R sus radios. Sean respectivamente, D, E, F, los
puntos en los que la circunferencia inscrita es tangente a los lados BC, CA y AB. Sean P
y Q los extremos del diámetro de la circunferencia circunscrita que pasa por I.

Sea A´ el punto en el que la bisectriz AI corta al lado E&F, y análogamente B´ y C´


los puntos de corte de BI, CI con FD y DE, respectivamente.

Consideremos la inversión de centro en I y razón el radio r. Al ser AI y EF


perpendiculares, es en el triangulo rectángulo AEI:

r 2 = ( EI ) 2 = IA ⋅ IA´ luego en la inversión A y A´ son puntos homólogos.

Análogamente los son B y B´, C y C´.

Por tanto, en la inversión, la circunferencia circunscrita al triangulo ABC se


transforma en la circunferencia circunscrita al triangulo A´B´C´. Como A´, B´ y C´ son
respectivamente los puntos medios de los lados del triangulo DEF, el radio de la
circunferencia circunscrita al triangulo A´B´C´ es la mitad del de la circunferencia
circunscrita al triangulo DEF, esto es r/2.

Si P´ y Q´ son los puntos homólogos de P y Q, respectivamente, P´Q´ es un


diámetro de una circunferencia de radio r/2.

Como, designando por d a la distancia entre I y O es:

r 2 = IP´⋅IP = IP´( R − d ); r 2 = IQ´⋅IQ = IQ´( R + d )

5/10
www.eltemario.com Oposiciones Secundaria – Matemáticas
© Antonio J. Abrisqueta Valcárcel, 2002 Hoja de Problemas – Geometría II

r2 r2
resultan: IP´= ; IQ´=
R −d R+d

r2 r2
luego sumando r = ( IP´+ IQ´) = + con lo que:
R −d R+ d

d 2 = R 2 − 2rR = R( R − 2r ) ⇔ d = R( R − 2 r )

147. Hallar las superficies en las que se verifique que el punto medio del
segmento de normal comprendido entre la superficie y el plano XY este sobre:

z2 = x + y.

Demostración.

Si F(x,y,z) es la ecuación de superficie, ( F X , F Y , F Z ) es un vector


normal a la superficie.

El punto de intersección de la normal en el punto (x,y,z) con el plano XY resulta


de hacer Z=0 en:
X −x Y −y Z−z
= = ,
FX FY FZ
F F
luego sus coordenadas son ( x − z X , y − z Y ,0) .
FZ FZ

Las coordenadas del punto medio del segmento de normal son:

zF X zF z
(x − , y − Y , ).
2 FZ 2 FZ 2

Sustituyendo en z 2 = x + y se obtiene:

z2 zF zF
= x − X + y − Y ⇔ 2 zFX + 2 zFY + ( z 2 − 4( x + y )) FZ = 0
4 2 FZ 2 FZ

ecuación en derivadas parciales cuyo sistema característico es:

dx dy dz
= = 2
2 z 2 z z − 4( x + y )
dx dy
= da la integral primera G1 ( x, y , z ) = x − y = C1 (1).
2z 2z

dx + dy dz
Por otro lado = 2 , luego haciendo x+y=t, resulta:
4z z − 4( x + y )

6/10
www.eltemario.com Oposiciones Secundaria – Matemáticas
© Antonio J. Abrisqueta Valcárcel, 2002 Hoja de Problemas – Geometría II

dt dz
= 2 . En consecuencia: ( z 2 − 4t )dt = 4 zdz.
4 z z − 4t
dz
Resolvamos pues la ecuación diferencial z 2 − 4 z = 4t .
dt

Haciendo el cambio z 2 = u se obtiene: u − 2u´= 4t .

Esta ecuación es lineal y su integral general es:


t
t
u = C 2 e 2 + 8(1 + ) .
2

Deshaciendo el cambio obtenemos como una segunda integral primera:

z 2 − 4( x + y + 2)
G2 ( x, y , z ) = x+ y
= C2 (2)
2
e

De (1) y (2) obtenemos como ecuación de la superficie buscada:

F ( x, y , z ) = φ(G1 , G 2 ) , donde φ es una función arbitraria.

148. Sean ρ,ψ, z coordenadas cilíndricas en el espacio. Se considera la


superficie tórica T de ecuación:

z 2 + ( ρ − R) 2 = r 2 (siendo R>r>0)

A cada punto de T se le hace corresponder un ángulo ω que satisface

ρ− R z
cos ω = , senω = .
r r

Designaremos por V el conjunto de los múltiplos de 2ππ por los números enteros.
Se pide:
a) Probar que ψ y ω son coordenadas curvilíneas ortogonales en T.
Expresar mediante ellas ds 2 así como el elemento de área ds.

b) Probar que las condiciones: ψ *-ω ω∈ V, ω *-ψψ ∈ V, establecen una


correspondencia biunívoca de T en si mismo: (ψ, ω) → (ψ
ψ *,ω
ω *) en las que
ciertas circunferencias se transforman en otras circunferencias. ¿Es
conforme esta transformación

c) Probar que el lugar geométrico de los puntos que se corresponden


consigo mismo en esa correspondencia es una curva cerrada en T. ¿En
cuantas regiones descompone a T esa curva?. Mostrar el aspecto de
dicha curva en un croquis de T realizado en cualquier perspectiva.

7/10
www.eltemario.com Oposiciones Secundaria – Matemáticas
© Antonio J. Abrisqueta Valcárcel, 2002 Hoja de Problemas – Geometría II

d) Calcular el área de las regiones en que queda dividida T por la curva


mencionada y por su simétrica respecto del plano z=0.

Demostración.

a) Al cortar la superficie tórica por un plano pasando por el eje OZ y perpendicular


al plano z=0, se obtiene una circunferencia en la que ω es el ángulo central. Las
ecuaciones paramétricas de T respecto de ψ y ω son por tanto:

x = ( R + r cos ω) cos ψ
y = ( R + r cos ω) sen ψ
z = rsen ω

Los vectores tangentes Pψ y Pω en cada punto son:

Pψ = ( −R + r cos ω) sen ψ, ( R + r cos ω) cosψ,0)


Pω = ( −rsenω ⋅ senψ, −rsenω ⋅ senψ, r cos ω)

Los coeficientes de ds 2 o primera forma cuadrática fundamental


I ( λPψ + µPω) = Eλ2 + 2 Fλµ + Gµ 2 son :

E = Pψ Pψ = ( R + r cos ω) 2
F = Pψ Pω = 0
G = Pω Pω = r 2

Así ds 2 = ( R + r cos ω) 2 λ2 + r 2 µ 2 .

Además, de F = Pψ Pω = 0 , se desprende que ψ y ω son coordenadas curvilíneas


ortogonales.

El elemento de área ds viene dado por :

ds = EG − F 2 dψdω = r ( R + r cos ω)dψdω .

b) Si f es la transformación se tiene que

f(ψ,ω)=(ψ*,ω*)=(ω+2kπ,ψ+2hπ), donde k,h ∈ Z .

Es evidente que f esta bien definida, pues es irrelevante la elección de k y h.

f es suprayectiva pues dado un punto P(ψ*,ω*), es:

f(ψ*,ω*)=(ψ*+2kπ,ω*+2hπ)=P

8/10
www.eltemario.com Oposiciones Secundaria – Matemáticas
© Antonio J. Abrisqueta Valcárcel, 2002 Hoja de Problemas – Geometría II

f es inyectiva, pues f(ψ,ω)=f(ψ´, ω´) ⇒ los puntos de coordenadas

(ω+2kπ,ψ+2hπ) y ( ω´+2k´π,ψ´+2h´π) son iguales

luego ( ω+2kπ)-(ω´+2k´π) ∈ V y (ψ+2hπ)-(ψ´+2h´π) ∈ V ,

luego también ω-ω´ ∈ V y ψ -ψ´ ∈ V , con lo que coinc iden los puntos de coordenadas
(ψ,ω) y (ψ´,ω´).

En suma f es una transformación biunívoca de T en si mismo.

Las circunferencias obtenidas cortando la superficie tórica por planos horizontales o


por planos verticales pasando por el eje OZ son , respectivamente, las circunferencias
ω=cte, ψ=cte. Por f estas circunferencias se transforman en ψ*=cte, ω*=cte,
respectivamente, es decir se transforman las de un tipo en las del otro y recíprocamente.

Si la transformación es conforme, los coeficientes de ds 2 en cada punto respecto de


(ψ,ω) y de (ψ*,ω*) han de ser proporcionales. Análogamente al apartado a) tendremos:
E*=(R+rcosω*) 2 , F*=0, G*= r 2

Como G*=G= r 2 , el coeficiente de proporcionalidad habria de ser 1, pero como


E*=(R+rcosω*) 2 = (R+rcosψ) 2 ≠ (R+rcosω)=E, la transformación no es conforme.

c) Para que un punto se corresponda con el mismo ha de ser ω-ψ ∈ V que es así la
ecuación del lugar.

En coordenadas cartesianas la ecuación del lugar se obtiene haciendo ω=ψ en las


ecuaciones de la superficie:

x = ( R + r cos ψ) cosψ
y = ( R + r cos ψ) sen ψ
z = rsen ψ

Es claramente una curva cerrada, pues para ψ+2kπ se obtiene el mismo punto que
para ψ.

La curva descompone a T en una única región. Ello se ve claramente observando la


representación de la curva en el plano (ψ,ω). Dos puntos cualesquiera se pueden unir
por una curva que no corta a ψ=ω. (téngase en cuenta que ω=2π ⇒ ω = 0 , y
ψ = 2π ⇔ ψ = 0 .

d) La representación de la curva simétrica respecto del plano z=0

x = ( R + r cos ψ) cosψ
y = ( R + r cos ψ) senψ
z = −rsen ψ

9/10
www.eltemario.com Oposiciones Secundaria – Matemáticas
© Antonio J. Abrisqueta Valcárcel, 2002 Hoja de Problemas – Geometría II

en el plano (ψ,ω), es el segmento que une los puntos (0,2π) y (2π,0). Teniendo en
cuenta que los puntos para ω=2π se identifican con los puntos para ω=0 y análogamente
se identifican los puntos para ψ=2π con los puntos ψ=0, resulta que ambas curvas
dividen a T en dos regiones. Como ds = r ( R + r cos ω )d ψ d ω , las áreas S1 y
S 2 buscadas son:

π ω π 2π
S1 = ∫ ∫ r ( R + r cos ω) dψdω +∫ ∫ r ( R + r cos ω) dψdω +
ω =0 ψ =0 ω =0 ψ = 2π −ω

2π 2 π −ω 2π 2π
+∫ ∫ r ( R + r cos ω) dψdω + = ∫ ∫ r ( R + r cos ω) dψdω =
ω =π ψ =0 ω =π ψ =ω

π 2π
= 2∫ r ( R + r cos ω)ωdω + 2∫ r ( R + r cos ω)( 2π − ω) dω =
0 π

 Rω2 π
2π Rω2


= 2r ( + rωsen ω + r cos ω) + 2π( Rω + rsenω) π − ( + rωsen ω + r cos ω)  =
 2 0
2 π 

π2 π2
= 2r ( R − 2r + 2 Rπ 2 − 2Rπ 2 + R − 2 r = 2r ( Rπ 2 − 4r ).
2 2

π 2 π −ω 2π ω
S2 = ∫ ∫ r ( R + r cos ω) dψdω + = ∫ ∫ r ( R + r cos ω) dψdω =
ω =0 ψ =ω ω =π ψ = 2 π −ω

π 2π
= ∫ r ( R + r cos ω)( 2π − 2ω) dω + ∫ r ( R + r cos ω)( 2ω − 2π) dω =
0 π

( 2πRω + 2πrsen ω − Rω2 − 2 rωsenω − 2r cos ω π + 


r =
0
 2π 
 + ( Rω + 2 rωsenω + 2r cos ω − 2πRω − 2πrsen ω π 
2

= r ( 2 Rπ 2 − Rπ 2 + 2 r + 2r + 4Rπ 2 + 2r − 4Rπ 2 − Rπ 2 + 2r + 2 Rπ 2 ) = 2 r ( Rπ 2 + 4r )

10/10
www.eltemario.com Oposiciones Secundaria – Matemáticas
© Antonio J. Abrisqueta Valcárcel, 2002 Hoja de Problemas – Geometría III

x2 y 2
149. Dada la elipse + = 1 , si tomamos el extremo B de ordenada positiva del
a2 b2
eje menor como centro, se describe una circunferencia de radio igual a dicho eje
menor, cortara a la elipse en dos punto P y P´. Determinar la relación que debe
existir entre los semiejes de la elipse para que el triangulo BPP´ sea equilátero.

Demostración.

Una condición necesaria y suficiente para que el triangulo PBP´ sea equilátero es
que el ángulo HBP´ sea 30º.

En consecuencia, las coordenadas de P´ son:

x=2b cos60=b
y=b-2b sen60=b(1- 3 )

puesto que P´ ha de estar en la elipse, se verificará:

b 2 b 2 (1 − 3 ) 2 b2
+ = 1 ⇒ = 2 3 − 3 que es la relación buscada.
a2 b2 a2

150. Hallar la ecuación de la superficie (conoide) formada por las rectas que son
paralelas al plano z=0 y se apoyan en la recta:

 y = 0  y=6
r x z y la curva P 
 4 + 2 = 1 4 − x = z
2

Calcular el volumen situado en el octante x ≥ 0 , y ≥ 0 , z ≥ 0 y limitado por la


superficie hallada, los planos coordenados y el plano y=6.

Demostración.

(4,0,0 ) es
un punto y (2,0,1) un vector de dirección de la recta r; así un punto
genérico de r es ( 4 − 2λ,6, µ).

Un punto genérico de la curva P es ( 4 − µ 2 ,6, µ)

Un vector de dirección de las rectas que apoyan en P y r es ( ( −2λ + µ 2 ,−6,−λ − µ)


Al estar intersectado por las rectas que son paralelas a z=0, ha de ser, λ − µ = 0 .

La ecuación del conoide buscada es:

1/8
www.eltemario.com Oposiciones Secundaria – Matemáticas
© Antonio J. Abrisqueta Valcárcel, 2002 Hoja de Problemas – Geometría III

x = 4 − 2λ + ρ( −2λ + λ2 ) 

y = −6 ρ  ⇔ 6 x − 24 + 12 z − 2 yz + yz = 0
2
(1)
z =λ 

Sea S O la parte del conoide situada en el primer octante y entre los planos y=0 e
y=6.

Si ( xO , y O , z O ) ∈ S O se tiene evidentemente, que 0 ≤ y O ≤ 6.

De (1), despejando x, se obtiene:

1
x= ( 2 − z )(12 + yz ) (2)
6

Si y O ∈ [0,6] y ( xO , y O , z O ) ∈ S O , por (2) se obtiene que z O ∈ [0,2 ] , ya que se ha de


verificar x ≥ 0 y z ≥ 0 .

Además para cada (y,z) ∈ [0,6 ] × [0,2] existe un único x, el dado por (2), tal que
( x, y , z ) ∈ S O .
En consecuencia el volumen V buscado es:

2
− 12 z + 24 + 2 yz − z 2 y 6 yz 2 z 3 y 
V =∫ ∫ dzdy = ∫  − z 2 + 4 z +
6 2
−  dy =
6  6 18  0
0 0 0

6
2y  y2
= ∫ (4 +
6
)dy =  4 y +  = 28
9  9 0
0

151. De un triangulo ABC rectángulo en A, se conocen el cateto Ac y la distancia


de B al incentro. Construir el triangulo.

Demostración.

En el triangulo BHI:

BI 2 = BH 2 + IH 2

Designando por r al radio de la circunferencia inscrita:

BI 2 = BH 2 + r 2 (1)

Es evidente que el perímetro 2p de ABC es:

2 p = 2 BH + 2CH + 2r = 2BH + 2CM + 2 MA = 2 BH + 2 AC

2/8
www.eltemario.com Oposiciones Secundaria – Matemáticas
© Antonio J. Abrisqueta Valcárcel, 2002 Hoja de Problemas – Geometría III

Luego: BH = p − AC y 2p = 2BH + 2CH + 2r = 2BC + 2r ⇒ r = p − BC

Sustituyendo en (1), y haciendo a=BC, b=AC, c=AB:


a+b+ c a+b+ c
BI 2 = ( p − AC) 2 + ( p − BC ) 2 = ( − b) 2 + ( − a) 2 =
2 2

a + c − b 2 b + c − a 2 a 2 + b 2 + c 2 − 2ab a 2 + b 2 + a 2 − b 2 − 2ab
=( ) +( ) = = = a(a − b) (2).
2 2 2 2

b
De todo lo anterior, si construimos una circunferencia de radio y trazamos la
2
tangente SR=BI, queda:

SR 2 = RP ⋅ PQ = ( RQ − PQ) ⋅ RQ = ( RQ − b) ⋅ RQ

es decir :
BI 2 = RQ ( RQ − b ) comprando con (2) queda RQ=a.

Conocidos la hipotenusa y un cateto, la construcción del triangulo es inmediata.

152 En el espacio euclídeo, se considera una recta variable s, que se apoya sobre
dos rectas fijas r y r´ en los puntos R y R´ respectivamente, de forma que s y r son
ortogonales. Llamando V al ángulo que forman los planos determinados por (r,R´)
y (r´,R), comprobar que d(R,R´)x tgV es constante. (d(R,R´) significa distancia
entre R y R´.

Demostración.

Sea u un vector de dirección de r, v un vector de dirección de r´ y w el vector


RR´. Supondremos además, por comodidad, u y v elegidos de tal manera que el ángulo
que forman, (u,v), verifique 0 < (u, v ) < π / 2 .

a) Supongamos 0<(u,v)<π/2.

Un vector perpendicular al plano determinado por r y R´ es u ∧ v . Un vector


perpendicular al plano determinado por r´ y R es v ∧ w .

Se tiene:

(u ∧ w) ⋅ (v ∧ w) = u ∧ w v ∧ w cos(u ∧ w, v ∧ w) (1)

(u ∧ w) ∧ (v ∧ w) = u ∧ w v ∧ w sen (u ∧ w, v ∧ w) (2)

por las propiedades del producto vectorial sabemos que:

(u ∧ w) ⋅ (v ∧ w) = (u ⋅ v )( w ⋅ w) − (u ⋅ w)( w ⋅ v)

3/8
www.eltemario.com Oposiciones Secundaria – Matemáticas
© Antonio J. Abrisqueta Valcárcel, 2002 Hoja de Problemas – Geometría III

(u ∧ w) ∧ (v ∧ w) = [u, w, w]v − [u , w, v]w

Al ser u ⋅ w =0, por ser r y s perpendiculares, y[u , w, w] = 0 resulta:

(u ∧ w) ⋅ (v ∧ w) = (u ⋅ v ) w = u v cos(u , v ) w
2 2
(3)

(u ∧ w) ⋅ ( v ∧ w) = [u, w, v] w (4)

Sustituyendo (3) y (4) en (1) y (2) respectivamente:

u v cos(u , v ) w = u ∧ w v ∧ w cos( u ∧ w, v ∧ w)
2

[u , v, w] w = u ∧ w v ∧ w sen (u ∧ w, v ∧ w)
por tanto el seno y el coseno del ángulo (u ∧ w, v ∧ w) son positivos.

Así (u ∧ w, v ∧ w) , esta comprendido entre 0 y π/2 y coincide con ello con V.

Teniendo en cuenta que V= (u ∧ w, v ∧ w) obtenemos de (1) y (2):

(u ∧ w) ∧ ( v ∧ w) [u, w, v ] w [u, w, v ]
tgV = = = (5)
(u ∧ w) ⋅ (v ∧ w) v u cos(u , v ) w
2
v u cos( u, v) w

por otro lado la distancia entre la rectas r y r´, d, viene dada por:

[u , v, w] [u, v, w]
d= = (6).
u ∧v v u sen (u, v )

De (5) y (6) se obtiene que:

[u , v, w] dsen(u , v )
d ( R, R´)tgV = w tgV = = = dtg (u , v ) de donde al ser
v u sen(u , v ) cos(u , v)
ángulo (u,v) constante, lo es d(R,R´)tgV.

b) Si (u,v)=0 ryr´ son paralelas y V=0. en consecuencia dtgV=0.

c) Si (u,v)= π/2, V es π/2 y no esta definido el producto d tgV.

153. Dadas las circunferencias C1 y C2 , cuyas ecuaciones respectivas son:


x 2 + ( y − 2) 2 = 4
( x − 4) 2 + y 2 = 16

4/8
www.eltemario.com Oposiciones Secundaria – Matemáticas
© Antonio J. Abrisqueta Valcárcel, 2002 Hoja de Problemas – Geometría III

se considera una inversión cuyo polo es el origen de coordenadas y la


circunferencia de puntos dobles tiene de radio la cuerda común de C1 y C2 .
Determinar analítica y gráficamente:

a) Las figuras inversas de C1 y C2 .


b) El ángulo bajo el que se cortan ambas circunferencias.
c) La figura inversa de la parte común a los círculos limitados por C1 y
C2 .

Demostración.

a) Por pasar C1 y C2 por el centro de inversión son rectas que no pasan por el
centro.
La recta r1 inversa de C1 pasara por el punto A, doble en la inversión, y será
perpendicular al radio o1 o .
Análogamente, la recta r2 inversa de C2 pasará por el punto A y será
perpendicular al radio o 2 o .

b) Por conservar la inversión los ángulos, el ángulo de corte es el que forman


las rectas r1 y r2 , esto es π/2 radianes.

c) En la inversión, los puntos interiores a C1 se transforman en puntos de


semiplano determinado por r1 que no contiene a o y recíprocamente.
Análogamente, los puntos interiores de C2 son homólogos con los del
semiplano determinado por r2 que no contiene a o.

La figura inversa pedida es, por lo tanto, la región angular limitada por r1 y
r2 opuesta a la que contiene a o.

Analíticamente:

Como C1 y C2 pasan por o(0,0), las coordenadas de A son la solución distinta


de (0,0) del sistema:

x 2 + ( y − 2) 2 = 4 x2 + y2 − 4y = 0 8 16
⇔ 2 ⇒ A( , )
( x − 4) 2 + y 2 = 16 x + y − 8y = 0
2
5 5

la razón de la inversión es (d(O, A)) 2 = 64 / 5 y el polo es O, luego son puntos


64 x 64 y
homólogos (x,y) y , ).
5 x + y 5 x + y2
2 2 2

a) Sea (x,y) el homologo de (α, β) ∈ C1 , (α, β) ≠ (0,0) . Es entonces:


64 x 64 y
(α, β) = ( , )
5 x +y
2 2
5 x + y2
2

5/8
www.eltemario.com Oposiciones Secundaria – Matemáticas
© Antonio J. Abrisqueta Valcárcel, 2002 Hoja de Problemas – Geometría III

2 2
 64 x   64 y  64 y
luego  2 
+ 2 
−4 =0⇔
5 x +y  5 x +y  5 x + y2
2 2 2

64 1  64 x 2 64 y 2  64 16
⇔ 2 
+ − 4 y = 0 ⇔ − 4y = 0 ⇔ y =
5 x +y 5 x +y
2 2 2
5 x +y
2 2
 5 5

que es la ecuación de una recta, r1 , inversa de C1 .

Análogamente, la figura inversa de C2 es la recta de la ecuación x=8/5.

b) Es el angulo que forman sus respectivas figuras inversas, las rectas y=16/5 y
x=8/5, es decir π/2 radianes.

c) Procediendo de forma análoga al apratdo a), si (α, β) es interior a C1 , esto


es tal que α2 + β 2 − 4 β < 0 su homologo (x,y) verifica y>16/5.
Análogamente si (x,y) es el homologo de (α, β) , interior a C2 , (x,y) verifica
x>8/5.
 8 16 
La figura inversa pedida es: ( x, y) ∈ R 2 x > , y > 
 5 5

154. Se toma un punto D en el lado Ab de un triangulo ABC dado. La recta CD se


corta en E a la paralela al lado BC trazada por A.
a) Determinar la posición del punto D para que la suma de las áreas de los
triangulo BCD y AED sea mínima.
b) Calcular el valor de este área.

Demostración.

a) Sean h=MP y z=DB, donde la recta MP es la perpendicular pasando por D a


las rectas paralelas EA y BC.

Los triángulos BCD y AED son evidentemente semejantes, luego


EA ED AD DM
= = = (1)
BC DC DB DP

La suma de las áreas de dichos triángulos es:

MD DP
S = EA + BC (2)
2 2

designado por a,b,c los lados del triangulo ABC queda:

EA DM AD c − z c−z
= = = ⇒ EA = a ( )
BC DP DB z z

6/8
www.eltemario.com Oposiciones Secundaria – Matemáticas
© Antonio J. Abrisqueta Valcárcel, 2002 Hoja de Problemas – Geometría III

c−z c−z a( c − z ) h(c − z )


DM = DP = ( h − DM ) ⇒ EA = , DM =
z z z c

Sustituyendo en (2) queda:

a (c − z ) h( c − z ) h − DM ah (c − z ) 2 ahz ah ( c − z ) 2
S= +a = + = ( + z) =
z 2c 2 2cz 2c 2c z

ah c2
= ( 2 z − 2c + )
2c z

La función z → S ( z ) esta definida para z variando en el intervalo (0,c).


ah c2 ah 2 z 2
Derivando : S = (2 − 2 ) =
2c z 2c z 2
c
En consecuencia: para todo z perteneciente al intervalo ( , c ) se verifica
2
c
que S´>0. análogamente si z varia en el intervalo ( 0, ) es S´<0.
2
c
Luego en z= la función z → S ( z ) presentan su mínimo absoluto.
2
c
Así el punto D ha de estar a la distancia de B.
2
b) El área mínima es:

ah 2c 2c 2 ah ah
S= ( − 2c + )= ( c 2 − 2c + 2 ) = ( c 2 − 2c ) = ah ( 2 − 1) = 2( 2 − 1) S ABC
2c 2 c 2c 2c

155. Dada una pirámide regular de base cuadrada y altura igual al doble del
lado de la base, se pide determinar la relación de volúmenes (superior e
inferior) en que queda dividida por un plano que pasando por un lado de la
base corta a la pirámide según un polígono cuyo perímetro sea mínimo.

Demostración.

La selección de la pirámide por un plano que pasa por el lado CD es el


trapecio CDEF. El perímetro de CDEF será mínimo cuando, considerando el
desarrollo lateral de la pirámide, sea mínima la longitud de la poligonal DEFC y
ello supone que esta poligonal es un segmento rectilíneo.
Por la semejanza de los triangulo OEF y OAB:
EF ON OD cos 3α 4 cos 3 α − 3 cos α
= = = = 4 cos 2 α − 3 y como
AB OM OA cos α cos α

7/8
www.eltemario.com Oposiciones Secundaria – Matemáticas
© Antonio J. Abrisqueta Valcárcel, 2002 Hoja de Problemas – Geometría III

1
AB
AM AM 2 2
senα = = = =
OA AH 2 + OH 2 3 6
AB
2

7
haciendo AB=a, resulta EF = ( 4 cos 2 α − 3)a = (1 − 4 sen 2α) a = a.
9

para obtener la altura, EP=h, del prismatoide ABCDEF:

OH − EP EF 14 4
= ⇒ ( 2a − h ) = a ⇒ h = a
OH AB 9 9

1 7 8 1
Como GJ= ( a + a) = a , JK= a , por la formula de los tres niveles el
2 9 9 2
volumen del prismatoide ABCDEF es:

1 14 4 50 3
h( S ABCD + 4 S GJKL ) = a (a 2 + 4 a 2 ) = a
6 69 9 243

2 3
Al ser el volumen de la pirámide a , la relación de volúmenes pedida es:
3

2 3 50 3
a − a
3 243 56
= .
50 3 25
a
243

8/8
www.eltemario.com Oposiciones Secundaria – Matemáticas
© Antonio J. Abrisqueta Valcárcel, 2002 Hojas de Problemas – Geometría IV

156. Construir un triángulo conocido el lado a, la mediana relativa al lado b y la


altura relativa al lado a.
Tomando como lados de un rectángulo los lados a, b del triángulo anterior
calcular los lados del rectángulo circunscrito a aquel rectángulo y cuya área sea
máxima.

RESOLUCION.-

a) Con centro en el vértice B del lado a se traza una circunferencia de radio la mediana
mb.

Se traza la paralela al lado a y distancia la altura ha.

La paralela media de estas dos rectas contiene el punto medio M del lado b, siendo
M el punto de corte de esta recta con la circunferencia anteriormente trazada.

El punto de corte de la recta que pasa por C y M con la paralela al lado a a distancia
ha nos da el tercer vértice, A, del triángulo.

Como existen dos posibles puntos M, (una recta corta a una circunferencia en dos
puntos), existen dos soluciones posibles distintas.

b) Designemos por c y d los lados


del rectángulo circunscrito.

Sea á el ángulo que forman los


lados a y c.

c1 = a cos α , d 1 = a sin α
c 2 = a sin α , d 1 = b cos α
c = a cos α + b sin α
d = a sin α + b cos α

El área del rectángulo


circunscrito es:

A = cd = ( a cos α + b sin α)( a sin α + b cos α) = ( a 2 + b 2 ) sin αcos α + ab =


a2 + b2
= sin 2α + ab
2
π
cuyo máximo se obtiene para el máximo de sin 2α , es decir, para 2α = , o bien,
2
π 2
α= . Entonces c = d = ( a + b ) .
4 2

1/8
www.eltemario.com Oposiciones Secundaria – Matemáticas
© Antonio J. Abrisqueta Valcárcel, 2002 Hojas de Problemas – Geometría IV

157. Transformar un triángulo en otro equivalente que sea semejante a un


triángulo dado.

RESOLUCIÓN.-

Primer método.-

Dado un triángulo ABC, construyendo el segmento AD de longitud la suma de AB y


CH (altura desde C a AB), la circunferencia de diámetro AD, y su corte, P, con la
perpendicular trazada por el punto de AD que disca AB de A determina un triángulo
cuya altura PB es el lado de un cuadrado de área el doble de la del triángulo ABC.

Aplicando esta
construcción al triángulo
dado, supongamos que es
ABC, y al que queremos
transformar, se obtienen,
respectivamente unos
segmentos PB y QB tales que
los cuadrados construidos
sobre ellos tienen doble área
que los triángulos de partida.

Llevando ahora QB sobre la recta PB y trazando por Q las paralelas a PA y PD se


encuentran los puntos A’ y D’.

El triángulo A’BC’, semejante al ABC, de


base A’B y altura un segmento de longitud BD’ es
claramente el triángulo buscado pues su área es
también la mitad de la del cuadrado de lado QB.

Segundo método.-

Sea S el área del triángulo de partida. Sea ABC el triángulo dado.

El triángulo que buscamos es el CB’A’, donde:

2/8
www.eltemario.com Oposiciones Secundaria – Matemáticas
© Antonio J. Abrisqueta Valcárcel, 2002 Hojas de Problemas – Geometría IV

a ' h ' = 2S 

a' a 
=
h' h 

Las condiciones anteriores


determinan a’ y por consiguiente B’.
Trazando una paralela por B’ al lado
AB obtendremos A’ en su
intersección con AC.

158. En un plano se dan un punto P y dos rectas r1 y r2 . Trazar por P una recta
que corte en A y B a las rectas dadas de forma que PA·PB = k 2 , siendo k un
segmento dado.

RESOLUCIÓN.-

Consideremos la inversión de centro P y razón k2 . Si en esta inversión la


circunferencia inversa de r1 corta a r2 en un punto B, el inverso de B, es decir A, está en
r1 y se verifica que PA·PB = k 2 .

Se traza C1 inversa de r1 en la inversión de centro P y razón k2 ; C1 cortará a r2 en dos


puntos (o en uno); cada uno de ellos unido con P determina una recta solución del
problema.

Es claro que si C1 no corta a r2 el problema no tiene solución.

159. Considérese en el plano tres pares de puntos distintos (p1 , q1 ), (p2 , q2 ), (p3 ,
q3 ). Demostrar que si por cada dos pares de puntos pasa una circunferencia o bien
pasa un circunferencia por los seis puntos o las rectas r1 = p1 q1 , r2 = p 2 q2 ,
r3 = p3 q3 son concurrentes.

RESOLUCIÓN.-

Sea C1 2 la circunferencia que pasa por p1 , q1 , p2 , q2 y sean análogamente C1 2 y C2 3 .

Si C1 2 = C1 2 = C2 3 , pasa una circunferencia por los seis puntos y no hay nada que
probar.

Es obvio que si dos circunferencias coinciden, entonces las tres coinciden.

Supongamos, por consiguiente, que las tres son distintas. En ese caso:

r1 es el eje radical de C1 2 y C1 3 .
r2 es el eje radical de C1 2 y C2 3 .

3/8
www.eltemario.com Oposiciones Secundaria – Matemáticas
© Antonio J. Abrisqueta Valcárcel, 2002 Hojas de Problemas – Geometría IV

r3 es el eje radical de C1 3 y C2 3 .

Así, si p es el punto de intersección de r1 y r2 , p tiene la misma potencia respecto de


las circunferencias C1 3 y C2 3 , luego está también en r3 . En suma r1 , r2 y r3 son
concurrentes.

160. El cubo de lado 6 cm, cuya base A1 A2 A3 A4 está situada en un plano


horizontal y cuya base superior B1 B2 B3 B4 no tiene superficie, está lleno de agua. Se
inclina subiendo el vértice A2 un centímetro, el A4 dos centímetros y el A3 se queda
en el suelo.

Se pide:
a) Altura a que queda el vértice A1 .
b) Volumen de agua que se derrama.

B1 B2

B4 B3

A1 A2

A4 A3

RESOLUCIÓN.-

a) Observando que el punto medio de A1 A3 y A2 A4 coinciden, la cota de A1 es la


suma de las cotas de A2 y A4 (por ser cero la cota de A3 ); es decir la altura de A1 es 3
cm.

b) La superficie del agua forma un rectángulo cuyos lados miden


B' 2 B3 = 6 2 + 12 = 37cm
B3 B' 4 = 6 2 + 2 2 = 40cm
Su área es por tanto 2 370cm 2 .

El volumen del agua derramada es por ello la mitad del volumen del prisma oblicuo
de bases rectángulos de área 2 370cm 2 y su altura 3 cm:

1
Volumen derramado = 3·2 370 = 3 370cm 3 .
2

4/8
www.eltemario.com Oposiciones Secundaria – Matemáticas
© Antonio J. Abrisqueta Valcárcel, 2002 Hojas de Problemas – Geometría IV

161. Sea una circunferencia de centro O y en ella el diámetro AB. Se traza por B
una recta que corta a la circunferencia en M. Sobre dicha recta se toma el
segmento
MC = MB. Las rectas OC y AM se cortan en H. Hallar el lugar geométrico de los
puntos H cuando BM gira alrededor de B y construir la figura nomotética del
3
lugar geométrico hallado en la homotecia de centro A y razón − .
2
RESOLUCIÓN.-

Dado que MC = MB es BC = 2BM, y el punto C describe una circunferencia, c1 ,


nomotética a la dada, c, en una homotecia de centro en B y razón 2; su centro es el
punto A y su radio AB.

Observando el triángulo ABC resulta que AM y CO son medianas, ya que MC =


MB y OA = OB, luego H es el baricentro del triángulo.

2
Se tiene así que AH = AM y, por tanto, que H describe una circunferencia, c2 ,
3
2
nomotética de c en una homotecia de centro en A y razón .
3

2
La figura nomotética de c2 en la homotecia de centro A y razón − , es, por
3
 2  3 
composición de homotecias del mismo centro  · −  = −1 , una circunferencia, c3 ,
 3  2 
simétrica a la dada, c, en una simetría central de centro en A (u nomotética de centro en
A y razón -1).

162. Dado el conjunto de circunferencias representadas por la ecuación


x 2 + y 2 − 2 aλx + λ2 − b 2 = 0
donde a, b son constantes, λ ∈ IR un parámetro, se pide:
a) La ecuación del lugar geométrico de los puntos de contacto de las tangentes
a estas circunferencias, paralelas al eje OX.
b) La naturaleza de la curva.

RESOLUCIÓN.-

a) Al ser las tangentes paralelas al eje OX, los puntos de contacto son los extremos
del diámetro paralelo al eje OY.
x 2 + y 2 − 2 aλx + λ2 − b 2 = 0 ⇔ ( x − aλ) 2 + y 2 = b 2 + a 2 λ2 − λ2

luego las coordenadas de los puntos de contacto verifican


 x = aλ
 2 [1]
y = b + a λ − λ
2 2 2 2

5/8
www.eltemario.com Oposiciones Secundaria – Matemáticas
© Antonio J. Abrisqueta Valcárcel, 2002 Hojas de Problemas – Geometría IV

que son ecuaciones paramétricas del lugar.

Si a = 0 , el lugar se reduce al segmento del eje OY limitado por los puntos (0, -b) y
(0, b).

Si a ≠ 0 , eliminando ë en [1], se obtiene

(1 − a) 2 x 2 + a 2 y 2 = a 2 b 2 [2]
que constituye la ecuación del lugar.

b) Si a = 0 se trata de un segmento.
Si a ≠ 0 y b = 0 entonces el lugar es:

a 2 −1
- Para a 2 > 1 , el par de rectas y = ± x.
a
- Para a 2 = 1 , la recta y = 0.
- Para a 2 < 1 , se reduce al punto (0, 0).
Si a ≠ 0 , a 2 ≠ 1 y b ≠ 0 , la ecuación [2] se puede escribir como:
x2 y2
+ =1
a2 b2 b2
1− a2
por lo que:
- Para a 2 > 1 se trata de una hipérbola.
- Para a 2 < 1 se trata de una elipse.

Si a 2 = 1 y b ≠ 0 se obtienen las rectas y = ±b .

163. Se supone en IR 2 un sistema de referencia ortogonal OXY. Hallar la


envolvente de los ejes radicales de dos circunferencias C y C’, donde la
circunferencia C es fija y tiene su centro en el eje OY, y la circunferencia C’ es
variable con centro en OX y tangente a la parábola y = 2x 2 .

RESOLUCIÓN.-

Sea x 2 + y 2 − 2by = m la ecuación de la circunferencia C, con centro en (0, b), y


m + b2 > 0 .

Supondremos que la circunferencia C’ es tangente a la parábola en un punto distinto


del origen. Sea x 2 + y 2 − 2 ax = k la ecuación de C’, con centro en (a, 0).

Eliminando y en el sistema
x 2 + y 2 − 2ax = k 
 ⇒ x + 2 x (1 − a) − k = 0
2

y = 2x
2


6/8
www.eltemario.com Oposiciones Secundaria – Matemáticas
© Antonio J. Abrisqueta Valcárcel, 2002 Hojas de Problemas – Geometría IV

queda una ecuación de 2º grado cuyas raíces han de ser iguales para que C’ y la
parábola sean tangentes. Así, ha de ser k = −(1 − a ) 2 .

La ecuación de la familia de las circunferencias C’ es:


x 2 + y 2 − 2 ax + (1 − a) 2 = 0

La ecuación del eje radical de C y C’ es:


( x 2 + y 2 − 2by − m) − ( x 2 + y 2 − 2ax + (1 − a ) 2 ) = 0 ⇔
[1]
⇔ 2ax − 2by = (1 − a ) 2 + m

Derivando respecto a en [1]:


2 x = −2(1 − a) ⇔ a = x + 1 [2]

Eliminando a entre [1] y [2] resulta la ecuación de la envolvente:

2( x + 1) x − 2by = x 2 + m ⇔ x 2 + 2 x − 2by − m = 0

Se trata de una parábola, salvo si el centro de C es el origen (b = 0), en cuyo caso se


trata de un par de rectas paralelas al eje OY.

164. Sea una pirámide recta de base cuadrada, cuyo lado de la base mide 2a y la
altura 4a. Cortamos dicha pirámide por un plano que pasa por uno de los lados de
la base. Calcular:
a) el perímetro mínimo de los polígonos de intersección que se forman, en
función de a.
b) el volumen de la cuña inferior para este perímetro mínimo, utilizando a = 9.

RESOLUCIÓN.-

a) Considerando el desarrollo lateral de la pirámide, el perímetro mínimo se tendrá


cuando la poligonal BEFC sea una línea recta. En este supuesto, al ser semejantes los
triángulos BEA y VDA, se tiene:

BE = AB = 2a
VA = VH 2 + AH 2 = 16a 2 + 2a 2 = 3 2a
AE AD 2a·2a 2 2
= ⇔ AE = = a
AB AV 3 2a 3
 2 2 
2a  3 2a − a 
⇔ EF = 
EF VE 3  = 14 a
=
AD VA 3 2a 9

14 68
El perímetro mínimo es: BE + EF + FC + CB = 2a + a + 2a + 2a = a.
9 9

7/8
www.eltemario.com Oposiciones Secundaria – Matemáticas
© Antonio J. Abrisqueta Valcárcel, 2002 Hojas de Problemas – Geometría IV

b) La cuña inferior se puede considerar un prismatoide de bases el cuadrado ABCD


h
y el segmento EF (base degenerada). Su volumen es V = ( S1 + S 2 + 4S m ) , donde h es
6
la altura del prismatoide, S1 y S2 las áreas de las bases y Sm el área de la sección media.

2
2a·4a
EK VH 3 8
= ⇔ h = EK = = a ; luego h = 8
AE AV 3 2a 9

El área del cuadrado ABCD es 4a2 , luego S1 = 324.

El área del segmento EF es 0, luego S2 =0.

0 + 2a
La sección media es un rectángulo, cuyos lados miden =a y
2

14
a + 2a
9 16 16
= a, luego S m = a· a = 16·9 = 144 .
2 9 9

8
El volumen de la cuña es: V= ( 324 + 0 + 576) = 1200 .
6

8/8
www.eltemario.com Oposiciones Secundaria – Matemáticas
© Antonio J. Abrisqueta Valcárcel, 2002 Hoja de Problemas – Geometría V

165. Clasificar la cónica:

f ≡ y 2 + 2 xy + x 2 − 2 y + x + 1 = 0

y hallar su ecuación reducida.

Demostración.

Formaremos el discriminante:

1 1 1/ 2
9 1 1
∆= 1 1 −1 = − ; A33 = =0
4 1 1
1/2 −1 1

como A33 =0 y ∆ ≠ 0 se trata de una parábola real.

Hallaremos los invariantes de la cónica:


I 1 = a11 + a 22 = 2

I 2 = a 33 = 0 
9 
I3 = ∆ = − 
4 

la ecuación reducida será

− I3
I1 y 2 ± 2 x=0
I1

Sustituyendo:

9 3 2
2y2 − 2 x=0 y2 = x
8 4

166. Clasificar las cónicas:

4 x 2 − 2 xy + y 2 − 14 x + 2 y + 13 = 0
4 y 2 − 4 xy + 2 x 2 − 8 y − 2 x + 9 = 0

Demostración.

1/9
www.eltemario.com Oposiciones Secundaria – Matemáticas
© Antonio J. Abrisqueta Valcárcel, 2002 Hoja de Problemas – Geometría V

1º)

4 −1 − 7
4 −1
∆ = −1 1 1 =0 A33 = =3>0
−1 1
−7 1 13

Teniendo en cuenta el cuadro de la nota 1 resulta que la cónica degenera en dos


rectas imaginarias conjugadas.

2º)

2 − 2 −1
2 −2
∆ = − 2 4 − 4 = −16 A33 = =4>0
−2 4
−1 − 4 9

como A33 >0 y además a11 ⋅ ∆ = 2 ⋅ ( −16) = −32 < 0 se trata de una elipse real.

167. Clasificar la cónica:

f ≡ 2 x 2 − 3 xy − 2 x + 5 y − 3 = 0

y hallar luego la ecuación reducida.

Demostración.

Formemos el discriminante de la cónica.

2 − 3/2 −1
7 2 − 3/2 9
∆ = − 3/ 2 0 5/ 2 = ≠ 0 A 33 = = − <
4 − 3 /2 0 4
−1 5/2 −3

Por consiguiente se trata de una hipérbola real.

Según vimos en la nota 5 hemos de hallar las raíces de la ecuación:

r 2 − I1 r + I 2 = 0

Los invariantes son:

I 1 = a11 + a 22 = 2

9
I 2 = A33 = −
4

2/9
www.eltemario.com Oposiciones Secundaria – Matemáticas
© Antonio J. Abrisqueta Valcárcel, 2002 Hoja de Problemas – Geometría V

7
I3 = ∆ =
4

 2 + 13
 α=
9  2
luego resulta: r 2 − 2 r − = 0
4  β = 2 − 13
 2
la ecuación reducida quedara:

− I3
αx 2 + βy 2 =
I2
2 + 13 2 2 − 13 2 7
x + y =
2 2 9

168. Clasificar la cónica:

y 2 + 2 xy − 6 x − 8 y + 15 = 0

Demostración.

El discriminantes es:

0 1 −3
0 1
∆= 1 1 − 4 =0 A33 = = −1 < 0
1 1
− 3 − 4 15

Se trata de dos rectas reales concurrentes.

Hallemos su intersección con los ejes.

y = 3
Para x=0 y 2 − 8 y + 15 = 0  1
y2 = 5
5
Para y=0 x=
2

Ecuación de AC: 6 x + 5 y = 15

Ecuación de BC: 2x + y = 5

Razonando de forma análoga que en el problema nº 322 resultara: y=3 2x+y=5

3/9
www.eltemario.com Oposiciones Secundaria – Matemáticas
© Antonio J. Abrisqueta Valcárcel, 2002 Hoja de Problemas – Geometría V

169. La curva intersección de la esfera x 2 + y 2 + z 2 − 2 x − 2 y − 4 z − 4 = 0 con el


plano x+y-z-1=0 se proyecta ortogonalmente sobre el plano coordenado XOY.
Estúdiese la cónica proyección. ¿Cuál es su ecuación reducida?

Demostración.

Al cortar la esfera por el plano obtenemos una circunferencia de ecuación:

x 2 + y 2 + z 2 − 2x − 2 y − 4z − 4 = 0

 x + y − z −1 = 0

Para proyectarla ortogonalmente al plano XOY basta eliminar la z.

x 2 + y 2 + ( x + y − 1) 2 − 2 x − 2 y − 4( x + y − 1) − 4 = 0
Resulta:
2 x 2 + 2 y 2 − 8 x − 8 y − 2 xy + 2 = 0

Estudiemos esta cónica:

2 −1 − 4
∆ = −1 2 − 4 = −90 ≠ 0
−4 − 4 2

2 −1
Como además A33 = = 3 > 0 , se trata de una elipse real, ya que
−1 2
a11 ⋅ ∆ = 2 ⋅ (−90) < 0

Para obtener la ecuación reducida hallaremos las raíces de la ecuación

r 2 − I1 r + I 2 = 0 .

Recordemos que los invariantes son:

I 1 = a11 + a 22 = 4
I 2 = A33 = 3
I 3 = ∆ = −90

r 2 + 4 r + 3 = 0 → α = 3; β = 1

− I3
La ecuación reducida es: αx 2 + βy 2 =
I2
90 x2 y 2 90 x2 y 2
3x 2 + y 2 = → + =1 o bien x 2 + 3 y 2 = → + =1
3 10 30 3 30 10

4/9
www.eltemario.com Oposiciones Secundaria – Matemáticas
© Antonio J. Abrisqueta Valcárcel, 2002 Hoja de Problemas – Geometría V

170. Consideremos la cónica

5 x 2 − 3 xy + y 2 − 3 x + 2 y − 5 = 0

hallar la ecuaciones de sus ejes así como también la ecuación del diámetro
conjugado con la recta y=4x+5.

Demostración.

Estudiaremos primero la naturaleza de la cónica.

5 − 3/2 − 3 /2
33
∆ = − 3/ 2 1 1 =− <0
2
− 3/ 2 1 −5

a11 a12 5 − 3 / 2 11
A33 = = = >0
a12 a 22 − 3 / 2 1 4

como A33 > 0 y además a11 ∆ = 5 ⋅  −  < 0 , se trata de una elipse real, de la cual
33
 2
vamos a determinar los ejes.

Resolvamos:

a12 m 2 + (a11 − a 22 ) m − a12 = 0

3 2 3
− m + (5 − 1) m + = 0
2 2

 m = 3
3m 2 − 8m − 3 = 0 ⇒ m = − 1
 3

1 δf 1 
fx = = (10 x − 3 y − 3) 
δx 2 
 sustituyendo en fx + mfy = 0
2
1 δf 1
fx = = (2 y − 3 x + 2) 
2 δy 2 

1 1
(10 x − 3 y − 3) + 3 ⋅ ( 2 y − 3x + 2) = 0
2 2

x + 3y + 3 = 0

1 1
(10 x − 3 y − 3) − (2 y − 3 x + 2) = 0
2 3

5/9
www.eltemario.com Oposiciones Secundaria – Matemáticas
© Antonio J. Abrisqueta Valcárcel, 2002 Hoja de Problemas – Geometría V

3x − y − 1 = 0

Una vez encontrados los ejes, se trata ahora de averiguar la ecuación del
diámetro conjugado con la recta y=4x+5.

Consideremos el punto del infinito de la recta y=mx+n. Es (1,m,0). En nuestro


caso (1,4,0).

Recordemos que el polo de la recta del infinito es el centro de la conica; por


consiguiente, las polares de puntos impropios han de pasar por el centro, es decir, seran
diámetros.

Hallaremos la polar del punto (1,4,0).

1
fx´= (10 x − 3 y − 3 z )
2
1
fy´= ( 2 y − 3x + 2 z )
2
1
fz´= ( −10 z + 2 y − 3x )
2

particularmente para el punto (1,4,0)

5 5
f α´= −1; f β´= ; f γ´=
2 2

la ecuación polar es: xfα´+ yfβ´+ zf γ´= 0

x 5y 5z
− + + =0
2 2 2
− x + 5 y + 5z = 0 .

171. Determinar los elementos de la curva

x 2 − y 2 + xyz 2 = 11
en el punto P(3,2,1)
x 3 + y 3 + z 3 − xyz = 30

Demostración.

2 xdx − 2 ydy + ( xdy + ydx ) z 2 + 2 xyzdz = 0 



3x 2 dx + 3 y 2 + 3z 2 − ( xdy + ydx) z − xydz = 0

dividiendo por dz y particularizando para el punto (3,2,1):

6/9
www.eltemario.com Oposiciones Secundaria – Matemáticas
© Antonio J. Abrisqueta Valcárcel, 2002 Hoja de Problemas – Geometría V

dx dy dy dx 
6 −4 +3 +2 + 12 = 0 
dz dz dz dz
dx dy dy dx 
27 + 12 +3−3 −2 − 6 = 0
dz dz dz dz 

dx dy   dx  − 105
8 − = −12    =
 dz  97
 ⇒ dy P − 324
dz dz
dx dy  
25 +9 = 3   =
dz dz   dz  P 97

volviendo a diferenciar resulta:

2 xd 2 x + 2( dx) 2 − 2( dy ) 2 − 2 yd 2 y + 2 zdz ( xdy + ydx) + z 2 (2 dxdy + xd 2 y + yd 2 x) +


+ 2 xyzd 2 z + 2dz ( xydz + xzdy + yzdx ) = 0

6 x (dx ) 2 + 3x 2 d 2 x + 6 y ( dy ) 2 + 3 y 2 d 2 y + 6 z (dz ) 2 + 3 z 2 d 2 z − z ( dxdy + xd 2 y + dydx + yd 2 x ) −


− ( xdy + ydx) dz − d 2 z ( xy) − xdy + ydx) dz = 0

teniendo en cuenta que d 2 z = 0

2 2
d2x  dx   dy  d2y dy dx dx dy
2 x 2 + 2  − 2  − 2 2 + 2 zx + 2 zy + 2z 2 ⋅ +
dz  dz   dz  dz dz dz dz dz
 dy dx 
2 2
2 d y 2 d x
+ xz + yz + 2 z  x + y  + 2xy = 0
 dz dz 
2 2
dz dz

 d2y d2x
2 2
 dx   dy 
2 2
2 d x 2 d y dx dy
6 x  + 3 x + 6 y   + 3 y + 6 z − 5 x
 dz 2 + 2 + y −
 dz  dz 2  dz  dz 2  dz dz dz 2 
dy dx
− 2x − 2y =0
dz dz

dx dy
si particularizamos las coordenadas de P y sustituimos los valores de y
dz dz
obtenidos antes, llegamos a un sistema de dos ecuaciones que nos permite encontrar

d2y d2x
 2  y  2 
 dz  P  dz  P

solo falta ya sustituir estos valores en las correspondientes formulas dadas a principio de
capitulo.

7/9
www.eltemario.com Oposiciones Secundaria – Matemáticas
© Antonio J. Abrisqueta Valcárcel, 2002 Hoja de Problemas – Geometría V

172. Averiguar si la curva

x = 3t 3 + 2t ; y = 2t 4 + t 3 + 2; z = t 4 − t 3 − t + 3 es alabeada.

Demostración.

Supongamos que la curva no fuese alabeada. Sus puntos estarían en un plano


Ax+By+Cz+D=0

Sustituyendo las expresiones de las tres coordenadas

A(3t 3 + 2t ) + B( 2t 4 + t 3 + 2) + C (t 4 − t 3 − t + 3 + D = 0

( 2 B + C )t 4 + ( B − C + A)t 3 + (2 A − C )t + 3C + 2 B + D = 0

Como debe verificarse para cualquier valor de t:

2B + C = 0 
B = −A
B − C + 3 A = 0 
⇒ C = 2A
2A − C = 0 
D = −4 A
3C + 2 B + D = 0 

Por consiguiente para A=1 tendremos:

x − y + 2z − 4 = 0

ecuación del plano que contiene a la curva.

173. Se da la recta x=y=z, y en el plano OXY la recta x-y=a. Hallar la ecuación de la


superficie de revolución engendrada por la rotación de la segunda recta
alrededor de la primera

Demostración.

Hallaremos las ecuaciones de un paralelo cualquiera de la superficie de


revolución que tiene por eje a la recta x=y=z.

Dicho paralelo vendrá dado por la intersección de la esfera x 2 + y 2 + z 2 = λ y


del plano x + y + z = µ .

 z =0
La generatriz considerada es 
x − y = a

Eliminando los parámetros λ y µ entre las cuatro ecuaciones resulta x-y=a;


x+y= µ

8/9
www.eltemario.com Oposiciones Secundaria – Matemáticas
© Antonio J. Abrisqueta Valcárcel, 2002 Hoja de Problemas – Geometría V

µ+ a µ− a
x= ; y=
2 2

Sustituyendo en x 2 + y 2 = λ llegamos a la expresión µ2 + a 2 − 2λ2 = 0

Finalmente basta eliminar λ y µ entre esta ultima ecuación y la ecuación del


paralelo para obtener la solución

( x + y + z ) 2 − 2( x 2 + y 2 + z 2 ) + a 2 = 0

Se trata de un hiperboide de revolución.

9/9
www.eltemario.com Oposiciones Secundaria – Matemáticas
© Antonio J. Abrisqueta Valcárcel, 2002 Hoja de Problemas – Geometría VI

174. Un lazo corredizo, formado por una cuerda, envuelve a una columna
cilíndrica de radio r perfectamente lisa, estando sujeto el extremos libre de la
cuerda. Averiguar a que distancia de la columna está el nudo corredizo en el
momento en que la cuerda está completamente tensa.

RESOLUCIÓN.-

Sea a la longitud de la cuerda. Se trata de encontrar bajo qué ángulo á es máxima la


distancia OP.

Para dicho á se verifica:

a = (π + 2α) r + 2 BA + AP
OP = OA + AP

Haciendo d = OP, y restando las igualdades anteriores:

= r  − π − 2α
OB OB 1 2
d − a = OA − (π + 2a ) r − 2 BA = − (π + 2α) r − 2 −
sin α tan α  sin α tan α 
de donde d = a + r  − π − 2α
1 2

 sin α tan α 

Derivando d respecto de á e igualando a cero la derivada:

 1
cos α cos α − 
 − cos α 2   2
0 = d ' = r + − 2 ⇔ =0
 sin α sin α  sin α
2 2

π π π
que se anula en el intervalo  0,  para α = y α = .
 2 2 3

1 + cos 2 α 4 cos α π π
Como d " = − , es positiva para α = y negativa para α = , la
sin α
3
sin α
3
2 3
π
distancia d = OP alcanza su máximo para α = .
3

Para este valor de á es:

1/9
www.eltemario.com Oposiciones Secundaria – Matemáticas
© Antonio J. Abrisqueta Valcárcel, 2002 Hoja de Problemas – Geometría VI

r r 2 3 
DA = OA − OD = −r = − r = r  − 1
sin α 3  3 
2
que es la distancia pedida.

175. Un triángulo equilátero de lado 6 m. tiene un lado sobre el eje OY. Su


proyección ortogonal sobre el plano XOY es un triángulo rectángulo. Determínese
la ecuación del plano que contiene al triángulo.

RESOLUCIÓN.

Sea A(0, 6, 0) y OB = AB = 6.

Al ser OB’A un triángulo rectángulo e isósceles es B’(3, 3, 0).

Como MB = 3 3 , en el triángulo rectángulo MB’B es B’B = 3 2 . Luego es


B(3, 3, 3 2 ).

El plano OAB tiene pues por ecuación 2 x – z = 0.

Por simetría, el punto B puede ser también cualquiera de los siguientes:

(3, 3, -3 2 ), (-3, 3, 3 2 ) ó (-3, 3, -3 2 )

Luego las ecuaciones del los posibles planos son:

2x–z=0ó 2x+z=0

176. Consideren un cubo ABCDA’B’C’D’ donde ABCD y A’B’C’D’ representan


respectivamente sus bases superior e inferior, siendo las aristas AA’, BB’, CC’,
DD’ paralelas entre sí.

Un punto X se mueve con velocidad constante sobre la línea del perímetro del
cuadrado ABCD en el sentido ABCDA. Igualmente, un punto Y se mueve a la
misma velocidad sobre la línea del perímetro del cuadrado B’C’CB en el sentido
B’C’CBB’.

Los puntos X e Y empiezan a moverse en el mismo instante desde las posiciones


de salida A y B’, respectivamente.

Determine y dibuje el camino descrito por el punto medio M del segmento XY.

RESOLUCIÓN.

Cuando X está en los vértices A, B, C, D e Y está respectivamente en los vértices


B’, C’, C, B, el punto M está en E, F, C, H, donde:

2/9
www.eltemario.com Oposiciones Secundaria – Matemáticas
© Antonio J. Abrisqueta Valcárcel, 2002 Hoja de Problemas – Geometría VI

E es el centro de la cara ABB’A’

F es el centro de la cara BCC’B’

H es el centro de la cara ABCD

Cuando X se mueve de A a B e Y lo hace de B’ a C’, se tiene que:

1 1 1
EM = EA + AX + XY = EA + AX + ( XA + AB' + B' Y ) = ( AX + B'Y )
2 2 2

y como AX = a AB , B' Y = a B' Y = a B' C ' , 0 ≤ a ≤ 1

a a
resulta que, EM = ( AB + B' C ') = AC
2 2

En consecuencia M describe el segmento EF, de E a F.

Análogamente, cuando X se mueve de B a C e Y lo hace de C’ a C, M desplaza


sobre el segmento FC, de F a C.

Cuando X se mueve de C a D e Y lo hace de C a B, M se mueve sobre el segmento


HE de H a E.

Al ser EF = HC y FC = EH , en el camino descrito por M, cuadrilátero EFCH es


un paralelogramo.

Como además, los lados del paralelogramo son iguales a la mitad de la diagonal de
la cara del cubo, el paralelogramo es un rombo.

No es un cuadrado ya que el ángulo HCF es de 60º.

177. a) Busque el valor del mayor de los ángulos que forman entre sí las
bisectrices interiores de los dos ángulos agudos de un triángulo rectángulo.

b) Si r y R representan respectivamente los radios de las circunferencias


inscrita y circunscrita a un triángulo rectángulo, demuestre que se tiene que
cumplir necesariamente la desigualdad: 0 < r ≤ R tan( π 8) .

c) Utilizando los resultados anteriores, explique qué camino se debe seguir


para construir con regla y compás un triángulo rectángulo conocidas las longitudes
de los radios r y R de sus circunferencias inscrita y circunscrita, respectivamente.

RESOLUCIÓN.

3/9
www.eltemario.com Oposiciones Secundaria – Matemáticas
© Antonio J. Abrisqueta Valcárcel, 2002 Hoja de Problemas – Geometría VI

a)
1 1 π 3π
BIC = π − ( IBC + ICB ) = π − ( ABC + ACB) = π − = .
2 22 4

El ángulo que forman es de radianes.
4


b) Puesto que el ángulo BIC es de radianes, el punto I debe estar en el arco capaz
4

del ángulo sobre el segmento BC. Se verifica que r es como máximo la distancia r0
4
entre M y K (donde M es el punto medio de B y C).

1 3  π
Los ángulos KBC y KCB son iguales a π − π  = radianes, resultando que
2 4  8
π r0
tan = , ya que BM = CM = R.
8 R

π r
Luego es tan ≥ ⇔ 0 < r ≤ R tan( π / 8) .
8 R

c) Para dibujar el triángulo se traza un segmento BC de longitud 2R, que será la


hipotenusa del triángulo, siendo su punto medio, M, el centro de la circunferencia
circunscrita.

Tomando como centro un punto de esta circunferencia, P, situado en la


perpendicular a BC por M, se traza el arco BC con radio PB = PC. Se tiene así, el arco

capaz de sobre BC, arco en el que está el incentro, según el apartado a).
4

Trazando, en el semiplano respecto del segmento BC en que está el arco BC, la


paralela a BC a distancia r, por el apartado b) esta paralela corta al arco BC en al menos
un punto, I, que es el incentro del triángulo.

Las intersecciones de las semirrectas que duplican los ángulos CBI y BCI es el
punto A, tercer vértice del triángulo.

4/9
www.eltemario.com Oposiciones Secundaria – Matemáticas
© Antonio J. Abrisqueta Valcárcel, 2002 Hoja de Problemas – Geometría VI

178. La tangente trazada en un punto A a una circunferencia de radio r lleva


marcado un segmento AN cuya longitud es igual a la del arco de la circunferencia
AM (tomados en el mismo sentido). La recta MN corta a la recta AO (siendo O el
centro de la circunferencia) en el punto B. Calcular lim OB .
N →A

RESOLUCIÓN.

Sea á la medida del arco AM.

Sea P la proyección de M sobre la tangente.

Se tiene que:
AP = r sin α
MP = r − r cos α
AN = rα
PN = r (α − sin α)

Por la semejanza de los triángulos NAB y PMN, es

MP. AN α(1 − cos α)


AB = =r
PN α − sin α
sin α − α cos α
OB = AB − r = r
α − sin α

Por lo tanto, resolviendo las sucesivas indeterminaciones por la regle de L’Hopital,

sin α − α cos α sin α + α cos α


lim OB = lim r = r lim =
N→ A α →0 1 − cos α α→0 sin α
2 cos α − αsin α
r lim = 2r
α→0 cos α

179. Dado el triángulo OAB cuyos vértices son los puntos O(0, 0), A(2 3 , 0) y
B(2 3 , 2), se construye la figura inversa AB’ del segmento AB respecto del punto
O como centro de inversión. Calcular:

a) Longitud del arco AB’


b) Volumen del cuerpo engendrado por el triángulo mixtilíneo OAB’ al girar
alrededor del eje OX.
c) Ecuación de la figura inversa de la recta C1 B’, siendo C1 el centro del arco
AB’.
d) Centros de homotecia directa e inversa que transforman la circunferencia
de arco AB’ en la figura inversa de la recta C1 B’.
Realizar la construcción gráfica correspondiente.

RESOLUCIÓN.

5/9
www.eltemario.com Oposiciones Secundaria – Matemáticas
© Antonio J. Abrisqueta Valcárcel, 2002 Hoja de Problemas – Geometría VI

Puesto que A es un punto doble en la inversión, la figura inversa de la recta AB es la


circunferencia de centro en C1 , punto medio del segmento OA, que pasa por O. B’ es el
punto de intersección de esta circunferencia con la recta OB.

a) El radio r1 del arco AB’ es


r1 = OC1 = C1 A = 3 .

π π
Como ∠AOB ' = es ∠AC1 B ' =
6 3

La longitud del arco AB’ es


π 3
L= .r1 = π.
3 3

b) Sea P la proyección perpendicular de B’ sobre OX.

El cuerpo se puede descomponer en el cono formado al girar el triángulo OPB’ más


el segmento esférico formado al girar PAB’.

Como el triángulo C1 AB’ es equilátero, las coordenadas de B’ son  3 ,  .


3 3
2 2
El volumen del cono es:
2
1 3 3 9 3
π  3= π
3 2 2 8

El volumen del segmento esférico es:

2
 3  
π   3 −1 3 = 5 3π
  3 2 
 2   8

El volumen buscado es:


9 3 5 3 7 3
π+ π= π.
8 8 8

c) Como la recta C1 B’ no pasa por O, su inversa es una circunferencia que pasa por O.
Al ser B el inverso de B’, también pasa por B. Su centro, C2 , es así el punto de
intersección de la perpendicular por O a C1 B’ con la perpendicular por el punto medio,
M, a la cuerda OB.

1
La ecuación de OC2 es: y=− x.
3

La ecuación de MC 2 es: y = − 3x + 4 .

Luego se tiene que C2 (2 3 , -2).

6/9
www.eltemario.com Oposiciones Secundaria – Matemáticas
© Antonio J. Abrisqueta Valcárcel, 2002 Hoja de Problemas – Geometría VI

Con ello, la ecuación de la circunferencia inversa de la recta C1 B’ es:

x 2 + y 2 − 4 3x + 4 y = 0
y su radio, r2 , es 4.

d) Trazando el diámetro por C2 paralelo al radio C1 A se obtienen los puntos extremos


A1 y A2 . Uniendo cada uno de ellos con A, la intersección con la recta C1 C2 determina
los puntos H1 y H2 , centros de las homotecias directa e inversa, respectivamente, que
transforman un circunferencia en la otra.

Sea H1 (á1 , â1 ); se tiene:


r2
H1C 2 = HC
r1 1 1

 10 3 − 12
α1 =
(
luego 2 3 − α1 ,−2 − β1 = ) 4
( )
3 − α1 ,− β1 ⇔  13
3 β = 8 3 + 6
 2 13

Sea H2 (á2 , â2 ); se tiene:


r2
H 2 C2 = − H C
r1 2 1

 10 3 + 12
α2 =
(
luego 2 3 − α2 ,−2 − β2 = − ) 4
( 3 − α2 ,− β2 ⇔ ) 13
3 β = 6 − 8 3
 2 13

180. Se considera el recinto plano R situado en el primer cuadrante, limitado por


el eje OX, una determinada curva C que pasa por el origen de coordenadas y una
recta r paralela al eje OY. Sean V1 y V2 los respectivos volúmenes de los cuerpos de
revolución engendrados por el recinto R al girar alrededor de OX y OY,
respectivamente. Se sabe que cualquiera que sea la recta r en las condiciones
dadas, V1 supera a V2 en el volumen del cilindro de altura unidad, engendrado por
r al girar alrededor de OY.

Hallar:
a) Ecuación de la curva C.
b) Cálculo de los volúmenes V1 y V2 en el caso en que el volumen del cilindro
es ð.

RESOLUCIÓN.

a) Sea y = f(x) la ecuación de la curva C y sea x = a la ecuación de la recta r.

7/9
www.eltemario.com Oposiciones Secundaria – Matemáticas
© Antonio J. Abrisqueta Valcárcel, 2002 Hoja de Problemas – Geometría VI

V1 = π ∫ ( f ( x) ) dx
a 2
Calculando V1 por integración por discos:
0

V2 = 2π ∫ xf ( x )dx
a
Calculando V2 por integración por tubos:
0

El volumen del cilindro de altura unidad engendrado por el giro de r alrededor de


OY es πa 2 = π∫ 2 xdx .
a

En suma

V1 − V2 = πa 2 ⇔ ∫ ( f ( x)) 2 dx − 2 ∫ xf ( x) dx =
a

0 0
a


0
a a

0
(
2 xdx ⇔ ∫ ( f ( x)) 2 − 2 xf ( x) − 2 x dx = 0 )
lo que se verifica si ( f ( x)) 2 − 2 xf ( x) − 2x = 0 , esto es si f ( x ) = x + x 2 + 2 x es la
ecuación de C, ya que f ( x ) = x − x 2 + 2 x no es una curva en el primer cuadrante.

c) Es V1 – V2 = ð, luego a = 1.

( )
2
(
V1 = π ∫ x + x 2 + 2 x dx = π∫ 2 x 2 + 2 x + 2 x x 2 + 2 x dx =
1

0
1

0
)
5
= 2π∫0 x x 2 + 2 x dx + π
1

( )
1
1
x + 2 x  −
13
∫0 x x + 2 x dx = ∫0 3 2 (2 x + 2) x + 2 x dx = 3 
1 1 3
2 2 2

y 0
−∫ x 2 + 2 xdx = 3 − ∫
1 1
x 2 + 2 x dx
0 0

Con el cambio de variable x 2 + 2 x = ( x + 1) 2 − 1 = ( x + 1) + t , se tiene:

( x + 1) 2
− 1 = ( x + 1) + 2( x + 1)t + t
2 2
x=−
(t + 1)2
2t

(t + 1) 2 t2 − 1 1− t2
x 2 + 2x = − +1+t = ; dx = dt
2t 2t 2t 2
luego:

t 2 − 11 − t 2 1 1 1
∫ x 2 + 2 x dx = ∫
2t 2t 2
dt = ∫ ( −t + 2t −1 − t −3 )dt = t 2 + ln t + t − 2 + C =
4 8 8
=−
1
8
( 2 1
)
x 2 + 2 x − ( x + 1) + ln x 2 + 2 x − ( x + 1) +
2
1
8
−2
x 2 + 2 x − ( x + 1) + C ( )
1

1
Así, x 2 + 2 x dx = 3 + ln( 2 − 3 ) con lo que
0 2

8/9
www.eltemario.com Oposiciones Secundaria – Matemáticas
© Antonio J. Abrisqueta Valcárcel, 2002 Hoja de Problemas – Geometría VI

1 1
∫x
1
x 2 + 2 x dx = − ln( 2 − 3 ) = ln( 2 + 3)
0 2 2

En consecuencia V1 = π + ln( 2 + 3 )  .


5
2 

Entonces V2 = π + ln( 2 + 3 )  .


2
3 

181. Determinar el triángulo isósceles de área máxima que tenga por base una
cuerda conjugada al eje de la parábola y 2 = 2 px , y cuyo vértice opuesto esté en el
eje de la curva a la distancia “a” del vértice de la misma.

RESOLUCIÓN.

Las rectas conjugadas con el eje, y = 0, son las polares de los puntos del eje, esto es,
las rectas paralelas a OY.

Si admitimos que la recta, x = b, sobre la que está la base pueda distar más de a del
origen, entonces no existe área máxima ya que cuando b crece, el área no está acotada.

Restringiéndonos a la situación 0 ≤ b ≤ a , la superficie del triángulo es

S (b ) = 2 pb ( a − b) .

El valor de b buscado es el que maximiza S2 (b).

d 2 a
Es S (b ) = 2 p ( a − b)( a − 3b) = 0 ⇔ a = b ó b = .
db 3

a
Como S ( b) ≥ 0 , S(0) = 0 y S(a) = 0, en b = la función presenta un máximo cuyo
3
2 2 p 32
valor es a .
3 3

9/9
www.eltemario.com Oposiciones Secundaria – Matemáticas
© Antonio J. Abrisqueta Valcárcel, 2002 Hoja de Problemas – Geometría VII

182. Dada la cónica 2 x 2 − 2 xy + y 2 + 2 x − 8 y + 21 = 0 obtener la ecuación de la


tangente en el punto (3,5).

Sol.:

Basta aplicar la fórmula a la ecuación en homogéneas:

f ≡ 2 x 2 + y 2 + 21z 2 − 2 xy + 2 xz − 8 yz = 0

1 δf
fx = = 2x − y + z
2 δx
1 δf
fy = = y − x − 4z
2 δy
fz =
1 δf
= 21z + x − 4 y
2 δz

particularizando para el punto dado: P(3,5,1)

fx1 = 2 ; fy1 = −2 ; fz1 = 4 ;

en consecuencia, obtenemos:

xfx1 + yfy1 + zfz1 = 0 2x-2y+4z=0

pasando de coordenadas homogéneas a ordinarias (z=1)

2x-2y=-4
y=x+2

183. Clasificar la cuádrica.

x 2 + 3 y 2 + 6 z 2 + 2 xy − 2 xz + 6 yz + 6 z − 4 = 0

Hallar la intersección de dicha superficie con el plano que pasa por el punto
(1,2,1) y es perpendicular a la recta

x + y =1

x − z = 2

Sol.:

Formemos el discriminante de la cuádrica:

1/8
www.eltemario.com Oposiciones Secundaria – Matemáticas
© Antonio J. Abrisqueta Valcárcel, 2002 Hoja de Problemas – Geometría VII

1 1 −1 0
1 3 3 0
A= = 6; A44 = −6
−1 3 6 3
0 0 3 −4

a11 a12 1 1
A´33 = = =2>0 a11 A44 = −6 < 0
a 21 a 22 1 3

Se trata de una hiperboloide de una sola hoja.

La recta que consideramos tiene como ecuación en forma continua:

x y −1 z + 2
= =
1 −1 1

x − xo y − y o z − z o
La ecuación de cualquier plano perpendicular a una recta = =
a b c
es:
ax + by + cz + λ = 0

en nuestro caso: x - y + z + λ = 0

Obligando a que pase por el punto (1,2,1) resulta λ = 0 . Hallemos la intersección


del plano x-y+z=0 con la cuádrica. Dicha intersección es una cónica cuyos puntos
tienen de coordenadas las soluciones del sistema que forman la ecuación de la cuádrica
y la ecuación del plano.

Para proyectar la cónica intersección sobre el plano XOY sólo es preciso eliminar la
variable z entre ambas ecuaciones:

Sustituyendo z=y-x resulta:

9 x 2 + 15 y 2 − 18 xy + 6 y − 6 x − 4 = 0

9 −9 −3
A= −9 15 3 = −270 ≠ 0
−3 3 −4

9 −9
A33 = = 54 > 0 a11 ⋅ A = 9(270) < 0
− 9 15

luego la curva intersección es una elipse real.

2/8
www.eltemario.com Oposiciones Secundaria – Matemáticas
© Antonio J. Abrisqueta Valcárcel, 2002 Hoja de Problemas – Geometría VII

184. Clasificar la siguiente cuádrica, determinando su centro.

F ( x, y , z ) ≡ x 2 − 2 y 2 + z 2 − 4 xy + 6 xz − 8 yz + 6 x + 8 y − 2 z + 3 = 0

Sol.:

Tenemos:

1 −2 3 3
−2 −2 − 4 4
A= = 736 ≠ 0
3 − 4 1 −1
3 4 −1 3

1 −2 3
A44 = − 2 − 2 − 4 = 44 ≠ 0
3 −4 1

a11 a12 1 −2
A´33 = = = −6 < 0
a 21 a 22 − 2 − 2

Se trata por consiguiente de un hiperboloide de una hoja. Vamos a hallar el centro


de la cuádrica:

1 δf 1 
fx = = (2 x − 4 y + 6 z + 6) = 0 
2 δx 2 
1 δf 1 
fy = = (−4 y − 4 x − 8 z + 8) = 0
2 δy 2 
1 δf 1 
fz = = (2 z + 6 x − 8 y − 2) = 0 
2 δz 2 

resolviendo el sistema resulta:

27 15 − 10
x0 = ; y0 = ; z0 =
11 11 11

y =2 x = −z
185. Dadas las rectas y , determinar la superficie engendrada por
x=z y =1
una recta que se apoya sobre ellas y además corta al eje X.

Solución.

El haz de planos que pasan por la primera recta tiene de ecuación:

3/8
www.eltemario.com Oposiciones Secundaria – Matemáticas
© Antonio J. Abrisqueta Valcárcel, 2002 Hoja de Problemas – Geometría VII

( y − 2) + β( z − x) = 0

análogamente el haz de planos que pasan por la segunda recta:

( y − 1) + α( z + x) = 0

Una recta que se apoye sobre ambas vendrá dada por la intersección de dos planos:

( y − 1) + α( z + x) = 0

( y + 1) + β( z − x) = 0

Para obligar a que corte al eje x basta hacer y=0, z=0. Resulta:

αx = 1  α 1
⇒ = − ; β = −2α
βx = −2 β 2

( y − 2) − 2α( z − x) = 0

en consecuencia: y − 2 = 2α( z − x )

y −2 1− y
α= =
2( z − x ) x + z

Sustituyendo resulta:

xy − 3 yz + 4 z = 0

Estudiemos esta cuádrica:

1
0 0 0 1
2 0 0
1 −3 2
0 0 1 −3
A= 2 2 = −2 0 =1> 0
−3 2 2
0 0 2 0 0 2
2
0 0 2 0

1
0 0
2
1 −3
A44 = 0 =0
2 2
−3
0 0
2

Es pues un paraboloide hiperbólico.

4/8
www.eltemario.com Oposiciones Secundaria – Matemáticas
© Antonio J. Abrisqueta Valcárcel, 2002 Hoja de Problemas – Geometría VII

186. Clasificar la cuádrica siguiente:

x 2 + 2 y 2 + 3z 2 − 4 xz + 2 y − 2 z − 1 = 0

Determínese también su ecuación reducida.

Solución.

Tenemos:

1 0 −2 0 1 0 0 0
0 2 0 1 0 2 0 1
A= = =1> 0
−2 0 3 −1 − 2 0 −1 − 1
0 1 −1 −1 0 1 −1 − 1

1 0 −2
1 0
A44 = 0 2 0 = −2 A33 = =2>0
0 2
−2 0 3
a11 ⋅ A44 = −2 < 0

Se trata de una hiperboloide de una hoja. Hallemos las raíces de la ecuación


secular.

1− x 0 −2
0 2− x 0 = 0 ⇒ (1 − x )( 2 − x )(3 − x) − 4( 2 − x) = 0
−2 0 3− x

Las solucione s son: x=2; x = 2 + 5 ; x= 2− 5

Los autovalores S1 = 2 + 5 S 2 = 2 S 3 = 2 − 5 los sustituimos en la ecuación


reducida.

A
S1 X 2 + S 2 Y 2 + S 3 Z 2 + =0
A44
1
(2 + 5 )x 2 + 2 y 2 + (2 − 5 ) z 2 =
2

187. Dada una línea x = λ2 − 1 ; y = λ3 ; z = λ + 2 hallar la ecuación de la


superficie que engendra al girar en torno al eje OZ.

Solución.

5/8
www.eltemario.com Oposiciones Secundaria – Matemáticas
© Antonio J. Abrisqueta Valcárcel, 2002 Hoja de Problemas – Geometría VII

Un punto cualquiera de la curva describe una circunferencia con centro en el eje


OZ y de radio
d = x 2 (λ) + y 2 (λ) = λ6 + (λ2 − 1) 2

 x = dsenα
Por otra parte 
 y = d cos α

x 2 + y 2 = d 2 = λ6 + λ4 + 1 − 2λ2

Sustituyendo el valor de λ = z − 2 resulta :

[ ]
x 2 + y 2 = ( z − 2) 2 ( z − 2) 4 + ( z − 2) 2 − 2 + 1

Esta es por tanto la ecuación de la superficie.

188. Determínese la ecuación del cono cuyo vértice es el punto (α, β, γ ) y cuya
directriz es la cubica alabeada:

αa 2 βb 2 γc 2
x= ; y= ; z=
a2 + t b2 + t c2 + t

Solución.

Vamos a eliminar t entre las tres ecuaciones:

x −α y−β z−γ
= =
αa 2
βb 2
γc 2
− α − β −γ
a2 + t b2 + t c2 + t

O lo que es igual:

( x − α)(t + a 2 ) ( y − β)( t + b 2 ) ( z − γ )(t + c 2 )


= =
α β γ
Designando − λ a estas tres razones iguales

αλ
t + a2 + =0
x −α
βλ
t + b2 + =0
y−β
γλ
t + c2 + =0
z−γ

Eliminando t y λ entre esas tres ecuaciones resulta:

6/8
www.eltemario.com Oposiciones Secundaria – Matemáticas
© Antonio J. Abrisqueta Valcárcel, 2002 Hoja de Problemas – Geometría VII

α
1 a2
x −α
β
1 b2 =0
y−β
γ
1 c2
z−γ

Desarrollando:

α(b 2 − c 2 ) β(c 2 − a 2 ) γ ( a 2 − b 2 )
+ + =0
x −α y−β z−γ

Finalmente llegamos a la ecuación buscada:

α(b 2 − c 2 )( y − β)( z − γ ) + β(c 2 − a 2 )( z − γ )( x − α) + γ (a 2 − b 2 )( x − α)( y − β) = 0

189. Determinar la ecuación de una superficie desarrollable definida como


envolvente de la familia de planos (t + 2)x + y + t 2 z + t 3 = 0

Solución.

Para hallar la ecuación de la superficie basta eliminar el parámetro t entre la


ecuación de la familia de planos y la ecuación obtenida derivando respecto a t. O sea:

(t + 2) x + y + t 2 z + t 3 = 0

 x + 2 zt + 3t 2 = 0

La generatriz que engendra la superficie se obtiene sin más que despejar x e y.

x = −2tz − 3t 2 = m(t ) z + p (t )
y = ( t 2 + 4t ) z + 2t 3 + 6t 2 = n(t ) z + q(t )

Comprobemos que la superficie es desarrollable:

p´ q´ − 6t 6t 2 + 12t
= =0
m´ n´ − 2 2t + 4

La arista de retroceso viene dada por

7/8
www.eltemario.com Oposiciones Secundaria – Matemáticas
© Antonio J. Abrisqueta Valcárcel, 2002 Hoja de Problemas – Geometría VII


 x = −2tz − 3t 2

 y = ( t + 4t ) z + 2t + 6t
2 3 2

 − p´ 6t
 z = q = − 2 = −3t

8/8
www.eltemario.com Oposiciones Secundaria – Matemáticas
© Antonio J. Abrisqueta Valcárcel, 2002 Hoja de Problemas – Geometría VIII

190. Dado el paralelepípedo OADBFCEG en el espacio afín ordinario, se considera el


→ → →
sistema de referencia afín ℜ = ( O, OA , OB , OC ).

Sean:

• OG la recta determinada por los puntos O y G.


• ABC y DEF dos planos, y
• I, J los puntos de intersección de la recta OG con los planos ABC y DEF,
respectivamente.

Se pide:

a) La ecuación vectorial de la recta OG y de los planos ABC y DEF.


→ → →
b) Demostrar que OI = IJ = JG viendo antes que ABC || DEF.
c) Demostrar que I, J son los baricentros de los triángulos ABC y DEF,
respectivamente.

SOLUCIÓN:

a) Consideramos la figura adjunta.

i) Ecuación de la recta OG.

→ → →
OX = λ OG (X punto de OG )
→ → →
= λ ( OA + OB + OC ) ( λ ∈R)

ii) Ecuación del plano ABC.

→ → → →
OX = OA + λ AB + µ AC (X punto de ABC)
→ → → → → → → →
= OA + λ ( OB - OA ) + µ( OC - OA ) = (1 - λ - µ) OA + λ OB + µ OC
( λ , µ ∈R)

iii) Ecuación del plano DEF

→ → → →
OX = OD + λ DE + µ DF (X punto del plano DEF)
→ → → → → →
= ( OA + OB ) + λ ( OE - OD ) + µ( OF - OD )

1/11
www.eltemario.com Oposiciones Secundaria – Matemáticas
© Antonio J. Abrisqueta Valcárcel, 2002 Hoja de Problemas – Geometría VIII

→ → → → → → → →
= ( OA + OB ) + λ ( OE + OA + OB ) + µ( OF - OA - OB )
→ → → → → → → → → →
= ( OA + OB ) + λ ( OA + OC - OA - OB ) + µ( OB + OC - OA - OB )
→ → →
= (1 - λ ) OA + (1- µ) OB + ( λ + µ) OC ( λ , µ ∈R)

b) i) Veamos que los planos ABC y DEF son paralelos. Para ello basta ver que

→ →
• DE es paralelo a BC y que
→ →
• AC es paralelo a DF

1 → 1 → → →
ii) Si hacemos en a) –ii) λ = µ= se tiene que OX = ( OA + OB + OC ) de donde
3 3
→ 1 → →
OX = OG = OI
3
1 → 2 → → →
iii) Si hacemos en a) –iii) λ = µ= se tiene que OX = ( OA + OB + OC ) de donde
3 3
→ 2 → →
OX = OG = OJ
3
de estas relaciones, ii) y iii) se tiene que

→ → → 1 → 
IJ = OJ − OI = OG  → → →
3
 ⇒ OI = IJ = JG
→ → → → 2 → 1 →
JG = OG − OJ = OG − OG = OG 
3 3 

c) i) Veamos que I es el baricentro del triángulo ABC. En efecto:

1 → → → 1 → →
( OA + OB + OC ) = OG = OI , de donde I es baricentro de ABC.
3 3

ii) Veamos que J es el baricentro del triángulo DEF. En efecto,

1 → → → 1 → → → → → →
( OD + OE + OF ) = ( OA + OB + OA + OC + OB + OC )
3 3
2 → → → 2 → →
= ( OA + OB + OC ) = OG = OJ .
3 3
de donde J es el baricentro de DEF.

191. Dado el tetraedro ABCD, se considera el sistema de referencia ℜ=


→ → →
( AB , AC , AD ). Se pide:
a) Ecuación vectorial del plano que pasa por el punto medio de CD y es paralelo a
las rectas AD y BC.
b) Demostrar que este plano corta a las aristas del tetraedro no paralelas al mismo,
en sus puntos medios.

2/11
www.eltemario.com Oposiciones Secundaria – Matemáticas
© Antonio J. Abrisqueta Valcárcel, 2002 Hoja de Problemas – Geometría VIII

c) Demostrar que los puntos medios de estas aristas forman un paralelogramo.

SOLUCIÓN:

a) Consideremos la figura adjunta.

→ →
Una determinación lineal del plano pedido es α ( I, AD , BC ) donde:

• I es el punto medio del segmento CD



• AD el vector direccional de AD

• BC el vector direccional de BC

La ecuación vectorial del plano es:

→ → → →
AM = AI + t AD + s BC t, s ∈R
1 → → → → →
= ( AD + AC ) + t AD + s( AC - AB )
2
→ 1 → 1 →
= -s AB + (s + ) AC + (t + ) AD
2 2
b) Si en la ecuación del plano IKJL hacemos,

1 1 → 1 → →
• s=- y t = - , entonces AM = AB = AJ , luego J ∈α
2 2 2
1 → 1 → →
• s = 0 y t = - , entonces AM = AC = AK , luego K ∈α
2 2
1 → 1 → 1 → →
• s = - y t = 0, entonces AL = AB + AD = AL luego L ∈α
2 2 2

→ →
c) IKJL es un paralelogramo si y solo si JL y KI son vectores equipolentes.

→ → →1 → 1 → 1 → 1 →
• JL = AL - AJ = AB + AD - AB = AD
2 2 2 2
→ → → 1 → 1 → 1 → 1 →
• KI = AI - AK = AD + AC - AC = AD
2 2 2 2

luego IKJL es paralelogramo.

3/11
www.eltemario.com Oposiciones Secundaria – Matemáticas
© Antonio J. Abrisqueta Valcárcel, 2002 Hoja de Problemas – Geometría VIII

192. Se considera el espacio vectorial euclídeo R 3 , y en él los siguientes subespacios


vectoriales:

a) W1 = Vect( (1, 1, 0), (0, 3, 6) )


b) W2 = Vect( (1, 2, 1) )
c) W3 = Vect( (7, 8, 5), (6, 3, 1), (1, 3, 6) )

Hallar los subespacios vectoriales ortogonales correspondientes.

SOLUCIÓN:

El subespacio vectorial W1⊥ está engendrado por el vector


r
n = (1, 1, 0) x (0, 3, 6) = (6, -6, 3)

1 r
o también por el vector n = (2, -2, 1). Luego W1⊥ = Vect( (2, -2, 1) ).
3 r
Puede obtenerse también imponiendo la condición de que el vector n sea ortogonal
a los vectores que engendran W1 .
r
Si w = (x, y, z) pertenece al subespacio vectorial W 2⊥ entonces:

(x, y, z) (1, 2, 1) = x + 2y + z = 0

que es precisamente la ecuación del plano vectorial W 2⊥ .

Puesto que rang( (7, 8, 5), (6, 3, 1), (1, 3, 6) ) = 3

los vectores son linealmente independientes y engendran todo el espacio R 3 .


Por tanto W3⊥ = (0)

193. En el espacio vectorial euclídeo R 3 se considera el subespacio vectorial W dado


por

2 x + y − z = 0

x − y + 3z = 0

Hallar las ecuaciones paramétricas de W ⊥ .

SOLUCIÓN:
r
El vector vr = (2, 1, -1) es ortogonal al primer plano vectorial.
El vector w = (1, -1, 3) es ortogonal al segundo plano vectorial.
Por tanto, el plano ortogonal a la recta dada W es:
r r
W ⊥ = Vect( v , w )

4/11
www.eltemario.com Oposiciones Secundaria – Matemáticas
© Antonio J. Abrisqueta Valcárcel, 2002 Hoja de Problemas – Geometría VIII

donde,

(x, y, z) = t(2, 1, -1) + s(1, -1, 3)

que son las ecuaciones paramétricas del plano.


O también:
x = 2t + s
y=t–s
z = -t + 3s
r r
194. Sean a y b dos vectores del mismo módulo, no nulos. Calcular en función de
r r
cos( a , b ), y por medio del producto escalar,
r r r
a) cos( a , a + b )
r r r
b) cos( a , a - b )
r r r
c) cos( b , a + b )
r r r
d) cos( b , a - b )

Interpretar geométricamente estos resultados en el plano euclídeo.

SOLUCIÓN:

Antes de resolver cada una de estas cuestiones calculemos:


r r r r r r r2 r r
i) a b = a b cos( a , b ) = a cos( a , b )
r r2 r r r r r r r r r2 r2 r r
ii) a + b = ( a + b )( a + b )= a 2 + 2 a b + b 2 = 2 a + 2 a cos( a , b )
r2 r r
= 2 a (1 + cos( a , b ))
r r2 r r r r r r r r r2 r2 r r
iii) a − b = ( a - b ) ( a - b ) = a 2 - 2a b + b 2 =2 a - 2 a cos( a , b )
r2 r r
= 2 a (1 - cos( a , b ))

r r r rr r r r2 r2 rr
r r r a (a + b ) a a + ab a + a cos( a.b )
a) cos( a , a + b ) = v r r = r r r = r
a a +b a a +b r2 r r =
a 2 a (1 + cos(a , b )
r2 rr r r
a (1 + cos( a.b )) 1 + cos( a + b )
r2 r r = 2
a 2(1 + cos( a , b )

r r r r r
fórmula que expresa que el ángulo de ( a , a + b ) es la mitad del ángulo ( a , b ).

r r r r r r r r r r r r
c) cos( a , a + b ) = cos( a , a + b ) ya que a b = b a y a = b

b) Produciendo como en a), y teniendo en cuenta iii) resulta:

5/11
www.eltemario.com Oposiciones Secundaria – Matemáticas
© Antonio J. Abrisqueta Valcárcel, 2002 Hoja de Problemas – Geometría VIII

r r
r r r 1 − cos( a + b )
cos( a , a - b ) =
2
r r r r r r r r r r r r
d) cos( b , a - b ) = - cos( a , a - b ) ya que b ( a - b ) = - a ( a - b )

La interpretación geométrica de estos resultados se da en la figura de arriba. Los


paralelogramos ABDC y ABNM son evidentemente rombos, ya que tienen los cuatro
lados iguales. Los segmentos AD y MN son diagonales de los rombos y dividen a los
ángulos en dos partes iguales. Por tanto:

1
• ángulo BAD = ángulo BAC
2
• ángulo NAC = π - ángulo MAN = π - ángulo NAB.

195. Hallar vectorialmente la mediana AM de un triángulo ABC cuando se conocen


los lados del mismo.

SOLUCIÓN:

Se tiene:

( )(  1 
2 
) 1 
2 
1
m 2 a = AM . AM = AB + BM . AB + BM =  c + a . c + a  = c.c + a.a + a.c =
 4
1
4
1
(
= c.c + a.a + b.b − a.a − c.c
2
)

6/11
www.eltemario.com Oposiciones Secundaria – Matemáticas
© Antonio J. Abrisqueta Valcárcel, 2002 Hoja de Problemas – Geometría VIII

1 2 1 2 1 2 1 2 2b 2 + 2c 2 − a 2
ya que b.b = ( a + c) = a.a + c.c + 2a.c = c 2 +
a + b − a − c = ,
4 2 2 2 4
siendo a, b y c los módulos de a, b y c respectivamente.

De donde

1
ma = 2b 2 + 2c 2 − a 2
2

NOTA.-

Análogamente se obtienen las restantes medianas, y resulta:

1
mb = 2a 2 + 2c 2 − b 2
2
1
mc = 2 a 2 + 2b 2 − c 2
2

196. Calcular el área de un triángulo ABC,

a) vectorialmente,
b) analíticamente.

SOLUCIÓN:

a) Consideremos el triángulo ABC.


Para hallar el área del triángulo tomemos el vector AB ortogonal a AB y del
mismo módulo. Entonces se tiene:

1
S= AB h , definición de área
2
1
S = AB AC ' , ya que AC' = h
2

7/11
www.eltemario.com Oposiciones Secundaria – Matemáticas
© Antonio J. Abrisqueta Valcárcel, 2002 Hoja de Problemas – Geometría VIII


AC. AB
1
S= AB ⊥
, proyección de AC
2 AB

1 ⊥
S= AB . AC , operando y teniendo en cuenta la conmutatividad del
2
producto escalar.
Por tanto,

1 ⊥
S= AB . AC
2

b) Supongamos ahora que las coordenadas de los vértices son: A(x1 , y1 ), B(x2 , y2 ) y
C(x3 , y3 ), entonces


- AB = ( x 2 − x1 , y 2 − y1 ) de donde AB = ( −( y 2 − y1 ), x2 − x1 )
- AC = ( x3 − x1 , y3 − y1 )
de donde,
1 ⊥ 1
S = AB . AC = ( −( y 2 − y1 )( x3 − x1 ) + ( x2 − x1 )( y 3 − y1 )) =
2 2
1
= ( − y 2 x3 + y 2 x1 + y1 x 3 − y1 x1 + x2 y 3 − x 2 y1 − x1 y3 + x1 y1 ) =
2
1
= ( x1 y 2 + x2 y 3 + x 3 y1 − x3 y 2 − x 2 y1 − x1 y3 ) =
2
x1 y1 1
1 1 x 2 − x1 y 2 − y1
= x2 y2 1 =
2 2 x 3 − x1 y 3 − y1
x 3 y3 1

que es la ecuación analítica del área de un triángulo.

Por tanto,

1
S= det( AB, AC )
2

197. Demostrar vectorialmente el teorema de Pitágoras.

SOLUCIÓN:

En el triángulo ABC

8/11
www.eltemario.com Oposiciones Secundaria – Matemáticas
© Antonio J. Abrisqueta Valcárcel, 2002 Hoja de Problemas – Geometría VIII

se consideran los vectores:

- CA = b
- CB = a = b + c
- AB = c

Entonces,
a.a = (b + c )(b + c) = b.b + c.c + 2b.c
de donde
2 2 2
a = b + c + 2b.c

y siendo b y c vectores ortogonales, b.c = 0 , luego

2 2 2
a = b + c

198. Resolver la siguiente cuestión en el plano euclídeo ordinario:

Dado un cuadrado de vértices consecutivos A(3, 1) y B(0, 5) hallar los otros dos.

SOLUCIÓN:

Sean A(3, 1), B(0, 5), C(x, y) y D(x’, y’) los cuatro vértices del cuadrado y sus
coordenadas.

Los vectores AB y DC son equipolentes y lo mismo sucede con los vectores AD y


BC .

Los vectores AB y AD son ortogonales y además tienen el mismo módulo.

AB = OB − OA = ( 0,5) − (3,1) = ( −3,4)

9/11
www.eltemario.com Oposiciones Secundaria – Matemáticas
© Antonio J. Abrisqueta Valcárcel, 2002 Hoja de Problemas – Geometría VIII

AD puede ser, por tanto, (4, 3) o (-4, -3), es decir, vectores opuestos.

Veamos las dos soluciones.

1) AD = (4, 3)

- OC = OB + BC = OB + AD = ( 0,5) + ( 4,3) = ( 4,8) , luego, C(4, 8).


- OD = OA + AD = (3,1) + ( 4,3) = (7, 4) , luego, D(7, 4).

2) AD = (-4, -3)

- OC = OB + BC = OB + AD = ( 0,5) + ( −4,−3) = ( −4,2) , luego, C(-4, 2).


- OD = OA + AD = (3,1) + ( −4,−3) = ( −1, −2) , luego, D(-1, -2).

199. Se consideran las rectas,

r: 3x − y − 8 = 0 y s: 2 x − y − 6 = 0 .

Hallar la ecuación de la recta que pasa por el punto de intersección de r y s, y dista


del punto P(1, 1) una unidad de longitud.

SOLUCIÓN:

Por el punto P pasan infinitas rectas, que son todas las del haz determinado por r y s.
De todas estas rectas solamente dos, s1 y s2 , distan una unidad de P.

a) Ecuación del haz de vértice P.

H p : 3x − y − 8 + t ( 2 x − y − 6) = 0 , t número real
es decir, (3 + 2t ) x − (1 + t ) y − (8 + 6t ) = 0

b) Distancia del punto P a las rectas s1 y s2 .

(3 + 2t ) − (1 + t ) − (8 + 6t )
1=
(3 + 2t ) 2 + (1 + t ) 2
de donde,
20t 2 + 46t + 26 = 0

10/11
www.eltemario.com Oposiciones Secundaria – Matemáticas
© Antonio J. Abrisqueta Valcárcel, 2002 Hoja de Problemas – Geometría VIII

y de aquí,
10t 2 + 23t + 13 = 0

Las raíces de esta ecuación son: t1 = -13/10, t2 = -1.

c) Ecuaciones de las rectas s1 y s2 .

13
- s1 : 3x − y − 8 − (2 x − y − 6) = 0 ⇔ 4 x + 3 y − 2 = 0
10
- s2 : 3x − y − 8 − 1( 2 x − y − 6) = 0 ⇔ x − 2 = 0

NOTA.-

Este problema se reduce al siguiente:

Trazar por un punto exterior las tangentes a una circunferencia. De todas las rectas
del haz HA, solamente existen dos que son tangentes.

11/11
www.eltemario.com Oposiciones Secundaria – Matemáticas
© Antonio J. Abrisqueta Valcárcel, 2002 Hoja de Problemas – Geometría IX

200. Hallar la ecuación de la simetría ortogonal respecto de la recta:

n.( x − a) = 0

SOLUCIÓN.

Sean:
- S la simetría ortogonal respecto de la recta n.( x − a) = 0 .
- P un punto cualquiera cuyo vector de posición es p .
- P’ el punto simétrico de P y cuyo vector de posición es p ' .

El vector de posición del punto medio de PP' es:


1
M : m = ( p + p')
2

Teniendo en cuenta que el punto M pertenece al eje de simetría y que los vectores n
y PP' son paralelos, se obtiene el sistema:

1  
n. ( p + p') − a  = 0
2  

p' − p = k n 
de donde,
n. ( p + p' + k n) − a  = 0
1
2 
y de aquí,
1
n. p + k ( n.n) − n.a = 0
2
n.( p − a)
y finalmente, k = −2
n.n

Sustituido este valor de k en


la ecuación p ' = p + k n , se obtiene la ecuación de la simetría, y es

n.( p − a )
p' = p − 2 2
n
n

NOTA.-

Si sustituimos estos vectores por sus coordenadas

p ' = ( x' , y ' ) , p = ( x , y ) , n = ( A, B)

1/10
www.eltemario.com Oposiciones Secundaria – Matemáticas
© Antonio J. Abrisqueta Valcárcel, 2002 Hoja de Problemas – Geometría IX

y el número n.( p − a) por

n.( p − a) = Ax + by + C

que es el primer miembro de la ecuación en forma general de la recta, se tienen las


ecuaciones de la simetría ortogonal:

Ax + By + C
x'= x − 2 A
A2 + B2
Ax + By + C
y'= y − 2B
A2 + B2

Estas ecuaciones se obtienen directamente en el siguiente ejercicio.

201. Hallar las ecuaciones de la simetría ortogonal respecto de la recta,

Ax + By + C = 0

SOLUCIÓN.

Sean:
- S la simetría ortogonal respecto del eje Ax + By + C = 0 .
- P(x, y) un punto cualquiera del plano afín euclídeo.
- P’(x’, y’) el punto homólogo, es decir, P’ = S(P).

Las coordenadas del punto


medio M de PP' son:
x + x' y + y' 
M  , 
 2 2 

Las coordenadas del vector PP'


son ( x'− x, y '− y) .

Las coordenadas del vector


normal al eje de simetría son
n ( A, B ) .

Teniendo en cuenta que el punto M pertenece al eje de simetría y que los vectores n
y PP' son paralelos, se obtiene el sistema:

2/10
www.eltemario.com Oposiciones Secundaria – Matemáticas
© Antonio J. Abrisqueta Valcárcel, 2002 Hoja de Problemas – Geometría IX

x + x' y + y' 
A +B + C = 0
2 2

x '− x = kA 
y '− y = kB 


x + x' y + y'
de donde, A +B +C = 0
2 2
Ax + By + C
y de aquí, k = −2
A2 + B 2

Sustituido este valor de k en las ecuaciones x ' = x + kA , y ' = y + kA se obtienen las


ecuaciones de la simetría respecto del eje Ax + By + C = 0 , y son:

Ax + By + C
x'= x − 2 A
A2 + B2
Ax + By + C
y'= y − 2B
A2 + B2

Estas son las ecuaciones analíticas de la simetría ortogonal respecto de una recta
dada. Las ecuaciones recíprocas se obtienen cambiando la x por la x’ y la y por la y’ ya
que se trata de una transformación involutiva.

202. En el plano euclídeo ordinario se considera la transformación que hace


corresponder al punto P(x, y) el punto P’(x’, y’) donde

 x' = − x cos 2θ − y sin 2θ + 2 cos θ


f :
 y ' = − x sin 2θ + y cos 2θ + 2 sin θ

Se pide:

a) Hallar el conjunto de puntos dobles por f.


b) Decir que transformación geométrica es f.

SOLUCIÓN.

a) Los puntos dobles se obtienen haciendo (x, y) = (x’, y’), luego

 x (1 + cos 2θ) + y sin 2θ − 2 cos θ = 0



 x sin 2θ + y(1 − cos 2θ ) − 2 sin θ = 0
x.2 cos θ + y.2 sin θ cos θ − 2 cos θ = 0
2

⇔
x.2 sin θ cos θ + y.2 sin 2 θ − 2 sin θ = 0
( x cos θ + y sin θ − 1) cos θ = 0
⇔
( x cos θ + y sin θ − 1) sin θ = 0

3/10
www.eltemario.com Oposiciones Secundaria – Matemáticas
© Antonio J. Abrisqueta Valcárcel, 2002 Hoja de Problemas – Geometría IX

de donde el sistema se reduce a la ecuación

x cos θ + y sin θ − 1 = 0 [1]

puesto que cos θ y sin θ no son nulos simultáneamente. Por tanto, los puntos dobles
vienen dados por la ecuación de la recta [1].

b) La aplicación lineal f asociada a la aplicación afín f es:

 x ' = − x cos 2θ − y sin 2θ



 y ' = − x sin 2θ + y cos 2θ

El determinante de la aplicación lineal f vale -1, luego se trata de una simetría


vectorial.

Por tanto, f es una simetría respecto a la recta [1].

203. Sean A, B, C y D cuatro puntos del espacio afín euclídeo orientado, se pide:

a) Demostrar que BC × BD = AB × AC + AC × AD + AD × AB
b) Interpretar esta relación cuando los 4 puntos son coplanarios.

SOLUCIÓN.

a)
BC × BD = ( BA + AC) × ( BA + AD) =
= BA × BA + BA × AD + AC × BA + AC × AD =
= BA × AD + AC × BA + AC × AD ⇔ BA × BA = 0
= − AC × AB + AC × AD − AB × AD ⇔ BA = − AB
= AB × AC + AC × AD + AD × AB

b)
Si los cuatro puntos son colineales es evidente que los productos vectoriales son
nulos, ya que los vectores son linealmente dependientes.

Supongamos ahora que no hay tres alineados, entonces

- BC × BD = λn , donde n es un vector normal a los vectores BC y BD tal que


(u1 , u 2 , u 3 ) y ( BC , BD , n) tienen la misma orientación.

- AB × AC = µn
- AC × AD = ν n

4/10
www.eltemario.com Oposiciones Secundaria – Matemáticas
© Antonio J. Abrisqueta Valcárcel, 2002 Hoja de Problemas – Geometría IX

- AD × AB = ξn

1 1 1 1
Siendo λ, µ, ν , ξ el área orientada de los triángulos BCD, ABC, ACD y
2 2 2 2
ADB, respectivamente.

De aquí se sigue que:


1 1
λ = ( µ + ν + ξ)
2 2
y por tanto,

“El área orientada del triángulo BCD es igual a la suma de las áreas orientadas de los
triángulos ABC, ACD y ADB”.

204. 1) Hallar la ecuación del plano que pasando por el punto A(1, -1, 2) tiene
por vector normal el vector v = ( 2,−2,3) .

2) Hallar la ecuación de la recta que pasando por el mismo punto A es


perpendicular al plano 3x − y + z − 1 = 0.

SOLUCIÓN.

1) Un plano queda determinado por un punto y su vector normal, es decir, en nuestro


caso por A y v .

La ecuación en forma normal es, siendo X un punto cualquiera del plano:

AX .v = 0 ⇔ ( x − 1).2 + ( y + 1).( −2) + ( z − 2).3 = 0


⇔ 2 x − 2 y + 3 z − 10 = 0

2) Si una recta es perpendicular a un plano tiene por vector direccional el vector


normal al plano, que en este caso es:
w = (3,−1,1) .

La determinación lineal de la recta pedida es ( A, w) .

La ecuación en forma continua es:

x −1 y +1 z − 2
= =
3 3 1

5/10
www.eltemario.com Oposiciones Secundaria – Matemáticas
© Antonio J. Abrisqueta Valcárcel, 2002 Hoja de Problemas – Geometría IX

205. Hallar la ecuación del plano que pasa por los puntos A(2, 0, -3) y B(1, -1, 1) y
es paralelo a la recta r dada por:

3x + 2 y − 3 z − 2 = 0

3x + 6 y − 6 z − 6 = 0
SOLUCIÓN:

Un plano queda determinado


r r por un punto y dos vectores linealmente
independientes. Sea α( P, v , w) una determinación lineal del plano

a) P(2, 0, -3) = A(2, 0, -3)

b) Puesto que el plano pasa por los puntos A y B, un vector direccional será
el vector AB . Luego
r
v = AB = OB − OA = (1, -1, 1) – (2, 0, -3) = (-1, -1, 4)
r
c) El plano α es paralelo a la recta dada, luego el vector direccional w será
perpendicular a los vectores normales de los planos que determinan la
recta. Por tanto,
r
w = (A, B, C) x (A’, B’, C’) ,A, B, C, A’,B’, C’ coeficientes de
los planos
= (3, 2, -3) x (3, 6, -6) = (6, 9 ,12)
r r
De a) , b) y c) se sigue que la ecuación paramétrica del plano α( P, v , w) es:

 x = 2 − t + 6s

 y = −t + 9s t, s ∈ R
 z = −3t + 4t + 12s

y en forma de determinante,

x−2 −1 6
r r r r
det( x − a , v , w) = y −1 9 = 0 ⇔ 16 x − 12 y + z − 29 = 0
z+3 4 12

206. Hallar,
r r enr el espacio afín euclídeo tridimensional, la distancia del punto M a la
recta x = a + tu .

SOLUCIÓN:

Consideremos la figura adjunta. Sea r rH lar proyección


ortogonal del punto M sobre la recta x = a + tu . Este punto

6/10
www.eltemario.com Oposiciones Secundaria – Matemáticas
© Antonio J. Abrisqueta Valcárcel, 2002 Hoja de Problemas – Geometría IX

es único, y MH es, por definición, la distancia del punto M a la recta r.

Se tiene:

r r
MA × u = ( MH + HA) × u , suma de vectores
r r
= MH × u + MA × u , propiedad distributiva
r r
= MH × u , ya que MA y u son colineales

y tomando módulos

r r r
MA × u = MH u , ya que MA es perpendicular al vector u

de donde,

r
MA × u
MH = MH = d ( M , r ) = r
u

que es la ecuación vectorial de la distancia de un punto a una recta.

207. Dadas las rectas del espacio afín euclídeo r: (x, y, z) = (2, 1+ t, t) y r’= (x, y, z)
= (2s, 0, s), se pide :

a) la distancia mínima entre estas rectas


b) la ecuación de la perpendicular común ( recta que contiene al segmento
mínima distancia.)

SOLUCIÓN:

a) Un punto cualquiera de la recta r viene dado por A(2, 1+ t, t) , t ∈R


Un punto cualquiera de la recta r’ viene dado pro B(2s, 0, s), s ∈ R

El vector que determina los puntos A y B es:

AB = OB - OA = (2s, 0, s) – (2, 1+ t, t) = (2s – 2, -1 – t, s- t)


r
Un vector direccional de la recta r es: vr= (0, 1, 1)
Un vector direccional de la recta r’ es: w = (2, 0, 1)
r r
Dado que el vector AB es perpendicular al vector v y w se verificará:

 AB.vr = 0 ⇔ ( 2 s − 2,−1 − t , s − t ).(0,1,1) = s − 2t − 1 = 0



 −→ r
 AB .w = 0 ⇔ (2 s − 2, −1 − t , s − t ).( 2,0,1) = 5s − t − 4 = 0

7/10
www.eltemario.com Oposiciones Secundaria – Matemáticas
© Antonio J. Abrisqueta Valcárcel, 2002 Hoja de Problemas – Geometría IX

7 1
Resolviendo este sistema se obtienen los siguientes valores: s = ,t= −
9 9
−→ 4 8 8
Por tanto, AB = ( − , − , )
9 9 9

−→ 4
de donde la distancia entre las rectas es: AB =
3

b) La perpendicular común tiene como determinación lineal ( A, AB) , y por tanto, la


ecuación de la recta es:

8 1 4 8 8 8 1 4 8 8 9 8 8 1 8
( (2, , ), ( − ,− , ) : ( x, y, z ) = ( 2, ,− ) + t ( − ,− , ) = ( 2 − t , − t , − + t )
142949 142
3 9 9439 9 9 9 9 9 4 9 9 9 9
A →
AB

que es la ecuación de la perpendicular común en paramétricas.

208. Se considera en el plano afín ordinario dos puntos fijos y distintos A y B; se


define la aplicación del plano en si mismo que hace corresponder a un punto P el
punto P’ tal que
OP ' = k AP + h BP ,

siendo k y h dos números reales dados.

a) Demostrar que esta aplicación es una homotecia o una traslación.


b) Estudiar los casos en que esta transformación geométrica se reduce a una
traslación.

SOLUCIÓN:
r r
Sea ℜ = ( 0, u1 , u 2 ) una referencia afín del plano ordinario, entonces si

A( x0 , y 0 ), B( x1 , y1 ), P( x , y ), P' ( x ' , y ' )

la ecuación vectorial OP ' = k AP + h BP

se traduce en R2 por
 x ' = k ( x − x 0 ) + h ( x − x1 )

 y ' = k ( y − y 0 ) + h ( y − y1 )
es decir,
 x ' = ( k + h ) x − ( kx0 + hx1 )

 y ' = ( k + h ) y − ( ky0 + hy1 )

8/10
www.eltemario.com Oposiciones Secundaria – Matemáticas
© Antonio J. Abrisqueta Valcárcel, 2002 Hoja de Problemas – Geometría IX

b) si k + h = 1, entonces las ecuaciones anteriores se reducen a:

 x ' = x − ( kx0 + hx1 )



 y ' = y − ( ky0 + hy1 )
que son las ecuaciones de una traslación.

a) si h + k ≠ 1, entonces las ecuaciones [1] se pueden escribir así:


 x '− a = ( k + h)( x − a)

 y '−b = ( k + h )( y − b )

donde a y b vienen dados por

hx 0 + hx1 ky0 + hy1


a= y b=
k + h −1 k + h −1

Se trata por tanto, de una homotecia de centro el punto H(a, b) y razón de homotecia k +
h.

• Nótese que las coordenadas del punto H, centro de homotecia, son precisamente
las coordenadas del centro de gravedad del triángulo OAB cuando los puntos O,
A, B están afectados por los coeficientes -1, k y h respectivamente.
• Nótese que el punto H no existe cuando k + h -1 = 0, es decir, cuando k + h =1,
que en este caso se trata de una traslación.

r r
209. Sea (E, V, θ) el plano afín ordinario y ℜ = ( 0, u1 , u 2 ) una referencia afín del
mismo. Determinar la transformada de la parábola y2 = 2px mediante:

a) la afinidad de eje XX’, dirección YY’ y característica k.


b) la afinidad de eje YY’, dirección XX’ y característica k.

SOLUCIÓN:

a) Las ecuaciones de la afinidad homológica Afh(XX’, YY’, k) vienen dadas por


(x’, y’) = (x, ky) como puede verse fácilmente en la figura.

Las ecuaciones inversas son entonces:


(x,y) = (x’, y’/k).
Sustituyendo estos valores en la ecuación de la parábola,
queda:
y’2 = 2k2 px

9/10
www.eltemario.com Oposiciones Secundaria – Matemáticas
© Antonio J. Abrisqueta Valcárcel, 2002 Hoja de Problemas – Geometría IX

b) Las ecuaciones de la afinidad homológica Afh(XX’, YY’, k) vienen dadas por


(x’, y’) = (kx, y) como puede verse fácilmente en la figura.

Las ecuaciones inversas son entonces:


(x,y) = (x’/k, y’).
Sustituyendo estos valores en la ecuación de la parábola,
queda:
1
y’2 = 2p x’
k

10/10
www.eltemario.com Oposiciones Secundaria – Matemáticas
© Antonio J. Abrisqueta Valcárcel, 2002 Hojas de Problemas – Estadística I

210. Se considera el experimento aleatorio consistente en tirar tres dados al aire y


anotar los puntos de las caras superiores.

a) ¿Cuantos elementos tiene el espacio de sucesos?


b) Calcular la probabilidad de sacar al menos dos 5.
c) Calcular la probabilidad de sacar dos 2 y un 3.

Sol.:

a) Como cada dado tiene seis posibilidades tenemos que

nº de Elementos = 6
nº de Repeticiones = 3

Ω = VRnm = 6 3 = 216

b) Sea A el suceso de que salgan al menos dos 5.

Casos Totales = 216


 3
Casos Favorables = 1 +  ·5
 2
 3
1 +  ·5
 2  = 1 + 15 = 2
P( A) =
216 216 27

c) Sea B el suceso correspondiente a que salgan dos 2 y un 3.

Casos Totales = 216


 3
Casos Favorables =  ·1
 2

 3
 ·1
P( B ) =   =
2 3 1
=
216 216 72

211. Dos jugadores A y B lanzan simultánea y respectivamente 3 y 2 monedas.


Gana el jugador que obtenga más caras, repitiéndose el lanzamiento si ambos
obtienen el mismo número.

a) Calcular la probabilidad de que gane el jugador A.

Sol.:

Caso n=1 (Sólo se lanzan una vez)

Sea el suceso A={El jugador A pierde o empata}

1/9
www.eltemario.com Oposiciones Secundaria – Matemáticas
© Antonio J. Abrisqueta Valcárcel, 2002 Hojas de Problemas – Estadística I

Los casos posibles se calculan sabiendo que el Jugador A tiene 4 posibilidades (0, 1,
2 ó 3 caras), y el Jugador B tiene 3 posibilidades (0, 1 ó 2 caras). Por tanto los casos
posibles son 4·3=12

Los casos favorables se calculan sumando las veces que A pierde con las veces que
A empata. Así tenemos:

Jugador A Jugador B Jugador A Jugador B


1 cara 2 caras 1 cara 1 cara
0 caras 1 cara 2 caras 2 caras
0 caras 2 caras 0 caras 0 caras

Por tanto, los casos favorables son 3+3 = 6

casos favorables 6 1
P( A) = = =
casos posibles 4·3 2

Una vez que calculamos P(A), como nos interesa su complementario, tenemos

1 1
Entonces P( A C ) = 1 − =
2 2

Caso n≠1 Supongamos que empatan en la primera tirada.

3 2

P(0 caras) = C   ·C 20   = · 5 · = 5
1
0 1 5! 1 2! 1
3
2  2 3! 2 2! 2

2 2 5

P(1 cara) = C31   ·C 12   =   ·2! = 4 ·3


1 1 1 3! 1
2  2   2  2! 2

P(2 caras) = C   · ·C 22 · ·  = · 5 = 3· 5


21 1 1 1 3! 1 1
3
 2   2  2   2  2! 2 2

1 1 1 5
P(Empate entre A y B) = 3· 5
+ 3· 4 + 5 =
2 2 2 16

Sea Ei el suceso de que A gane en la i-ésima tirada

P(E3 )=   ⋅
1 5 1 5 1
Sabemos que P(E1 )= P(E2 )= · y así sucesivamente
2 16 2  16  2

i −1
5 5
i −1 i −1 1−   ⋅

5 1 1 ∞ 5 1  16  16
P(A gane)= ∑   ⋅ = ⋅ ∑  = ⋅ Lim =
i = 1  16  2 2 i = 1  16  2 i → +∞ 5
1−
16

2/9
www.eltemario.com Oposiciones Secundaria – Matemáticas
© Antonio J. Abrisqueta Valcárcel, 2002 Hojas de Problemas – Estadística I

1 1 1 16 8
= ⋅ = ⋅ =
2 5 2 11 11
1−
16

212. En una bolsa hay 4 bolas negras y 5 blancas. En otra bolsa hay dos negras y 3
blancas. Se elige al azar una bolsa y se extrae de ella una bola.

a) Hallar la probabilidad de que la bola extraída sea negra.


b) Hallar la probabilidad de que la bola extraída sea blanca.

Sol.:

a) P(N) = P(N∩Ω) = P(N∩(Bolsa1∪Bolsa2)) = P((N∩Bolsa1)∪(N∩Bolsa2))=

= P(N∩Bolsa1)+P(N∩Bolsa2) = P(Bolsa1)·P(N|Bolsa1) + P(Bolsa2)·P(N|Bolsa2) =

1 4 1 2 2 1 19
= ⋅ + ⋅ = + =
2 9 2 5 9 5 45

b) P(B) = P(B∩Ω) = P(B∩(Bolsa1∪Bolsa2)) = P((B∩Bolsa1)∪(B∩Bolsa2))=

= P(B∩Bolsa1)+P(B∩Bolsa2) = P(Bolsa1)·P(B|Bolsa1) + P(Bolsa2)·P(B|Bolsa2) =

1 5 1 3 5 3 26
= ⋅ + ⋅ = + =
2 9 2 5 18 10 45

213. Supongamos que hay una prueba para diagnosticar cáncer que da positivo en
el 95% de los casos cuando se aplica a personas que poseen esta enfermedad, y da
negativo en el 95% de los casos cuando se aplica a personas que no la poseen. Si la
probabilidad de que una persona tenga realmente cáncer es 0’005. ¿Cuál es la
probabilidad de que una persona tenga realmente cáncer cuando la prueba le haya
dado positiva?

Sol.:

Sean los siguientes sucesos

E = {Tener Cáncer}
S = {No tener Cáncer}
+ = {Prueba Positiva}
– = {Prueba Negativa}

Lo que nos pide el problema es el cálculo de la probabilidad P(E|+)

Para ello, primero hemos de calcular la probabilidad de que la prueba de positiva

3/9
www.eltemario.com Oposiciones Secundaria – Matemáticas
© Antonio J. Abrisqueta Valcárcel, 2002 Hojas de Problemas – Estadística I

1 95 199 5 19 199 109


P(+) = P(E)·P(+|E) + P(S)·P(+|S) = ⋅ + ⋅ = + =
200 100 200 100 200 ⋅ 20 200 ⋅ 20 2000

1
donde = 0'005
200

1 95
+
P( E) ⋅ P( + | E ) 200 100 95 19
P(E|+) = = = =
P (+ ) 109 1090 218
2000

214. Sean A y B sucesos de probabilidad no nula. Demostrar que si son


incompatibles entonces no son independientes. Ilustrar con un ejemplo que si son
incompatibles no tienen porqué ser dependientes.

Sol.:

Sabemos que si A∩B = ∅ ⇒ Son Incompatibles

y que si P(A∩B) = P(A)·P(B) ⇒ Son Independientes

Supongamos que P(A∩B) = P(A)·P(B) ≠ 0 ⇒ No pueden ser Incompatibles


porque si lo fueran tendríamos que A∩B = ∅ y por tanto P(A∩B) = 0.

Luego si A y B son incompatibles entonces no son independientes.

Ejemplo. Lanzar una moneda dos veces.


• Son Compatibles (puede salir dos veces cara/cruz)
• Son Independientes.

215. Se dispone de una urna que contiene 5 bolas rojas y 4 bolas blancas. Se
extraen de su interior 5 bolas y estamos interesados en definir la variable aleatoria
X=nº de bolas blancas extraídas. Completar la tabla adjunta en los casos
siguientes:

X=r 0 1 2 3 4 5
P(X=r)

a) Las 5 bolas se extraen de 1 en 1 devolviéndolas cada vez a la urna.


b) Las 5 bolas se extraen a la vez.

Sol.:

 5
P(X=0) =  
5

= 0'0529 P(X=1) =
(5 ·4)·5 = 0'2116
4

 9 95

4/9
www.eltemario.com Oposiciones Secundaria – Matemáticas
© Antonio J. Abrisqueta Valcárcel, 2002 Hojas de Problemas – Estadística I

(5 ·4 )· 52 
3 2

  = 0'3387 5 2 43  5 
P(X=2) = P(X=3) = · ·  = 0'2709
95 9 2 9 3  3 

 4   5   5
4 5

P(X=5) =   = 0'0173
4
P(X=4) =   ⋅   ⋅   = 0'1083
 9   9   4 9

X=r 0 1 2 3 4 5
P(X=r) 0’0529 0’2116 0’3387 0’2709 0’1083 0’0173

 9  9!
b) Ω =   = = 252
 5  5!⋅4!

 4  5
  ⋅  
P(X=0) =     =
0 5 1
= 0'00396
252 252

 4 5 
  ⋅  
P(X=1) =     =
1 4 4·5! 5
=
252 4!·252 88

 4  5
  ⋅  
P(X=2) =     =
2 3 15
252 176

 4 5 
  ⋅  
P(X=3) =     =
3 2 5
252 44

 4  5
  ⋅  
P(X=4) =     =
4 1 5
252 252

P(X=5) = 0

X=r 0 1 2 3 4 5
P(X=r) 1 5 15 5 5
0
252 88 176 44 252

216. La probabilidad de que un jugador de tenis gane un punto cuando le entra el


primer servicio es de 0’8 y de que lo gane con el segundo servicio es de 0’5.

5/9
www.eltemario.com Oposiciones Secundaria – Matemáticas
© Antonio J. Abrisqueta Valcárcel, 2002 Hojas de Problemas – Estadística I

Teniendo en cuenta que la probabilidad de que le entre el primer servicio es de 0’3,


calcular:

a) La probabilidad de que gane un punto.


b) La probabilidad de que gane un juego.

Sol.:

a) Sea A el suceso A={Gana un punto}

  G1 Gana 0'8
 A1 Pr imer Servicio 0'3 
 P1 Pierde 0'2

 A2 Segundo Servicio 0'7  G2 Gana 0'5
 P2 Pierde 0'5

P(A) = P(A1 )·P(G1 |A1 ) + P(A2 )·P(G2 |A2 ) = 0’3·0’8 + 0’7·0’5 = 0’24 + 0’35 = 0’59

b) Sea B el suceso B={Gana el Juego}

Sean los sucesos siguientes:

C1 = {Gana los 4 puntos}


C2 = {Gana 4 puntos y pierde 1}
C3 = {Gana 4 puntos y pierde 2}
C4 = {Gana 5 puntos y pierde 3}
C5 = {Gana 6 puntos y pierde 4}
C6 = {Gana 7 puntos y pierde 5}

Como la probabilidad de ganar un punto es 0’59, tenemos que su complementario es


la probabilidad de perderlo, 0’41.

P(C 1 ) = 0’594 = 0’1211


 4
P(C 2 ) = 0’594 ·0’41·   = 0’1987
 1
El número combinatorio expresa la posibilidad de perder uno cualquiera de los
cuatro primeros puntos de los cinco que se juegan. Es evidente que el último lo gana el
jugador que gana el juego.

 5
P(C 3 ) = 0’594 ·0’412 ·   = 0’2035
 2

Igualmente, el número combinatorio representa las distintas formas de perder dos


puntos de los cinco primeros en juego.

 6
P(C 4 ) = 0’593 ·0’413 ·   ·0’592 = P(D0 )
 3

6/9
www.eltemario.com Oposiciones Secundaria – Matemáticas
© Antonio J. Abrisqueta Valcárcel, 2002 Hojas de Problemas – Estadística I

Los tres primeros factores corresponden a acabar los seis primeros puntos en
empate, siendo el último factor el que corresponde a ganar los dos últimos puntos en
juego para ganar el juego.

 6
P(C 5 ) = 0’593 ·0’413 ·   ·0’593 ·0’41·2 = P(D1 )
 3

 6
P(C 6 ) = 0’593 ·0’413 ·   ·0’594 ·0’412 ·22 = P(D2 )
 3

.......

 6
P(Dn ) = 0’593 ·0’413 ·   ·0’592+n ·0’41n ·2n
 3

+∞
P(B) = P(C 1 ) + P(C 2 ) + P(C 3 ) + ∑ P (D ) = i
i= 0

+∞
 6
= 0’1211 + 0’1987 + 0’2035 + ∑ 0'59 3
⋅ 0'413 ⋅   ⋅0'59 2 + i ⋅ 0'41i ⋅ 2 i =
i= 0  3

 6  +∞
= 0'1211 + 0'1987 + 0'2035 + 0'59 5 ⋅ 0'413 ⋅   ⋅ ∑ ( 0'59 ⋅ 0'41 ⋅ 2) i =
 3  i= 0

n
= 0'1211 + 0'1987 + 0'2035 + 0'0985465 ⋅ Lim ∑ ( 0'59 ⋅ 0'41 ⋅ 2) i =
n → +∞
i= 0

n
= 0'1211 + 0'1987 + 0'2035 + 0'0985465 ⋅ Lim ∑ ( 0'48) i =
n → +∞
i= 0

1 − 0'48 n +1
= 0'1211 + 0'1987 + 0'2035 + 0'0985465 ⋅ Lim =
n → +∞ 1 − 0'48

1
= 0'1211 + 0'1987 + 0'2035 + 0'0985465 ⋅ =
1 − 0'48

= 0'1211 + 0'1987 + 0'2035 + 0'1895 = 0'713

217. Distribuimos al azar n bolas blancas y n bolas negras en n urnas.

a) Hallar la probabilidad de que una urna especificada contenga i bolas


blancas y j bolas negras.
b) Calcular la probabilidad de que en cada urna haya 1 bola de cada color.

7/9
www.eltemario.com Oposiciones Secundaria – Matemáticas
© Antonio J. Abrisqueta Valcárcel, 2002 Hojas de Problemas – Estadística I

Sol.:

a) Sea A el siguiente suceso

A = {Una urna contenga i bolas blancas y j bolas negras}

Para calcular la probabilidad P(A) tenemos que:

Casos Totales: VRn2 n = n 2 n

Casos Favorables: Se obtiene como producto de

• Las i bolas blancas Cni


• Las j bolas negras C nj
• Repartimos el resto de bolas en el resto de urnas VRn2−n1− i − j

 n  n 
  ⋅   ⋅ (n − 1)2 n −i − j
C ni ⋅ C nj ⋅ VRn2−n1− i − j
=     2n
i j
P(A) =
VRn2 n n

b) Sea B el suceso

B = {Cada urna tiene una bola de cada color}

Casos Totales: VRn2 n = n 2 n

Casos Favorables: Se obtiene como producto de:

• Las n bolas blancas en las n urnas Pn


• Las n bolas negras en las n urnas Pn

Pn2 n!·n!
P(B) = =
n 2 n n 2n

218. Supóngase una urna con a bolas blancas y b bolas negras. Calcular la
probabilidad de obtener exactamente n bolas blancas cuando se han sacado N
≤ N≤
bolas, con n≤ ≤ a de la urna:

a) Simultáneamente.
b) Sin reemplazamiento.

Sol.:

a) Sea A el suceso A = {Sacar n bolas blancas}

8/9
www.eltemario.com Oposiciones Secundaria – Matemáticas
© Antonio J. Abrisqueta Valcárcel, 2002 Hojas de Problemas – Estadística I

Casos Totales: C aN+ b

Casos Favorables: C an ⋅ CbN − n

Can ·CbN −n
P( A) =
C aN+ b

b) Casos Totales: VaN+ b

Casos Favorables: Van ·VbN − n ·C Nn Donde el último factor corresponde a las formas de
ordenar las bolas blancas tomadas.

VNn
P( A) =
VaN+b

9/9
www.eltemario.com Oposiciones Secundaria – Matemáticas
©Antonio J. Abrisqueta Valcárcel.2002. Hojas de Problemas – Estadística II

219.- Sean a y b ∈ Ð *, sea X una variable aleatoria discreta con valores enteros
estrictamente positivos de manera que:
1 1
P( X = x ) = − si 1 ≤ x ≤ ab
a b
P( X = x ) = 0 si x > ab

a) ¿Qué condición debe satisfacer a y b para que la sucesión, de término gene-


ral p(x)=P(X=x), pueda ser considerada como la ley de la probabilidad de X.

b) Determinar la función de distribución F(x). ¿Cuáles son las soluciones de la


ecuación F(x)= 12 ? (con las medianas de X)

c) Calcular la esperanza de X. ¿Qué valores habrá que dar a a y a b para que


E(X)=7/2?
Sol.:

a) ∑ P( X = x) = 1 (para que sea función de probabilidad)
x= 1


1 − 1 =1  1 − 1  = 1 − 1  ab 1 = 1 − 1  ab =  b − a  ab = b − a
ab ab

∑   ∑    ∑    
x= 1  a b x= 1  a b   a b  x =1  a b   ab 

1=b - a

b) F(x)=P(X=x)

x x
1 1 1 1 x 1 1
Si 1 ≤ x ≤ ab ⇒ F ( x) = ∑ P( X = j ) = ∑  −  = − ∑1 = x − 
j =1 j =1  a b   a b  j =1 a b

x ab x
Si x>ab ⇒ F ( x) = ∑ P( X = j ) = ∑ P( X = j ) + ∑ P( X = j) = 1
j =1 j =1 j =ab +1

 1 1
 x −  si 1 ≤ x ≤ ab
F ( x)   a b 
 1 si x > ab

F ( x) = x  −  =
1 1 1 1 ab 1 ab ab
⇒ x= × = × =
a b 2 2 a −b 2 1 2

b − a ab(1 + ab ) 1 + ab
E ( x ) = ∑ xP( x ) =∑  −  x =  − ∑ x =
ab ab
1 1 1 1 ab
c) × =
x =1 x =1  a b   a b  x= 1 ab 2 2

7 7 1 + ab  ab = 6
E (x ) = ⇒ = ⇒  sustituyendo
2 2 2  b − a = 1 → b = a +1

1/9
www.eltemario.com Oposiciones Secundaria – Matemáticas
©Antonio J. Abrisqueta Valcárcel.2002. Hojas de Problemas – Estadística II

− 1 ± 1 + 24 −1 ± 5 a1 = 2
a ( a + 1) = 6 ⇒ a2 + a −6 = 0 a= = =
2 2 a2 = −3 No válida

resultando: a=2 b=3

220.- Sea m un 3 +. Se considera la función definida ∀ x∈


∈Ð :
x
m
f (x ) = e − m distribución de POISSON
x!
a) Probar que f(x) puede ser considerada como la ley de probabilidad de una
variable aleatoria X

b) Calcular la E(x) y Var(X)


Sol.:
+∞ +∞
mx xn
a) 1 = ∑ e −m = e − me m = e 0 = 1 ya que ex = ∑
x= 0 x! n =0 n!

+∞ +∞
mx +∞
mx +∞
m x− 1
b) E ( X ) = ∑ x.P( x ) =∑ xe −m = e−m ∑ = me − m ∑ =me − m e m = m
x= 0 x =0 x! x =0 ( x − 1)! x =0 ( x − 1)!

Var(X)=E(X2 )-(E(X))2
+∞ +∞ +∞
mx mx
E(X2 ) = ∑x 2
f ( x) = ∑ x 2 e −m
x!
= e−m ∑ x
( x − 1)!
= ...
x= 0 x= 0 x =1

 +∞ mx +∞
m x  −m  +∞ m x +∞
m x −1 
…= e  ∑ ( x − 1) +∑  = e  ∑ + m∑  = …
−m

 x =1 ( x − 1)! x=1 ( x − 1)!   x =2 ( x − 2)! x =1 ( x −1)! 

 2 +∞ m x −2 m x −1  − m 2 m
( )
+∞
…= e  m ∑
 + m∑  = e m e + me m = m2 + m
−m

 x =2 ( x − 2)! x =1 ( x − 1)! 

Var(X) = m2 + m − m2 = m

221.- La variable aleatoria X tiene la función de densidad:


f ( x ) = C( x − a ) p −1 (b − x ) q −1 para a< x<b
f (x ) = 0 en el resto donde p>0, q>0 y a<b
Calcular: a) la constante C y b) E(X)
Sol.:

2/9
www.eltemario.com Oposiciones Secundaria – Matemáticas
©Antonio J. Abrisqueta Valcárcel.2002. Hojas de Problemas – Estadística II

β( p, q) = ∫ t p −1 (1 − t )
1 q −1
dt
0

1 = ∫ f ( x )dx = ∫ 0.dx + ∫ C (x − a ) (b − x )q−1 dx + ∫b


+∞ a b p −1 +∞
a) 0.dx = ...
−∞ −∞ a

p −1 q −1
x −a  b − a − x + a (b − a) 
…= C ∫ (x − a ) (b − x ) dx = C ∫  ( b − a) 
b p −1 q −1 1
  dx = ...
0 b− a b−a
a
   

p −1 q −1 t = ( x − a ) ( b − a) 
x−a   x − a    
…= C ∫ 
1
(b − a)   1 −  (b − a )  dx = ...  x → a ⇒ t → 0
0 b−a
   b − a   x → b ⇒ t → 1 
 

…= C ∫ t p −1 (b − a ) (1 − t )q −1 (b − a ) q−1 dt .(b − a) ) = ...


1 p −1

…= (b − a ) C ∫ t p −1 (1 − t ) dt . =(b − a )
p + q −1 1 q −1 p + q −1
Cβ( p, q ) = 1 de donde:
0

1
C=
β( p, q ).(b − a )
p + q −1

× x ( x − a ) p −1 (b − x )q −1 dx = ...
1
E ( X ) = ∫ x.P( X = x )dx = ∫
b b
b)
β( p, q ).(b − a )
a a p +q −1

t = ( x − a ) ( b − a) 
1  
∫ x ( x − a ) (b − x )
b p −1 q −1
…= dx =…  x → a ⇒ t → 0
β( p, q ).(b − a )
p + q −1
x → b ⇒ t → 1 
a

 

1
∫ t (b − a ) (1 − t ) (b − a )
1 p −1 q −1 q −1
p −1
…= xdx =…
β( p, q ).(b − a )
p + q −1 0

y siendo: x = (b − a )t + a y dx = (b − a )dt ⇒ xdx = [(b − a )t + a ]dt (b − a )

∫ t (b − a ) (1 − t ) (b − a ) [(b − a )t + a]dt (b − a ) =
1 1
p −1 p −1 q −1 q −1
…=
β( p, q ).(b − a )
p + q −1 0

(b − a ) p+ q−1
p + q −1 ∫0
t p −1 (1 − t ) [(b − a )t + a]dt = …
1 q −1
…=
β( p, q ).(b − a )

1  1 t p −1 (1 − t ) q −1 (b − a )tdt + 1 t p −1 (1 − t ) q −1 adt  = …
β( p, q )  ∫0 ∫0
…=

3/9
www.eltemario.com Oposiciones Secundaria – Matemáticas
©Antonio J. Abrisqueta Valcárcel.2002. Hojas de Problemas – Estadística II

b−a 1 p
t (1 − t ) q −1 dt + t p −1 (1 − t )q −1 dt = …
a
∫ ∫
1
…=
β( p, q ) 0 β( p, q) 0

b−a b−a
β( p + 1, q ) + β( p, q ) = β( p + 1, q ) + a = …
a
…=
β( p, q ) β( p, q) β( p, q)

p ( p + 1). p (q )
β( p + 1, q ) p ( p + q + 1)
…= (b − a )
p
+ a = (b − a) + a = (b − a ) + a =…
β( p, q) p ( p). p (q ) p+q
p( p + q)

bp − ap + ap + aq bp + aq
…= =
p+q p+q

222.- Sea la función de densidad: f (x1 , x 2 , x3 , x4 ) = 16 x1 x 2 x 3 x 4 0 ≤ xi ≤ 1 de


la variable aleatoria (X1 ,X2 ,X3 ,X4 ). Calcular:

a) Probabilidad de obtener un punto en la región x1 <1/2 , x4 >1/3

b) Distribución marginal de x1

c) Distribución marginal de la variable (X2 ,X3 ,X4 ).

d) Distribución de (X1 ,X2 ) condicionada por X3 =a y X4 =b.

e) Distribución de X1 condicionada a (X2 ,X3 ,X4 )

f) E(X1 | (X2 ,X3 ,X4 ))

Sol.:

P( x1 < 1 2 , x4 > 1 3) = ∫ ∫∫∫


1/ 2 1 1 1
a) 16 x1 x2 x 3 x 4 dx4 dx 3 dx2 dx1 = …
0 0 0 1/3

1
 x42  64
…= ∫ ∫0 ∫0  1 2 3 2  .dx 3dx 2 dx1 = ∫ ∫∫
1/ 2 1 1 1/ 2 1 1
16 x x x x1 x 2 x3 dx3 dx2 dx1 = …
0 0 0 0 9
x3

1
 64 x32  32 16
…= ∫ ∫0  9 1 2 2  dx 2 dx1 = ∫ ∫0 9 x1 x 2 dx 2dx1 = ∫
1/ 2 1 1/ 2 1 1/ 2
x x x1 dx1 = …
0
0
0 0 9

1/2
16  x 2  16 1 4 16 1 2
…=  1  = × = × =
9  2 0 9 21 9 8 9

b) Distribución marginal de X1 :

4/9
www.eltemario.com Oposiciones Secundaria – Matemáticas
©Antonio J. Abrisqueta Valcárcel.2002. Hojas de Problemas – Estadística II

∫∫ ∫ 16 x1 x 2 x3 x4 dx 4 dx3 dx 2 = ∫ ∫ 8 x1 x2 x 3dx 3dx 2 = ∫ 4 x1 x 2 dx 2 = 2 x1


1 1 1 1 1 1
f x1 =
0 0 0 0 0 0

c) Distribución marginal de (X2 ,X3 ,X4 )


1
 x2 
f ( x 2 .x 3 , x4 ) = ∫16 x1 x 2 x3 x 4 dx1 = 16 1 x 2 x3 x 4  = 8 x2 x3 x4
1

0
 2 0

d) Distribución de (X1 ,X2 ) condicionada por X3 =a y X4 =b.

f (( x1 , x 2 )  ( x3 = a, x4 = b ))

f (x1 , x2 ) = ∫ ∫ 16 x1 x 2 x 3 x4 dx2 dx1 =∫ 8x1 x3 x4 dx1 = 4 x3 x4


1 1 1

0 0 0

f (( x1 , x 2 )  ( x3 = a, x4 = b )) =
f ( x1 , x2 , a , b) 16 x1 x 2 ab
= = 4 x1 x2
f ( x 3 = a, x 4 = b) 4ab

e) Distribución de x condicionada (X2 ,X3 ,X4 ).

f ( x1 , x 2 , x 3 , x 4 ) 16 x1 x2 x3 x 4
F(x 1 |(x 2 ,x3 ,x4 )) = = = 2 x1
f ( x2 , x3 , x4 ) 8 x 2 x3 x 4

1
 x3  2
E(X1 |(X2 ,X3 ,X4 )) = ∫0 x1 (2 x1 ) dx1 = 2 1  =
1
f)
 3 0 3

223.- Una máquina fabrica ejes de diámetro x y otra cojinetes de diámetro interior
y. Suponiendo que la función de densidad conjunta sea de la forma:
f(x,y)=e− (x+y) x>0 , y>0

Calcular: a) Distribución marginal de x d) E(y)


b) Distribución marginal de y e) V(x)
c) E(x) f) V(y)
Sol.:

a)
+∞
f X = ∫0 e −( x + y ) dy = lim ∫e
t → +∞ 0
t
−( x + y )
t → +∞
[ ]
t
dy = lim − e −( x + y ) 0 = lim − e −( x+ t ) + e − x = e − x
t →+∞

b)
+∞
f Y = ∫0 e −( x + y ) dx = lim ∫e
t → +∞ 0
t
−(x+y)
t → +∞
[ ]
t
dx = lim − e −( x+ y ) 0 = lim − e −( t + y ) + e − y = e − y
t →+∞

+∞
E ( x ) = ∫0 x.e −( x+ y ) dx = lim ∫ xe
t
e) −x
dx = (*)
t →+∞ 0

5/9
www.eltemario.com Oposiciones Secundaria – Matemáticas
©Antonio J. Abrisqueta Valcárcel.2002. Hojas de Problemas – Estadística II

u = x du = dx 
resolviendo la integral: ∫0
t
xe −x
dx =  −x 
= − xe − x [ ] +∫ e
t t
−x
dx
dv = e dx v = −e 
−x 0
0

y sustituyendo en (*), tendremos:

. (*) …= lim − xe − x
t →+∞
[ ] + lim ∫ e
t
0
t →+∞ 0
t
−x
dx =…(**)

Primer término: [
lim − xe − x ]
t −t
= lim  t 

=   l' Hôpital = lim − t = 0
1
∞
0
t →+∞
 
t → +∞ e t → +∞ e

Segundo término: lim


t →+∞ 0 ∫e
t
−x
dx = lim − e − x
t →+∞
[ ]t
0 = lim − e −t + 1 = 1
t → +∞

y sustituyendo en (**), resulta: [


E ( x ) = lim − xe − x 0 + lim
t → +∞
] t
∫e
t → +∞ 0
t
−x
dx = 0 + 1 = 1

+∞ u = y du = dy 
E ( y ) = ∫0 y.e − y dy = lim ∫
t
d) ye −y
dy = …  −y 
=…
t → +∞ 0
 dv = e −y
dy v = − e 

t →+∞
[
…= lim − ye − y 0 + lim ]
t

t → +∞ 0 ∫e
t
−y
dy = lim
t →+∞
−t
t
+ lim e −t + 1 = 0 + 0 + 1 = 1
e t →+∞

e) Var(X)=E(X2 )−(E(X))2


E(X2 )= ∫0 x 2 e − x dx = lim ∫xe
t
2 −x
dx = …(#)
t →+∞ 0

 x 2 = u 2 xdx = du 
I = ∫ x e dx = … 
t
2 −x
resolviendo la integral:  =…
0
dv = −e − x dx v = e − x 

2 x = u 2dx = du 
…= ∫ x 2 e − x dx = x 2 e − x
t
[ ] +∫ e
t t
−x
2 xdx =…  −x 
=…
dv = −e dx v = e 
0
0
0 −x


[
…= − t 2 e −t +  − 2 xe − x ] +∫ e
t
0
0
t
−x
2dx  = − t 2 e −t − 2t e − t + 2 − e − t + 1

( )
y sustituyendo en (#): ( )
… = lim − t 2 e −t + lim − 2te − t + lim − 2e − t + 2 =… (##)
t →+∞ t → +∞
( ) t →+∞
( )

t →+∞
(
lim − t e 2 −t
) −t2 − 2t
= lim t = l' Hôpital = lim t = lim t = 0
t →+∞ e t →+∞ e
−2
t → +∞ e

− 2t −2
lim (− 2te − t ) = lim t
= l' Hôpital = lim t = 0
t →+∞ t →+∞ e t → +∞ e
−2
lim (− 2e −t ) = lim t
=0
t →+∞ t →+∞ e

6/9
www.eltemario.com Oposiciones Secundaria – Matemáticas
©Antonio J. Abrisqueta Valcárcel.2002. Hojas de Problemas – Estadística II

t →+∞
( )
(##) …= = lim − t 2 e −t − 2t e −t − 2e −t + 2 = 0 + 0 + 0 + 2 = 2 ⇒ Var(X)=2-1=1

 n   n −1   n − 2   m  n +1 
224.- Demostrar que:   +   +   + ......... +   =  
m  m   m   m   m + 1
Sol.:

  n   n − 1  m   n + 1 
Sea: A= n ∈ N   +   + ......... +   =  
 m  m   m   m + 1

a) Caso n=m+1

 m + 1  m  (m + 1 )! (m + 1 )m!+m! m!( m + 1 + 1) m + 1 ( m + 2)!  m + 2 


  +   = +1 = = × = = 
 m  m 1!.m! m! m! m + 1 ( m + 1)!  m + 1 

 n   n −1   n − 2   m  n +1 
* Supuesto válido para n ⇒   +   +   + ......... +   =  
m  m   m   m   m + 1

veamos si se cumple para n+1:

 n + 1  n   n − 1  n − 2  m n+ 2
  +   +   +  ......... +   = (?) =  
 m  m  m   m  m  m + 1

considerando la anterior:

 n + 1  n + 1  ( n + 1)! ( n + 1)!
  +   = + =…
 m   m + 1 (n + 1 − m)!m! ( n − m)!( m + 1)!

(m + 1)( n + 1)!+( n + 1 − m)( n + 1)! ( n + 1)! ( m + 1 + n + 1 − m)


…= = =…
( n + 1 − m)! (m + 1)! ( n + 1 − m)! ( m + 1)!

( n + 1)! ( n + 2) (n + 2)! n + 2
…= = =  
(n + 1 − m)!( m + 1)! ( n + 1 − m)! ( m + 1)!  m + 1

 n   n −1   n − 2   m  n +1 
⇒   +   +   + ......... +   =   ∀n∈Ð
m  m   m   m   m + 1

225.- La variable aleatoria (X,Y) tiene una distribución uniforme en el recinto


∈ 32
C = {(x,y)∈ 0<x<1 , x<xy<2}

7/9
www.eltemario.com Oposiciones Secundaria – Matemáticas
©Antonio J. Abrisqueta Valcárcel.2002. Hojas de Problemas – Estadística II

Y−X
Se introduce la variable aleatoria (U,V) por la transformación U=X y V=
2−Y
Calcular: a) la función de densidad de Y, f2 (Y) y la condicionada de Y por X=x,
f2/1 (Y/X)
b) la función de densidad g(U,V)

Sol.:

y=2 k si 0 < x < 1 x < y < 2


y=2 a) f ( x, y ) = 
y=x 0 en el resto
+∞ +∞

∫ ∫ kdydx = ∫ ∫
1 2
1= kdydx = …
y=1 −∞ − ∞ 0 x

1
 x2  3
…= k ∫ (2 − x) dx = k  2 x −  = k
1

0
 2 0 2
x=1 2
⇒ k=
3

2 / 3 si 0 < x < 1 x< y<2


f ( x, y ) = 
0 en el resto

Para hallar f 2 (y) tenemos que hacer un cambio en los límites de integraciónporque
tenemos que barrer la x en función de la y

y
2 2  2
f 2 ( y) = ∫
y
1 y 2 0<y<1 dx =  x  = y
0 x 1 0 3  3 0 3

0 y 1 1
2 2  2
f 2 ( y) = ∫ dx =  x  =
0 x y 1
0<y<2
0 3
3 0 3

 2
 y si 0 < y < 1
3
f 2 ( y) = 
 2
si 1 < y < 2
 3

f ( X ,Y ) 23 1
f 2 / 1 = (Y / X = x) = = =
f ( X = x ) 2 3 (2 − x ) 2 − x

4 − 2 x 2(2 − x )
dy = [ y ]2x = [2 − x ] = − x =
2 2 2 4 2
f1 (x ) = ∫
2
=
x 3 3 3 3 3 3 3

b) 0<u<1 (porque u=x)

8/9
www.eltemario.com Oposiciones Secundaria – Matemáticas
©Antonio J. Abrisqueta Valcárcel.2002. Hojas de Problemas – Estadística II

y− x y −u
v= = (*)
2− y 2− y
0−0
Lo más pequeño que puede ser v es para y→0 ⇔ x→0 luego: v= =0
2−0
y−x y−x
Lo más grande que puede ser v es para y→2 pues v= = =∞
2−2 0

2v + u
2v − vy = y − u ⇒ 2v + u = y + vy = y(1 + v ) y=
1+v

2v + u
u< <2 ⇒ 0 < v < +∞
1+ v

 (# ) 0 < u <1 0 < v


g ( u, v) = 
 0 en el resto

∂( x , y )
(#) g ( u, v) = f ( x, y ) •
∂(u , v )

1 0
2 −u 2 2 −u
y= 1 2(1 + v ) − ( 2v + u ) = ⇒ g ( u, v) = ×
(1 + v ) 2 3 (1 + v) 2
1+ v (1 + v) 2

9/9
www.eltemario.com Oposiciones Secundaria – Matemáticas
©Antonio J. Abrisqueta Valcárcel.2002 Hoja de Problemas – Estadística III

226.- Se considera una pareja de v.a. X,Y de la que se conoce la densidad f(x,y).
4
 (x + 3 y )e x≥0 y ≥0
−x−2 y

f ( x, y ) =  5
0 en otro caso

a) Probar que f(x,y) es una función de densidad.


b) Determinar las leyes marginales de X y de Y, así como las leyes condicionales
de Y si X=x y de X si Y=y
c) Calcular las E y las Var de las leyes marginales.
d) Calcular E(X.Y) obteniendo así la covarianza.

NOTA: Utilizar los siguientes resultados:


+∞ +∞ +∞ +∞

∫ 0
ue −u du = ∫ e − u du = 1
0 ∫
0
u 2 e − u du = 2! ∫0
u 3 e− u du = 3!

--------------------------------------------------------------------------------------------------------

+∞ +∞ +∞ 4
a) I= ∫
−∞
f ( x, y )dydx =∫
0 ∫
0 5
( x + 3 y )e − x −2 y dydx (*)

( x + 3 y )e − x− 2 y dy =  x ∫ e − x − 2 y dy + ∫ 3 ye − x − 2 y dy + = [x.I 11 + I 12 ]
+∞ 4 4 +∞ +∞ 4
I1 = ∫
0 5 5 0 0 
 5

+∞ +∞ 2 y = u  +∞ du
I11 = ∫ e − x − 2 y dy = e − x ∫ e − 2 y dy =   = e−x ∫ e−u =…
0 0
2dy = du  0 2

1 − x +∞ − u 1
…= e ∫ e du = e − x
2 0 2

+∞ +∞ 2 y = u  +∞ u du
I12 = ∫ 3 ye − x − 2 y dy = 3e − x ∫ ye −2 y dy =   = 3e ∫0
−x
e −u =…
0 0
2dy = du  2 2

3 +∞ 3
…= e − x ∫ ue − u du = e − x luego:
4 0 4

4  1 −x 3 −x  4  1 −x 3 −x 
I1 = x. e + e  =  xe + e 
5  2 4  5 2 4 

y sustituyendo en (*) resultará finalmente:

+∞ 4  1 −x 3 −x  4  1 +∞ − x 3 +∞ −x  4  1 3  4 5
I =∫  xe + e  dx =  ∫0 xe dx + ∫0 e dx  =  +  = × = 1
0 52 4  52 4  52 4 5 4

( x + 3 y )e − x − 2 y dy =  ∫ xe − x −2 y dy + ∫ 3 ye − x− 2 y dy 
+∞ +∞ 4 4 +∞ +∞
f x (X ) = ∫
0
f ( x , y ) dy = ∫ 0 5 5 0 0 

1/11
www.eltemario.com Oposiciones Secundaria – Matemáticas
©Antonio J. Abrisqueta Valcárcel.2002 Hoja de Problemas – Estadística III

4 +∞ 4 +∞ 4 1 4 3
…= xe − x ∫ e − 2 y dy + e − x ∫ 3 ye − 2 y dy = xe − x × + e − x ×
5 0 5 0 5 2 5 4

 4  1 − x 3 −x 
  xe + e  x ≥ 0
f x(X ) =  5  2 4 
 0
 en el resto

( x + 3 y) e − x −2 y dx =  ∫ xe − x −2 y dx + ∫ 3 ye − x − 2 y dx  = …
+∞ +∞ 4 4 +∞ +∞
f x (Y ) = ∫
0
f ( x , y ) dx = ∫
0 5 5 0 0 

4 +∞ 4 +∞ 4 4
…= e − 2 y ∫ xe − x dx + 3 ye −2 y ∫ e − x dx = e − 2 y ×1 + 3 ye − 2 y × 1
5 0 5 0 5 5

 4 −2 y

f x (Y ) =  5
(
e + 3 ye − 2 y y ≥0)
 0 en el resto

f ( x, y ) f ( x, y)
b) f y x (Y X = x ) = f x y ( X Y = y) =
f x(X ) f y (Y )

4 +∞  1 2 − x 3 − x 
x e + xe dx = 1 +  = × =
4 3 4 7 7
c) E( X ) = ∫ 
5 0 2 4  5  4 5 4 5

E (Y ) = ∫ ( ye + 3 y 2 e −2 y )dy =  +
4 +∞ −2 y 4 3 1 4 5
= × =1
5 0 5 4 2  5 4

d) Var(X) = E(X2 ) − E(X)2

--------------------------------------------------------------------------------------------------------

227.- Sean a y b ∈ 9 +. Sea X una variable aleatoria discreta con valores enteros
estrictamente positivos de manera que:
1 1
P( X = x ) = − si 1 ≤ x ≤ ab
a b
P( X = x ) = 0 si x > ab

a) ¿Qué condición deben verificar a y b para que la sucesión de término general


p ( x ) = P( X = x )
pueda ser considerada como ley de probabilidad de X.
b) Determinar la función de distribución F(X). Trazar su gráfica. ¿Cuáles son
las soluciones de la ecuación F(X)=1/2 (medianas de X).
c) Calcular E(X). ¿Qué valores habrá que dar a a y b para que se tenga
E(X)=7/2 ¿

2/11
www.eltemario.com Oposiciones Secundaria – Matemáticas
©Antonio J. Abrisqueta Valcárcel.2002 Hoja de Problemas – Estadística III

--------------------------------------------------------------------------------------------------------

1 1 1 1
a) P( X = x ) = − ≥0 ⇒ ≥ ⇒ b≥a
a b a b

b−a
∑1  a − b  = ab a − b  = ab ab  = 1
ab
1 1 1 1
∑X P ( X = x) = ⇒ b = a +1

k
1 1  b − a  b = a + 1 k
b) F ( X ) = P( X ≤ k ) = ∑  −  = k  = =
1  a b  ab  b-a = 1  ab

1/ab

k=1 k=2 k=3 k=ab

1 x ab a( a + 1) a 2 + a
F( X ) = = ⇒ ab = 2 x Mediana x= = =
2 ab 2 2 2

b − a ab 1 (ab )( ab + 1) ab + 1 7
E ( X ) = ∑ x −  =
ab
1 1
c) ∑ x= × = =
x= 1  a b ab x =1 ab 2 2 2

de donde: ab = 6 siendo b = a + 1 ⇒ a2 + a = 6

− 1 ± 1 + 24 − 1 ± 5 a = 2
a2 + a −6 = 0 ⇒ a= = =
2 2 a = −3 no válida

resultando finalmente: a= 2 y b =3

----------------------------------------------------------------------------------------------------------
228.- Sea ξ una variable aleatoria con distribución uniforme en [0,1]. Calcular la
distribución de la media para muestras de tamaño 2.
1 si 0 ≤ x1 ≤ 1 0 ≤ x2 ≤ 1
f ( x1 , x 2 ) = 
0 en el resto

X1 + X 2
Calcular la distribución para Z =
2
----------------------------------------------------------------------------------------------------------

 x + x2 
F ( z ) = P( Z ≤ z ) = P 1 ≤ z  = P( x1 + x 2 ≤ 2 z )
 2 

3/11
www.eltemario.com Oposiciones Secundaria – Matemáticas
©Antonio J. Abrisqueta Valcárcel.2002 Hoja de Problemas – Estadística III

x2
(0,1)
x1 + x 2 ≤ 2 z

x 2 ≤ 2 z − x1
z=1/2
1
x1 = 1 − x2 ⇒ z=
x1 2
0 2z (1,0)

Si z ≤0 ⇒ F ( z) = 0

Si 0 < z ≤ 1/ 2 ⇒

2z
 x12  7z2
(2 z − x1 )dx1 = 2 zx1 −  =
2 z − x1

∫ ∫ ∫
2z 2z
⇒ F ( z) = dx2 dx1 =
 2 0 2
0 0 0

0 ≤ x 1 ≤ 2 z − 1
A1 : 
0 ≤ x2 ≤ 1

2z - 1 ≤ x 1 ≤ 1
x 2=2z-x 1 A2 : 
0 ≤ x2 ≤ 2 z − x1
A1 A2

1
2z-1 si < z ≤1 F ( Z ) = A1 + A2
2
2 z −1 1 2 z −1
A1 = ∫ ∫ dx 2 dx1 = ∫ dx1 = 2 z − 1
0 0 0

1
2 z − x1  x2 
A1 = ∫ ∫ dx 2 dx1 = ∫ ( 2 z − x1 ) dx1 = 2 zx1 − 1 
1 1
= ...
2 z −1 0 2 z −1
 2  2 z −1

 1  ( 2 z − 1) 2  4z 2 + 1 − 4z
…=  2 z −  −  2 z (2 z − 1) −
1
 = 2 z − − 4 z 2 + 2 z + =…
 2  2  2 2

1 1
…= 2 z − − 4 z 2 + 2 z + 2 z 2 + − 2 z = −2 z 2 + 2 z
2 2


0 z≤0
 2
7 z 1
F (Z ) =  0< z ≤
 2 2
 1
4 z − 2 z − 1 2 < z ≤ 1
2

4/11
www.eltemario.com Oposiciones Secundaria – Matemáticas
©Antonio J. Abrisqueta Valcárcel.2002 Hoja de Problemas – Estadística III


0 z≤0

 1
f ( z ) = 7 z 0< z <
 2
 1
4 − 4 z 2
< z <1

----------------------------------------------------------------------------------------------------------
229.- Una máquina fabrica ejes de diámetro X y otra cojinetes de diámetro interior
Y. Suponiendo que la función de densidad conjunta sea de la forma:
f(x,y)=e− (x+y) x>0 , y>0

Calcular: a) Distribución marginal de X.


b) Distribución marginal de Y.
c) Distribución de Y condicionado por X=a.
d) Distribución de X condicionado por Y=b.
e) Curva de regresión de Y sobre X.
f) Curva de regresión de X sobre Y.
g) Coeficiente de correlación.
----------------------------------------------------------------------------------------------------------

a) f (x ) = ∫0
+∞
e − ( x + y ) dy = lim
t → +∞ 0 ∫e
t
−(x+ y)
t → +∞
[ ] t
dy = lim − e −( x + y ) 0 = lim − e −( x+ t ) + e − x = e − x
t →+∞

b)
+∞
f 2 ( y) = ∫0 e − ( x + y ) dx = lim
t → +∞ 0 ∫e
t
−( x + y )
t → +∞
[ ]t
dx = lim − e − ( x + y ) 0 = lim − e −( t + y ) + e − y = e − y
t →+∞

f ( x, y ) e −( x+ y ) e − y si y > 0
c) f 2 1 ( y X = x) = = −x = e −y f 2 1 ( y X = a) = 
f1 (x ) e 0 en el resto

f (x , y ) e − ( x + y ) e − x si x > 0
d) f 2 1 ( x Y = y) = = − y = e−x f 2 1 ( x Y = b) = 
f 2 ( y) e 0 en el resto

e) Curva de regresión de Y sobre X: y = m2 1 ( x)

+∞
m2 1 ( y x ) = ∫ ydF2 1 ( y x) = ∫ yf 2 1 ( y x ) dy = ∫ ∫
t
ye − y dy = lim ye − y dy = (*)
0 t → +∞ 0

u = y du = dy 
I = ∫0 ye − y dy = 
t

−y 
= − ye − y [ ] − ∫ −e
t t
−y
dy = − te −t + ∫ e − y dy = …
t

dv = e dy v = −e 
−y 0
0 0

…= − te −t − e −t + e 0 = −te − t − e − t + 1

(*)
t →+∞
(
…= lim − te − t − e − t + 1 = 1 ) ⇒ y=1

f) Curva de regresión de X sobre Y: x = m1 2 ( y)

5/11
www.eltemario.com Oposiciones Secundaria – Matemáticas
©Antonio J. Abrisqueta Valcárcel.2002 Hoja de Problemas – Estadística III

+∞
m1 2 ( x y ) = ∫ xdF1 2 ( x y) = ∫ xf1 2 ( x y) dx = ∫0 xe − x dx = lim ∫ xe
t
−x
dx = 1 ⇒ x=1
t → +∞ 0

σ xy Cov ( x, y )
g) Coeficiente de correlación de x,y : ρ= =
σx σ y (VarX .VarY )1 2
+∞
E ( x ) = ∫0 x. f 1 ( x ) dx = lim ∫ xe
t
−x
dx = 1 (ver problema 223)
t → +∞ 0

+∞
E ( y ) = ∫0 y. f 2 ( y ) dy = lim ∫
t
ye − y dy = 1
t →+∞ 0


E(X2 )= ∫0 x 2 e − x dx = lim ∫xe
t
2 −x
dx = …(#)
t →+∞ 0

 x 2 = u 2 xdx = du 
I = ∫ x 2 e − x dx = … 
t
 = ∫ x 2 e − x dx = x 2 e − x
t
[ ] +∫ e
t t
−x
2 xdx =…
dv = −e dx v = e 
− x − x 0
0 0 0


[
…= − t 2 e −t +  − 2 xe − x ] +∫ e
t
0
0
t
−x
2dx  = − t 2 e −t − 2t e − t + 2 − e − t + 1

( )
y sustituyendo en (#): ( )
… = lim − t 2 e −t + lim − 2te − t + lim − 2e − t + 2 =… (##)
t →+∞ t → +∞
( ) t →+∞
( )

t →+∞
(
L1 = lim − t 2 e − t = lim ) −t2
t → +∞ e t
− 2t
= l' Hôpital = lim t = lim t = 0
t → +∞ e
−2
t → +∞ e

− 2t −2
L2 = lim (− 2te − t ) = lim t
= l' Hôpital = lim t = 0
t →+∞ t →+∞ e t → +∞ e

−2
L3 = lim (− 2e − t ) = lim =0 y sustituyendo en (##)
t →+∞ t → +∞ et

(
(##) … = lim − t 2 e −t − 2t e −t − 2e −t + 2 = 0 + 0 + 0 + 2 = 2
t →+∞
)

E(Y2 )= ∫0 y 2 e − y dy = lim ∫
t
y 2 e − y dy = 2
t → +∞ 0

+∞ +∞ +∞
E(X Y)= ∫ ∫ xye −( x + y ) dydx = ∫ Idx = ... siendo
0 0 0

+∞ +∞
I= ∫0 xye − ( x + y ) dy = lim ∫ ∫
t
xye − x e − y dy = xe − x lim ye − y dy = xe − x
t → +∞ 0 t →+∞ 0

+∞ +∞ +∞
luego: E(X Y)= ∫ ∫ xye −( x+ y ) dydx = ∫ xe − x dx = 1
0 0 0

Cov(X,Y)=E(XY)−E(X).E(Y)=1−1=0 ⇒ ρ= 0

----------------------------------------------------------------------------------------------------------

6/11
www.eltemario.com Oposiciones Secundaria – Matemáticas
©Antonio J. Abrisqueta Valcárcel.2002 Hoja de Problemas – Estadística III

230.- Cierta empresa produce dos artículos A y B. Sea x una variable aleatoria que
mide la demanda mensual, en miles de unidades, de A e y la de B. Se sabe que la
función de densidad conjunta es:

 3x − y
 1< x < 2 1< y <3
f ( x, y ) =  5
0 en el resto

a) Estudiar la independencia entre la demanda mensual de A y B.


b) Calcular P(X-Y>0)
c) Sabiendo que la demanda de B es igual a 1500 unidades, calcular la
probabilidad de que la demanda de A sea al menos de 500 unidades.

----------------------------------------------------------------------------------------------------------

3
3x − y 3x 3 1  y 2 
[ y ]1 −   = …
+∞ 3x 3 1 3
f1 (x ) = ∫ f ( x, y ) dy = ∫
5 ∫1 5 ∫1
3
a) dy = dy − ydy =
−∞ 1 5 5 5  2 1

…=
3x
(3 − 1) − 1  9 − 1  = 3 x × 2 − 1 × 4 = 6 x − 4 = 6 x − 4
5 5 2 2  5 5 5 5 5

2
3x − y 3  x2 
dx = ∫1 xdx − ∫1 dx =   − [x ]1 = …
+∞ 3 2 y 2 y 2
f 2 ( y) = ∫−∞ f ( x , y ) dx = ∫1
2

5 5 5 5  2 1 5

9 y 9 − 2y
…=  −  − (2 − 1) = × − × 1 =
3 4 1 y 3 3 y
− =
5  2 2 5 5 2 5 10 5 10

f 1 ( x ) f 2 ( y ) =  − . −  =
6x 4 9 y 6x 9 6x y 4 9 4 y
× − × − × + × = ...
 5 5   10 5  5 10 5 5 5 10 5 5

27 6 18 4 27 x + 4 y − 6 xy − 18
…= x− xy − + y= ≠ f ( x, y)
25 25 25 25 25

⇒ X e Y independientes
b)
P( x − y > 0 ) = P( x > y ) = …
3
3x − y
…= ∫ ∫ dydx = ∫ I .dx
2 x 2
(*)
2 1 1 5 1
1<x<2
1<y<x
1 3x − y x 3x x y
I =∫ dy = ∫ dy − ∫ dy =...
x
siendo
1 5 1 5 1 5

1 2
x
3x y2  3x 2 x 2 3 x 1 5 x 2 − 6 x + 1
... =  y −  = − − + =
5 10  1 5 10 5 10 10

7/11
www.eltemario.com Oposiciones Secundaria – Matemáticas
©Antonio J. Abrisqueta Valcárcel.2002 Hoja de Problemas – Estadística III

2
5x 2 − 6 x + 1 x2 2 3x 2 1  x 3 3x 2 x 
luego: (*) = ∫ ∫ dx − ∫ dx + ∫
2 2
dx = dx =  − + =…
1 10 1 2 1 5 1 10
6 10 10  1

80 − 72 + 12   10 − 18 + 6  22 11
…=  − +  −  − +  = 
8 12 2 1 3 1
− = =
 6 10 10   6 10 10   60   60  60 30

3x − y 3x − 1'5
10(3 x − 1'5)
P( X ≥ 0'5 y = 1'5) = = 5 = { y = 1'5} =
f ( x, y ) 5 1'5
c) = = x−
f 2 ( y) 9 − 2y 9 −3 30 3
10 10

2
 1'5  2 1  x2 x  4 2 1 1
∫1  x − 3 dx = ∫1  x − 2 dx =  2 − 2 1 =  2 − 2  −  2 − 2  = 1
2

----------------------------------------------------------------------------------------------------------

231.- Sea la variable aleatoria con función de densidad:

k + xy 0 ≤ x ≤1 0 ≤ y ≤1
f ( x, y ) = 
0 en el resto

Calcular: a) f(x,y) y b) E(x+y)

----------------------------------------------------------------------------------------------------------

1 1
 xy2   x  x2 
1 = ∫0 ∫0 (k + xy)dydx = ∫0  ky +
1
∫0  2   4  = k + 4
1 1 1 1
dx =  +  = + ⇒
2  0
a) k dx kx
 0

1 3
⇒ k =1− =
4 4

1
3   3 x yx 2  3 y 3 + 2y
f 2 ( y) = ∫  + xy dx =  +
1
 = + =

0 4
 4 2 0 4 2 4

f ( x, y ) 4
3
+ xy
1
(3 + 4 xy ) 3 + 4 xy
f ( x y) = = = 4 =
3+ 2y
f 2 ( y) 1
(3 + 2 y ) 3 + 2 y
4 4

b) E ( x + y) = E ( x) + E ( y )

1
3 x 1 3x 1 x
2
 3  x 2  1  x3 
E ( x ) = ∫ xf1 ( x ) dx = ∫ x + dx = ∫ dx + ∫
1 1
dx =    +    = …
0 0
4 2 0 4 0 2
 4  2  2  3  0

8/11
www.eltemario.com Oposiciones Secundaria – Matemáticas
©Antonio J. Abrisqueta Valcárcel.2002 Hoja de Problemas – Estadística III

3 1 9 + 4 13
…=  (1 − 0 ) + (1 − 0 ) = + =
3 1
=
8 6  8 6 24 24

1
3 y  13y 1 y
2
 3  y 2  1  y 3 
E ( y ) = ∫ yf 2 ( y) dy = ∫ y  + dy = ∫ dy + ∫
1 1
dy =    +   = …
0 0
4 2  0 4 0 2
 4  2  2  3  0

3 1 9 + 4 13
…=  (1 − 0 ) + (1 − 0 ) = + =
3 1
=
8 6  8 6 24 24

13 13 26 13
E ( x + y ) = E ( x) + E ( y ) = + = =
24 24 24 12

----------------------------------------------------------------------------------------------------------

232.- Probar que no pueden existir dos variables x e y tales que E(X)=3, E(Y)=2,
E(X 2 )=10, E(Y 2 )=29 y E(X,Y)=0.

----------------------------------------------------------------------------------------------------------

Cov( X , Y ) −6 −6
ρ= = = < −1
(Var ( X ).Var (Y ) ) 12
(1× 25) 12
5

Cor ( X , Y ) = E ( XY ) − E ( X ) E (Y ) = 0 − 2 × 3 = −6

Var ( X ) = E ( X 2 ) − E( X ) 2 = 10 − 3 2 = 1

Var (Y ) = E(Y 2 ) − E (Y ) 2 = 29 − 2 2 = 25

ρ ≤1

----------------------------------------------------------------------------------------------------------

ξ ,η
233.- Sea (ξ η ) una variable aleatoria bidimensional con función de densidad:
 1 si 0 ≤ x ≤ y 0 ≤ y ≤ 2

f ( x, y ) =  2
0 otro valor

Calcular el coeficiente de correlación ρ y las rectas de regresión

----------------------------------------------------------------------------------------------------------

Cov( X , Y )
ρ=
(Var ( X ).Var (Y ) )1 2

9/11
www.eltemario.com Oposiciones Secundaria – Matemáticas
©Antonio J. Abrisqueta Valcárcel.2002 Hoja de Problemas – Estadística III

1 1 2
αn , m = ∫ ∫ dxdy = ∫ I .dy =…
2 y
xn ym (*)
0 0 2 2 0

y
1 1 m y n 1 m  x n +1  1 m y n +1 1 y m +n +1
I = ∫0 dx = y ∫0 x dx = y 
y
 = y n +1 = 2 × n +1
n m
siendo: x y
2 2 2  n + 10 2

2
1 1 2 m + n +1 1  y m+ n+ 2 
2 n + 1 ∫0
…= × = =…
2( n + 1)  m + n + 2  0
sustituyendo en (*): y dy

1 2 m+ n+ 2 1 2 m +n +1
…= × = ×
2(n + 1) m + n + 2 n + 1 m + n + 2

1 23
E ( XY ) = α11 = × =1
2 4

1 22 2
E ( X ) = α10 = × =
2 3 3

1 22 4
E (Y ) = α01 = × =
1 3 3

2 4 8 1
Cov( x, y ) = E ( XY ) − E( X ) E (Y ) = 1 − × =1− =
3 3 9 9

E ( X ) 2 =   =
1 23 2 2 4
E ( X 2 ) = α20 = × = y
3 4 3  3 9

2 4 6−4 2
Var ( X ) = E ( X 2 ) − E( X ) 2 = − = =
3 9 9 9

E (Y ) 2 =   =
1 23 4 16
E (Y 2 ) = α02 = × =2 y
1 4 3 9

16 2
Var (Y ) = E(Y 2 ) − E (Y ) 2 = 2 − =
9 9

1
9 1
Coeficiente de correlación: ρ= 1
=
2
 2 2 2
 × 
9 9

X − E( X ) =
Cov( X , Y )
(Y − E(Y ) )
Var (Y )

10/ 11
www.eltemario.com Oposiciones Secundaria – Matemáticas
©Antonio J. Abrisqueta Valcárcel.2002 Hoja de Problemas – Estadística III

2 19 4 2 1
=  y − 
4 1
x− = y −  ⇒ x− ⇒ x= y
3 2 9 3 3 2 3 2

Y − E (Y ) =
Cov( X , Y )
( X − E (X ))
Var ( X )

4 1
=  x − 
2 4 1 1 1
y− ⇒ y= + x− ⇒ y =1+ x
3 2 3 3 2 3 2
----------------------------------------------------------------------------------------------------------

11/ 11
www.eltemario.com Oposiciones Secundaria – Matemáticas
© Antonio J. Abrisqueta Valcárcel, 2002 Hojas de Problemas – Estadística IV

234. Tenemos tres monedas que llamaremos M1 , M2 , M3 . La moneda M 1 esta


equilibrada. La moneda M2 esta cargada y, al lanzarla, es dos veces más fácil que
salga cara que cruz. La moneda M3 esta cargada y, al lanzarla, aparece cara con
probabilidad ¼.
Si elegimos una moneda al azar y la lanzamos, ¿Cuál será la probabilidad de
que salga cara?

RESOLUCION.-

Llamemos M i al suceso “la moneda elegida es la i”, donde i puede ser 1, 2 ó 3.


Entonces, M 1 , M 2 y M 3 es una serie exhaustiva de sucesos incompatibles, puesto que
forzosamente alguno de ellos ocurre

M1 ∪ M 2 ∪ M 3 = Ω

y no pueden darse dos a un tiempo

M i ∩ M j = 0/ si i≠ j para i, j = 1, 2, 3

Si C es el suceso “al lanzar la moneda sale cara”, de la fórmula de la probabilidad total


se sigue
P(C ) = P ( M 1 ) P(C | M 1 ) + P( M 2 ) P (C | M 2 ) + P( M 3 ) P(C | M 3 ) (3.2)

Ahora bien, puesto que la elección entre monedas es al azar, se tendrá

1
P( M 1 ) = P( M 2 ) = P( M 3 ) =
3
Por otra parte, la moneda M 1 esta equilibrada, luego

1
P(C | M 1 ) =
2
La moneda M 2 es tal que resulta dos veces más fácil que salga cara que cruz, esta
condición significa que

P( C | M 2 ) = 2 P( C c | M 2 )

y como P( C | M 2 ) + P(C c | M 2 ) = 1 resultará

2
P( C | M 2 ) =
3
Por último, aplicando un razonamiento semejante aplicado a la moneda M 3 , revela que

1
P( C | M 3 ) =
4
Si se reemplazan estos valores en la ecuación 3.2, resulta

1/11
www.eltemario.com Oposiciones Secundaria – Matemáticas
© Antonio J. Abrisqueta Valcárcel, 2002 Hojas de Problemas – Estadística IV

11 12 11
P(C) = + +
32 33 34

235. Una urna, digamos I, contiene tres bolas blancas y cuatro bolas negras.
Extraemos al azar dos bolas de la urna I y, sin mira r su color las introducimos en
otra urna, digamos II, que está vacía. A continuación elegimos una bola al azar de
la urna I y otra, también al azar de la II. ¿Cuál es la probabilidad de que la bola
elegida de la urna II sea blanca? ¿Cuál es la probabilidad de que la bola elegida de
la urna I sea blanca? ¿Cuál es la probabilidad de que ambas bolas sean blancas?

RESOLUCION.-

El desarrollo de este experimento aparece descrito de manera gráfica en la figura.


Llamemos BI y BII , respectivamente, a los sucesos “la bola elegida de la urna I es
blanca” y “la bola elegida de la urna II es blanca”. La circunstancia que influye en la
facilidad de elegir una bola blanca de la urna II, es el número de bolas blancas entre las
dos traspasadas de I a II, puesto que determina la composición de la urna II.

Si llamamos A0 , A1 y A2 respectivamente a los sucesos “entre las bolas traspasadas


de la urna I hay 0, 1, 2 bolas blancas”, resulta evidente que, forzosamente, alguno de los
tres sucesos ocurre, y no pueden darse, dos cualesquiera, simultáneamente. Tenemos así
que A0 , A1 y A2 son una serie exhaustiva de sucesos mutuamente incompatibles.

Por otra parte, para que ocurra A0 , es preciso que la primera bola elegida de la urna
I sea negra –lo que ocurre con probabilidad 4/7 – y que extraída la primera negra, la
segunda también lo sea –lo que sucede con probabilidad 3/6 - Por lo tanto

43 2
P( A0 ) = =
76 7
de manera semejante se tienen

43 34 4 32 1
P( A1 ) = + = , P( A2 ) = =
76 76 7 76 7
Ahora, cuando ocurre A0 , la probabilidad de extraer una bola blanca de la urna II es 0.

P( B II | A0 ) = 0

De igual manera se tienen

1
P( B II | A1 ) =
, P( B II | A2 ) = 1
2
Todos estos cálculos se resumen en el esquema de la figura 3.5. Se tiene así:

3
P( B II ) = P( A0 ) P( BII | A0 ) + P( A1 ) P ( BII | A1 ) + P( A2 ) P( BII | A2 ) =
7

2/11
www.eltemario.com Oposiciones Secundaria – Matemáticas
© Antonio J. Abrisqueta Valcárcel, 2002 Hojas de Problemas – Estadística IV

También el número de bolas blancas traspasadas de la urna I a la II determina la


probabilidad de extraer, a continuación, una bola blanca de I. El caso, creemos, no
merece más detalles. No hay dificultad en obtener:
3 2 1
P( B I | A0 ) = , P( B I | A1 ) = , P( B I | A2 ) = ;
5 5 5
por lo tanto, se tiene:

3
P( B I ) = P ( A0 ) P( B I | A0 ) + P( A1 ) P( BI | A1 ) + P( A2 ) P( B I | A2 ) =
7

La última cuestión presenta, aparentemente, más interés. El suceso “ambas bolas son
blancas” se simboliza por C = BI ∩ BII . Afortunadamente la fórmula de la probabilidad
total no hace distinciones, C es un suceso más, cuya probabilidad -condicionada- de
ocurrir, está determinada por la aparición de A0 , A1 o A2 . El cálculo se hace mecánico,

P(C ) = P ( A0 ) P(C | A0 ) + P( A1 ) P(C | A1 ) + P( A2 ) P(C | A2 )

La pequeña dificultad adicional reside en el cálculo de las probabilidades


condicionadas. No es preciso pensar mucho para concluir:

P(C | A0 ) = 0

ya que no puede ocurrir que ambas bolas sean blancas si en la urna II sólo hay bolas
negras.

Calculemos las restantes. Supongamos que A1 ha ocurrido. Entonces la urna I


contiene dos bolas blancas y tres negras y, la urna II una blanca y otra negra. El
experimento consiste en elegir, al azar, una bola de cada urna. Otra vez imaginamos que
las bolas están numeradas. Hay 5 maneras distintas de elegir una bola de la urna I, y
elegida ésta, por cada elección de una bola de la urna I, hay dos maneras de elegir una
bola de la urna II. En total tenemos 5x2 casos posibles. ¿Cuántos serán favorables a que
ambas bolas sean blancas? Para que la bola elegida de la urna I sea blanca, es preciso
que sea una de las dos blancas que contiene. Hay pues dos maneras distintas de elegir
una bola blanca de la urna I. Hecha esa elección, por cada manera de elegir una bola
blanca de I, ha y una única manera de tomar una bola blanca de II. Por lo tanto, 2 casos
son favorables a BI ∩ BII . Se tiene así:

2 1
P(C | A1 ) =
=
10 5
Un razonamiento semejante permite concluir que se cumple:

2 1
P(C | A2 ) = =
10 5

Todos estos cálculos se resumen en el esquema de la figura 3.6, y la fórmula de la


probabilidad total produce:

3/11
www.eltemario.com Oposiciones Secundaria – Matemáticas
© Antonio J. Abrisqueta Valcárcel, 2002 Hojas de Problemas – Estadística IV

2 41 11 1
P( B I ∩ BII ) = 0+ + =
7 75 75 7

236. Dos personas A y B juegan alternativamente en una máquina recreativa,


hasta que uno de los dos obtiene premio. Las probabilidades de conseguir premio
en cada partida son pa y pb, respectivamente, y la primera partida la juega A.
¿Cuál es la probabilidad de que el juego termine en la k-ésima partida?

RESOLUCIÓN.-

Llamemos P(N = k) a la probabilidad de que el juego termine en la k-ésima partida.


Desde luego si k es impar y el juego termina, A ha ganado; mientras que si k es par, B
habrá ganado.

Ahora bien, A gana en la partida 2n – 1 si ninguno de los jugadores logra ganar en


ninguna de las 2n – 2 primeras partidas y gana en la (2n – 2) primeras partidas y gana en
la (2n – 1)-ésima. Esto ocurre con la probabilidad

(1 − p a )(1 − pb )...(1 − p a )(1 − p b ) pa = (1 − p a ) n −1 (1 − pb ) n −1 p a

n – 1 veces

Mientras que, por análoga razón, B gana en la partida 2n-ésima con probabilidad

(1 − p a )(1 − pb )...(1 − pa )(1 − pb )·(1 − p a )· p a = (1 − pa ) n (1 − p b ) n −1 pb

n – 1 veces

Luego
(1 − pa ) n −1 (1 − p b ) n −1 pa si k = 2n - 1
P( N = k ) = 
(1 − pa ) n (1 − p b ) n −1 pb si k = 2n

237. Un experimentador desea comprobar la eficacia de un tratamiento. Elige


diez parejas de individuos lo más parecidos entre si que sea posible y prueba del
tratamiento en uno de los individuos de cada pareja. El resultado de los tratados es
mejor ue el de los no tratados en 7 de las 10 parejas. ¿Justifica este resultado la
suposición de que el tratamiento es eficaz, al nivel de significación del 5%?

RESOLUCION.-

4/11
www.eltemario.com Oposiciones Secundaria – Matemáticas
© Antonio J. Abrisqueta Valcárcel, 2002 Hojas de Problemas – Estadística IV

Si el azar decidiera qué individuo, de cada pareja, iba a presentar e resultado más
favorable, escogiendo con igual probabilidad uno u otro, la probabilidad de que escoja a
7 o más individuos tratados es

 7  −10 10  −10 10  −10 10  −10


  2 +   2 +   2 +   2 ≅ 0.1719
 2 8  9  10 

Por lo tanto, aún cuando el tratamiento fuera ineficaz, sólo por causa del azar, casi un
20% de las veces esperamos resultados tan buenos o mejores que los obtenidos. El
resultado del experimento no aporta una evidencia estadísticamente significativa a favor
del tratamiento.

238. Dos candidatos A y B se presentan a una elección, si A recibe a votos y B


recibe b votos, a>b, ¿Cuál será la probabilidad de que en todo momento del
escrutinio A vaya por delante de B?

RESOLUCION.-

Entendamos bien el procedimiento del escrutinio. Los electores han depositado sus
votos en una urna. Sabemos que hay a votos favorables a A y b votos favorables a B. El
escrutinio consiste en extraer los votos uno a uno. La pregunta hace referencia al suceso
“en todo momento A tiene mas votos escrutados que B”.

Hay tantas maneras distintas de extraer los votos uno a uno como formas de elegir a
lugares entre a+b para que en ellos aparezcan los votos favorables de A. Por lo tanto hay

a + b
 
a 

maneras de extraer los votos. Además, es razonable suponer que todas son igualmente
probables.

En términos de trayectorias, si consideramos que cada vez que un voto es favorable


a A se suma +1 y cada vez que es favorable a B se suma -1, cada extracción posible se
identifica con una trayectoria que conduce de (0,0) a (a+b, a-b), puesto que,
necesariamente, al final son a+b los votos escrutados y la diferencia favor de A es a-b.

Por ejemplo, si a = 5, b = 3, la secuencia ABAAABBA, que significa: el primer voto


es favorable a A, el segundo a B, etc., se identifica con la trayectoria de la figura 4.6.
Una trayectoria será favorable a que A vaya siempre por delante de b si no toca ni cruza
al eje de las abscisas (todos los resultados parciales son positivos). En la figura 4.7, la
trayectoria representada a la izquierda es favorable al suceso y la de la derecha no.

Obsérvese que toda trayectoria favorable a que a vaya siempre ganando,


necesariamente, pasa por el punto (1, 1), (es necesario que el primer voto sea favorable
a A), pero no todas las trayectorias que pasan por (1, 1) son favorables, tan sólo aquellas
que no tocan al eje de abscisas.

5/11
www.eltemario.com Oposiciones Secundaria – Matemáticas
© Antonio J. Abrisqueta Valcárcel, 2002 Hojas de Problemas – Estadística IV

Por el principio de reflexión, hay tantas trayectorias que van desde (1, 1) hasta (a+b ,
a-b) y tocan al eje como trayectorias van desde (1, -1) hasta (a+b, a-b), es decir

 a + b − 1
 
a −1 

Por lo tanto, el número de trayectorias favorables será igual a las que conducen de (1, 1)
a (a+b, a-b), esto es:

 a + b − 1
 
a −1 

menos las que van de (1, -1) a (a+b, a-b), es decir

 a + b − 1  a + b − 1
  -  
a −1  a −1 

Luego la probabilidad de que A vaya siempre ganado es:

 a + b − 1  a + b − 1
  −  
 a −1   a  = a−b
a +b a+b
 
a 

Observemos que si A tiene el α % de los votos, entonces hay un 2 α -100% de


posibilidades de que vaya ganando en todos los resultados parciales.

Por el contrario hay 2(100- α )% de posibilidades de que alguna vez B esté ganando
o estén empatados. Esto nos previene contra las conclusiones apresuradas: la simple
información de que en algún momento B va ganando, o que ambos candidatos están
empatados, no permite deducir que los resultados finales serán favorables a B o parejos.
Aún cuando A tenga el 90% de los votos hay una probabilidad igual a 0,2 (uno de cada
cinco casos) de que en algún resultado parcial del escrutinio los dos candidatos estén
empatados o que tenga ventaja el candidato que ha recibido menos votos.

239. De una urna que contiene a bolas blancas y b bolas negras, dos jugadores
hacen extracciones con reemplazamiento alternativamente. Gana aquel jugador
que consigue extraer primero una bola blanca. Calcular las respectivas
probabilidades de que gane cada uno de los jugadores.

RESOLUCION.-

Sea Ai el suceso “la primera bola blanca aparece en la extracción i”. Se tendrá que

6/11
www.eltemario.com Oposiciones Secundaria – Matemáticas
© Antonio J. Abrisqueta Valcárcel, 2002 Hojas de Problemas – Estadística IV

i −1

P( Ai ) = 
b  a

a+b a+b

La probabilidad de que gane el jugador que extrae las bolas impares es:

2k

 b  a a 1 a( a + b ) b a+b
∑  
k =0  a + b 
=
a + b a + b 1 − (b / a + b) 2
=
(a + b) − b
2 2
=1− =
a + 2b a + 2b

b
y la probabilidad de que gane su contrincante es .
a + 2b

240. Al lanzar una moneda aparece cara con probabilidad p. Calcular la


probabilidad del suceso

Ak = {aparecen k cruces antes de la n - ésima cara}

RESOLUCIÓN.-

Evidentemente, este suceso depende tan sólo de lo que ocurra en los n + k primeros
lanzamientos. De las 2n+k realizaciones posibles de lanzar una moneda n + k veces, son
favorables a Ak las que tienen la estructura:

................................... C

k cruces y n – 1 caras

Esto es, el último resultado (el (n + k)-ésimo) debe ser cara, precisamente la n-ésima
cara. Mientras que en los n + k – 1 primeros resultados deben aparecer n – 1 caras y k
cruces. Habrá tantas realizaciones con esta estructura como maneras distintas de elegir k
lanzamientos, entre los n + k – 1 primeros, para que en ellos aparezcan las cruces. Así
pues, hay
 n + k − 1
 
 k 

trayectorias favorables.

Encontramos aquí la misma circunstancia que en el cálculo de las probabilidades


binomiales: todas las trayectorias favorables tienen la misma probabilidad. En efecto,
sea cual sea el orden en que aparecen las k cruces entre los n + k – 1 primeros
lanzamientos, la probabilidad de que aparezca tal secuencia es pn (1 – p)k . Luego la
probabilidad de Ak será igual al número de trayectorias favorables por la probabilidad
común, esto es

 n + k − 1 n
P( Ak ) =   p (1 − p) k
 k 

7/11
www.eltemario.com Oposiciones Secundaria – Matemáticas
© Antonio J. Abrisqueta Valcárcel, 2002 Hojas de Problemas – Estadística IV

Por último, al lanzar repetidas veces la moneda tarde o temprano aparecerá la


n-ésima cara (recordemos que todo suceso con probabilidad positiva de ocurrir, tarde o
temprano ocurre). Cuando tal cosa suceda habrán aparecido cierto número k, k = 0,1,…,
de cruces, en este razonamiento probabilístico prueba la interesante identidad
combinatoria:


 n + k − 1 n
P(aparezca la n-ésima cara) = ∑   p (1 − p) k = 1 (1)
k =0  k 

El conjunto de probabilidades

 n + k − 1 n
  p (1 − p ) k para k = 0,1,...
 k 

se denomina probabilidades binomiales negativas. Esta denominación se explica si


recordamos que los coeficientes binomiales pueden extenderse a los números reales; por
ejemplo, si x es un número real y k un entero positivo, se tiene

 x  x( x − 1)( x − 2)...( x − k + 1)
  =
k  k!

Con esta convenció n puede darse la fórmula generalizada del binomio de Newton.


r
Si r es real y x < 1, entonces (1 + x ) r = ∑   x k
k =0  x 

También con esta convención, la expresión de las probabilidades binomiales


negativas puede escribirse

 n + k − 1 n − n
  p (1 − p ) k =   p n ( p − 1) k
 k   k 

de donde se deriva el término bino miales negativas.

La identidad anterior es útil. Por ejemplo, permite deducir que la suma de las
probabilidades binomiales negativas es uno, sin emplear el razonamiento probabilística
que demostró la identidad (1). En efecto;


 n + k − 1 n ∞
− n
∑ 
 k   p (1 − p ) k
= p n
∑  ( p − 1) k = p n [1 + ( p − 1)]− n = 1
k=0   k =0  k 

A la identidad (1), la denominaremos identidad binomial negativa.

241. Dos jugadores disputan un torneo de ajedrez, que concluye cuando uno de
ellos alcanza c victorias. En cada partida el jugador A tiene la probabilidad p de

8/11
www.eltemario.com Oposiciones Secundaria – Matemáticas
© Antonio J. Abrisqueta Valcárcel, 2002 Hojas de Problemas – Estadística IV

ganar, mientras que B tiene la probabilidad q, donde p + q = 1. Calcular la


probabilidad de que A gane el torneo.

RESOLUCIÓN.-

Para que A gane el torneo es preciso que la c-ésima victoria de A ocurra antes de la
c-ésima victoria de B. Así pues, es conveniente descomponer el suceso {A gana},
atendiendo al número i de victorias obtenidas por B 0 ≤ i < c y al número k (0 ≤ k < ∞ )
de empates ocurridos cuando A logra su c-ésima victoria.

Llamemos Ai,k al suceso “A gana el torneo, habiendo hecho k veces tablas y dejando
a B con i victorias”, 0 ≤ i < c .

Las realizaciones del torneo que son favorables a Ai,k, constan de i + k + c


resultados; tienen una victoria de A en la última posición y, entre las i + k + c
resultados; tienen una victoria de A en la última posición y, entre las i + k + c – 1
posiciones anteriores, hay i victorias de B y k tablas.

Por lo tanto hay

 i + k + c − 1 i + c − 1
  
 k  i 

realizaciones favorables, cada una con probabilidad p c q i (1 − p − q ) k . Luego

 i + k + c − 1 i + c − 1 c i
P( Ai , k ) =    p q (1 − p − q ) k
 k  i 

y la probabilidad de que A gane el torneo será

c− 1 ∞ c −1
 i + c − 1 c i  ∞  i + k + c − 1 
P( A gana ) = ∑∑ P( Ai , k ) = ∑   p q ∑   (1 − p − q) k 
i =0 k = 0 i= 0  i   k =0  k  

Ahora bien, si en la identidad binomial negativa del ejemplo anterior tomáramos


i + c como n, y p + q como p, resulta


 i + k + c − 1 ∞
 i + k + c − 1
∑ ∑ 
1
 (1 − p − q) k = (1 − p − q ) k ( p + q ) i +c =
k =0  k  ( p + q) i +c k=0  k 
1
=
( p + q ) i+ c

luego

 i + c − 1 c i c −1 i + c − 1
  c i p 
c
 q 
i
c− 1
1
P( A gana ) = ∑   p q = ∑   p q    
i =0  i  ( p + q) i +c
i= 0  i   p +q  p+ q

9/11
www.eltemario.com Oposiciones Secundaria – Matemáticas
© Antonio J. Abrisqueta Valcárcel, 2002 Hojas de Problemas – Estadística IV

242. Una urna contiene 3 bolas marcadas con los números 0, 1 y 2. Si extraemos
n + 1 bolas con reemplazamiento, hallar la probabilidad de que en k ocasiones
0 ≤ k ≤ n , el resultado de una extracción sea mayor que el de la primera.

RESOLUCIÓN.-

Llamaremos X al resultado de la primera extracción, y Ak al suceso “en k


extracciones resulta un número mayor que X”, entonces

n −k
n 2
k

P( Ak X = 0) =    
1
para k = 0, 1, ..., n
 k  3   3 
n −k
 n  1   2 
k

P( Ak X = 1) =     para k = 0, 1, ..., n


 k  3   3 
0 para k = 0, 1, ..., n
P( Ak X = 0) =
1 para k = 0

Luego para cada k, k = 1, 2, …, n, se tiene

n− k n− k
1  n  2   1  1  n  1   2   n  2 k + 2 n −k
k k

P( Ak ) =      +      =   n +1
3  k  3   3  3  k  3   3  k  3

n n

P( A0 ) =   +   +
1 1 1 2 1
3 3 3  3  3

243. De una urna que contiene inicialmente una bola blanca y otra negra, un
jugador debe realizar dos extracciones con reemplazamiento. Si obtiene dos bolas
blancas gana. En caso contrario, vuelve a intentarlo después de añadir una nueva
bola negra a la urna si se continúa el juego indefinidamente, hallar la probabilidad
de que el jugador consiga ganar.

RESOLUCIÓN.-

Sea Bn el suceso “se obtienen 2 bolas blancas en el n-ésimo par de extracciones”; es


decir, en las que se realizan cuando la urna contiene n + 1 bolas, n negras y una blanca.
Entonces, se tiene:

1
P( Bn ) =
( n + 1) 2

y la probabilidad de ganar el juego en el n-ésimo par de extracciones es:

10/11
www.eltemario.com Oposiciones Secundaria – Matemáticas
© Antonio J. Abrisqueta Valcárcel, 2002 Hojas de Problemas – Estadística IV

P(Gn ) = P ( B1c ∩ B2c ∩ ... ∩ Bnc−1 ∩ B nc ) =


 1  1   1   1  1
= 1 − 2 1 − 2 ... 1 − 2 ...1 − 2  =
 2  3   k   n  ( n + 1)
2

1·3 2·4 ( k − 1)( k + 1) ( n − 1)( n + 1) 1 1


= 2 2 ... ... =
2 3 k 2
n 2
( n + 1) 2
2n (n + 1)

Por lo tanto, la probabilidad de ganar el juego es:

N
1 1 N 1 1 
lim ∑ = lim ∑  − =
( n + 1) 
n =1 2n( n + 1) n =1  n
N→∞ N→∞ 2

1 1  1
= lim 1 − =
N→∞ 2  N +1  2

11/11
www.eltemario.com Oposiciones Secundaria – Matemáticas
© Antonio J. Abrisqueta Valcarcel, 2002 Hojas de Problemas – Estadística V

244. Consideremos tres urnas que llamaremos I, II y III. Cada una de ellas
contiene a bolas blancas y b bolas negras. Extraemos una bola al azar de la urna I
y la introducimos en la urna II, a continuación extraemos una bola al azar de la
urna II y la introducimos en la urna III. ¿Cuál es el número esperado de bolas
blancas en la urna III después de las dos operaciones?

RESOLUCION.-

Desde luego, en la urna III habrá a ó a + 1 bolas blancas. Para hallar el número
esperado nos resta encontrar las probabilidades con que aparecen cada uno de esos
valores. El cálculo no es muy complicado, basta seguir el esquema de la figura 5.1. Para
que en la urna III haya a bolas blancas es preciso que la bola elegida de la urna II sea
negra, lo que tiene una probabilidad igual a:

a b b b +1
· + ·
a + b a + b +1 a + b a + b +1

(elegir una blanca de I y una negra de II o elegir una negra de I y una negra de II)

Mientras que la urna III tendrá a + 1 bolas blancas si la elegida de la urna II es


blanca, lo que tiene una probabilidad de ocurrir igual a:

a a +1 b a
· + ·
a + b a + b +1 a + b a + b +1

Por lo tanto, el número esperado de bolas blancas en III será

a (a + 1) + ba ab + b (b + 1) a( a + 1) ab a ( a + b + 1)
( a + 1) +a = + =
( a + b)( a + b + 1) ( a + b)( a + b + 1) a+b a +b a+b

245. (Más general que el ejercicio anterior) n urnas contienen, cada una, a bolas
blancas y b negras. Se toma una bola al azar de la primera urna y se introduce en
la segunda; luego, se toma una de la segunda y se introduce en la tercera y así
sucesivamente, hasta que se extrae una bola de la urna (n – 1)-ésima y se introduce
en la n-ésima. Hallar el número esperado de bolas blancas que habrá en la última
urna.

RESOLUCION.-

Desde luego en la urna n-ésima habrá a ó a + 1 bolas blancas. La probabilidad de


que haya a bolas blancas es igual a la probabilidad de extraer una bola negra de la urna
(n – 1)-ésima. Análogamente, la probabilidad de tener a + 1 bolas blancas en la última
urna es igual a la de extraer una bola blanca de la urna (n – 1)-ésima. Calcularemos
primero estas probabilidades:

en la primera extracción resulta blanca con probabilidad

1/12
www.eltemario.com Oposiciones Secundaria – Matemáticas
© Antonio J. Abrisqueta Valcarcel, 2002 Hojas de Problemas – Estadística V

a
p1 =
a+b

La probabilidad de obtener bola blanca en la segunda extracción es

a +1 a
p 2 = p1 + (1 − p1 ) = p1
a + b +1 a + b +1

Puesto que con probabilidad p1 hay a + 1 bolas blancas, y en tal caso la probabilidad de
que resulte blanca en la segunda extracción es (a + 1)/(a + b +1), y con probabilidad (1 -
p1 ) hay a bolas blancas en la segunda urna antes de la segunda extracción.

La igualdad p1 = p 2 sugiere que la probabilidad de extraer una bola blanca de una


urna es constante, independientemente de la urna. Tratemos de probarlo por inducción.
Supuesto que la probabilidad de obtener bola blanca en la extracción de la urna (k – 1)-
ésima es p k −1 = p1 , la probabilidad de extraer una bola blanca de la urna k-ésima será

a +1 a
p k = p k −1 + (1 − p k −1 )
a + b +1 a +b +1
a +1 a
= p1 + (1 − p1 )
a + b +1 a + b +1
a a +1 b a
= · + ·
a + b a + b +1 a + b a + b + 1
= p1

Luego la probabilidad de extraer una bola blanca de cualquier urna es constante e igual
a a/(a + b).

Por lo tanto, en la urna n-ésima habrá a + 1 bolas blancas con probabilidad a/(a + b)
y a bolas blancas con probabilidad b/(a + b), de donde el número esperado de bolas
blancas será

a b a( a + b + 1)
( a + 1) +a =
a+b a+b a+b

246. Una urna contiene a bolas blancas y b bolas negras. Se hacen extracciones sin
reemplazamiento hasta que aparece la primera bola blanca. Hallar el número
esperado de bolas extraídas.

RESOLUCION.-

Sea N el número de la extracción en que se obtiene la primera bola blanca. Se tendrá


N = i, si las i – 1 primeras son negras y l i-ésima es blanca. Por lo tanto, para i = 1, 2,…,
b + 1 será

2/12
www.eltemario.com Oposiciones Secundaria – Matemáticas
© Antonio J. Abrisqueta Valcarcel, 2002 Hojas de Problemas – Estadística V

a + b −i 
 
b b −1 b −i + 2 a  a − 1 
P( N = i ) = L =
a + b a + b −1 a + b − i + 2 a + b − i +1 a +b
 
 a 

y el número esperado, µ, de bolas extraídas será igual a

−1
 a + b b +1
a + b − i
µ =   ∑  ·i
a − 1 
 a  i =1

Ahora bien

b +1 a + b − i
1   1 b +1
a + b − i
∑ 
 a + b  i =1  a − 1 
·i = ∑
 a + b  i =1
(( a + b + 1) − ( a + b − i + 1))  
a − 1 

   
 a   a 
b +1 
1 a + b − i a + b − i
= ∑
 a + b  i =1 
 ( a + b + 1) 
a −1 
 − ( a + b − i + 1) 
a − 1  
 
 
 a 
−1
a + b  b +1 a + b − i
  b +1 a + b − i + 1 
 
=   ( a + b + 1) ∑   − a ∑  
 a   i =1  a −1  i =1  a 

Cambiamos los índices de las sumas: en la primera hacemos j = a + b –i y en la segunda


j = a + b –i +1. Entonces se tiene

−1
 a + b  a +b −1
 j  a +b  j  
µ =    (a + b + 1) ∑   − a ∑   

 a   j = a −1  a − 1 j= a  a  

y, puesto que

n
i  n +1
∑  k  =  k + 1
i= k

resulta:

−1
 a + b   a + b   a + b + 1 
µ =    (a + b + 1)   − a  

 a    a   a + 1 

a + b +1
de lo que se sigue µ = .
a +1

3/12
www.eltemario.com Oposiciones Secundaria – Matemáticas
© Antonio J. Abrisqueta Valcarcel, 2002 Hojas de Problemas – Estadística V

247. k bolas se colocan al azar en n urnas. Calcular el número esperado de urnas


que quedan vacías.

RESOLUCIÓN.-

Para cada urna, por ejemplo la i- ésima, consideramos una variable X i que toma el
valor 1 cuando está vacía y el valor 0 cuando está ocupada. El número N de urnas vacías
será igual a
N = X1 + X 2 + K + X n

Ahora, la probabilidad de que una urna fijada, por ejemplo la i-ésima, esté vacía es igual
a:
n −1
k

P( X i = 1) =  
 n 

ya que cada bola puede ser colocada con igual probabilidad en cualquier urna, hay

n1 L3n
·n2
k _ veces

maneras posibles de colocar las k bolas en las n urnas; ahora bien, para que la urna i-
ésima esté vacía, es preciso que las k bolas se coloquen en las n – 1 urnas restantes, lo
que puede hacerse de (n – 1)k maneras.

(n − 1) k
Así, el valor esperado de X i es , y el número esperado de urnas vacías será
nk

n −1 
k

n· 
 n 

248. Una urna contiene a bolas blancas y b bolas negras, extraemos k bolas sin
reemplazamiento, 0 ≤ k ≤ a + b , y sin mirarlas las apartamos. A continuación
elegimos al azar una bolas de la urna, ¿Cuál es la probabilidad de que sea blanca?

RESOLUCION.-

Llamemos N al número de bolas blancas que hay entre las k elegidas y µ al número
medio de bolas blancas elegidas. Entonces, la urna contiene a – N bolas blancas y b – k
+ N bolas negras.

La probabilidad de que la bola elegida sea blanca es igual a

a a −1 a µ
P( N = 0) + P ( N = 1) + L = −
a+b−k a+b−k a +b − k a + b − k

4/12
www.eltemario.com Oposiciones Secundaria – Matemáticas
© Antonio J. Abrisqueta Valcarcel, 2002 Hojas de Problemas – Estadística V

en consecuencia, el razonamiento en media es válido.

Entre las k bolas elegidas habrá en media ka /( a + b ) bolas blancas. Por lo tanto, en
media, la urna contiene a − (ka /( a + b)) = a( a + b − k ) /( a + b) bolas blancas, de un total
de a + b – k bolas. Por lo tanto, la probabilidad de que la bola elegida sea blanca es

a( a + b − k ) /( a + b ) a
=
a +b−k a+b

249. Al lanzar una moneda aparece cara con probabilidad p. Se lanza k veces la
moneda y se introducen en una urna tantas bolas blancas como caras se hayan
obtenido y tantas bolas negras como cruces. Después, se extraen r (r < k) bolas de
la urna si reemplazamiento. Hallar la distribución del número de bolas blancas
que se obtienen. Si se ha obtenido i bolas blancas, hallar la probabilidad de que en
la urna queden j bolas blancas.

RESOLUCION.-

Llamemos B1 al número de bolas blancas que contiene la urna y B2 al número de


bolas blancas que resultan de la extracción. Será

k 
P( B1 = b) =   p b (1 − p) k − b , para b = 0, ..., k
b

y, si i = 0, …, r, se tendrá
 b  k − b 
  
 i  r − i 
P( B2 = i B1 = b ) =
k
 
r 

siempre que r + b − k ≤ i ≤ b , para que haya suficientes bolas de cada color en la urna;
pues en caso contrario la probabilidad sería cero.

Entonces

−1
k 
k − r+ i
 b  k − b  k 
P( B2 = i ) = ∑   p b (1 − p) k − b    
b =i  b   i  r − i  r 
  k − b  b
k − r+ i r
= ∑    p (1 − p ) k −b
b =i  i  r − i 

r k−r
k − r l
=   p i (1 − p ) r −i ∑   p (1 − p) k − r − l
i  l =0  l 
r
=   p i (1 − p ) r −i
i 

5/12
www.eltemario.com Oposiciones Secundaria – Matemáticas
© Antonio J. Abrisqueta Valcarcel, 2002 Hojas de Problemas – Estadística V

de manera que la distribución del número de bolas blancas extraídas de la urna es


binomial de parámetros r y p.

El resultado puede razonarse sin ningún cálculo; supongamos que primero se


sortean los números de las r, entre las k bolas, que serán extraídas; una vez
determinados, se lanza una moneda por cada bola para decidir su color. La probabilidad
de que i sean blancas es igual a la probabilidad de que aparezcan i caras al lanzar r veces
una moneda. Por tanto:

r 
P( B2 = i ) =   p i (1 − p ) r − i
i

Para calcular la probabilidad de que en la urna hub iese i + j bolas blancas, supuesto
que en la extracción se han obtenido i, es decir

P( B1 = i + j B2 = i )

aplicaremos la definición de probabilidad condicionada.

P ( B1 = i + j ) P( B2 = i B1 = i + j )
P( B1 = i + j B2 = i ) =
P ( B2 = i )
 k  i+ j  i + j  k − i − j
  p (1 − p ) k −i − j   
i+  i  r − i
=
j 
r i k 
  p (1 − p ) r −i  
i  r 
k − r  j
=   p (1 − p) k − r − j
 j 

Lo que demuestra que el número de bolas blancas que quedan en la urna es


independiente del número de bolas blancas obtenidas y tiene distribución binomial de
parámetros k – r y p. Otra vez, la razón es evidente: si se sortean de antemano las k – r
bolas que quedarán en la urna, y se les atribuye color lanzando la moneda, el número de
bolas blancas en la urna será binimial e independiente del número de bolas blancas entre
las extraídas.

250. Un jugador lanza un dado y gana, en la primera tirada, si obtiene un


resultado múltiplo de 3. En caso contrario, tiene que continuar tirando hasta que
se repita el resultado del primer lanzamiento o se obtenga un múltiplo de 3. En el
primer caso el jugador gana y en el segundo pierde. Si llamamos N al número de
lanzamientos que realiza, hallar E{N}.

RESOLUCIÓN.-

6/12
www.eltemario.com Oposiciones Secundaria – Matemáticas
© Antonio J. Abrisqueta Valcarcel, 2002 Hojas de Problemas – Estadística V

El jugador gana en la primera tirada (N = 1) con probabilidad 1/3. Si no es así, desde


la segunda tirada hay probabilidad igual a 1/2 de que el juego termine en ese
lanzamiento (probabilidad 1/6 de que gane y 1/3 de que pierda); la distribución de N es:

k−1

P(N =1) =1/ 3, P(N =k) =   paracadak,k ≥ 2


21
32

por tanto, la esperanza matemática de N será igual a:

1  
k −1 k −1
1 ∞ 21 ∞
 1
E{ X } = + ∑k   = 1 + 2∑ k   
3 k=2 3  2  3  k=2  2 


El cálculo de la suma ∑k2 − ( k −1)
, es un ejercicio bastante elemental, pero no está de
K =2
más recordar algunos métodos. El más simple utiliza la misma idea que permite calcular
la suma de una progresión geométrica.

Consideremos la suma


S = ∑ nx n
n =1
donde 0 < x < 1, entonces

∞ ∞
S − xS = ∑ nx n − ∑ nx n +1
n =1 n =1

Si restamos los coeficientes de los términos en xn , resulta;

∞ ∞
(1 − x) S = x + ∑ ( n − ( n − 1)) x n = ∑ x n
n= 2 n =1

serie geométrica que suma x/(1 – x), luego se tiene:


x
S = ∑ nx n =
n =1 (1 − x ) 2

Otro procedimiento; consideremos la serie geométrica siguiente como función de x


x
f (x ) = ∑ x n = para 0 < x < 1
n =1 1− x

Puede demostrarse que la derivada de esta función es igual a la serie de las


derivadas de cada término. Se tiene así

7/12
www.eltemario.com Oposiciones Secundaria – Matemáticas
© Antonio J. Abrisqueta Valcarcel, 2002 Hojas de Problemas – Estadística V


f ' ( x ) = ∑ nx n −1 para 0 < x < 1
n =1

luego

x
f ' (x ) = =
(1 − x ) n=1
2 ∑ x n −1

de donde

x
(1 − x ) 2
= ∑
n =1
nx n

En cualquier caso, si aplicamos el resultado anterior a nuestra serie, resulta:


1/ 2
∑k2
k =1
−k
=
(1 − 1 / 2) 2
= 2;


por lo tanto, ∑ k2 − ( k −1)
= 3 y E{ X } = 7 / 3 .
k=2

Para una variable aleatoria X, que tome tan sólo valores enteros no negativos,
podemos dar una expresión alternativa para la esperanza matemática que resulta útil en
numerosas ocasiones. La prueba es bien simple, basta reordenar los términos de la serie:

∞ ∞
E{ X } = ∑ nP ( X = n) = ∑ nP( X = n)
n= 0 n =1
es decir

E{ X } = P( X = 1) + 2 P( X = 2) + 3P( X = 3) + ...

Los términos de esta suma pueden reordenarse

E{ X } = P( X = 1) + P( X = 2) + ... + P( X = 2) + P( X = 3) + ... + ...


luego
E{ X } = P( X > 0) + P ( X > 1) + P( X > 2) + ...

En resumen: Si X es una variable aleatoria que toma valores en los enteros no


negativos 0, 1, …, se tiene:


E{ X } = ∑ P( X > n)
n= 0

251. Una urna contiene n bolas numeradas de 1 a n. Tomamos r bolas sin


reemplazamiento. Si X es el menor de los números que aparecen en las bolas
extraídas, calcular la distribución de X y su esperanza matemática.

RESOLUCIÓN.-

8/12
www.eltemario.com Oposiciones Secundaria – Matemáticas
© Antonio J. Abrisqueta Valcarcel, 2002 Hojas de Problemas – Estadística V

Los valores posibles de la variable X son 1, 2, …, n – r + 1. Calcularemos primero


P( X > k ), k = 1,2,..., n − r . Para que X > k, ninguna de las r bolas que extraemos debe
pertenecer al conjunto {1, 2, …, k}, esto es: todas deben pertenecer al conjunto k + 1,
k + 2, …; en consecuencia, hay

n − k 
 
 r 

maneras distintas de elegir las r bolas, de suerte que el menor de los números que llevan
impresos sea mayor que k, y dado que hay

 n
 
r 

casos posibles –tantos como maneras distintas de elegir las r bolas que extraemos entre
las n que hay en la urna–, resulta

n − k 
 
 r 
P( X > k ) = , para 0 ≤ k ≤ n - r
n
 
r 
Por otra parte

P( X = k ) = P( X > k − 1) − P( X > k )
 n − k + 1  n − k 
  − 
 r   r 
=
 n
 
r 
luego la distribución está dada por

n − k 
 
 r − 1 
P( X = k ) = para 1 ≤ k ≤ n - r
n
 
r 

Para calcular la media de la variable emplearemos la expresión anterior, se tiene

9/12
www.eltemario.com Oposiciones Secundaria – Matemáticas
© Antonio J. Abrisqueta Valcarcel, 2002 Hojas de Problemas – Estadística V

n− r
E{ X } = ∑ P( X > k ) =
k=0
−1
 n n− r
n − k 
=   ∑  r 

r  k=0
−1
 n   n + 1
=    
 r   r + 1
n +1
=
r +1

puesto que

 r   r + 1  n   n + 1
  +   + ... +   =  
r  r   r   r + 1

252. Una urna contiene una bola blanca. Se lana un dado y se introducen en la
urna tantas bolas negras como indique el resultado. A continuación se extraen al
azar dos bolas, simultáneamente, de la urna. ¿Cuál es el número esperado de bolas
blancas entre las dos extraídas?

RESOLUCIÓN.-

Llamemos Xi a la variable aleatoria definida por

1 si la bola i - ésima es blanca


Xi
2 si la bola i - ésima es negra

El número de bolas blancas entre las dos extraídas será X1 + X2 . El número esperado
de bolas blancas será

E{ X 1 + X 2 } = E{X 1 } + E{X 2 }

Para calcular E{X1 } emplearemos la esperanza condicionada. Llamemos Y al


resultado de lanzar el dado. Si Y = k, entonces la urna contiene una bola blanca y k
negras, luego

E{ X 1 Y = k} = 0· P( X 1 = 0 Y = k ) + 1·P( X 1 Y = k )
k 1
= 0· +1
k +1 k +1
1
=
k +1
Por lo tanto

10/12
www.eltemario.com Oposiciones Secundaria – Matemáticas
© Antonio J. Abrisqueta Valcarcel, 2002 Hojas de Problemas – Estadística V

6
E{ X 1 } = ∑ E{ X Y = k}P(Y = k )
k =1
6
1 1
=∑
k =1 k + 1 6

=  + + ... + 
1 1 1 1
6 2 3 7

Por simetría, E{X2 } = E{X1 }. Se tiene así que el número esperado de bolas blancas es

1 1 1 1
 + + ... + 
3 2 3 7

Obsérvese cómo empleamos la esperanza condicionada para reducir la complejidad.


El problema se subdivide en varios más simples, la fórmula 6.4 permite obtener la
esperanza incondicional a partir de los resultados parciales.

253. (Sumas de un número aleatorio de variables) Se lanza un dado y, a


continuación, tantas monedas como indique el resultado del lanzamiento. ¿Cuál es
el número esperado de caras que se obtendrán?

RESOLUCIÓN.-

Llamemos N al resultado que aparece al al nzar el dado. La variable N indica el


número de monedas que se lanzarán, por ello, si llamamos S al número total de caras
que aparecen, S se puede descomponer como

S = X 1 + X 2 + ... + X N

donde Xi es una variable que toma el valor 1 si al lanzar la moneda i-ésima aparece cara
y el valor si aparece cruz.

Un error frecuente consiste en aplicar la linealidad de la esperanza a la


descomposición de S. Esto es, la expresión

E {S } = E{ X 1 + X 2 + ... + X N } = E { X 1 } + E{ X 2 } + ... + E{ X N }

es falsa. La linealidad de la esperanza es cierta cua ndo el número de sumandos es fijo.


Sin embargo la suma

S = X 1 + X 2 + ... + X N

tiene un número de sumandos aleatorio. Condicionemos por el valor de N. Si N = k,


entonces el número de sumandos es fijo: vale k, luego podemos aplicar la linealidad de
la esperanza y resulta

11/12
www.eltemario.com Oposiciones Secundaria – Matemáticas
© Antonio J. Abrisqueta Valcarcel, 2002 Hojas de Problemas – Estadística V

E{S N = k } = E{X 1 + X 2 + ... + X k }


= kE{X 1 }
= kp

Por lo tanto

6
E{S} = ∑ E{S N = k}P( N = k )
k =1
6
= p ∑ kP( N = k )
k =1

= pE{N}
= 3.5 p

Este resultado se puede hacer general.

12/12
www.eltemario.com Oposiciones Secundaria – Matemáticas
© Antonio J. Abrisqueta Valcárcel, 2002 Hojas de Problemas – Estadística VI

254. Calcular el número de veces que se debe lanzar una moneda de manera que se
tenga una probabilidad superior a 0,9 de que el cociente entre el número de caras y
el de lanzamientos esté comprendido entre 0,4 y 0,6.

SOLUCIÓN:

1 si sale cara.


Sea Xi = 

0 si sale cruz.

1
Xi se distribuye según la ley de Bernouilli de parámetro p = ; con ello
2
admitimos que la moneda es perfecta.

Si lanzamos la moneda n veces, sea


n
x = ∑xi
i =1
la variable aleatoria “número de caras que aparecen”.

1
X se distribuirá según la ley binomial B(n, ), luego:
2
n n
E[X] = y var[X] =
2 4
X
Para la variable se tiene:
n
E   = var   = 2 * =
X 1 X 1 n 1
n 2 n n 4 4n

X
Hemos de determinar n de modo que P( 0, 4 ≤ ≤ 0,6) ≥ 0,9
n
X X
⇒ P (−0,1 ≤ − 0,5 ≤ 0,1) ≥ 0,9 ⇒ P ( − 0,5 ≤ 0,1) ≥ 0,9
n n
Primer método.

Por la desigualdad de Bienaymé-Tchebychef que viene dada por


1
P( Y − µ ≤ λσ) ≥ 1 − 2 donde
λ
µ = E[Y] y σ = var [Y ] , λ = cte , λ > 0 .

Luego:

1/9
www.eltemario.com Oposiciones Secundaria – Matemáticas
© Antonio J. Abrisqueta Valcárcel, 2002 Hojas de Problemas – Estadística VI

1
1  0,01 = λ2 * ⇒ λ2 = 0,04 * n 
0,1 = λ  4n 
4n   10
 ⇒ ⇒n= = 250
1
0,9 = 1 − 2   0,04
 1 = λ2 * 0,1 ⇒ λ2 = 10 
λ

Segundo método.

Por el Teorema de De-Moivre-Laplace tenemos que si X sigue una ley Binomial


 1 ,n
 
2 

n
X −µ X − 2
= ∈ N (0,1)
σ n
4

n
X−
2 ∈ N ( 0,1) ⇒ P (0,4 ≤ X ≤ 0,6) = P (0,4 ≤ X ≤ 0,6n) = P (−0,1n ≤ X − n ≤ 0,1n =
n n 2
4
 n   n 
 0,1n X−  X− 
0 ,1n 0 ,1n
= P − ≤ 2 ≤  = P 2 ≤  ≥ 0,90 ⇒ por las tablas de la ley
 n4 n n 4  n n 4
 4   4 
   
N(0,1):

0,1n 1,64
≥ 1,64 ⇒ 0,2 n ≥ 1,64 ⇒ n ≥ ⇒ n ≥ 8,2
n4 0, 2
⇒ n ≥ (8, 2) 2 ⇒ n ≥ 67,24 ⇒ n ≥ 68

255. En un proceso de fabricación se sabe que la probabilidad de fabricar una


pieza defectuosa es 0,0001. Si se fabrican 20.000 piezas por semestre ¿cuál es la
probabilidad de que el número de piezas defectuosas en la producción de un
determinado semestre sea al menos 3?.

SOLUCIÓN:

Aproximado Por una distribución de Poisson de λ = np = 20.000 * 10 −4 = 2 queda:

 2 0 21 2 2 
p = 1 − e − 2  + +  = 1 − 5e −2 = 0,3233235
 0! 1! 2! 

2/9
www.eltemario.com Oposiciones Secundaria – Matemáticas
© Antonio J. Abrisqueta Valcárcel, 2002 Hojas de Problemas – Estadística VI

256. Sobre una circunferencia se toma un punto fijo A y otros n-1 al azar, que
son los vértices de un polígono convexo de n lados que deja en su exterior al centro
de la circunferencia ( n ≥ 3 variable aleatoria). Determinar el número de lados del
polígono esperanza matemática de los considerados.

SOLUCION:

El diámetro que pasa por A determina dos semicircunferencias.


El primer punto lo elegimos en cualquiera de las dos semicircunferencias, y los otros
n-2 en la semicircunferencia que contenga el primer punto.

Designado por Pn , n ≥ 3 , la probabilidad de obtener un polígono de n lados que


verifique el enunciado es:
1 ( n− 2) 1 1
Pn = 1 ............ = n − 2
2 2 2

La esperanza matemática es:



1 ∞
1  ∞ 1 
E = ∑ n n −2 = 2 ∑ n n −1 =2 ∑ n n −1 − (1 + 1)  =
n =3 2 n =3 2  n =1 2 
 
   
  d  1    1 
= 2    − 2  = 2 − 2  = 2( 4 − 2) = 4
  dx  1 − x  x= 1 
2

   1 − 1  
2
 2 
 

257. Dos hormigas están situadas respectivamente en los extremos A y B de la


diagonal de una cuadrícula que consta de pxp
cuadrados ( p ∈ N , p ≥ 1) .
La hormiga situada en A se encamina a B
siempre siguiendo las aristas de los cuadrados
de la cuadrícula. Lo hace siempre adelante,
nunca hacia atrás – siempre hacia arriba o a la
derecha – y en cada uno de los cruces en los que
puede elegir el camino, lo hace aleatoriamente
dando la misma probabilidad a las dos
posibilidades que se le presentan.

De forma similar, la hormiga situada en B se dirige al punto A andando


siempre abajo o bien a la izquierda.
Las dos hormigas caminan a la misma velocidad y salen de los puntos A y B
en el mismo instante.
Se pide:
a) Busque la probabilidad de que las dos hormigas se encuentren en
uno de los cruces.
b) Generalice el resultado anterior en la hipótesis de que la
cuadrícula conste de pxq cuadrados ( p, q ∈ N , p ≥ 1, q ≥ 1).

3/9
www.eltemario.com Oposiciones Secundaria – Matemáticas
© Antonio J. Abrisqueta Valcárcel, 2002 Hojas de Problemas – Estadística VI

SOLUCIÓN:

a) Cada hormiga ha de recorrer 2p aristas para completar su recorrido, luego se


pueden encontrar en el mismo cruce en la mitad de sus recorridos, es decir, al cabo de p
aristas.

En ese momento, cada hormiga ha efectuado un camino de entre los 2p caminos


posibles, p elecciones entre dos posibilidades.

Se encontrarán en un cruce si la hormiga que va de A a B ha elegido k aristas


verticales y la que va de B a A ha elegido p – k aristas verticales, para 0 ≤ k ≤ p .

Es decir, se encontrarán si de un total de 2p elecciones ha n hecho conjuntamente


p elecciones de arista vertical.
2p
 
La probabilidad pedida es  2 p 
p
2

b) Si p + q es impar las hormigas no pueden encontrarse en un cruce ya que ello


supondría que en un mismo tiempo una ha recorrido un mismo par de aristas y la otra
uno impar.
Supondremos , por tanto que p + q es par y sea m = ( p + q ) .
1
2
Suponiendo q ≤ p , al cabo de m elecciones, cada hormiga ha podido recorrer
m m m
c =   +   + ........ +   = 2 m
 0  1  m

caminos distintos (suponiendo que ambas hormigas pueden salir de la cuadrícula


manteniendo el modo de desplazamiento y la probabilidad de cada elección).

Se encuentran en un nudo de la cuadrícula si cuando una ha elegido k aristas


verticales, la otra recorre q - k aristas verticales, con 0 ≤ k ≤ q

La probabilidad de que se encuentren es:

m  m   2 m  p + k
       
k  q − k  = q  = q 
q

∑ c c c2 c 2
k=0

Si es p ≤ q , un razonamiento similar aplicado a los desplazamientos


 p + q
 
 c  , donde
horizontales, conduce a la probabilidad 2
c
   
m m  
m
c =   +   + ..... +   = 2 m
 0  1  m

4/9
www.eltemario.com Oposiciones Secundaria – Matemáticas
© Antonio J. Abrisqueta Valcárcel, 2002 Hojas de Problemas – Estadística VI

258. El aparato de Galton-Pearson es una serie de clavos dispuestos en un tablero


inclinado en la forma que indica la figura adjunta. Al dejar caer bolas desde el
clavo superior, la probabilidad de desviarse a la izquierda en cada choque se
supone que es 1 2 .
a) Calcular la probabilidad de que una bola caiga en B.
b) Calcular la probabilidad de que una bolsa que ha caído en B haya golpeado
el clavo A.

Aparato de Galton-Pearson

SOLUCION:

a) Para que caiga en B han de producirse cuatro giros a la izquierda y siete a la


derecha, luego la probabilidad pedida es:
 11 1 
11

p =   
 4  2 
b) Sea S el suceso que consiste en que la bola llegue a B.
Sea T el suceso que consiste en que la bola llegue a A.
La probabilidad buscada es P(T S ) y resulta:

 6  1   5  1   6  5 
6 5

        
P (T )P(S T )  2  2   2  2   2  2  = 5
P(T S ) = = =
P (S ) 11 1 
11
11 11
    
 4  2  4 

259. Al tomar aleatoriamente un punto del interior de un cuadrado, calcular la


probabilidad de que esté más próximo a algún vértice que al centro de gravedad del
mismo.

5/9
www.eltemario.com Oposiciones Secundaria – Matemáticas
© Antonio J. Abrisqueta Valcárcel, 2002 Hojas de Problemas – Estadística VI

SOLUCION:

Sea ABCD el cuadrado que nos dan.

Supuesto que el punto se toma en el interior


del cuadrado OMCN, un punto equidistará de O y C
si está en MN, y estará más próximo de C que de O
si pertenece al interior del triángulo MCN.

La probabilidad pedida es, por simetría:


4área _ triangulo ( MCN ) 1
p= =
4 área _ triangulo ( OMCN ) 2

260. Suponiendo que se eligen al azar dos valores reales x e y que satisfagan las
condiciones:

 ( x, y ) punto int erior a la curva y = ± − x 2 + 4 x − 3



 x− y−2< 0
 x+ y−2> 0

hallar la probabilidad que el valor y, elegido al azar, sea tal que 2y> 2

SOLUCION:

La curva y = − x 2 + 4 x − 3 ⇔ x 2 + y 2 − 4 x + 3 = 0 es la circunferencia de centro


en C(2,0) y radio 1. Las ordenadas de los puntos de corte de las rectas x − y − 2 = 0 y
x + y − 2 = 0 , perpendiculares entre sí, con la circunferencia son ± 2 2 .

Interpretando geométricamente el problema, la probabilidad buscada es:


π 2 1 2
  r − r
2  Area _ segmento _ del _ arco _ AB 4 π−2

P y > = = 2 = = 0,36
 π 2 π
 2  Area _ sec tor _ AB r
4

6/9
www.eltemario.com Oposiciones Secundaria – Matemáticas
© Antonio J. Abrisqueta Valcárcel, 2002 Hojas de Problemas – Estadística VI

261. Sean n variables aleatorias independientes X 1 , X 2 ,...., X n todas con la


misma ley de probabilidad. Se designa respectivamente por F(x) y por f(x) la
función de distribución y la densidad de probabilidad común a cada una de las
variables X i . ¿Cuáles son las funciones de distribución y de densidad de
probabilidad de las variables Y = Máx( X 1 , X 2 ,...., X n ) y Z = Mín ( X 1 , X 2 ,...., X n ) ?.

SOLUCIÓN:

n
a) Para todo z∈R es P(Y ≤ z ) = Π P ( X i ≤ z ) = F n ( z ) y
i =1

[
P( Z ≤ z ) = 1 − Π[1 − P ( X i ≤ z ) ] = 1 − 1 − F n ( z ) ]
n n

i =1

En efecto:
n
P(Y ≤ z ) = P ( X 1 ≤ z, X 2 ≤ z ,... X n ≤ z ) = Π P ( X i ≤ z ) = F n ( z )
i =1

P( Z ≤ z ) = 1 − ( P( Z > z ) = 1 − P( X 1 > z , X 2 > z ,....., X n > z ) =

= 1 − Π P( X i > z ) = 1 − Π [1 − P( Xi ≤ z )] = 1 − [1 − F ( z ) ]
n n
n

i =1 i =1

b) Sean g y h las funciones de densidad de Y y Z respectivamente. Entonces:

g ( z ) = nF n−1 ( z ) f ( z )
h ( z ) = n[1 − F ( z ) ]n−1 f ( z )
en todos los puntos de continuidad de f.

En efecto:
P(Y ≤ z ) = F n ( z ) ⇒ g ( z ) = nF n −1 ( z ) F ' ( z ) = nF n −1 ( z ) f ( z )
P( Z ≤ z ) = 1 − [1 − F ( z ) ]n ⇒ h ( z ) = −n[1 − F ( z )]n −1 ( − f ( z )) = n[1 − F ( z )]n −1 f ( z )

262. Un plano esta rayado por rectas paralelas separadas entre sí a la distancia
2a.
a) Sobre el plano se lanza al azar una aguja de longitud 2b (b<a). Hallar la
probabilidad de que la aguja corte una recta cualquiera.
b) Si se lanzan n agujas, todas ellas de longitudes menores que 2b, hallar la
probabilidad de que al menos una de las agujas corte una recta cualquiera.

SOLUCIÓN:

a) Designaremos por ρ la distancia del centro de la aguja a la línea más cercana. Y


sea α el ángulo que forma la aguja con la línea más cercana.

7/9
www.eltemario.com Oposiciones Secundaria – Matemáticas
© Antonio J. Abrisqueta Valcárcel, 2002 Hojas de Problemas – Estadística VI

Es evidente que si ρ ∈ [0, a ] y α ∈ [0, π) ,obtenemos todos los resultados


posibles.
Para que la aguja interseque a la línea más próxima, se ha de verificar que
α ∈ [0, π) y 0 ≤ ρ ≤ bsenα .
Luego la probabilidad pedida es:

p=
∫ 0
bsenα∂α

2b
πa πa

b) Sean 2b1 ,2b2 ,...2bn las longitudes de las agujas. La probabilidad buscada es:

 2b  2b   2b 
p = 1 − 1 − 1 1 − 2 ......1 − n 
 πa  πa   πa 

263. Determinar la probabilidad para que elegidos al azar dos valores de “x” e
“y”, que satisfagan las condiciones 2-x-y<0; 3y-4x-6<0;y+2x-7<0; resulte que el
valor “y”, elegido al azar, sea <0.

SOLUCIÓN:

8/9
www.eltemario.com Oposiciones Secundaria – Matemáticas
© Antonio J. Abrisqueta Valcárcel, 2002 Hojas de Problemas – Estadística VI

Operando se obtiene las siguientes coordenadas:

3 7
A(0,2), B( ,4) ,C(5,-3),D(2,0),E( ,0),F(0,7)
2 2
3 3
AB( ,2), AC (5, −5), DE ( ,0) y DC (3, −3)
2 2

Calculemos ahora las áreas de los triángulos ABC y DEC en el espacio.


r r r
i j k
r 35 r
0 =  − −10 k = − k
15
AB ∧ AC = 3 2 2
 2  2
5 −5 0

1 35 35
Luego S(ABC)= =
2 2 4
r r r
i j k
9r 19 9
Análogamente DE ∧ DC = 3 2 0 0 = − k y S(DEC)= =
2 22 4
3 −3 0

La probabilidad pedida es:


9
9
p= 4 =
35 35
4

9/9
www.eltemario.com Oposiciones Secundaria – Matemáticas
© Antonio J. Abrisqueta Valcárcel, 2002 Hojas de Problemas – Estadística VII

264.- La distribución de probabilidad de una variable aleatoria discreta es la


siguiente:

xj 1 2 3 … n
Pj 1 1 1 … 1
n4 +1 n4 + 24 n4 + 34 n4 + n 4

Se pide:
1º) Esperanza matemática o valor medio probable de la variable.
2º) Límite de la esperanza cuando n tiende a infinito.

RESOLUCIÓN.-

1º) Evidentemente ∑P i ≠ 1 , luego X no es una variable aleatoria. Si lo fuera:

1 1 1
E( X ) = 1 +2 + ... + n
n +1
4
n +2
4 4
n + nn
4

k
n n
k 1
2º) lim E ( X ) = lim ∑ = lim ∑ n =
n →∞ n →∞ 4 n →∞ n k =1 4
k =1 k  k
n2 1+   1+  
n n

k
n 2
1 1 1 2x 1
= lim ∑ n = ∫ dx = ( Arg sh 1 − Arg sh 0) =
n →∞ 2n
k =1  k 2 
2 2 0 1 + x2 ( ) 2 2
1 +    
 n  
1
= Arg sh 1 = 0,4407
2

Observando que el valor obtenido es menor que 1, se confirma que la distribución


dada no es una distribución de probabilidad puesto que el valor de la esperanza cae
siempre dentro del recorrido de la variable.

265.- Se dispone de una urna que contiene cinco bolas rojas y cuatro bolas blancas.
Se extraen de su interior cinco bolas y estamos interesados en definir la variable
aleatoria X = número de bolas blancas extraídas. Completar la adjunta tabla en los
dos casos siguientes:
a) Las cinco bolas se extraen de una en una, devolviéndolas cada vez a la urna.
b) Las cinco bolas se extraen de una sola vez.

r 0 1 … … …
P(X = r) P0 P1 … … …

1/10
www.eltemario.com Oposiciones Secundaria – Matemáticas
© Antonio J. Abrisqueta Valcárcel, 2002 Hojas de Problemas – Estadística VII

c) Calcular en cada uno de los casos X y σ x2

RESOLUCIÓN.-

a)
 5  4   5 
0 5

P0 =      = 0'05292 , siendo r = 0


 0  9   9 
 5  4   5 
1 4

P1 =      = 0'21169 , siendo r = 1


 1  9   9 
 5  4   5 
2 3

P2 =      = 0'33870 , siendo r = 2


 2  9   9 
 5  4   5 
3 2

P3 =      = 0'27096 , siendo r = 3


 3  9   9 
 5  4   5 
4 1

P4 =      = 0'10838 , siendo r = 4


 4  9   9 
 5  4   5 
5 0

P5 =      = 0'01734 , siendo r = 5


 5  9   9 

P1 + P2 + P3 + P4 + P5 = 0'99999 = 1

b)
 4  5 
  
P0 =    =
0 5 1
= 0'00793 , siendo r = 0
 9 126
 
 5
 4  5 
  
P1 =    =
1 4 20
= 0'15873 , siendo r = 1
9  126
 
5 
 4  5 
  
P2 =    =
2 3 60
= 0'47619 , siendo r = 3
 9 126
 
 5

 4  5 
  
P3 =    =
3 2 40
= 0'31746 , siendo r = 3
 9 126
 
 5

2/10
www.eltemario.com Oposiciones Secundaria – Matemáticas
© Antonio J. Abrisqueta Valcárcel, 2002 Hojas de Problemas – Estadística VII

 4  5 
  
P4 =    =
4 1 5
= 0'03968 , siendo r = 4
 9 126
 
 5
P5 = 0 , siendo r = 5

P1 + P2 + P3 + P4 + P5 = 0'99999 = 1

c) En el caso a) estamos ante distribución binomial de parámetros

4
( n, p) = (5, )
9
Por consiguiente
4 20
X = n· p = 5· = = 2'22
9 9

4 5 100
σ x2 = np(1 − p) = 5 = = 1'2345
9 9 81

En el caso b) estamos en una distribución hipergeométrica, luego, siendo el número


de bolas N = 9, el de bolas blancas n = 4 y el número de bolas extraídas M = 5, se tiene
que:

M 5 20
X =n =4 = = 2'22
N 9 9

M N −M N −n 5 4 5 400
σ x2 = n =4 = = 0'6172
N N N −1 9 9 8 684

266.- Disponemos de dos urnas con N bolas cada una, numeradas de 1 a N en


ambas. Se extrae simultáneamente una bola de cada urna y sin devolverlas
repetimos esta operación, hasta vaciar las urnas.
a) Hallar la probabilidad de que en ninguna de las extracciones los números de
las bolas coincidan.
b) Hallar el límite de dicha probabilidad cuando N tiende a infinito.

RESOLUCIÓN.-

Puesto que las N! ordenaciones que resultan al extraer las N bolas de la primera urna
son equiprobables, el problema queda simplificado en su notación si suponemos fijada
la ordenación que resulta de las extracciones en la primera urna.
Así pues, sea Ai (1 ≤ i ≤ N ) es suceso que consiste en que haya coincidencia en la
extracción i.
( N − 1)! 1
Evidentemente p ( Ai ) = =
N! N

3/10
www.eltemario.com Oposiciones Secundaria – Matemáticas
© Antonio J. Abrisqueta Valcárcel, 2002 Hojas de Problemas – Estadística VII

Sea H = A1 ∪ A2 ∪ ... ∪ AN . Evidentemente H es el suceso que consiste en que al


menos haya una coincidencia.
Se tiene:
( N − 2)! 1
p ( Ai ∩ A j ) = = (i ≠ j )
N! N ( N − 1)
( N − 3)! 1
p ( Ai ∩ A j ∩ Ak ) = = (i, j, k diferentes)
N! N ( N − 1)( N − 2)
M
( N − ( N − 1))! 1 ( subindices
p ( Ai1 ∩ Ai 2 ∩ ... ∩ Ai N −1 ) = =
N! N ( N − 1)...2 diferentes )
( N − ( N − 0))! 1
p ( Ai1 ∩ Ai 2 ∩ ... ∩ Ai N ) = =
N! N!
En consecuencia:
N 1  N  1 N 1
p ( H ) = p ( H = A1 ∪ A2 ∪ ... ∪ AN ) =   −   +   + ... +
 1  N  2  N ( N − 1)  3  N ( N − 1)( N − 2)
 N  1 N 1
+ ( −1) N   + (−1) N +1   =
 N − 1 N ( N − 1)...2  N  N!
1 1 1 1 1
= 1 − + − + ... + (−1) N + ( −1) N +1
2! 3! 4! ( N − 1)! N!

a) El suceso que consiste en que no haya coincidencias es H . Así:

1 1 1 1
p ( H ) = 1 − (1 −
+ − + ... + ( −1) N +1 ) =
2! 3! 4! N!
1 1 1 1
= − + − ... + ( −1) N
2! 3! 4! N!

1 1 1
b) Puesto que e −1 = 1 − + − + ..., se tiene que lim p ( H ) = e −1
1! 2! 3! N →+∞

267.- Dos personas A y B, juegan una competición de ajedrez, la cual será ganada
por el primero de los dos jugadores que gane dos partidas. Las probabilidades que
tiene A de ganar, hacer tablas o perder en una partida son a, b, c, respectivamente.
¿Cuál es la probabilidad de que A venza en la competición?.

RESOLUCIÓN.-

Para que A gane la competición se ha n de celebrar n + 2 partidas con n = 0, 1, 2,…


La probabilidad de que gane A la competición en la partida n + 2 exactamente, es:

 n + 1 2 n  n + 1 n  2 n −1
 a b +   a b c si n ≥ 1
 1   1  1 

4/10
www.eltemario.com Oposiciones Secundaria – Matemáticas
© Antonio J. Abrisqueta Valcárcel, 2002 Hojas de Problemas – Estadística VII

a2 si n = 0

La probabilidad buscada es:


 n + 1 2 n ∞  n + 1 n  2 n −1 ∞ ∞
p = a 2 + ∑   a b +∑    a cb = a 2 + a 2 ∑ ( n + 1) b n +a 2 c∑ ( n + 1)nb n −1 =
n =1  1  n =1  1  1  n =1 n =1

∞ ∞
= a 2 ∑ ( n + 1) b n +a 2 c∑ ( n + 1)nb n −1
n= 0 n =1


1
Teniendo en cuenta que ∑x n
=
1− x
si x < 1 , derivando:
n= 0

∞ ∞ ∞
1 2
∑ nx n−1 = ∑ (n + 1) x n =
n =1 n= 0 (1 − x ) 2
y ∑ (n + 1)nx
n =1
n −1
=
(1 − x) 3

Por consiguiente

1 2 a 2 (1 − b) + 2a 2 c a 2 ( a + c) + 2a 2 c a 3 + 3a 2 c
p = a2 + a 2
c = = =
(1 − b) 2 (1 − b ) 3 (1 − b) 3 ( a + c) 3 (a + c )3

268.- Dado un segmento cualquiera, hallar la probabilidad de obtener, por


trisección, los tres lados de un triángulo.

RESOLUCIÓN.-

Entenderemos por trisección la elección de dos puntos al azar del interior del
segmento. Es evidente que no supone restricción alguna identificar al segmento con el
intervalo [0, 1]. La elección de dos puntos en el segmento es entonces la elección de dos
números x ∈ (0,1) , y ∈ (0,1) , x ≠ y .
La condición necesaria y suficiente para que con tres segmentos se pueda formar un
triángulo es que la longitud de cada uno de los segmentos sea menor que la suma de los
otros dos. Refiriendo estas condiciones a los números x e y obtenemos:

1 1 1
Si x < y, 0< x< , y−x< , < y <1
2 2 2
1 1 1
Si y < x, 0< y< , x−y< , < x <1
2 2 2

Interpretando geométricamente estas condiciones, identificando (x , y) con un punto


del plano, resulta que los puntos que verifican la condic ión pertenecen a la parte rayada
de la figura. La probabilidad es por tanto:

5/10
www.eltemario.com Oposiciones Secundaria – Matemáticas
© Antonio J. Abrisqueta Valcárcel, 2002 Hojas de Problemas – Estadística VII

1 1
+
p= 8 8=1
1 4

269.- En el interior de un cuadrado OABC cuyo lado es igual a L = 10 cm. Se


elige un punto P(x, y) al azar. Hallar la probabilidad de que se pueda formar un
triángulo cuyos lados tengan longitudes 4, x, y.

RESOLUCIÓN.-

Se podrá formar un triángulo si y solo si

4< x+y
x< 4+ y
y< 4+ x

Geométricamente la probabilidad equivale a la de elegir en el cuadrado OABC un


punto situado en la zona no rayada.

Dicha probabilidad es:

100 − 8 − 18 − 18 56
P= = = 0,56
100 100

definida en (− ∞ ,+∞) , se pide:


k
270.- Dada la función f ( x ) =
1 + x2
a) Determinar k para que f(x) sea una función de densidad.
b) Hallar la función de distribución.
c) Calcular la probabilidad P( −2 ≤ X ≤ 2) .

RESOLUCIÓN.-

+∞ k
a) Por ser ∫−∞ 1 + x 2
dx una integral convergente, resulta:
+∞ 1 +r 1
1= k∫ dx = k lim ∫ dx =
−∞ 1+ x 2 r →+∞ − r 1 + x 2

= k lim (arctan r − arctan( −r ) ) = k (π / 2 − ( −π / 2) ) = kπ


r →+∞

Así, k = 1/ π

1/ π
dt = (arctan x − ( −π / 2) ) = (arctan x + π / 2 )
1 1

x
b) ∀x ∈ IR : F ( x) =
−∞ 1+t 2
π π

6/10
www.eltemario.com Oposiciones Secundaria – Matemáticas
© Antonio J. Abrisqueta Valcárcel, 2002 Hojas de Problemas – Estadística VII

P( −2 ≤ X ≤ 2) = F ( 2) − F (−2) =
1
(arctan 2 + π / 2 ) − 1 (arctan( −2) + π / 2) =
π π
c)
=
1
(arctan 2 − arctan( −2) ) = 2 arctan 2
π π

271.- Disponemos de dos urnas con N bolas cada una, numeradas de 1 a N en


ambas. Se extrae simultáneamente una bola de cada urna y sin devolverlas
repetimos esta operación, hasta vaciar las urnas.

a) Hallar la probabilidad de que en ninguna de las extracciones los números


de las bolas coincidan.
b) Hallar el límite de dicha probabilidad cuando N tiende a infinito.

RESOLUCIÓN.-

Puesto que las N! ordenaciones que resultan al extraer las N bolas de la primera urna
son equiprobables, el problema queda simplificado en su notación si suponemos fijada
la ordenación que resulta de las extracciones en la primera urna.

Así pues, sea Ai (1 ≤ i ≤ N ) el suceso que consiste en que haya coincidencia en la


extracción i.

( N − 1)! 1
Evidentemente p ( Ai ) = = .
N! N

Sea H = A1 U A2 U ... U AN . Evidentemente H es el suceso que consiste en que al


menos haya una coincidencia.

Se tiene:

( N − 2)!
p ( Ai ∩ A j ) = =
1
(i ≠ j )
N! N ( N − 1)
( N − 3)! 1
p ( Ai ∩ A j ∩ Ak ) = = (i, j, k diferentes)
N! N ( N − 1)( N − 2)
.
.
.
( N − ( N − 1))! 1
p ( Ai1 ∩ Ai 2 ∩ ... ∩ Ai N −1 ) = = (subíndices
N! N ( N − 1)...2
diferentes)
( N − ( N − 0))! 1
p ( A1 ∩ A2 ∩ ... ∩ AN ) = =
N! N!

En consecuencia:

7/10
www.eltemario.com Oposiciones Secundaria – Matemáticas
© Antonio J. Abrisqueta Valcárcel, 2002 Hojas de Problemas – Estadística VII

 N  1 N  1 N 1
p ( H ) = p ( A1 ∩ A2 ∩ ... ∩ AN ) =   −   +   + ... +
 1  N  2  N ( N − 1)  3  N ( N − 1)( N − 2)
 N  1 N  1
+ ( −1) N   + ( −1) N +1   =
 N − 1 N ( N − 1)... 2  N  N!
1 1 1 1 1
= 1− + − + ... + (−1) N + ( −1) N +1
2! 3! 4! ( N − 1)! N!

a) El suceso que consiste en que no haya coincidencias es H .

Así:

p ( H ) = 1 − 1 − + − + ... + ( −1) N +1  =


1 1 1 1
 2! 3! 4! N! 
1 1 1 1
= + − − ... + (−1) N
2! 3! 4! N!

1 1 1
b) Puesto que e −1 = 1 − + − + ... se tiene que lim p ( H ) = e −1
1! 2! 3! N →+∞

272.- Obtener la probabilidad de que un cartero que lleva tres cartas con
distintos destinatarios entregue al menos una de ellas correctamente al efectuar
dicha entrega al azar sin mirar dirección alguna.

RESOLUCIÓN.-

Usaremos el método general para resolver este tipo de problemas.

Sea Ai el suceso que consiste en que el destinatario i recibe su carta. Habrá que
calcular:

p ( A1 ∩ A2 ∩ A3 )

p ( A1 ∩ A2 ∩ A3 ) = p ( A1 ) + p ( A2 ) + p ( A3 ) − p ( A1 ∩ A2 ) − p ( A1 ∩ A3 ) −
2! 2! 2! 1 1 1 1
− p ( A2 ∩ A3 ) + p( A1 ∩ A2 ∩ A3 ) = + + − − − + =
3! 3! 3! 3! 3! 3! 3!
2! 1 1 1 1 2
= 3 −3 + =1− + =
3! 3! 3! 2! 3! 3

273.- Se dispone de dos urnas, blanca y negra, conteniendo cada una de ellas
bolas blancas y negras en las proporciones siguientes:

Urna blanca: 1-r bolas blancas y r bolas negras.

8/10
www.eltemario.com Oposiciones Secundaria – Matemáticas
© Antonio J. Abrisqueta Valcárcel, 2002 Hojas de Problemas – Estadística VII

Urna negra: s bolas blancas y 1-s bolas negras.

Cuando en una extracción se saca bola blanca, la extracción siguiente se efectúa


en la urna blanca, y en la urna negra en el caso contrario. Después de cada tirada
se repone la bola.

La primera extracción se realiza en la urna blanca. ¿Cuál es la probabilidad pn


para que la n-ésima bola sacada sea blanca?. Calcular lim p n .
n →∞

RESOLUCION.-

Sea pn la probabilidad de que la n-ésima bola sacada sea blanca. ¿Cuál es la


probabilidad pn para que la n-ésima bola sacada sea blanca y qn la probabilidad de que
sea negra. Se verirfica:

p n = (1 − r ) p n −1 + sq n −1   p n  1 − r s 
n
1 
⇔ q  =  r 1 − s  0  , para que n>1.
q n = rp n −1 + (1 − s ) q n −1   n   

Como la primera extracción se realiza en la urna blanca, ello equivale a suponer una
extracción 0 en la que se hubiera sacado bola blanca; así:

 p1  1 − r s  1   p n  1 − r s 
n
1 
q  =  r 1 − s  0 . Luego, ∀n ∈ IN : q  =  r 1 − s  0 .
 1     n    

1 − r s 
Los valores propios de la matriz 
1 − s 
son 1 y 1-r-s; los correspondientes
 r
vectores propios filas son [1, 1], [r, -s], suponiendo r ≠ 0 ó s ≠ 0 . Así:

−1
1 − r s  1 1  1 0  1 1 
 r = =
1 − s  r − s  0 1 − r − s  r − s 
  

− 1  − s − 1 1 0  1 1 
=
r + s − r 1  0 1 − r − s   r − s 

Por tanto:

−1
 p n  1 − r
n
s  1  1 1  1 0  1 1  1 
 q  =  r 1 − s  0 =  r − s  0 (1 − r − s ) n   r − s  0 =
 n          
− 1  − s − −1 1 0  1 1  1 
=  =
1  0 (1 − r − s )   r − s   0
  n 
r + s − r
1
− 1  − s − (1 − r − s) n  1  1  s + r (1 − r − s ) n 
=   =  
r + s − r (1 − r − s) n  r  r + s r − r (1 − r − s ) n 

9/10
www.eltemario.com Oposiciones Secundaria – Matemáticas
© Antonio J. Abrisqueta Valcárcel, 2002 Hojas de Problemas – Estadística VII

s + r (1 − r − s ) n s
En suma p n = y lim p n = , si 1 − r − s ≠ 1 , ya que de
r+s n → +∞ r+s
0 ≤ r ≤ 1 , 0 ≤ s ≤ 1 se deduce que 1 - r - s ≤ 1 .

r = s = 0 ⇒ p n = 1

Como 1 − r − s = 1 ⇒  1 + ( −1) n
r = s = 1 ⇒ p n = ∀n ∈ IN
 2

Resulta que si r = s = 0, lim p n = 1 y si r = s = 1, no existe lim p n .


n →∞ n →∞

10/10

También podría gustarte